Está en la página 1de 230

ABG

Filosofía y Lógica Jurídica


FLJ · n o v e n o c u at r i m e s t r e
Estimado Alumno:

Códigos QR Esta materia se ha diseñado con Códigos QR para agilizar el acceso a los
contenidos audiovisuales que el docente ha desarrollado especialmente
para Usted.
Para poder visualizar los videos utilice el lector de código QR que trae por
defecto su teléfono o dispositivo móvil. En caso de no disponer de uno,
puede descargarlo desde Google Play si su sistema operativo corresponde
a Android o desde la App Store si utiliza IOS.

Una vez instalado el software en su dispositivo móvil:


1) Abra la aplicación
2) Apunte con la cámara de su dispositivo hacia los códigos QR que
encuentre en el interior de su materia.

Los códigos estarán identificados con el ícono corresdondiente a los videos


elaborados por sus docentes.
Índice

Presentación 4

Macroobjetivos 8

Programa 8

Agenda 10

Mapa Conceptual 10

Material 11

Glosario 11

Módulos
Módulo 1 22
Módulo 2 34
Módulo 3 57
Módulo 4 81
Módulo 5 99
Módulo 6 111
Módulo 7 129
Módulo 8 158

Anexo: Lecturas básicas 176

Referencias

Ícono para indicación de Lectura Básica (disponible en ANEXO): ∙

Ícono para indicación de Lectura Complementaria (disponible en plataforma):

Impresión total del documento 230 páginas

EDUBP | Abogacía | Filosofía y Lógica Jurídica - pag. 3


Estimados alumnos:
Presentación
Comienzo dándoles una cordial bienvenida a la materia Filosofía y Lógica
Jurídica que, como la experiencia docente indica, produce cierta incertidum-
bre y temor en algunos de ustedes, probablemente por el carácter abstracto
de este ámbito del conocimiento.

Una manera adecuada para emprender el estudio de estas disciplinas parte


de aceptar que toda simplificación y esclarecimiento de problemas de dis-
cernimiento requiere cierto grado de abstracción. La filosofía contemporánea
considera que la tarea del filósofo es el análisis lógico del lenguaje, la clarifica-
ción lógica de los conceptos, la determinación del significado de una expre-
sión lingüística. Para ello debe, entre otras cosas, registrar usos lingüísticos
vigentes, ambigüedades e indeterminaciones de las expresiones lingüísticas.
En ese mismo sentido, puede decirse que la teoría del derecho emprende el
análisis del mismo desde un punto de vista general, formal y estructural.
Veamos que implican cada una de estas perspectivas o enfoques.

General: porque analiza conceptos comunes a los diversas partes de un


mismo ordenamiento. Formal: porque analiza el derecho como un conjunto
de expresiones del legislador, subespecie de lenguaje prescriptivo y final-
mente estructural, porque analiza el conjunto de conceptos relativos a la
introducción y eliminación de normas y a la resolución de conflictos normati-
vos, en síntesis, al conjunto de fuentes de derecho y sus mutuas relaciones.

La práctica del lenguaje jurídico, como la de todo lenguaje natural, genera un


entramado de creencias que expresan la forma en que pensamos la realidad
jurídica. Ahora bien, ustedes pueden preguntarse ¿Para qué poner en cues-
tión tópicos que pueden ser comprendidos apelando a nuestras intuiciones
más comunes? ¿Por qué es necesario someter a revisión conceptos que,
para estudiantes avanzados como ustedes, resultan tan obvios y claros? Una
respuesta posible sería: ¿qué seguridad tienen de que el derecho es como
lo piensan?, otra respuesta más optimista podría ser ¿Acaso no podemos
acceder a conceptos mejores que los del sentido común para pensar la
realidad jurídica?

En la presente asignatura, el objetivo es clarificar aquellos conceptos más


generales con los que pensamos la realidad jurídica y esclarecer las interco-
nexiones de esos conceptos. Aprendemos a operar con una amplia gama de
nociones como, por ejemplo, sanción, responsabilidad, deber. Es cierto que
aprendemos el uso de las palabras que expresan estos conceptos de formas
diversas durante el cursado de las diferentes asignaturas de la carrera; pero
las aprendemos sin la ayuda de una reflexión teórica.

La lógica proporciona el medio para establecer los conceptos fundamen-


tales. En el mayor nivel de abstracción esa disciplina nos indica una caracte-
rística estructural de nuestro uso del lenguaje, una dimensión sintáctica, la
cual consiste en predicar algo sobre algo, – estructura sujeto-predicado. Un
paso ulterior consiste en interesarnos por saber qué es lo que predicamos
y de qué cosas lo predicamos – dimensión semántica y, también, para qué
otros propósitos, además de la predicación, usamos el lenguaje – dimensión
pragmática.

EDUBP | Abogacía | Filosofía y Lógica Jurídica - pag. 4


Así como el gramático analiza las partes que componen una oración para
hacer explícitas las reglas y principios que hacen que un conjunto de pala-
bras, constituya una oración portadora de sentido y no una simple agrupa-
ción de palabras sin él, podría decirse, que el filósofo analiza las partes que
componen un concepto. Una analogía similar podría hacerse con la labor de
un cartógrafo que elabora mapas útiles para orientar al viajero al punto de
llegada sin correr el riesgo de extraviarse.

Otros suelen comparar la labor del filósofo con la de un terapeuta especiali-


zado en resolver desórdenes de índole intelectual, suministrando una técnica
que ayude a restablecer el orden, a liberar la mente de confusiones o falsas
concepciones, capacitando a las personas para pensar con claridad.

Para ambas perspectivas, la fuente principal de confusión reside en el hecho


de desvincular las palabras de su uso habitual, de las cuestiones que les
proporcionan significación.

Las analogías del gramático y del terapeuta reflejan diferentes presupuestos.


La primera de ellas sugiere que existe una estructura subyacente general a
nuestro uso de los conceptos y, que es función de la filosofía hacer explícita
dicha estructura, esto es, saber qué hacemos cuando usamos los conceptos.
La analogía con la terapia, en cambio, sugiere que es función de la reflexión
liberarnos de las perplejidades que suelen tener lugar cuando nuestros con-
ceptos se entrecruzan. El propósito del filósofo no es explicar, pues no hay
nada que explicar o hacer explícito, sino tan sólo indicar la fuente u origen de
las confusiones. Pero, por más humilde que parezca, esta última perspectiva
es suficiente para hacer explícita la necesidad de distinguir entre el lenguaje
del derecho y el lenguaje de los juristas estando, el primero, destinado a
satisfacer propósitos prácticos y, el segundo, a dar respuesta a inquietudes
o intereses teóricos.

A modo de síntesis y como respuesta a nuestros cuestionamientos sobre el


interés que puede tener la reflexión filosófica aplicada al dominio del dere-
cho, podríamos decir que no nos resulta satisfactorio limitarnos al uso del
lenguaje del derecho, sino que aspiramos a establecer conexiones para
obtener una visión unificada del fenómeno jurídico.

Hay una propensión natural de búsqueda de una visión unificada del mundo
que puede conducir a un modelo deformado y parcial, a un despotismo inte-
lectual. La filosofía entendida como tarea de revisión de nuestros esquemas
conceptuales ordinarios puede resultar útil planteando problemas, exigiendo
aclaraciones y ajustes de dichos esquemas, contribuyendo, de ese modo, a
abrir espacios para nuevos desarrollos de nuestros esquemas. La tarea tanto
de la lógica como de la filosofía, tal como es abordada en este curso, es des-
componer ideas complejas, a fin de analizar sus partes y reconstruirlas de un
modo más claro y racional.

Por último, intentaré esclarecer de modo sintético la relación entre la lógica y


el derecho, dando respuestas a dos cuestiones preliminares: ¿de qué trata la
lógica? y, ¿cuál es su relevancia práctica para el estudio del derecho?

EDUBP | Abogacía | Filosofía y Lógica Jurídica - pag. 5


La respuesta a la primera de las cuestiones planteadas tropieza con la con-
fusión generada por el modo ambiguo en que se usa la palabra “lógica”. En
efecto, en el lenguaje ordinario el término es empleado para designar una
cualidad equivalente a “razonable”. Este sentido ordinario hace referencia a
una cualidad asociada con nuestra forma de pensar habitual o normal, según
la cual, afirmamos que algo “es lógico” porque “tiene sentido”, “es obvio” o
“es razonable”. Por el contrario, en un sentido teórico que surge de la con-
cepción aristotélica, la lógica no describe cómo normalmente la gente razona
o piensa, sino que realiza un análisis normativo del razonamiento, un análisis
de cómo se debe pensar para pensar correctamente. A su vez, dicha con-
cepción, sostiene que un razonamiento es correcto cuando no es contradic-
torio y, es lógicamente válido cuando es apto para preservar la verdad desde
las premisas a la conclusión. El propósito de capturar inferencias válidas es
considerado más importante, en este sentido de “lógica”, que el grado de
persuasión que puedan poseer los argumentos para algún auditorio.

La lógica aristotélica se ocupó de elaborar representaciones formales y sis-


temáticas de argumentos válidos consistentes en esquemas silogísticos del
tipo “Todos los A son B, todos los B son C; por tanto, todos los A son C”. Toda
instancia de un esquema de argumento válido que contenga premisas ver-
daderas tendrá una conclusión verdadera. A su vez, las instancias se obtie-
nen reemplazando cada letra del esquema de argumento por una expresión
de la categoría lógica apropiada, por ejemplo: “Todos los aviones pueden
estrellarse, Todos los DC-10 son aviones, por tanto, Todos los DC-10 pueden
estrellarse. Pero esta perspectiva tradicional de la lógica no proporcionaba las
razones por las cuales ciertos esquemas de argumentos resultaban válidos,
simplemente indicaba estructuras de argumento que poseían esa cualidad.

En épocas más recientes se ha restringido aún más el uso del término lógica
para aludir, exclusivamente, a la disciplina que se ocupa de sistemas lógico-
formales. Esta concepción moderna de la lógica, que tiene su origen en la
obra de Frege, es denominada concepción deductiva - formal. Se considera
que es “deductiva” y “formal” porque trata de capturar tanto las clases de infe-
rencias que preservan necesariamente la verdad, como las reglas de forma-
ción de argumentos que garantizan la validez de tales inferencias. Cada letra
de un esquema de argumento válido representa una oración, como surge
claramente del ejemplo citado. Pero, la validez de los esquemas de argu-
mento no surge de la verdad o falsedad de premisas y conclusión, sino de la
forma en que, tanto premisas como conclusión, se hallan conectadas dentro
del argumento. De ahí, la importancia que posee para la lógica moderna la
determinación del significado de ciertas expresiones que cumplen la función
de conectar las oraciones o enunciados que forman parte de un argumento,
expresiones tales como la negación, conjunción, disyunción incluyente o
excluyente, condicional, bicondicional y cuantificadores. La determinación
del significado de las conectivas o constantes lógicas se realiza por medio
de las conocidas tablas de verdad.

Es importante remarcar que Frege no proporcionó un análisis sintáctico del


lenguaje natural, sino que elaboró un lenguaje formalizado cuyas sentencias
tuvieran, a diferencia de las sentencias del lenguaje natural, una estructura
sintáctica precisa. Parece acertado suponer que consideró al lenguaje natu-
ral como resistiendo todo intento de sistematización definida y coherente.

EDUBP | Abogacía | Filosofía y Lógica Jurídica - pag. 6


Pese a ello, muchos lógicos y filósofos posteriores pensaron que era posible
analizar el lenguaje natural de un modo satisfactorio exhibiendo sus senten-
cias como si tuvieran una estructura análoga a las sentencias del lenguaje
formalizado. La idea de que el lenguaje formalizado era la herramienta ade-
cuada para la construcción de teorías científicas llegó a ser indiscutible y, fue
adoptada por los profesores Carlos Alchourrón y Eugenio Bulygin, autores
de la obra Introducción a la Metodología de las Ciencias Jurídicas y Sociales,
en la cual emplean las herramientas de la lógica contemporánea con el pro-
pósito de llevar a cabo una reconstrucción racional del discurso de la cien-
cia jurídica. Algunos capítulos relevantes de esta obra serán objeto de trata-
miento en el último módulo del programa de la asignatura.

El propósito de utilizar las herramientas lógicas para el análisis de sentencias


del lenguaje natural, motivó que algunos lógicos afirmaran que las formas de
composición de sentencias ideadas por Frege resultaban insuficientes, pues
existen otras formas importantes de sentencias que no están incluidas en su
lenguaje formalizado. De ese modo, indagaron el comportamiento lógico de
palabras modales, tales como “necesario” “obligatorio”, etc., y, extendieron
el método de formalización hasta incluir el tratamiento lógico de enuncia-
dos que contienen esos términos, dando lugar a las denominadas lógicas
modales. Uno de sus precursores fue C. I. Lewis a comienzos del siglo XX.
Estas lógicas, como ya dijimos, incorporan operadores tales como “nece-
sariamente” y “posiblemente” y, axiomas que gobiernan el comportamiento
lógico de esos operadores. La lógica deóntica, o lógica del deber, surge
como un desprendimiento de los cálculos de lógica modal a partir de los
análisis contenidos en el artículo Deontic Logic de G.H. von Wright publicado
en 1950. La lógica deóntica es objeto de análisis en la segunda parte del
séptimo módulo del programa.

En cuanto al segundo interrogante, relativo a la cuestión de la relevancia


práctica del análisis lógico, cabe afirmar que los cálculos de lógica modal
deóntica constituyen un método de lógica formal aplicable a sentencias pres-
criptivas, esto es, sentencias que no poseen valor de verdad. El propósito de
esos cálculos es dar cuenta de relaciones lógicas entre normas que sumi-
nistren algún grado de soporte a las conclusiones que resultan deducibles
a partir de las mismas. Un problema central de la teoría del derecho es pro-
porcionar una explicación del vínculo lógico existente entre la norma general
aplicable y la sentencia judicial resultante de su aplicación.

Los profesores Carlos Alchourrón y Eugenio Bulygin fueron, de modo indis-


cutible, los que proporcionaron la elaboración más precisa y sofisticada del
modelo deductivo de fundamentación de las sentencias judiciales. Para ello,
utilizaron los métodos de análisis de la lógica contemporánea para la refor-
mulación y sistematización de los contenidos del derecho y, determinaron
las condiciones que tiene que satisfacer una fundamentación racional de la
decisión judicial.

Surge de estas observaciones preliminares que consideramos el conoci-


miento del modelo teórico propuesto importante a los fines de mejorar nues-
tro modo de entender lo que hacen quienes operan con el derecho. A modo
de conclusión, estimamos valioso todo el esfuerzo y voluntad que se invierta
para lograr dicho objetivo y, toda la buena disposición para disfrutarlo.

EDUBP | Abogacía | Filosofía y Lógica Jurídica - pag. 7


Macroobjetivos • Desarrollar hábitos de lectura reflexiva y crítica de textos sobre cuestio-
nes lógico-filosóficas, con el propósito de mejorar la capacidad de análi-
sis y explicación del fenómeno jurídico.
• Adquirir habilidades en el uso de herramientas lógicas, para lograr una
articulación racional de nuestros argumentos frente a futuros clientes e
instituciones públicas.
• Obtener un conocimiento sistemático del complejo fenómeno jurídico y
de las dificultades involucradas en su implementación, con el objetivo de
incrementar nuestra capacidad de delimitar problemas jurídicos y pro-
nosticar soluciones al ejercer el derecho.
• Integrar los conocimientos adquiridos y aplicar los criterios y principios
generales proporcionados por la asignatura, con el propósito de resolver
casos prácticos.
• Evaluar críticamente los contenidos del derecho vigente, con la finalidad
de mejorar la normativa y modificar prácticas deficitarias en el ejercicio
de la profesión.

MÓDULO I. LO QUE LOS JURISTAS SABEMOS Y LO QUE CREEMOS


Programa SABER. PREGUNTAS PERSISTENTES
Unidad 1
• Perplejidades de la teoría jurídica.
• El derecho como órdenes coercitivas.

MÓDULO II. NORMAS, MANDATOS, ÓRDENES Y AMENAZAS. NUESTRO


HÁBITO DE OBEDIENCIA Y LAS OBLIGACIONES JURÍDICAS
Unidad 2
• La diversidad de normas jurídicas.
• La teoría del soberano y el hábito de obediencia.

MÓDULO III. UNA CONCEPCIÓN ALTERNATIVA. EL DERECHO COMO


PRÁCTICA SOCIAL DE UN CONJUNTO DE INDIVIDUOS
Unidad 3
• Un nuevo punto de partida: el derecho como unión de reglas primarias
y secundarias.

Unidad 4
• La idea de obligación y los elementos del derecho.
• Los fundamentos de un sistema jurídico.

EDUBP | Abogacía | Filosofía y Lógica Jurídica - pag. 8


MÓDULO IV. FORMALISMO (LOS JUECES SON LA “BOCA DE LA LEY”)
Y ESCEPTICISMO ANTE LAS REGLAS (EL DERECHO ES LO QUE LOS
JUECES DICEN QUE ES)
Unidad 5
• La textura abierta del derecho
• Formalismo y escepticismo ante las reglas.

MÓDULO V. LO BUENO Y LO CORRECTO. UNA DISTINCIÓN ENTRE


SISTEMAS JURÍDICOS Y SU VALOR
Unidad 6
• Derecho y Moral: a) La justicia como segmento de la moral. b) Obligación
moral y jurídica. c) Derecho natural y positivismo jurídico.

MÓDULO VI. DISCUSIONES CENTRALES EN LA TEORÍA DEL DERECHO


Unidad 7
• El debate Hart-Dworkin
a) Los derechos en serio: el neo-iusnaturalismo de Dworkin.
b) Normas y principios.
c) Hart y la defensa del positivismo jurídico conceptual o metodológico.

MÓDULO VII. LÓGICA PROPOSICIONAL Y LÓGICA DEÓNTICA


Unidad 8
• Lenguaje artificial: variables, conectivos y tablas de verdad.
• Implicación y equivalencia.

Unidad 9
• Modalidades aléticas y deónticas: analogías y desanalogías. Ley de
Hume.
• Interdefinibilidad de operadores deónticos.
• Lógica de normas y lógica de proposiciones normativas.

MÓDULO VIII. APLICACIÓN DE LA LÓGICA DEÓNTICA A LOS


PROBLEMAS JURÍDICOS.
Unidad 10
• Problema normativo: aspectos fácticos y normativos.
• Concepto de caso y concepto de solución.
• Casos genéricos y casos individuales.

EDUBP | Abogacía | Filosofía y Lógica Jurídica - pag. 9


Agenda Porcentaje estimativo por módulo según la cantidad y complejidad de
contenidos y actividades.

10% 7% 10% 13% 13% 13% 17% 17%


Módulos 1 2 3 4 5 6 7 8

Representación de porcentajes en semanas.


1 2 3 4 5 6 7 8 9 10 11 12 13 14 15
M1 M2 M3 M4 Módulo 5 M 6 Módulo 7 Módulo 8

Primera parte Segunda parte Tercera parte Cuarta parte


del parcial del parcial del parcial del parcial

Mapa conceptual

EDUBP | Abogacía | Filosofía y Lógica Jurídica - pag. 10


Material Material Básico:
• Guibourg, Ricardo y otros; Lógica, proposición y norma. Bs. As. Ed.
Astrea, 1980.
• Hart, H.L.A.; El Concepto de Derecho. Bs.As. Abeledo Perrot, 1968.
• Alchourrón, Carlos y Bulygin, Eugenio; Introducción a la Metodología de
las Ciencias Jurídicas y Sociales, Bs. As. Ed. Astrea, 1974.
(De este libro estudiaremos los primeros 4 capítulos, los cuales encon-
trará en: https://bit.ly/3mVv85n)

Material Complementario:
• Alchourrón, Carlos y Bulygin, Eugenio; “Lógica de normas y lógica de
proposiciones normativas” en Análisis Lógico y Derecho, Centro de
Estudios Constitucionales, Madrid, 1991, ps.25-50.
- - - - - - - - - - - - - - - - - - - - - - “Normas, proposiciones normativas y
enunciados Jurídicos”, en Análisis Lógico y Derecho, ps. 169-194.
- - - - - - - - - - - - - - - - - - - -“Sentencia judicial y creación de derecho”,
en Análisis Lógico y Derecho, ps. 355-370.
• Calsalmiglia, Albert; ¿“Por qué es importante Dworkin?”, en Doxa Revista
de Filosofía del Derecho, 1985. Nº 2, ps.159-65.
• Dworkin, Ronald; Los Derechos en Serio, Barcelona, Bs. As., Caracas.
Ed. Planeta, 1993.
• Garzón Valdés, Ernesto; “Algo más acerca de la relación entre derecho y
moral”, Doxa 8, 1990, ps. 111-30.
• Hart, H.L.A.; “Poscriptum” en La Decisión Judicial: el debate Hart Dworkin.
Bogotá, Colombia. Siglo del Hombre Editores, Universidad de los Andes,
5ª reimpresión, 2005.
• Kelsen, Hans; Teoría Pura del Derecho. México, 2º edición, UNAM, 1979.
• Nino, Carlos Santiago; Introducción al Análisis del Derecho. Buenos
Aires. Ed. Astrea, 2ª edición ampliada y revisada, 1992.
• Rodríguez, Jorge L.; “Naturaleza y lógica de las proposiciones normati-
vas”, Doxa 26, 2003, ps.87-108.

Glosario Acción: Acto que produce un cambio en el mundo, u omisión que impide un
cambio en el mundo.
Argumentar: Es la actividad de dar razones a favor o en contra de cierta
tesis.
Aspecto fáctico del problema: Clase de circunstancias fácticas o casos
abarcada por el problema a analizar.
Aspecto normativo del problema: El relativo a la determinación deóntica
de la acción o acciones que forman parte del universo de acciones abarcado
por el problema.

EDUBP | Abogacía | Filosofía y Lógica Jurídica - pag. 11


Austin, John: Autor de The Province of Jurisprudence Determined. Su teoría,
básicamente, reconstruye al derecho como un conjunto de órdenes emana-
das de un soberano, respaldadas por amenazas y dirigidas a un grupo de
individuos (súbditos) que están habituados a obedecer.
Autoridad: Se refiere generalmente al ejercicio de autoridad como algo dis-
tinto a la mera posibilidad de ejercer un mal o amenazar a una persona para
que haga o deje de hacer algo. Quien detenta autoridad, aunque pueda tener
o no el poder de ejercer un mal, detenta además una cierta posición social
e institucional, en virtud de la cual, creemos que lo que ella dice cuenta en
forma distinta a lo que dice cualquier persona común.
Carácter estático de las reglas: Cuando no pueden ser modificadas por
actos deliberados de expresión de voluntad debido a la ausencia de órgano
legislativo; falta de institucionalización de actividad creadora de reglas.
Caso: Se determina por una combinación de propiedades. Se define por la
conjunción de cada una de las propiedades, o su complementaria (–P).
Caso individual: Se denomina a los elementos del Universo de Discurso.
Caso claro de aplicación de una regla: Es aquél en que los términos gene-
rales parecen no necesitar interpretación y el reconocimiento de los ejem-
plos parece ser “automático”. Son casos familiares que se repiten en forma
constante en contextos semejantes, respecto de los cuales existe acuerdo
general sobre la aplicabilidad de los términos clasificatorios.
Caso difícil: Son aquellos en los que se produce una crisis en la comunica-
ción: hay razones tanto a favor como en contra de que usemos un término
general, y no existe convención firme o acuerdo general que dicte su uso o
su rechazo a la persona ocupada en clasificar.
Caso elemental: Es aquél cuya propiedad definitoria es una conjunción que
contiene todas las propiedades relevantes del UP o sus negaciones.
Caso genérico: Es toda propiedad que define un subconjunto del Universo
de Discurso.
Ciencia: Conjunto de enunciados relacionados lógica y temáticamente.
Coherencia: Ausencia de contradicción.
Completitud: Un sistema normativo α es completo en relación a un UCj y
un USmax k, si y sólo si, α no tiene lagunas en UC j en relación al USmax k.
Concepción iusnaturalista: Sostiene la tesis de la conexión necesaria entre
el derecho y la moral, según la cual, la determinación de lo que es Derecho
depende de su adecuación a la moralidad.
Concepción iuspositivista: Sostiene dos tesis, a saber: una tesis negativa,
según la cual, la determinación de lo que es Derecho no depende de su ade-
cuación a la moralidad, y una tesis positiva, según la cual, la existencia del dere-
cho depende de un conjunto de hechos sociales – tesis de las fuentes sociales.
Condición de aplicación de una norma: Es la condición que tiene que
darse para que haya oportunidad de hacer aquello que constituye el conte-
nido –acción- de una norma.
Conectivos lógicos: Expresiones que conectan los enunciados simples o
atómicos que forman para de un enunciado compuesto o molecular.

EDUBP | Abogacía | Filosofía y Lógica Jurídica - pag. 12


Consistencia: Esquema veritativo funcional que es verdadero para alguna
interpretación de sus variables proposicionales.
Constituyente deóntico: Es un enunciado formado por una descripción de
estado precedida por el operador P o –P.
Contenido de la norma: Es la acción deónticamente modalizada.
Contradicción: Conjunción de la afirmación y la negación simultánea de una
misma proposición.
Contradicción deóntica: Que un acto sea prohibido y permitido simultá-
neamente, sea obligatorio y esté permitida su omisión, o sea obligatorio y
prohibido de consuno.
Deducción: Proceso de obtención de un enunciado a partir de otros.
Derecho: Técnica de control social que utiliza el lenguaje como vehículo
para motivar a los individuos a que se comporten de una manera determi-
nada (hagan o dejen de hacer algo).
Descripción de estado: Es una conjunción de cada uno de los contenidos
atómicos de un UA o su negación, pero no ambos.
Difusa presión social para hacer cumplir las reglas: Ausencia de órganos
centralizados para dirimir conflictos relativos a la trasgresión de reglas y apli-
cación de sanciones.
Directrices: Estándares que proponen un objetivo colectivo político, econó-
mico o social.
Discrecionalidad: Es un acto de la naturaleza de una elección entre alter-
nativas abiertas consistente en resolver si el caso se asemeja “en grado sufi-
ciente” al caso típico, en aspectos “relevantes”.
Dogmática jurídica: Disciplina encargada de describir y sistematizar las
normas jurídicas de un derecho positivo particular, por ejemplo el derecho
positivo argentino.
Enunciado declarativo: Es aquél que tiene una función informativa, pues
afirma o niega eventos o estados de cosas del mundo.
Equivalencia: Es la validez del bicondicional. Si dos enunciados tienen siem-
pre el mismo valor de verdad se dice que son equivalentes en el sentido de
que con ellas puede formarse un bicondicional tautológico.
Escepticismo extremo ante las reglas: Concepción que niega que el dere-
cho consista en reglas. El derecho es lo que los jueces dicen que es cuando
resuelven controversias.
Escepticismo moderado: Las reglas son sólo fuente de derecho y no derecho.
Existencia de una regla: (p.136-37) Para sostener que existe una determi-
nada regla en términos jurídicos es necesario constatar una cierta regulari-
dad (v.g. que los individuos se detienen ante el semáforo) y que ellos adop-
tan una cierta actitud hacia esa regla. Ven en ella una razón para actuar y
para criticar a quien se desvía.
Explicandum: Concepto que se quiere explicar.

EDUBP | Abogacía | Filosofía y Lógica Jurídica - pag. 13


Explicatum: Nuevo concepto que se obtiene como resultado de la recons-
trucción racional.
Facultad o libre permisión: es un modalizador deóntico. Un acto es faculta-
tivo o libre cuando no es obligatorio, ni está prohibido, sino que está permi-
tida no sólo su realización, sino también su omisión.
Falta de certeza: Ausencia de criterio para identificar las reglas jurídicas y
diferenciarlas de otras reglas sociales.
Formalismo: Concepción que considera al derecho como un conjunto de
normas generales que posee la propiedad de ser completo – proporciona
solución para todos los casos, coherente –no contiene normas contradicto-
rias y preciso– su formulación lingüística es totalmente precisa –no contiene
términos vagos.
Generalidad: Condición de moralidad crítica según la cual las protecciones
frente al daño que la moral y el derecho acuerdan a través de las acciones y
omisiones que requieren, deben extenderse por igual a todos los hombres
que puedan y quieran aceptar tales restricciones.
Hábito: Actitud convergente de un grupo de personas (v.g. ir al cine).
Compartir o converger en un hábito no necesita de quien lo realiza ni un
acuerdo con el resto de personas que convergen en esa práctica, ni compar-
tir la razón por la que las otras personas hacen lo que hacen. Se diferencia
del seguimiento de una regla porque en ésta, los sujetos que convergen en
la práctica de seguir una regla: a) desarrollan una actitud crítica o reflexiva
con respecto al seguimiento de la regla; b) consideran que la violación de la
pauta de conducta es una buena razón para formular la crítica; c) la actitud
crítica no sólo se limita a formular un reparo u objeción sino que, además,
pretende exigir conformidad de quien se apartó de la pauta establecida.
Imperativo (p.24): En términos generales puede decirse que un imperativo
expresa el deseo de que alguien haga o deje de hacer algo. Este deseo puede
ser manifestado más o menos vehementemente. De ello dependerá que un
imperativo sea tomado como un pedido (cuando nos piden que alcancemos
algo que está sobre la mesa) una súplica (cuando alguien pretende que no le
hagan algo; por ejemplo, imagínese el caso de quien le pide a un policía que
no le cobre la multa) una advertencia (cuando alguien nos dice con un grito
que se avecina un peligro). El aspecto más importante de los imperativos, a
nuestros efectos, es el que se relaciona con la orden o exigencia de llevar
adelante una determinada conducta.
Implicación: Se dice que un esquema veritativo funcional implica otro, si no
hay forma de interpretar las variables o letras proposicionales de ambos de
modo tal que el primer esquema sea verdadero y el segundo sea falso.
Incertidumbre de la regla de reconocimiento: Dudas sobre el ámbito y
alcance del poder soberano y, en consecuencia sobre la validez de una norma.
Inconsistencia: Esquema veritativo funcional que es falso para toda interpre-
tación de sus variables proposicionales.
Intención: propósito, voluntad o fin.
Intensión: Conjunto de características definitorias de los objetos referidos
por el término. Criterio de uso del término o palabra.

EDUBP | Abogacía | Filosofía y Lógica Jurídica - pag. 14


Juegos y reglas de tanteo. Tanteador: La figura del tanteador o persona
que cuenta los tantos es utilizada por Hart a lo largo de su libro para trazar un
paralelo con lo que un tribunal representa en el derecho. El ejemplo del fútbol
y la persona que cuenta los tantos sirve para mostrar que existe una posición
contra-intuitiva cuando se sostiene que “el partido va como el tanteador dice
que va”. Análogamente, existe algo contra-intuitivo en decir que el derecho
es lisa y llanamente lo que los jueces dicen que es.
Justicia compensatoria: Se aplica para evaluar situaciones en las que se ha
causado algún daño y se evalúa moralmente la compensación o indemniza-
ción reclamada por la/s víctima/s.
Justicia distributiva: Se aplica para evaluar situaciones en que juzgamos la
distribución de cargas o beneficios dentro de una clase o grupo de individuos.
Laguna: Un caso Ci de universo UC j es una laguna de un sistema normativo
α en relación a un USmax k, porque α no correlaciona Ci con ninguna solu-
ción del USmax k.
Laguna de conocimiento: Problema de subsunción o clasificación de los
casos individuales que surge por falta de conocimiento de algún rasgo o
característica del hecho.
Laguna de reconocimiento: Problema de clasificación de casos individua-
les que surge por la indeterminación semántica de los conceptos que carac-
terizan a un caso genérico.
Laguna normativa: Es de índole conceptual o lógico. Cuando en una matriz
no aparece correlacionado algún caso elemental de un UC con una solución
normativa decimos que el sistema normativo es incompleto. Por lo tanto, este
concepto de laguna está relacionado con la completitud o falta de completi-
tud del sistema normativo.
Lenguaje: Sistema de símbolos utilizados para la comunicación entre los
seres humanos.
Ley lógica: Es un enunciado, en general molecular, que tienen la caracte-
rística de ser verdadero en todos los casos, esto es, no puede ser falsa. Se
suele denominar tautología.
Lógica: Disciplina que estudia las condiciones de validez de la inferencia o
relación de consecuencia lógica.
Mandatos: Se utiliza en el texto para dar cuenta de un tipo especial de orden
expresada por quien detenta, dentro de una cierta organización jerárquica,
una posición de preeminencia. Hart se refiere con esta expresión a un cierto
tipo de orden característica de los ámbitos militares (commands). No es
seguro que la expresión castellana mandato refleje todo el alcance que en
inglés refleja la expresión referida.
Matriz: Tabla que une los casos elementales y las normas que correlacionan
cada uno de ellos con una solución normativa.
Modalidades aléticas: Son modos en que puede afirmarse la verdad de un
enunciado.
Modalidades deónticas o del deber: Son modos del deber, los de uso más
frecuente son obligatorio, prohibido y permitido.

EDUBP | Abogacía | Filosofía y Lógica Jurídica - pag. 15


Moral crítica: Conjunto de criterios o estándares objetivos aptos para eva-
luar la corrección de las acciones o instituciones humanas.
Moral positiva: Conjunto de estándares o criterios que los miembros de un
grupo social creen que son los correctos para evaluar los comportamientos
e instituciones humanas.
Necesariamente verdadero, posiblemente verdadero, imposible o necesa-
riamente falso.
Norma: 1) Enunciado condicional que correlaciona un caso con una solu-
ción. 2) Enunciado usado para prescribir conductas y, que no es ni verda-
dero ni falso. Su forma lógica consiste en un modalizador deóntico, por ejem-
plo ‘O’, que se antepone a una variable proposicional. 3) Las normas tienen
una estructura disyuntiva, en el sentido de que son o no son funcionalmente
importantes según sean válidas o no, se identifican por su origen y, es posi-
ble precisar y enumerar las excepciones a su aplicación.
Norma categórica: Es la que regula un cierto contenido incondicionalmente.
Norma negativa: La que regula omisiones.
Norma positiva: La que regula actos.
Normas hipotética o condicionada: Es aquella que establece que una cierta
acción es obligatoria, está prohibida o es permitida, dadas ciertas condiciones.
Oración: Estructura gramatical o sintácticamente bien formada y, en conse-
cuencia, apta para transmitir un sentido que pueda ser calificado de verda-
dero o falso.
Orden respaldada por amenaza (u órdenes coercitivas): Expresión utilizada
para describir el acto propio del asaltante o quien no posee autoridad puesto
que, en el caso del asaltante, si éste no respalda su orden con la amenaza
de coacción, no decimos que nos ha ordenado algo. En cambio, en el caso
de la autoridad, ella nos puede ordenar algo (y nos creemos obligados a
hacerlo) aunque no haya ejercicio de la coacción. La expresión orden refiere,
además, a la idea de un requerimiento o mandato dado cara a cara (como
el que da el asaltante a la víctima). Las órdenes (en cuanto a requerimientos
dados cara a cara –v.g. “déme el dinero”–) deben ser cuidadosamente dife-
renciadas de las reglas generales (que precisamente no son dadas a los indi-
viduos cara a cara sino mediante algún tipo de promulgación legislativa). La
orden respaldada por amenaza carece de las características de generalidad
y permanencia que detentan, por ejemplo, las reglas generales.
Positivismo exclusivo: Constituye la versión más fuerte de la tesis de las
fuentes sociales, pues afirma que la determinación del contenido del dere-
cho depende exclusivamente de su origen en ciertos hechos sociales sin
referencia a argumentación moral alguna.
Positivismo inclusivo: Sostiene que la validez jurídica de las normas sólo
puede depender de su validez moral de un modo contingente, cuando una
norma jurídica incorpore en su formulación términos morales y, por ende,
requieran para su determinación y alcance de argumentación moral.
Positivismo jurídico como ideología: Sostiene que el derecho positivo es
justo y, en consecuencia, debe ser obedecido.

EDUBP | Abogacía | Filosofía y Lógica Jurídica - pag. 16


Positivismo jurídico como teoría: Se caracteriza por los siguientes rasgos:
monopolio de la fuerza coercitiva, imperativismo, supremacía de la ley res-
pecto de otras fuentes de derecho, consistencia y completitud del derecho,
separación de funciones o poderes de creación de derecho y de aplicación,
concepción de la aplicación de derecho como actividad puramente deductiva.
Positivismo metodológico: Propone estudiar el derecho que es desde una
perspectiva valorativamente neutral.
Positivismo normativo: Afirma que la legalidad de una norma no debe
depender de su mérito moral.
Principios en sentido estricto: Estándares que son exigencias de justicia,
equidad o alguna otra dimensión moral.
Principios: Estándares que no son reglas. No son concluyentes, son muy
generales e inespecíficos y compiten entre sí orientandos en diferentes senti-
dos o direcciones la decisión en un caso. No determinan ni las condiciones,
ni las consecuencias de su aplicación, las excepciones a su aplicación no
son susceptibles de enumeración y, por último, se identifican por su peso
argumentativo o importancia relativa al competir con otros principios en un
caso en mano y, no por su origen.
Problema conceptual: Determinación de los casos genéricos y las normas
generales.
Problema de aplicación: Es un problema de clasificación del caso individual.
Propiedad relevante: Propiedad definitoria del conjunto de situaciones que
conforman el universo del discurso.
Proposición atómica: Describe un hecho o estado del mundo.
Proposición compuesta o molecular: Combinación de proposiciones ató-
micas, verdaderas o falsas por medio de conectivos lógicos.
Proposición: Sentido o significado verdadero o falso atribuido a un enun-
ciado u oración.
Proposiciones normativas que no se refieren a normas, sino a acciones,
o estados de cosas resultantes de acciones: Describen la propiedad que
poseen acciones de ser obligatorias, permitidas o prohibidas por ser conte-
nidos de normas.
Punto de vista externo extremo: Hecho acerca de la conducta observable
de la mayoría de un grupo social, registrable por un observador externo.
Desde este punto de vista sólo describimos regularidades observables –
hechos brutos.
Punto de vista externo moderado: Asume el punto de vista de los partici-
pantes (el punto de vista interno del seguimiento de una regla), y describe
cómo los individuos ven en las reglas no sólo un hecho sino una señal
(orden) de lo que deben hacer y, cómo atribuyen a esta señal no sólo la
capacidad de ordenarles qué deben hacer sino, además, la de justificar qué
están haciendo (dar razones de por qué actúan de determinada manera).
Punto de vista interno: Específico de los que siguen reglas, pues ven en
la conducta una pauta de conducta social correcta que se manifiesta en la

EDUBP | Abogacía | Filosofía y Lógica Jurídica - pag. 17


crítica frente a la conducta desviada y en el reconocimiento de la legitimidad
de la crítica y de las exigencias de conformidad.
Racionalidad: Condición de moralidad crítica según la cual las estructuras
sociales no deben descansar en creencias cuyo error pueda demostrarse.
Razón para actuar: Un individuo tiene una razón para hacer o no hacer
algo, si está dispuesto a justificar su conducta o su decisión de actuar de una
determinada manera en base a esa razón.
Regla: Su existencia depende de que algunos vean en la conducta prescripta
una pauta o criterio general de comportamiento a ser seguido por el grupo
como un todo. Requiere que su aceptación se manifieste en una actitud crítico-
reflexiva frente a ciertos modelos de comportamiento en tanto pautas de con-
ducta comunes, y que se despliegue en forma de crítica, exigencia de confor-
midad, y en reconocimiento de que tales críticas y exigencias están justificadas.
Regla de formación de una norma: Consiste en la aplicación de un opera-
dor modal deóntico a un enunciado; el enunciado resultante es una norma.
Regla de inferencia: Principios o reglas que elabora la lógica para estable-
cer las condiciones de validez de una relación de consecuencia lógica.
Regla de reconocimiento: Indica criterio/s – o fuentes de derecho - para
identificación de reglas primarias de un sistema jurídico.
Regla de reconocimiento: No es simplemente un test para determinar “lo
que ellos consideran como derecho” (que es la forma en que un profesor
de derecho comparado utiliza este tipo de enunciados) sino que también
es usada para indicar lo que “nosotros consideramos derecho”. Es decir, no
sólo se señala desde el punto de vista externo qué es derecho para la comu-
nidad tal o cual, como lo hace el científico, sino que también se la utiliza para
indicar qué es derecho según nuestras convicciones (punto de vista interno).
Reglas de adjudicación: Confieren facultad para determinar con carácter
incontrovertible cuando ha sido violada una regla primaria del sistema jurí-
dico y la sanción correspondiente.
Reglas de cambio: Reglas que confieren habilitación o facultad para crear,
modificar o derogar reglas jurídicas primarias.
Reglas o directivas generales: Son pretensiones de conducta dictadas por
la autoridad, que se aplican a una cierta clase amplia de individuos, que
viven en un cierto territorio. Esta característica de generalidad (en cuanto a
los sujetos y el territorio) es un elemento central que permite diferenciar al
derecho de la mera orden de un asaltante. Es importante tener en cuenta
que una norma es general, no en el sentido de que se dirige a todos los
ciudadanos, o en el sentido de que todos los ciudadanos la conocen si no,
más bien, en el sentido de que pretende regular la conducta de un amplio
sector de la ciudadanía, aunque sea cierto que éstos no conocen lo que dice
la ley. H. Hart hace un importante esfuerzo en distinguir entre la noción de
normas que se dirigen a un grupo de personas y normas que se aplican a
un grupo de personas. Sólo éste último debe ser considerado el contenido
o significado de regla general. La regla general se diferencia de una orden
respaldada por amenaza, ya sea por su carácter de generalidad, así como
también por su propiedad de permanencia. Consideramos que una norma

EDUBP | Abogacía | Filosofía y Lógica Jurídica - pag. 18


continúa existiendo aunque la autoridad de la cual emanó ya haya dejado de
ejercer sus funciones. No ocurre lo mismo con el asaltante.
Reglas primarias: Indican a los individuos qué hacer y qué omitir, lo quieran
o no (imponen deberes). Se refieren a acciones que implican movimientos o
cambios físicos.
Reglas secundarias: Establecen que los individuos, haciendo o diciendo
algo en el contexto adecuado pueden crear, extinguir o modificar las reglas
anteriores (otorgan potestades públicas o privadas); las denominamos
secundarias en razón de que en algún sentido se refieren a (o versan sobre)
las reglas primarias. Se refieren a actos que implican la creación o modifica-
ción de deberes u obligaciones.
Reglas. Punto de vista interno: El punto de vista interno es la perspectiva
que tiene que adoptar un conjunto de individuos para poder hablar de la exis-
tencia de reglas sociales, y para diferenciar al seguimiento de ellas del mero
hábito o regularidad externamente constatable (véase la palabra hábito). El
punto de vista interno consiste en una cierta actitud crítico-reflexiva frente a
ciertos modelos de comportamiento. Esta actitud se despliega en la forma
de crítica. Esto es, en la predisposición de ciertos individuos a exigir confor-
midad a ciertos cursos de acción y a criticar a quien no respeta esas pautas.
Este punto de vista interno no puede reducirse o analizarse simplemente en
términos de la probabilidad que existe de que se ejerza presión social (que
se critique). Quien desarrolla un punto de vista interno frente a una regla
(v.g. el semáforo en rojo), no desarrolla frente a ella una reacción física sino
que ve en ella una señal, i.e. algo que indica qué debe hacerse, y una razón,
tanto para hacer lo que la regla indica, como para criticar a quien se desvía.
Mencionar o dar cuenta del punto de vista interno de las reglas permite mejo-
rar la explicación de lo que los individuos hacen socialmente cuando siguen
reglas. Quien adopta un punto de vista interno no se limita a predecir la con-
ducta que se adecua a las reglas, sino que usa la regla para evaluar y criticar
su conducta y la de los demás.
Simple deber: Se da cuando su exigencia social no es insistente y la presión
ejercida para hacerlo cumplir no es seria.
Soberanía continuada: El poder soberano del soberano actual está limitado
jurídicamente por igual poder de soberanos futuros.
Soberanía omnicomprensiva: El poder soberano es ilimitado jurídicamente.
Soluciones: Son caracterizadas como modalizadores deónticos que se ante-
ponen a variables proposicionales que representan contenidos normativos o
descripciones de elementos del universo de acciones (UA) –actos u omisiones.
Tautología: Esquema veritativo funcional que es verdadero para toda inter-
pretación de sus variables o letras proposicionales
Tener una obligación: Es una afirmación que tiene un característico tras-
fondo normativo pues se afirma que el caso particular de un sujeto determi-
nado cae bajo al ámbito de aplicación de una regla
Tener una obligación y sentirse obligado: Sostener que existe una norma
jurídica o una obligación jurídica equivale a decir que un conjunto mínimo
de conductas no son optativas. Para la teoría de las obligaciones coerci-
tivas, A tiene una obligación con B, si B respalda su orden con una ame-

EDUBP | Abogacía | Filosofía y Lógica Jurídica - pag. 19


naza. Pero esto en realidad es más bien verse obligado o sentirse obligado.
Cuando decimos que alguien se vio obligado a algo, hacemos una afirma-
ción acerca de sus temores y creencias, acerca de sus estados psicológicos.
Sostenemos, p. ej. que él creía conveniente entregar el dinero dado el mal
que sufriría. Decir que alguien tiene una obligación, en cambio, no necesita
de las creencias del agente. Alguien está obligado a hacer A aunque no lo
sepa o crea. A su vez, mientras que el enunciado de que alguien tiene una
obligación no implica necesariamente que esta persona realice la acción, el
enunciado de que alguien se vio obligado o se sintió obligado suele implicar
que la persona efectivamente hizo lo que se le pedía.
Teoría general del derecho: Analiza los problemas de la ciencia jurídica
en un nivel de mayor abstracción. Su tarea es el análisis lógico del lenguaje
para el esclarecimiento lógico de los conceptos fundamentales de la ciencia
jurídica. Es una meta jurisprudencia o filosofía de la ciencia jurídica.
Término técnico: Abreviatura de expresiones más complejas.
Términos de clase: Son aquellos que designan un conjunto de elementos
que comparten una determinada propiedad (pasta, mamíferos, tigres, meta-
les, argentinos, mayores de edad, etc.). Para formar una clase hace falta un
criterio o propiedad con la cual dividir el mundo (al respecto ver la distinción
hecha en el módulo 3 entre conjuntos por intensión y por extensión).
Tesis de la discrecionalidad judicial: Sostiene que indefectiblemente habrá
casos imprevistos y no regulados legalmente, es decir, casos para los cuales
el derecho es parcialmente indeterminado e incompleto y en los cuales los
jueces tendrán la posibilidad de ejercer discreción (esto es, deberán crear
nuevo derecho en un ámbito de libertad restringida por el núcleo de signifi-
cado claro de las reglas jurídicas).
Tesis de la respuesta correcta: El derecho da una solución normativa a
todos los casos, aún los casos difíciles generados por la textura abierta de
términos contenidos en la formulación de las reglas.
Tesis de la Separación: No existe una conexión necesaria entre el derecho
y la moral. El derecho es un fenómeno social cuya identificación no necesita
apelar a las nociones de “justicia”, “bondad” o “corrección”.
Tesis de las fuentes sociales: Sostiene que es posible identificar lo que el
derecho indica atendiendo a lo que dice un grupo de personas (legisladores,
jueces) o la forma en que actúa un grupo considerable de personas. En defi-
nitiva, se afirma que los últimos tests de validez jurídica descansan en una
práctica social.
Textura abierta: Surge del carácter abierto de un término y consiste en la
imposibilidad de prever por adelantado todas las propiedades extrañas que
pueden presentarse y la lista de las circunstancias que no deben darse para
que ese término o palabra sea aplicable.
Universo de Soluciones Maximales: Contiene el conjunto de todas las res-
puestas posibles a la pregunta sobre el carácter normativo de la acción o
acciones elementales que constituyen el universo de acciones.
Universo del Discurso: Conjunto de situaciones abarcadas por un determi-
nado problema normativo.

EDUBP | Abogacía | Filosofía y Lógica Jurídica - pag. 20


Variable proposicional: Símbolo o representación esquemática de enuncia-
dos o proposiciones.
Verdad por correspondencia: Verdad atribuible a una proposición atómica.
Veritativo funcional: Es la función de inferir verdades a partir de otras ver-
dades que cumplen las expresiones que conectan enunciados simples para
formar enunciados compuestos o moleculares.
Verse obligado a hacer algo: En general, se trata de una afirmación acerca
de los motivos o creencias que acompañan a una acción, o acerca de pro-
babilidad de que un individuo que ha realizado una acción contraria a la
ordenada, sufrirá un mal.

EDUBP | Abogacía | Filosofía y Lógica Jurídica - pag. 21


Módulos MÓDULO 1

M1 Microobjetivos

• Intentar responder a la pregunta básica de un abogado: ¿qué es el dere-


cho?, con el propósito de identificar las principales respuestas que le han
ido proponiendo durante el estudio de la carrera de abogacía.
• Desarrollar una reconstrucción conceptual de las intuiciones acerca del
derecho, para precisar nuestro empleo del lenguaje jurídico.
• Conceptualizar, de forma esquemática y simple, las preguntas filosófi-
cas básicas que se suscitan en el estudio del derecho para descubrir el
origen de su complejidad.
• Desarrollar habilidades para llevar a cabo búsquedas teóricas en el área
de la Filosofía Jurídica, a partir de las dudas o perplejidades que pueda
generarle la pregunta ¿qué es el derecho? como motor impulsor, con el
fin de revisar sus propias intuiciones, dividiendo lo que realmente sabe
sobre la abogacía o el derecho, de lo que creía saber.

M1 Contenidos

Lo invito a ver el video de presentación del Módulo 1

La pregunta básica y las perplejidades persistentes. ¿Qué es el derecho?


En el presente módulo le proponemos un desafío crucial en el marco del
futuro abogado. Dicho desafío consiste básicamente en intentar construir
una respuesta adecuada para la pregunta ¿qué es el derecho? Es nuestro
propósito acompañarlo en la indagación sobre lo que significa la expresión
derecho o bien, en la búsqueda del concepto de derecho.

En los años que lleva estudiando esta carrera posiblemente


esté habituado a disparar respuestas frente a la pregunta citada.
Seguramente, ha escuchado o leído expresiones del tipo:
i) el derecho es lo que los jueces dicen que es;
ii) el derecho es la predicción de lo que los tribunales harán;
iii) las leyes son fuentes del derecho, no el derecho mismo;
iv) la constitución no es más que un conjunto de valores
morales puestos en un gran libro. El derecho y la moral se
funden en la constitución;
v) el derecho es sólo aquello que establece sanciones.

Sin embargo, aunque las respuestas señaladas nos permiten


salir fácilmente del entuerto frente a un lego o una persona no
muy avezada sobre temas jurídicos, la cuestión no es tan fácil en
el caso de que nos encontremos con alguien que conoce más
o menos el derecho o tiene un sentido agudo de los problemas
conceptuales que rodean la pregunta ¿qué es el derecho?

EDUBP | Abogacía | Filosofía y Lógica Jurídica - pag. 22


Supongamos que nuestro interlocutor avezado dice frente a nuestra res-
puesta “i”:

“o.k. el derecho es lo que los jueces dicen que es. Pero si esto es así,
¿qué es lo que hace a estas personas, jueces? Obviamente, no lo que
ellos mismos dicen. Las leyes orgánicas y la constitución indican cómo
se designan jueces y, a partir de que condiciones y procedimientos,
quien ayer era sólo mi vecino del frente hoy pasa a ser el Juez Pérez. Si
esto es así, la frase el derecho es lo que los jueces dicen que es, es
falsa en dos sentidos. En primer lugar, porque consideramos derecho a
las normas orgánicas y constitucionales que dicen cómo se designan a
los jueces y, sin embargo, esas normas que consideramos claramente
derecho no han sido dictadas por ningún juez. En segundo lugar, la
expresión es falsa porque si el derecho se limitase a ser sólo lo que los
jueces dicen que es, ¿qué es lo que los haría jueces a ellos mismos?
Si no existe una norma que los declare jueces, no habría forma de dis-
tinguir a un simple vecino del que realmente está juzgando. Pero para
que ello ocurra, no puedo decir que lo que lo hace juez es su propia
decisión, salvo que me vuelva a enrolar en argumentos falsos y circula-
res. Tiene que haber habido un primer momento en que el juez no era
juez (era sólo Juan) y haya dictado una norma que lo nombre a sí mismo
como juez. Pero ¿por qué esa primera norma que lo nombra como juez
habría de ser válida jurídicamente? Si sólo es derecho para nosotros
lo que el juez dice –y claramente lo que dijo esa primera vez no era la
expresión de un juez, ya que no era magistrado todavía -, ¿por qué val-
dría ello como condición suficiente para nombrarlo juez?”.

Lo anterior, muestra que la expresión “el derecho es lo que los jueces dicen
que es” no puede dar cuenta de qué es lo que hace que los Juanes Pérez
que se postulan para jueces, pasen a ser de un momento a otro Su señoría
Juan Pérez.

Por lo tanto, habría que desechar esta primera tentativa de explicar el derecho
con esta expresión (convincente, persuasiva, pero incompleta y falsa al fin).

Lo mismo cabe decir de expresiones como “ii”. Esta afirmación sostiene


que el derecho en realidad es un conjunto de predicciones sobre lo que los
jueces harán. Como ha señalado Carlos Santiago Nino, esta es una horrible
confusión entre el objeto de estudio (el conjunto de hechos que considera-
mos derecho, tales como la actividad de los legisladores y jueces, sus leyes
y sentencias) y el aparato conceptual o herramienta que uso para analizarlo.
Puede ser totalmente cierto que la ciencia del derecho se dedique a diseñar
datos probabilísticos de cómo actuará tal o cual Juez o cuerpo colegiado.
Por ejemplo, si yo tengo un buen conocimiento de la Corte Suprema de
Perú, ella sigue conformada por los mismos jueces desde hace 20 años y
ya se ha pronunciado varias veces sobre el tema del aborto de forma nega-
tiva, es muy probable que vuelva a pronunciarse de la misma manera en el
caso que acaba de ingresar caratulado XXX por supuesto autor de aborto
consentido. Esto es lo que haríamos si fuésemos científicos del derecho,
observadores del mismo, juristas o abogados. Observaríamos un conjunto
de regularidades y de allí podríamos prever qué es lo que va a hacer la Corte
Suprema o el Juez Pérez con nuestro caso. Pero de ello no se sigue que el

EDUBP | Abogacía | Filosofía y Lógica Jurídica - pag. 23


derecho consista en la previsión de lo que se hará en el futuro. Lo que hace
el Juez cuando dice “condeno a X a pagar 1000 pesos” no es predecir lo que
hará mañana, sino precisamente decir lo que de hecho, dadas las circuns-
tancias fácticas del caso y las leyes existentes, corresponde dictaminar. No
está diciendo cómo se comportará en el futuro. Más bien, de hecho se está
comportando de una manera particular.

Una cosa muy diferente es que, como abogados o teóricos acerca del dere-
cho, nos dediquemos a tomar estas decisiones como signos de lo que suele
hacer el Juzgado tal o cual. Como abogados, generalmente, utilizamos nues-
tro conocimiento de los juzgados para decidir si ingresamos nuestra demanda
o recurso de amparo frente a tal o cual Juez. Sabemos que frente a un mismo
caso algunos jueces probablemente hagan una cosa y el resto otra.

Que usemos las decisiones de los jueces para hacer predicciones sobre
cómo actuarán en el futuro, no implica que esas decisiones sean juicios pro-
babilísticos. El juez, cuando falla, no está diciendo lo que hará mañana con
el caso que le toca juzgar, sino lo que considera es el derecho aplicable al
caso de hoy. Similares argumentos pueden desarrollarse con respecto al
resto de las respuestas más frecuentes a “qué es el derecho”.

El motivo de este primer módulo es precisamente desandar ese camino y


descubrir cómo esas afirmaciones algo livianas encierran algunas verdades
atendibles, pero no dan cuenta suficiente de todo el fenómeno jurídico.

Algunas, como en el caso “i”, nos llevan a la indeseable conclusión que la


constitución o las leyes orgánicas y procesales no son derecho, otras –como
la “ii”– nos conducen a tomar a los jueces como una especie de funcionarios
del servicio meteorológico los cuales siempre están anunciando lo que ocu-
rrirá mañana, dejando afuera la descripción del fenómeno de lo que están
haciendo hoy cuando dicen que Juan debe ir a la cárcel. El desafío preci-
samente es tratar de refinar nuestras intuiciones de modo que la respuesta
a qué es el derecho, nos permita dar cuenta del complejo fenómeno que
implica el derecho como práctica social.

No obstante, uno podría seguir empecinado en sostener que tiene la res-


puesta correcta a la pregunta que nos aqueja. ¿Cuántas veces hemos estu-
diado que el derecho civil, el penal, el comercial, la constitución, etc. son en
definitiva un conjunto de reglas? Pues bien, ante la pregunta ¿qué es el dere-
cho?, esta respuesta sólo es correcta de manera parcial. ¿Qué significa con-
junto de reglas? ¿No es acaso el ajedrez un conjunto de reglas? Y la receta
de ñoquis de Utilísima Satelital ¿no es conjunto de normas o reglas sobre
cómo hay que hacer los ñoquis del 29? (1. Debe poner 250 gr. de harina en
una taza, 2. Se le prohíbe terminantemente comprar otra harina que no sea
‘...harinas candeal, ssssu harina....’ dice la locutora. 3. Hierva el agua, 4...).

La respuesta “el derecho es un conjunto de reglas” nos deja igual que al


principio. Nos acerca un poco a la respuesta, pero no del todo. Con esa res-
puesta no habría forma de diferenciar el programa de cocina del derecho y
nosotros hemos estudiado derecho con la convicción de que podemos dar
cuenta de una diferencia tan significativa como la que media entre las reglas
del ajedrez o la cocina y las que dicta el Congreso. Veamos juntos todas las

EDUBP | Abogacía | Filosofía y Lógica Jurídica - pag. 24


cosas que pueden ser un conjunto de normas o reglas de modo de terminar
de convencernos de que la respuesta está mal:
a) las reglas de la gramática y de la matemática (Ud. debe utilizar el
verbo en pretérito si quiere explicar lo que hizo ayer; en la división no
debe olvidarse de agregar un cero cuando elimina la coma). Estas
reglas son las llamadas definitorias o determinativas;
b) las reglas de uso, o diseñadas para obtener un fin con un procedi-
miento (no enchufe este artefacto a 220 v.; si desea un mejor plan-
chado rocíe su ropa con Braid; para grabar en versión Hi Fi Ud. debe
apretar el botón rojo de la izquierda). Estas reglas se denominan vul-
garmente reglas de uso y, más técnicamente, reglas anankásticas.

La pregunta sigue sin ser contestada, pero hemos concedido dos argumentos:
I) que hay algo de verdad (y algo de exageración) en las afirmaciones
iniciales;
II) que hay un punto central, pero aún necesitado de refinamiento, en la
idea de que el derecho es un conjunto de reglas.

En este punto puede acudirse a alguien un poco más prevenido sobre algu-
nos temas jurídicos (quizá algún estudiante de derecho avanzado, o alguien
con particular sensibilidad para los problemas prácticos). Este personaje,
haciendo uso de las concesiones hechas precedentemente, dirá: el rasgo
distintivo del derecho es que posee un conjunto de normas y que ellas están
siempre asociadas con una sanción. Dicho de una manera más sencilla, sólo
es derecho el que prescribe una sanción.

Esta última, sigue siendo una respuesta insatisfactoria. ¿Qué hay por ejemplo
de las normas o reglas para realizar un contrato? ¿Tienen ellas prevista alguna
sanción para quien, por ejemplo, decide no pagar lo debido? Claramente no.
Prevén una indemnización, pero ella no puede contarse como una sanción
(ya veremos más adelante cómo sólo con intentos algo artificiosos puede
equipararse una indemnización u otro tipo de consecuencias jurídicas a una
sanción). ¿Qué hay con respecto a las normas o reglas que dicen cómo se
sanciona una ley? No existe ninguna sanción si el legislador no sigue esas
reglas. Simplemente, si el legislador no siguió los pasos previstos, por ejem-
plo, no levantando la mano (y la mayoría no lo hizo), entonces la ley no será
sancionada, ni promulgada, ni publicada, pero no habrá sanción alguna para
el legislador.

De este modo, vemos cómo el conjunto de reglas que nos hemos abocado
a estudiar (las reglas jurídicas) reúne en una misma bolsa una fauna tan
variada y compleja que se resiste a ser clasificada simplemente bajo la uti-
lización de una y tan solo una propiedad. Del mismo modo que la fauna de
la selva misionera no puede ser clasificada sólo como conjunto de animales
cuadrúpedos, el derecho no puede reunirse (sin incurrir en una falsa descrip-
ción) en un conjunto de reglas dotadas de carácter sancionatorio.

EDUBP | Abogacía | Filosofía y Lógica Jurídica - pag. 25


Recorramos rápidamente la fauna jurídica. En ella encontramos:
a) reglas que prohíben o hacen obligatorio ciertos tipos de conductas
bajo amenaza de aplicar una pena (sanción);
b) reglas que exigen que indemnicemos el daño causado;
c) reglas que especifican qué es lo que tenemos que hacer para otorgar
testamentos, contratos, etc.;
d) reglas llamadas “individuales” (sólo dirigidas a un individuo, cono-
cidas como sentencias) que obligan a un individuo a pagar tal o cual
dinero y lo envían a la cárcel;
e) reglas que indican cómo cambiar una vieja ley, o promulgar una nueva.

La imposibilidad de caracterizar satisfactoriamente algo con lo que estamos


altamente familiarizados (como es caso del derecho) no debe ser tomado
como una razón válida para pensar que no podremos llegar a saber nunca qué
es el derecho. Más bien, debería ser tomado sobriamente, como muestra que
nuestras intuiciones teóricas y conceptuales aún están esperando allí para ser
refinadas. En este camino hacia ese refinamiento de nuestras intuiciones, nos
encontraremos con tres preguntas que han sido recurrentes en el derecho.

Tres problemas recurrentes


Históricamente, el derecho entendido como el fenómeno práctico que que-
remos analizar en esta materia, esto es, como un conjunto de reglas que
pretende imponer obligaciones a los individuos, ha tratado de diferenciarse,
por un lado, de las bandas de mafiosos. A su vez, ha reclamado siempre
para sí una especie de legitimidad o halo de pulcritud que no puede recla-
mar o alegar un delincuente o una banda de delincuentes. Por el otro, tam-
bién como producto histórico, ha ido lentamente tratando de secularizarse
de diferentes órdenes morales como la religión y algunas concepciones de
la educación. Esto ha dado lugar a dos preguntas permanentes:
1) ¿En qué se diferencian las bandas de delincuentes armados del
derecho? Es decir, el problema de la coerción y el derecho.
2) ¿En qué se diferencia el derecho de la moral?

La primera pregunta tiene sentido si se tiene en cuenta que, visto objeti-


vamente (como quien viniese de Marte y no conociese la cultura occidental
y tratara de explicar el derecho), no hay ninguna diferencia evidente entre
una persona que me pone un revólver en la cabeza (ladrón) y me pide que
le entregue el dinero y otra que me lo reclama, pero vestido de saco y cor-
bata, con cuatro policías atrás y luciendo un escudito de la AFIP en la solapa
(agente fiscal).

Si es que ha de existir una diferencia entre una banda de asaltantes y el dere-


cho, ella deberá mostrar cómo hay una diferencia entre la amenaza cruda
del revólver bajo el grito “entregue el dinero o lo mato” y aquellas de “entré-
gueme el dinero o sufrirá de 3 a 10 años de prisión, y estos muchachos lo
llevarán al calabozo –no precisamente en una limosina – ni pidiéndole en
francés que suba al móvil”.

EDUBP | Abogacía | Filosofía y Lógica Jurídica - pag. 26


La clave en este punto será distinguir entre la idea de tener una obligación,
que es lo que solemos creer con respecto al Estado (creemos que tenemos
la obligación de pagar) y la de sentirse obligado, que es la sensación que
tenemos frente al asaltante, a quien no creemos deberle nada pero, dada su
furia y el revólver que detenta, preferimos hacerlo (sentimos que debemos
hacerlo, pero no creemos que estemos obligados). Esto que hemos venido
planteando, ocupa una parte importante de la materia. Más bien, frente al
asaltante, la expresión correcta es que nos vemos forzados, mientras que
frente al Estado, en algún sentido, nos vemos obligados.

La segunda pregunta (¿en qué se diferencia el derecho de la moral?), tiene


una central relevancia si se tiene en cuenta que históricamente el derecho
ha nacido (por diversas razones) como un competidor natural frente a otros
órdenes o sistemas prácticos, como la religión o la educación, o cualquier
otro tipo de orden que tenga alguna concepción determinada sobre lo que
es bueno para un ser humano y de qué manera deberían comportarse los
individuos de acuerdo a estos valores.

Una de las razones principales por las cuales el derecho ha tratado de eman-
ciparse de determinados órdenes morales, como la religión, se ha debido a
que el derecho trata de afianzar (entre otras cosas) la seguridad jurídica.

Es decir, el derecho trata que el individuo, una vez que conoce cuáles son
las reglas del Estado de Derecho (conoce, digamos, cómo se contrata, que
no puede matar, que el rojo en el semáforo obliga a detenerse, etc.) pueda
diseñar su plan de vida prevenido sobre qué cursos de acción lo beneficiarán
y cuáles lo perjudicarán, y con la confianza en que las reglas de juego no se
cambiarán en el camino. Uno de los grandes riesgos (aunque no necesario)
de aceptar la subordinación del derecho a la moral (p. ej. religiosa) es que
siempre sería posible que el Estado sorprendiera al individuo modificando
las reglas preexistentes, bajo la excusa de que las mismas son inmorales.

La idea de contar con un sistema práctico (un sistema que me dice qué
tengo que hacer) independiente de lo que dice la moral tal o cual (sea judía,
mahometana, católica, etc.) es que puedo saber a qué atenerme y confiar
en que no seré sorprendido en el cambio de las reglas de juego, bajo el pre-
texto de “...mire el derecho decía que Ud. podía donar sangre, pero ahora los
jueces consideran que donar una parte del cuerpo es inmoral y, por lo tanto,
como el derecho depende de la moral, aunque ello no era ilegal, lo vamos a
declarar así y vamos a proceder a detenerlo...”.

Este debate suele ser llevado adelante en el terreno del positivismo versus el
iusnaturalismo. El mismo ha tomado diversos caminos. Desde el que sostiene
que el derecho no es derecho si no está de acuerdo con la moral tal o cual,
hasta aquél que sostiene que el derecho nunca es injusto, pasando por quien
propugna que el derecho puede ser inmoral pero en ese caso los jueces siem-
pre deberán apartarse de lo que él diga y aplicar directamente la moral.

Esta discusión, la relativa al derecho y su relación con la moral, será tra-


tada en los módulos sucesivos. Por ahora cabe retener que existe un interés
genuino, cuando se responde a la pregunta ¿qué es derecho? en tratar de
mostrar si el derecho se diferencia de las órdenes respaldadas por ame-

EDUBP | Abogacía | Filosofía y Lógica Jurídica - pag. 27


nazas (si se diferencia de la banda de delincuentes) y si se diferencia de la
moral o de alguna moral (como la religiosa).

El tercer problema recurrente: ¿qué son las reglas?

Una tercera pregunta frente a la que, de alguna manera, ya nos enfrentamos


es la relativa a qué son las reglas y en qué medida el derecho es una cuestión
de reglas.

En primer lugar, ya vimos que hay un universo de reglas enormes y decir


que el derecho es un conjunto de reglas no ilumina demasiado. La segunda
cuestión es que existe un problema sostenido en saber o determinar cuándo
existe una regla.

En este sentido, puede tender a pensarse que existe una regla cuando ella es
generalmente obedecida o existe un comportamiento general y regular de un
conjunto de personas que suele llevar adelante cierto tipo de acción (como
p. ej., ir a la cancha los domingos). Un aspecto importante, al momento de
caracterizar lo que una regla es y cuándo existe, es tratar de diferenciar ésta
de los meros hábitos o conductas convergentes. Los hábitos son tipos de
conductas regularmente realizados por personas que se comportan de una
determinada manera (hacen repetitivamente lo mismo).

Sin embargo, a diferencia del seguimiento de una regla, en el hábito se


manifiesta una conducta peculiar. Sólo existe una convergencia casual de
comportamientos entre los individuos que comparten el hábito. Así, la
razón por la que yo voy a la cancha los domingos no está apoyada en que el
resto de los individuos lleva adelante esa conducta, ni en la creencia de que
esa es la pauta de conducta correcta a seguir. Los individuos que convergen
en la cancha de fútbol no van allí motivados por la existencia de una regla.

La convergencia en la cancha entre miles de personas es casual y no


causal. Es decir, no está originada por la aceptación de alguna regla
social, según la cual “es correcto ir a la cancha los domingos”; y en
la elección de este hábito (las razones por las cuales el sujeto sigue esta
costumbre) no interviene la consideración de que él estime correcto lo que
hacen los demás yendo a la cancha. Así, existe una diferencia radical entre
decir que existe una regla –p. ej. dar el asiento a las embarazadas en el colec-
tivo – y el hábito de concurrir los domingos a la cancha de fútbol. Mientras
que en el hábito sólo tenemos un conjunto de personas que se comportan
homogéneamente, en las reglas intervienen otros fenómenos.

Paradigmáticamente en el caso de la regla, se considera que el patrón de


conducta que ella marca (lo que dice que hay que hacer) es correcto. De
este modo, estamos dispuestos a reprochar (no necesariamente sancionando)
a quien se aparta de ese patrón de conducta (supongamos que alguien no
deja el asiento a la embarazada) y vemos en la transgresión lo que justifica
o convalida nuestro reproche. El juez, por ejemplo, no ve en la regla simple-
mente la descripción de que muchos harán lo mismo (como quien observa al
hábito diciendo habitualmente todos van a la cancha) sino que ve en ella una
guía para su conducta y, cuando alguien la transgrede, ve en ese acto la razón
que justifica su reproche. Nada de esto ocurre con el hábito. Casualmente

EDUBP | Abogacía | Filosofía y Lógica Jurídica - pag. 28


vamos o van miles a la cancha, pero no ven en el hábito una guía para su
conducta, cuya violación estarían dispuestos a reprochar a alguien.

Aunque todavía no hemos avanzado demasiado, podemos sacar algunas


conclusiones parciales de nuestro primer paso en la materia:
i) Hemos descubierto que algunas descripciones del derecho eran algo
insatisfactorias y apresuradas y,
ii) hemos recordado que el desafío central de la teoría del derecho es
reconstruir conceptualmente en qué se diferencia el derecho de cualquier
sistema o ámbito práctico respaldado por amenazas (como la banda de
asaltantes) o ámbitos prácticos más amplios (como la moral).

Una salida que aún no hemos explorado es la que pretende evitar la pre-
gunta qué es el derecho y buscar una solución al problema echando mano
a una definición. Hasta ahora nuestro derrotero ha sido el tratar de seña-
lar qué cosas existen en la realidad que sean llamadas derecho y de qué
manera ellas se diferencian fenomenológicamente de otros fenómenos prác-
ticos (como los asaltantes y la moral). Quienes quieren evitar este problema,
suelen recurrir a las definiciones, es decir, a tratar de explicar una palabra
con palabras.

Este tipo de estrategia, aunque usualmente productiva, se encuentra con el


siguiente inconveniente: es necesario contar con un concepto más o menos
claro de los caracteres que en la realidad rodean al concepto que quiero
definir (cómo se usa, quiénes lo usan, qué hacen cuando lo usan, etc.). Pero
es justamente éste el problema que hemos tenido y tenemos con la pregunta
acerca de qué es el derecho. Quienes recurren a las definiciones echan
mano a terminología y clasificaciones que nos son familiares, de modo de
acercarnos por género y diferencia a lo que se nos quiere explicar o al fenó-
meno del que se nos quiere brindar una explicación (p. ej. para poder definir
“cuadrúpedo” debo contar con información mínima acerca de lo que es un
bípedo, qué diferencia hay entre tal y cual especie de animal, qué caracterís-
ticas tiene el género, y cuáles son algunas de las especies dentro del género
animal etc.). Pero si a quien yo le estoy tratando de definir la palabra, también
carece de esas herramientas conceptuales (no puede traducir las palabras
que uso en la definición), el problema sigue siendo el inicial: no tengo la
menor idea de qué significa o en qué consiste lo que quiero definir. Es como
buscar el significado de una palabra del alemán, en un diccionario inglés-
alemán, sin saber tampoco inglés. Todo lo que se me diga, aún cuando se
me trate de dar una descripción en términos de género y diferencia, resultará
terriblemente infructuoso y desalentador. Le proponemos que a partir de
aquí, sigamos siempre con la pregunta inicial: ¿qué es derecho? El desa-
fío será entonces emprender este camino hacia una respuesta. Pero ello ya
será motivo de los módulos siguientes y de los capítulos II, III y IV del texto
de Hart, El Concepto de Derecho.

Como complemento del materia presentado, lo invito a profundizar estu-


diando los Capítulos I y II de HART, Herbert: El concepto de derecho. Buenos
Aires, Abeledo Perrot, 1992.

EDUBP | Abogacía | Filosofía y Lógica Jurídica - pag. 29


Si usted desea ampliar sobre estos temas, también puede consultar la
siguiente bibliografía:
• NINO, Carlos Santiago: Introducción al Análisis del Derecho. Buenos
Aires, Astrea, segunda edición ampliada y revisada, 1987. Capítulo I.
actividades m1
• ATIENZA, Manuel: Introducción al Derecho. Barcelona, Barcanova, pri-
mera edición, 1985. Capítulo 1.
• Dabove, María Isolda: El concepto de derecho en la teoría HL Hart pers-
pectiva tridimensional. Investigación y docencia, 2003 – Recuperado de
http://www.cartapacio.edu.ar/ojs/index.php/iyd/article/viewFile/859/685

Con la finalidad de transferir los conceptos teóricos estudiados, lo invito a


realizar las actividades del módulo.

M1 Actividades
Actividad 1

“Haciéndole caso a mi Rey: Un estudio antropológico”


Ud. es antropólogo de un país en el que no existe ni el fútbol ni el derecho, ni
el rock and roll Como buen antropólogo, Ud. sólo se limita a describir lo que
ve. A su vez, es una persona que está investigando nuevas costumbres que
no conoce, por lo que sólo se dedica a describir lo que hace el grupo social,
tribu o comunidad.

Así, ya ha realizado innumerables investigaciones con el propósito de expli-


carle a la comunidad científica de su reino qué es esto del rock and roll y de
los recitales. Por ejemplo en su último libro, titulado El rock and roll, una
danza tribal de los países de occidente, Usted escribió lo siguiente:

En las comunidades analizadas, la práctica que ellos llaman rock and roll
consiste básicamente en que una persona o personas se paran arriba de
una especie de altar. Suelen recibir el nombre de “músicos” (no he podido
averiguar si “músicos” es o no un sinónimo de lo que nosotros llamamos
en nuestro reino “dioses”). El resto de la gente que no está arriba del altar
conoce lo que estas personas gritan. Incluso repiten sin cesar lo que los
llamados “músicos” gritan, como si ya hubiesen sido educados en esos ser-
mones. Los llamados músicos se paran frente a un palo vertical en el que
gritan fuertemente. Le suelen llamar micrófono (es posible que el micrófono
sea una especie de báculo o artefacto divino). Este rito suele terminar con la
gente gritando una rara expresión casi impronunciable (¡otra!, ¡otra!). He lle-
gado a la conjetura de que esa expresión es una especie de reconocimiento
o cierre del acto de la tribu (....) Otro hecho a destacar es que realizan estas
danzas y oraciones en una especie de templos muy grandes llamados “esta-
dios”. Mi corta estancia en este lugar no me ha permitido averiguar cuál es

EDUBP | Abogacía | Filosofía y Lógica Jurídica - pag. 30


la función de estos “estadios”. Es posible que sean usados también para una
actividad que tiene lugar una vez a la semana que aquí denominan “fútbol”.
Allí la gente ya no está parada sobre el césped sino sobre la parte “dura” de
estos templos. Ellos llaman a esa parte dura (el material tampoco es cono-
cido en nuestro reino) “tribunas”. En el césped hay 22 personas que corren
desesperadamente tras un esfera de cuero........”.

Intrigado por los resultados de la investigación, su rey decide encomendarle


que prosiga indagando sobre la práctica del fútbol. Además, a su rey tam-
bién le ha resultado interesante conocer sobre lo que estos indígenas llaman
“derecho”, por lo que Ud. es enviado al país occidental en cuestión a con-
tinuar la investigación. Fundamentalmente, le ha ocasionado mucha intriga
diferentes fenómenos del llamado “derecho” de los que ha escuchado por
versiones de viajeros. Por ejemplo ha escuchado palabras extrañas como
“tribunales”, “policía”, “códigos”, “jueces”, “asaltantes”, “contratos”, “pri-
sión”... Luego de dos meses de estadía en el lugar, usted tiene alguna con-
jetura sobre el fenómeno del fútbol y del derecho.

Redacte, en clave de científico antropológico, una descripción de las caracte-


rísticas centrales de estas prácticas de la tribu occidental y hágaselas saber
a su jefe (rey) en una carta.

El antropólogo como un observador de una práctica ajena a su cultura sólo


registra regularidades de hecho. ¿Qué descripción de la práctica del derecho
de una sociedad extraña puede darse desde tal perspectiva?

Actividad 2

“La tesis de división entre el derecho y la religión o moral” y “La tesis de


diferenciación entre la banda de asaltantes y los funcionarios del Estado”
Usted ha terminado su viaje de investigación como antropólogo por los
países occidentales. Le ha escrito innumerables cartas a su rey tratando
de explicarle los puntos centrales de las prácticas de estas tribus. Sin
embargo, Ud. no ha quedado del todo satisfecho con la descripción o grado
de conocimiento que ha logrado de la práctica relativa al derecho. El pro-
blema más significativo que usted visualiza es que no ha llegado a identificar
claramente en qué consiste lo que las tribus analizadas llaman derecho.

En especial, le ha costado encontrar diferencias entre lo que los indígenas


de esas tribus llaman derecho y otras supuestas instituciones. Por ejemplo,
algunas partes de lo que ellos llaman código penal se parecen en muchos
aspectos a lo que ellos llaman “los diez mandamientos”. Aunque estos libros
(Código Penal y Biblia) difieren en muchos aspectos, Ud. genuinamente ha
encontrado innumerables coincidencias, por lo que aún no está en condicio-
nes de saber si ambos son parte del mismo fenómeno (derecho) o no.

EDUBP | Abogacía | Filosofía y Lógica Jurídica - pag. 31


Similares dificultades ha tenido Ud. para distinguir entre dos grupos de per-
sonas. Unas que son llamadas “agentes o funcionarios públicos” y otras
que son denominadas “ladrones o asaltantes”. Según su experiencia, bási-
camente, los llamados “funcionarios” que Ud. tuvo oportunidad de observar
se visten de azul, usan armas y exigen determinadas cosas a los individuos
(les piden documentos, usan la fuerza cuando no se les obedece y hasta
llegan a disparar).

Sin embargo, también tuvo oportunidad de conocer un grupo muy bien ves-
tido y educado de personas que también tenía armas y exigían a la gente que
haga o deje de hacer cosas. Estos últimos eran denominados “asaltantes”.
Ud. no ha podido llegar a una conjetura acabada de la diferencia sustancial
entre el grupo de los llamados “asaltantes” y el grupo de los llamados “fun-
cionarios” (policías).

Decide, no obstante estas dudas, exponer a su rey cuáles son los hechos
que para Ud. diferencian o no claramente a una Biblia de un código penal,
y cuáles son los que le parecen que diferencian al grupo vestido de azul del
que es considerado una banda de asaltantes. Recuerde que siempre existe
el peligro de que el rey objete su análisis, con el argumento según el cual
“...lo que Ud. me dice no es suficiente para demostrar que las instituciones
observadas son diferentes....” Dado que Ud. ya ha avanzado en sus conoci-
mientos de la comunidad, puede introducir algunos términos más técni-
cos para explicar las diferencias o similitudes.

Usted se propone emprender un trabajo donde exponer su hipótesis. El


mismo consta de dos partes. Una de ellas titulada: “La tesis de división entre
el derecho y la religión o moral”; y la otra, “La tesis de diferenciación entre
la banda de asaltantes y los funcionarios del Estado”

Se sugiere ocupar como máximo dos páginas para la elaboración del trabajo,
y tener presente las razones que hemos sostenido en los contenidos del pre-
sente módulo que avalan las dudas sobre la identidad entre un código penal
y la Biblia; y las dudas que genera la identificación de los llamados “agentes
o funcionarios públicos y aquellos que son denominados asaltantes.

Actividad 3

“Demostrando”
A esta altura, usted ya ha obtenido bastante experiencia por medio de las
actividades realizadas en su período de investigación, propuestas en las acti-
vidades 1 y 2. Como uno de sus descubrimientos centrales usted ha caído
en la cuenta que el fenómeno del derecho consiste particularmente en
una cuestión de reglas. Sin embargo, en su investigación ha surgido un
problema. Un antropólogo de la corte, (con quien Ud. disiente), en función
de su propias investigaciones, ha podido concluir que nuestra tesis de que el
derecho es fundamentalmente una cuestión de reglas es una afirmación algo
apresurada e incompletamente verdadera. Cabe resaltar que este antropó-
logo estuvo unos meses en el país donde usted ha realizado su investigación.

EDUBP | Abogacía | Filosofía y Lógica Jurídica - pag. 32


Según nuestro opositor, en el país observado hay innumerables prácticas
que envuelven reglas y que no son llamadas “derecho”. Básicamente sos-
tiene que las reglas del deporte y de la cocina son, en sí mismas, reglas
y nadie las confunde con el derecho. Además, esta persona sostiene que,
como no hemos realizado muy bien nuestra investigación, no estamos en
condiciones de señalar instancias paradigmáticas o casos claros de reglas
jurídicas. Según este argumento, dado que hemos sido “malos investigado-
res”, podemos llegar a tomar por jurídicas reglas que no lo son.

Usted, en un argumento de una página, decide contestarle a su opositor


tratando de demostrarle lo siguiente:
I) que Ud. está al tanto de que no toda regla es una regla jurídica;
II) que existe una clasificación básica entre diferentes tipos de reglas;
III) que es capaz de señalar instancias paradigmáticas de reglas jurídicas.

En este contexto, ofrezca un ejemplo de una regla constitucional, una proce-


sal, una penal y otra civil.

EDUBP | Abogacía | Filosofía y Lógica Jurídica - pag. 33


MÓDULO 2

M2 Microobjetivos

• Conocer la forma de análisis de las normas jurídicas, de modo de identi-


ficar la diferencia entre las órdenes coercitivas, simplemente respaldadas
por amenazas, y las obligaciones jurídicas.
• Diferenciar las normas primarias de las secundarias, con el fin de poder
reconstruir la diferencia entre dos grandes grupos de normas jurídicas:
las que prohíben acciones y las que permiten identificar, cambiar o apli-
car normas primarias.
• Comprender los aspectos centrales de un sistema jurídico, identificando las
diferencias principales entre un sistema de reglas primarias y secundarias.
• Revisar las propias nociones de autoridad legítima/ilegítima, leyes,
normas y hábitos, para entender cuáles son las claves de la práctica
social llamada Derecho.

M2 Contenidos

Lo invito a ver el video de presentación del Módulo 2

Normas, mandatos, órdenes y amenazas. Nuestro hábito de obediencia


y las obligaciones jurídicas.
En el módulo I, delineamos algunos puntos centrales para nuestro estudio de
la materia. En primer lugar, concluimos que nuestra pregunta básica, a la que
intentaríamos dar respuesta es, en definitiva, ¿qué es el derecho?

Luego, anotamos en nuestra “agenda académica” tres problemas fundamen-


tales que procurarían ser trabajados en orden a responder estas cuestiones:
i) la del derecho y su relación /diferencia con las órdenes respaldadas
por amenazas;
ii) el problema de la relación entre el derecho y la moral;
iii) la elucidación del concepto, idea o noción de regla.

Comenzaremos por analizar el problema de la relación entre nuestra noción


de regla, regla jurídica y órdenes respaldadas por amenazas. Es decir, anali-
zaremos en qué sentido las órdenes del legislador pueden ser reducidas
a la idea de orden respaldada por una amenaza. Parecería ser plausible
sostener que las órdenes respaldadas por amenazas se asemejan, en un
punto, a los “mandatos” (commands). Esto implica que no habría diferencia
entre una orden de quien me grita “dame la plata sino te mato”, de aquella
orden dada por mi jefe en el trabajo o por el policía de tránsito. Éstos últi-
mos suelen proferir órdenes o mandatos del siguiente tipo “entrégueme X
(el expediente, el automóvil, la tarjeta verde) o sufrirá las consecuencias”.
Las semejanzas entre ambas “órdenes” (por ahora entrecomillaremos este

EDUBP | Abogacía | Filosofía y Lógica Jurídica - pag. 34


término) son fáciles de advertir: ambas requieren de mí una conducta (que
haga u omita algo) y respaldan (refuerzan) esa “orden” o “pedido” con el
anuncio de un mal.

¿En dónde reside, entonces, nuestra intuición según la cual tendemos a


creer que hay una diferencia sustancial entre la orden del asaltante y la de
nuestro jefe o el empleado municipal?.

Y si continuamos pensando, podríamos preguntarnos si no hay acaso una


diferencia entre aquél que me apunta con un arma y mi jefe que está parado
frente a mí con una taza de café en la mano, ordenándome que termine
rápido el trabajo sino me va a reemplazar por alguien más eficiente. Nuestra
intuición no es para nada débil.

En determinados contextos donde consideramos a los “mandatos” u


“órdenes” como válidas, existe una autoridad jerárquicamente estable-
cida, sobre la que depositamos algún tipo de responsabilidad o respeto.
Creemos, legítimamente, que debemos hacer lo que ella manda, no necesa-
riamente por el mal que acaecerá si no lo hacemos, sino más bien porque, en
algún sentido de la palabra, creemos que tiene “legitimidad” para decirnos lo
que debemos hacer.

En estos contextos (como los del jefe o el empleado municipal), no es nece-


sario incluso que exista una amenaza.

Mandar es característicamente ejercer autoridad sobre hombres y no por


tener el poder de causar daño. Aunque es posible que se combinen las ame-
nazas de daños con un mandato u “orden” (en el sentido que le damos a la
orden del jefe y no a la del asaltante), no es primariamente una apelación
al miedo lo que caracteriza a las reglas jurídicas o institucionales, sino
el respeto a la autoridad. Por lo tanto, no parece muy adecuado identificar
las órdenes respaldadas por amenazas con las órdenes o reglas jurídicas
(o institucionales de cualquier tipo), ya que sólo éstas últimas están relacio-
nadas de una manera especial con el principio de autoridad de quien las
emite. De todos modos, la noción de mandato u “orden de trabajo”, “orden
de mando”, etc. está demasiado cerca de la idea de autoridad; y la idea
de autoridad demasiado cerca de la idea de derecho como para ser útil en
nuestra empresa de caracterizar al derecho. Aunque esto nos puede arrojar
alguna luz sobre lo que queremos contestar (sobre qué es el derecho, cuál
es la diferencia entre la orden del asaltante y la del legislador, etc.), la idea
de “autoridad” está tan ligada a la idea de “orden” y derecho que corremos
el riesgo de presuponer lo que hay que probar. Es decir, corremos el riesgo
de contentarnos con decir “el derecho detenta autoridad mientras que el
asaltante no”, sin haber contestado a la pregunta que queríamos respon-
der. Porque si ahora uno se conformase con decir “el derecho es aquel que
detenta autoridad”, la pregunta se reduciría a “¿en qué consiste el mágico
proceso por el cual Ud. dice que el derecho tiene autoridad y no el asaltante?

Si bien hasta aquí resulta legítima nuestra asociación de la idea de “orde-


nar” con un cierto rasgo de autoridad, legitimidad o poder de quien la emite
(como el caso del jefe o el empleado policial) y no con la amenaza del asal-
tante, es conveniente seguir nuestra búsqueda por otros rumbos.

EDUBP | Abogacía | Filosofía y Lógica Jurídica - pag. 35


Aunque por ahora no ingresemos en el análisis del derecho a partir de la
idea de autoridad, sí podemos tener en cuenta que cuando reconocemos
a alguien autoridad o poder jurídico, le atribuimos a esa persona una cierta
posición institucional y un cierto valor a sus expresiones (órdenes) que no
atribuimos a un asaltante.

Un camino alternativo sería analizar el contenido de la “orden” del asaltante:


a quién se dirige; cómo es dada a conocer a su destinatario; y tratar de ver
en qué sentido ella se diferencia de las “órdenes” jurídicas. Como el modelo
del asaltante es bastante tosco, trataremos más bien de analizar el modelo
de quienes sostienen que el derecho es básicamente un conjunto de
órdenes respaldadas por amenazas. La estrategia que seguirá nuestro aná-
lisis es, a partir del modelo de derecho como órdenes coercitivas (el que se
diferencia, en algunos puntos, de la simple idea del asalto), detectar si éste
es capaz de dar cuenta de la complejidad del fenómeno jurídico.

J. Austin es quien, a partir de nociones muy simples (órdenes respalda-


das por amenazas, hábito de obediencia y el concepto de soberano) ha
intentado (con bastante éxito) elucidar el concepto de derecho. Y es a partir
de esta propuesta teórica, que Hart efectúa el análisis que emprenderemos
a partir de aquí.

El derecho como órdenes coercitivas

Aunque continuamos con nuestra empresa de distinguir el derecho de las


órdenes respaldadas por amenazas, ya hemos concedido innumerables
veces que un amplio grupo de las órdenes jurídicas se parecen –en gran
medida– a las órdenes coercitivas. Veamos, por ejemplo, las normas del
derecho penal. En principio, no hay ninguna diferencia entre decirle a alguien
“dame la plata o te disparo” (como diría el asaltante) y “entrégueme el dinero
o será llevado a juicio por evasión impositiva y podrá ser condenado a 10 años
de prisión”. Aunque el estilo de las órdenes sea diferente (el ladrón lo dice a
los gritos y el agente de la AFIP lo dice muy tranquilo sentado tras un escritorio
y con música clásica de fondo), el contenido parece ser bastante similar.

Sin embargo, existe entre ambas una diferencia que cabe destacar: la gene-
ralidad.Mientras que las órdenes de un asaltante se dirigen directamente a
la persona de la que se pretende el dinero, el derecho suele dirigirse a los
sujetos pasivos de la orden de forma general. Basta imaginarse lo desca-
bellado que sería tratar que cada funcionario estatal hiciese conocer a cada
ciudadano lo que el derecho pretende de él. Por ello, resulta fácil detectar
que el derecho detenta, como rasgo central, pretensiones de generalidad
(erga omnes, como nos gusta decir a los juristas).

Así, aunque hay normas, como las del derecho penal, que se asemejan cla-
ramente a las órdenes respaldadas por amenazas del asaltante, difieren de
éstas en el sentido de prescribir un tipo general de conducta y de aplicarse
a una clase general de personas. La norma penal, p. ej., espera lo siguiente:
i) que los individuos adviertan que la orden rige para ellos (se les aplica);
ii) que cumplan con lo prescripto.

EDUBP | Abogacía | Filosofía y Lógica Jurídica - pag. 36


En conclusión, puede señalarse el punto de la generalidad como un primer
rasgo distintivo de las normas jurídicas frente a las órdenes respaldadas por
amenazas del asaltante. Estamos aquí frente a una doble generalidad. La
norma jurídica se refiere a una clase general de individuos, e indica una clase
general de conductas.

El remitente y el destinatario. La dirección de las normas.


Una segunda diferencia importante entre la orden del asaltante y la del legis-
lador es aquella relativa a la dirección que tiene una norma o al conjunto
de individuos a los que ella se dirige. Según el carácter de generalidad que
vimos antes, el derecho suele hacer conocer sus pretensiones de manera
colectiva y general. Sería demasiado costoso ir casa por casa, contándoles
a los ciudadanos que “...a partir de mañana el monotributo ascenderá a 100
pesos...”. Así, es bastante usual sostener que las normas se dirigen hacia un
grupo de individuos. Veamos qué se quiere decir con esto.

Si lo que se quiere enfatizar es que la orden va dirigida a un conjunto de per-


sonas a las cuales se las tratará, juzgará, condenará, de acuerdo al contenido
de la norma, la utilización de la expresión es correcta. Lo único que estamos
diciendo, en definitiva, es que la norma se aplica al conjunto de individuos
a los que ella refiere. Si ella dice “los menores de 21 años deberán realizar
servicios sociales durante el lapso de 3 meses en hospitales públicos”, los
menores de 21 años en Argentina, tendrán que ir a los hospitales públicos
durante el tiempo estipulado. Sin embargo, a veces se pretende decir que
el derecho se dirige hacia un grupo de individuos, en el sentido de que si el
derecho no es conocido por los individuos, no existe. Aquí la expresión “diri-
girse a un grupo de individuos” no significa “se aplica a un conjunto de indi-
viduos” sino más bien “se le ha hecho conocer a un conjunto de individuos”.

Sin embargo, estamos más que acostumbrados a sostener que existe el


derecho o una norma jurídica con independencia del conocimiento que de él
tienen los individuos.

Es importante destacar que las normas jurídicas detentan determinadas pro-


piedades que nos permiten sostener que las normas existen, están dirigidas
a un conjunto de individuos o se aplican a ellos, aún cuando no han
sido dadas a conocer ampliamente. Aunque siempre es deseable que las
normas sean conocidas por todos, nos manejamos con la “ficción” de
que las normas jurídicas existen, aunque no se conozcan (nadie podría
aducir, por ejemplo, que pasó el semáforo en rojo porque no estaba enterado
de que estaba prohibido por una norma jurídica) y esto constituye un rasgo
central para distinguir las reglas jurídicas de otro tipo de órdenes.

De este modo, mientras que solemos decir “existe esta norma jurídica sobre
quiebras, aunque pocos la conocen”, sería muy raro decir “Juan intentó robar
a su vecino, gritando en soledad, encerrado en el baño, dame la cortadora
de césped”. Nadie sostendría sensatamente que existe un robo (una orden
de entregar algo respaldada por amenazas) en el caso de una persona orde-
nando en soledad la entrega de una cosa, mientras que es usual sostener
que ya existe la norma jurídica aunque nadie o muy pocos la conozcan.
Todo esto indica que no debemos asociar la idea de que existe una norma

EDUBP | Abogacía | Filosofía y Lógica Jurídica - pag. 37


jurídica a la idea de que la norma ha sido puesta en conocimiento de los
individuos. Es posible (y de hecho ocurre) que haya normas escasamente
conocidas por los individuos, pero de ello no se sigue que no consideremos
a esas normas como existentes. Un fenómeno central en los sistemas jurídi-
cos modernos (que nos permite diferenciarlos de la orden de los asaltantes)
radica en que puede haber órdenes o normas jurídicas, sin que ellas sean
estrictamente conocidas por los sujetos a quienes se les aplican. Existe una
fuerte e importante diferencia entre la persona a quien la norma se aplica
y la persona a quien la norma se ha hecho conocer. Mientras que en el
caso del asaltante no hay orden de robo sin que la víctima conozca la preten-
sión de robo, es perfectamente factible pensar que existe una norma jurídica
que se aplica a un sujeto (como el caso de los menores de 21 años) aunque
éstos no conozcan todavía que están o caen dentro del alcance de la norma.

Un punto central relacionado con el tema del “conocimiento” que acabamos


de ver es el atinente a la perduración de la orden respaldada por ame-
naza del asaltante versus la perduración de la orden o norma jurídica.
Nosotros asumimos que la orden que nos da el asaltante perdura mien-
tras dure el “asalto” (me apunte con el arma, tenga amenazado al cajero,
etc.), pero una vez que ello finaliza no seguimos actuando “bajo las órde-
nes del asaltante” en el resto de los aspectos de la vida. De hecho, cuando
regresamos al banco donde nos robaron, no seguimos entregando dinero a
cuanta persona se nos acerque, ni consideramos que deberíamos hacerlo
(salvo que aparezca nuestro amigo ladrón y venga por más).

Asumimos, en este contexto, que la relación de superioridad que detenta


el asaltante sobre el asaltado es efímera. Por el contrario, no considera-
mos efímera la influencia que el derecho ejerce sobre nuestro accionar
cotidiano. Seguimos actuando bajo el influjo de las normas jurídicas aunque
estemos fuera de tribunales o la comisaría, e incluso aunque el legislador o
legisladores que las dictaron se hayan muerto. El código penal es un caso
claro. Los artículos centrales fueron legislados en 1921 y es altamente pro-
bable que no quede vivo ni uno de los que participaron en su sanción. ¿Por
qué, entonces, no actúo de igual manera a como actuaría si el asaltante del
barrio falleciera?

Nos encontramos aquí, en resumen, con un fenómeno interesante. Las órde-


nes del asaltante no perduran en el tiempo, ni son habitualmente obedeci-
das. Lo que no sucede con el derecho. No se pretende sostener aquí que
estos rasgos sean necesarios y suficientes para identificar el derecho. Sólo
se pretende resaltar la idea de volatilidad de las órdenes de los asaltantes
frente a la idea de perduración de las órdenes jurídicas y de cómo éstas
últimas habitualmente son obedecidas (la práctica persiste) incluso en
ausencia de quien ordena. Hasta aquí, podría decirse que hemos clarifi-
cado una pequeña parte del panorama.

Sabemos que donde sea que exista un sistema jurídico encontraremos


alguna persona o cuerpo de personas que emite órdenes generales que
perduran en el tiempo, y que existen y se aplican a los individuos a pesar
de que no se las haya hecho conocer a cada uno de los sujetos pasivos
de las mismas (destinatarios).

EDUBP | Abogacía | Filosofía y Lógica Jurídica - pag. 38


Aún nos resta (para clarificar la idea de “derecho” y “sistema jurídico”) com-
prender por qué estas normas son vistas por los individuos como supre-
mas y por qué, además, quien las dicta no está sometido, a su vez, a otro
órgano supremo. Es decir, sabemos que hay un conjunto de individuos a los
que consideramos legitimados para dictar órdenes (legisladores) y también
sabemos que ellos no tienen a otro órgano superior sobre sus cabezas.

Por lo tanto, aunque hemos explicado y explorado rasgos centrales del dere-
cho como son la generalidad de las normas y la habitualidad con las que
ellas son obedecidas, aún resta dar cuenta de otro aspecto central de estas
reglas: la supremacía del órgano del que emanan, y la independencia de
este órgano de otros órganos externos. En palabras más simples, por qué
nuestro Congreso y no el de Rusia, por ejemplo, rige nuestros destinos.

Sumado a esto, aún nos quedan otros desafíos importantes: la noción de


regla es un punto central, y elucidarla no es tan sencillo como aparenta.

El zoológico de las normas jurídicas. Un montón de especies con apa-


rentes rasgos en común.
El “modelo de derecho como órdenes coercitivas”, a pesar de que lo hemos
diferenciado del asalto, veremos que no resulta suficiente para caracterizar
la complejidad del fenómeno jurídico. Así, aunque existe una tendencia a
sostener que todas las reglas comparten una sola y única propiedad o carac-
terística (son coercitivas), ello sólo puede lograrse mediante artificios que
desfiguran el fenómeno jurídico, tergiversando el material jurídico disponible.
Así, por ejemplo, no sólo hay normas que prescriben una sanción (como
las del derecho penal), sino también hay otras especies en nuestra fauna:
existen normas para contratar, para contraer matrimonio, las normas cons-
titucionales que indican cómo sancionar otras normas, etc. que no parece
que puedan caracterizarse como normas que prescriben una sanción y
claramente se las califica como jurídicas.

Veremos a continuación cómo es conveniente diferenciar las normas de


acuerdo a su contenido (no sólo hay normas que imponen deberes a través
de una sanción para su incumplimiento) y, además, cómo el modelo simple
de órdenes coercitivas no da cuenta de los diferentes modos de origen de
las normas y de su ámbito de aplicación.En el desarrollo del texto de Hart
estos puntos centrales de su crítica al sistema de Austin quien consideraba a
la derecho como ordenes respaldas por amenazas.

El contenido de las normas jurídicas


Como estudiante de abogacía, posiblemente usted tenga bastante claro que
las normas penales y algunas del derecho extracontractual detentan como
contenido un hecho o acción que debe ser omitido o realizado por el sujeto
a quien se dirige la regla. Así, se pretende que el ciudadano omita matar, que
ayude a tal o cual persona, que no lesione, etc. Este contenido (lo que se
manda a hacer u omitir) se enlaza o relaciona casi siempre con una sanción
o el pago de una suma de dinero. En este sentido, es bastante claro que este
tipo de normas se adecuan perfectamente al modelo de órdenes coercitivas.

EDUBP | Abogacía | Filosofía y Lógica Jurídica - pag. 39


De esta manera, uno podría decir que las reglas penales o extracontractuales
exigen o persiguen que los individuos actúen de un modo determinado lo
quieran o no (les impone un deber).

Sin embargo, a poco que seguimos introduciéndonos en la selva jurídica,


nos encontramos con otras especies de normas que no pretenden que los
individuos realicen una acción u omitan hacer algo en contra de su voluntad.
Tampoco enlazan esa conducta con una amenaza de sanción. Este es típi-
camente el caso de las normas contractuales, matrimoniales o testamenta-
rias. Estas normas no pretenden forzar o imponer deberes u obligaciones,
sino que se refieren a las condiciones bajo las cuales se pueden crear o
modificar derechos. Parece que existe la tentación a violentar la realidad de
manera tal de construir artificios que, en definitiva, muestren que todas las
reglas jurídicas se caracterizan por un solo elemento: la sanción. Un artificio
de este tipo sostiene que, en última instancia, siempre que encontremos una
regla “sin sanción”, ésta constituye sólo un pedazo de una norma completa
enlazada, de una u otra manera, a una regla con sanción y que siempre será
posible mostrar que todas las reglas buscan que alguien haga u omita algo.

Sería conveniente que, en este punto, nos detengamos y releamos normas


claves en el ámbito contractual, del derecho de familia o procesal. Allí obser-
varemos que estas normas no dicen (como lo hace una norma penal) “haga
esto, lo quiera o no”, sino más bien “si quiere hacer esto, esta es la manera
de hacerlo”. No pretenden empujarnos a que contratemos, nos casemos,
sancionemos leyes, sino más bien a que, si decidimos hacerlo, sigamos
un cierto procedimiento.

Por ejemplo, una declaración testimonial que carece de firma (del testigo, el
secretario del juzgado, etc.) o un contrato de compraventa que no ha sido
hecho por escrito, cuando la ley lo indica así. El objeto de la norma que
impone la nulidad no es disuadir a las personas de realizar actos impro-
pios, sino definir las condiciones y límites bajo los cuales sus actos ten-
drán los efectos buscados (contratar, casarse, etc.).

Veamos otro ejemplo, un juez que permanentemente realice actos sin tener en
cuenta las normas de procedimiento que le indican cómo actuar. Ingresando
un poco más en detalle, es posible que el juez sea llevado frente a un jury
de enjuiciamiento o que sea condenado por prevaricato. Pero no hay una
conexión necesaria entre la norma que prescribe la nulidad de un acto judi-
cial y la sanción que recae sobre el juez por incumplimiento de los deberes
de magistrados. Es perfectamente factible que se declaren nulos uno y mil
juicios por defectos formales (falta de notificación de las partes, ausencia del
abogado defensor, etc.) y que, sin embargo, no recaiga una sanción sobre
el juez. Esto muestra claramente que las condiciones bajo las cuales un acto
se declara nulo o inválido son totalmente diferentes a las condiciones bajo
las cuales un acto se declara prohibido u obligatorio (bajo amenaza de san-
ción). También muestra que el contenido de tales normas (las que acarrean
nulidad e invalidez, y las del derecho penal o extracontractual) es totalmente
diferente y persigue fines sociales distintos. Unas pretenden que el individuo
haga o deje de hacer algo, lo desee o no. Otras regulan el modo en que
un cierto juego debe jugarse. Esto, sí y sólo sí, el sujeto a quien se dirige la
norma quiere jugarlo (p. ej., si quiere jugar al juego de contratar, de casarse,

EDUBP | Abogacía | Filosofía y Lógica Jurídica - pag. 40


de sancionar una ley, etc.). Por lo tanto, sólo forzando extremadamente las
cosas puede equipararse la sanción de prisión o el contenido de una norma
penal al de una contractual o a algunas de las normas de procedimiento de
un código procesal. Además, las normas que prescriben la invalidez o nuli-
dad de una norma o acto no tienen por qué ser vistas necesariamente como
un mal para la persona contratante o para el juez que quiso dictar la senten-
cia. Tampoco deben ser analizadas como normas que pretenden desalentar
todo lo que no sea realizar un contrato válido o un testamento válido o una ley
válida. Mientras que la norma penal pretende claramente desalentar nuestras
intenciones de matar (diciendo no mates) o de estafar (diciendo no adminis-
tres fraudulentamente), las normas contractuales, por ejemplo, no pretenden
que sólo te dediques a realizar contratos, del mismo modo que las normas
que indican cómo se sanciona una ley no pretenden que el legislador se
dedique sólo a involucrarse en la sanción de leyes. Un ejemplo bastante
claro es el que da Herbert Hart con respecto a las reglas del fútbol. En él
existen algunas reglas que pretenden que los individuos hagan o dejen de
hacer determinadas conductas bajo amenaza de sanción (como por ejemplo
cuando se prohíbe los codazos, las patadas de atrás y demás prácticas que
alejan al juego del fair play). Pero también existen otras que establecen las
condiciones bajo las cuales se considera que existe o no un gol. Así, imagi-
nemos una regla paradigmática para cada uno de estos casos:

N1: el jugador que golpease a su contrincante de atrás será expulsado.

(Una forma más habitual de decir lo mismo sería: prohibido golpear).

N2: se considerará gol cuando la totalidad de la esfera haya transpuesto


la línea imaginaria trazada por los tres postes y la línea blanca que bajo
el poste horizontal cruza el césped –arco, (en criollo: sino entra no es gol.
No vale como gol).

Parece bastante claro que el primer caso es una instancia clara de una norma
penal. Pretende desalentar conductas. La segunda, en cambio, no pretende
nada de eso. Sólo pretende indicarnos que si queremos ganar o hacer goles,
la pelota tiene que entrar al arco. Mientras que N1 pretende desalentar toda
conducta violenta, N2 no pretende declarar inválida toda conducta que no
sea hacer goles.

De igual manera, las normas procesales o contractuales no pretenden des-


alentar el resto de las conductas que no se compadecen con la norma, sino
sólo indicarnos qué tipo de procedimientos debemos seguir si queremos
determinados resultados. Para seguir con el ejemplo del fútbol, la regla N2
no pretende que los jugadores sólo se dediquen a hacer goles, ni declara
que cualquier cosa que no sea entrar la pelota dentro del arco está prohi-
bida. Si equiparásemos las normas penales a las procesales o contractuales,
correríamos este riesgo de tergiversación. Terminaríamos afirmando que las
normas que otorgan facultades (públicas o privadas) pretenden que el juez
sólo se dedique a fallar, o que los sujetos sólo se dediquen a contratar. Es
muy importante comprender la función central que en un sistema jurídico
cumplen las normas potestativas. Si prescindiéramos de ellas (de las normas
contractuales, de matrimonio, de las que indican cómo se sanciona una ley
o se dicta una sentencia) no sólo nos quedaríamos sin instituciones centra-

EDUBP | Abogacía | Filosofía y Lógica Jurídica - pag. 41


les para la vida jurídica (como las transacciones comerciales o la unión de
parejas bajo un amparo jurídico), sino que también careceríamos de la posi-
bilidad de que el derecho cambie, se transforme y mejore. En este sentido,
las reglas que estamos analizando permiten (ya veremos esto más adelante)
que el derecho cambie, avance y se transforme. Así, mediante un contrato,
yo puedo cambiar mi situación jurídica (puedo pasar a ser propietario, ven-
dedor, locatario, concesionario, etc.). Mediante la firma frente al funcionario
del registro civil, puedo modificar mi status de soltero, lo cual implica que
automáticamente se modificarán los derechos y deberes que me circundan
y se me aplican. Del mismo modo, así como con las reglas contractuales
puede cambiar la situación jurídica propia o de un conjunto de sujetos, las
reglas que indican cómo dictar o sancionar una ley permiten cambiar o modi-
ficar la situación jurídica de una clase o conjunto más o menos amplio de
individuos. Piénsese en el caso de las leyes de inmigración que transforman
de un día para el otro a extranjeros en ciudadanos, o las leyes penales que
transforman una conducta que antes era neutral o inocua en algo que es
considerado delito.

De este modo, estamos en condiciones de conceder que no todas las


normas jurídicas tienen el mismo contenido, persiguen el mismo fin y aca-
rrean las mismas consecuencias para sus destinatarios. Unas quieren que
hagamos algo aún en contra de nuestra voluntad, otras persiguen que cree-
mos o extingamos derechos de una manera particular. Algunas nos dicen
dónde no podemos entrar, cuánto debemos pagar; mientras que otras nos
dicen cómo debemos entrar y de qué manera debemos pagar. En otras pala-
bras, mientras que las reglas semejantes a las del derecho penal impo-
nen deberes, las reglas que confieren potestades son fórmulas para la
creación de ellos.

Las reglas que confieren potestad como fragmentos de normas jurídicas.

Un argumento alternativo al anterior es aquel que pretende unificar las


normas jurídicas sosteniendo que todas y cada una de ellas se dirigen en
definitiva a los funcionarios, exigiéndoles que apliquen tal o cual sanción o
tal o cual consecuencia jurídica a los individuos. Mientras que el primer argu-
mento que analizamos en el punto anterior pretendía sostener “...en definitiva
todas las normas prescriben una sanción....” éste pretende defender que “...
en definitiva todas las normas se dirigen a funcionarios (jueces, fiscales)
a quienes se les impone la obligación de aplicar la normativa vigente a
los ciudadanos...”.

Del argumento que sólo son normas las que estipulan una sanción, se pasa
al argumento según el cual sólo son normas “genuinas” aquellas que
prescriben que los funcionarios apliquen ciertas sanciones en ciertas
circunstancias a aquellos que, por ejemplo, cometan homicidio. De este
modo, las normas tendrían el siguiente contenido: “si una persona realiza
tal o cual conducta, entonces Ud. como juez debe aplicar tal o cual san-
ción”. Toda norma contractual puede ser reformulada de esta manera
sosteniendo que “si dos personas expresan solemnemente su consen-
timiento, firman en frente a dos testigos, entonces Ud. Sr. Juez tiene la
obligación de hacerle pagar a quien figura como locatario la suma que
se estipula en el escrito”. Esto supone un cambio central respecto a la

EDUBP | Abogacía | Filosofía y Lógica Jurídica - pag. 42


concepción original, según la cual sólo eran derecho las órdenes respalda-
das por amenazas. Ahora, las órdenes están dirigidas a los funcionarios
para que se apliquen sanciones. Una forma menos extrema de esta ver-
sión dice que sólo algunas de estas normas no necesitan ser reconstruidas
como órdenes dirigidas a los individuos. Sin embargo, esta versión menos
extrema reconstruye las reglas potestativas (las que permiten hacer un con-
trato) como reglas que imponen o dan lugar a un deber y que poseen un
condicional: “si se dan las condiciones tal y tal y los individuos detentan tal y
tal propiedad, se da lugar al nacimiento del deber, es decir, dan lugar al naci-
miento de un deber impuesto a las partes”.

Esta nueva forma de reconstruir el derecho olvida algunos aspectos funda-


mentales de los sistemas jurídicos.

En primer lugar, surge bastante claro que el derecho, como técnica de


control social, no sólo está dirigido a los jueces o funcionarios, sino
también a los individuos. Se pretende que éstos últimos se rijan diariamente
por los mandatos jurídicos con independencia de que algún funcionario esté
dispuesto a golpearle la puerta o llamarlo a juicio. El derecho dirige un con-
junto de pretensiones generales a una clase de individuos y sólo en el caso
de que éstas fallen (sólo en el caso de que los individuos no jueguen al juego
del derecho respetando las reglas) será factible que intervenga un funcio-
nario. Así como es posible que juguemos a un juego sin árbitro, el derecho
pretende que sus deseos (lo que espera de los individuos) se realicen sin
la necesidad de un juez o policía que nos tire de la oreja cada vez que nos
pasamos de la línea.

Una doctrina como la que estamos analizando olvida este aspecto central
del derecho: que tiene pretensiones generales de control social y que, como
tal, es un juego donde los individuos suelen jugar sin árbitro y siguiendo las
normas generales. Si sólo reconstruyéramos las normas como dirigidas a
los funcionarios, perderíamos una dimensión importante del derecho como
herramienta de control social. La principal función del derecho no debe ser
vista dentro de los tribunales, sino en su capacidad para controlar, guiar y
planear la vida fuera de los tribunales. En palabras de Hart: “la protesta natu-
ral es que la uniformidad impuesta sobre las reglas por esta transformación
de ellas oculta los modos en que las mismas operan y las maneras en que los
jugadores las usan para guiar actividades orientadas por propósitos, y oscu-
rece así la función de esas reglas en la empresa social cooperativa, aunque
competitiva que es el juego...”.

Podríamos catalogar las teorías que hemos visto como reduccionistas, ya que
pretenden reducir el fenómeno del derecho, o bien, a la existencia de amena-
zas (sanciones), o bien, a órdenes dirigidas a un grupo menor de personas.

Una última forma de reduccionismo es aquella que analiza el fenómeno que


surge a partir de los contratos (en el que las partes modifican por propia
voluntad su situación jurídica) sólo desde el punto de vista de los deberes
que surgen para los individuos involucrados en esa transacción o acto jurí-
dico. Este enfoque centra su atención sólo en la forma en que los individuos
que ejercen sus potestades deben actuar. Es decir, el efectivo deber que
surge para las partes del ejercicio de sus potestades. Este análisis pierde una

EDUBP | Abogacía | Filosofía y Lógica Jurídica - pag. 43


dimensión importante de lo que significa “deber”. Lo importante de las reglas
potestativas es que no pueden ser analizadas bajo la noción estándar del
deber que tienen los individuos, sino más bien bajo el fenómeno normativo
del ciudadano como “legislador privado”. Cuando dos sujetos se “otorgan”
un contrato, están modificando su situación jurídica, creando derechos y –
en algún sentido– imponiendo obligaciones. Todo este complejo fenómeno
al que Hart alude como el fenómeno del “legislador privado” no puede ser
reducido simplemente al estudio o tratamiento sobre qué deberes tienen los
individuos a partir de un contrato. Lo importante en la reconstrucción de un
sistema jurídico es tener en cuenta que algunas reglas (como las que confie-
ren potestades) son utilizadas y actúan de manera muy diferente a como lo
hacen las normas que imponen deberes. Igual ocurre con la reconstrucción
del ejercicio de las potestades por parte de los jueces y los legisladores.
Estas actividades no son guiadas por el cumplimiento de un deber, sino más
bien guiadas por un propósito que difiere totalmente del cumplimiento de
deberes o de la sumisión al control coercitivo.

El ámbito de aplicación.
Ya hemos analizado cómo el conjunto de normas jurídicas a las que estamos
acostumbrados a enfrentarnos detenta una variedad de contenidos muy diversa.

Ello nos sirvió para ver que las normas no son simplemente órdenes res-
paldadas por amenazas, y también que no puede reducírselas a órdenes
dirigidas al funcionario. Resta aún analizar un aspecto central de las normas
de los sistemas modernos.

La teoría según la cual el derecho es un conjunto de órdenes respalda-


das por amenazas no puede dar cuenta de un aspecto central del dere-
cho: la idea de que el legislador queda, muchas veces, “atrapado” por
la propia norma que dicta. Mientras que resulta bastante extraño que un
ladrón quede bajo el alcance de su propia orden, no es para nada extraño
que el mismo legislador que legisló sobre el impuesto a las ganancias, deba
luego tributar en ese sentido.

Dos alternativas se presentan como disponibles para explicar este fenómeno


de la auto-obligación: i) recurrir a la idea artificial de la existencia de dos
legisladores en uno: el legislador como persona privada y el legislador como
funcionario público y ii) que, en algún sentido, el derecho o la emisión de
órdenes jurídicas se parece a la idea de promesa: la misma persona que
hace la promesa (emite la orden) queda vinculada por lo que dijo (o prome-
tió). Ambas ideas parecen ser más que artificiales y no nos servirán porque
el legislador, a veces, queda implicado por sus propias normas y a veces no.
Tampoco nos dará ninguna pista de qué es lo que hace que un órgano que
se considera supremo sea, a pesar de ello, limitado.

EDUBP | Abogacía | Filosofía y Lógica Jurídica - pag. 44


Modos de origen
Por último, habiendo analizado por qué el modelo simple de órdenes coerci-
tivas “se queda corto” para dar cuenta de la diversidad de las normas jurídi-
cas (contenido) y para explicar el fenómeno de la auto-obligación (ámbito de
aplicación), resta analizar algo respecto al origen de las normas (o lo que
los juristas estamos acostumbrados a llamar fuente). Hay una cierta inclina-
ción a sostener que las leyes, al igual que las órdenes coercitivas, tienen un
origen cierto, que emanan de una autoridad y están sometidas a un proce-
dimiento. Pero existe un tipo de norma que contradice estos requisitos: la
costumbre. Si hay un elemento que suele estar presente entre los juristas y
el derecho es la costumbre, pero ¿dónde se origina la costumbre?, ¿es dere-
cho la costumbre? Si es así, ¿por qué lo es?, y si no es así, ¿por qué tiene
tanto peso? Que la costumbre sea o no derecho, encubre dos discusiones.
En un sentido obvio, no toda costumbre es derecho. Esto demuestra que
sólo es derecho un subconjunto de las costumbres si es reconocida como
derecho. La otra discusión radica en qué significado tiene que una costum-
bre sea reconocida jurídicamente. ¿Significa que el soberano ha ordenado
que la costumbre sea obedecida, de modo que su status como derecho se
debe a algo que, en este aspecto se asemeja a un acto de legislación?

A veces significa que algo es derecho si una determinada autoridad suprema


o un dependiente de él lo han ordenado. Un dependiente de él puede orde-
nar con fuerza de ley una conducta si la autoridad suprema se lo ha orde-
nado o si, tácitamente, le ha encargado que regule esa conducta. Existe
una tendencia a pensar que la costumbre es una suerte de legislación
que es sancionada tácitamente por un órgano supremo que permite que
sus inferiores la tengan en cuenta con fuerza de ley. El órgano supremo
o soberano consentiría tácitamente la introducción de la costumbre
dentro del sistema jurídico, permitiendo que los tribunales inferiores la
tengan en cuenta.

Las objeciones a esta forma de justificar la juridización de la costumbre son


múltiples:
i) No es verdad que hasta que las costumbres no sean usadas en los
litigios, carecen de status jurídico (¿por qué razón las costumbres son
válidas a partir de que los jueces las usan y no pasa lo mismo con las
leyes?, ¿por qué se asume cuasi dogmáticamente que son leyes lo que
dice la legislatura y no la costumbre?). Una salida es sostener que nada
es ley mientras no haya sido “ordenado” y la costumbre no ha sido orde-
nada por un órgano establecido democráticamente. Sin embargo aquí es
importante comenzar a dividir entre características necesarias y contin-
gentes de los sistemas jurídicos. Es perfectamente posible pensar un
sistema jurídico en que la costumbre sea ley, aún antes de haber sido
reconocida por el Parlamento o utilizada por el juez en una sentencia.
ii) Por otra parte, no hay por qué asumir la idea de orden tácita, ya que
del hecho de que el subordinado esté haciendo cosas y el soberano
no interfiera, no se sigue que esas cosas que está dejando hacer,
sean ordenadas por el soberano u órgano supremo.

EDUBP | Abogacía | Filosofía y Lógica Jurídica - pag. 45


iii) La no interferencia de la legislatura o el soberano en determinadas
costumbres no puede considerarse, sin más, una orden tácita. La
razón por lo que la costumbre (algunas costumbres) son derecho (o
fuente de derecho) se comprenderá más adelante cuando analice-
mos las cuestiones referidas al sistema jurídico.

Un breve resumen de lo trabajado hasta aquí…


En nuestro intento de describir el derecho como órdenes coercitivas (respal-
dadas por amenazas) enfrenta el problema general que hay innumerables
normas que no tienen estas características y que, sin embargo, considera-
mos jurídicas. Además de lo anterior, es importante tener en cuenta:
i) en cuanto al ámbito de aplicación, las normas suelen imponer deberes
incluso a quienes las dictan. No son órdenes dadas a otros;
ii) muchas leyes no requieren que las personas hagan algo, sino que le
confieren potestades, es decir, que ofrecen facilidades para la libre crea-
ción de derechos subjetivos y deberes jurídicos dentro de la estructura
coercitiva del derecho;
iii) aunque sancionar una ley es, en ciertos aspectos, algo análogo a dar una
orden, algunas reglas de derecho se originan en la costumbre y no deben
su status jurídico a ningún acto consciente de creación de derecho;
iv) para evadir estos problemas se ha intentado sostener que la nulidad es
una especie de sanción; que las normas potestativas son fragmentos de
normas y que las normas que auto-obligan al legislador responden a una
doble “personalidad” del mismo o a través de la idea de promesa.

Asociemos la caracterización dada precedentemente, del derecho como


práctica social constituida por un conjunto conformado por diversos
tipos de reglas, con el siguiente ejemplo:

El novio de la nena, las tradiciones de la familia y las órdenes del abuelo.


Un sistema jurídico en miniatura.

Pedro era un chico como cualquiera. Tenía 19 abriles recién cumplidos. En


1998 se puso de novio con María. Se conocieron en el verano, casi de casua-
lidad. Los presentó una amiga en común. María quería que Pedro conozca
a su familia. La familia Reyna. La primera cena con la familia de María se
adivinó difícil, crucial, tensa. Pedro sabía que en esa instancia la frase “cual-
quier cosa que digas puede ser usada en su contra” estaría más vigente que
nunca. Aceptar un vaso de vino de más significará ser un borracho; recha-
zar educadamente una segunda porción de carne podía leerse como “no
le gustó la comida”; los nervios al contestar preguntas podían interpretarse
como inseguridad, timidez, reticencia a hablar, falta de educación, oligofre-
nia, síntoma de drogadicción, tartamudez o dislexia.

Pedro conocía perfectamente estas variables. Trató de rechazar varias veces


la invitación pero, finalmente, tuvo que acceder. Sabía que seguir rechazando
la invitación también podía contar en su contra. Siempre era posible que el
rechazo a la cena sea leído como “por algo no querrá venir, este chico quiere
ocultar algo”. El viernes siguiente por la noche sería la cena.

EDUBP | Abogacía | Filosofía y Lógica Jurídica - pag. 46


Lo esperaba la familia en pleno; la comida: carne al horno; los comensales:
seis (madre y padre, María, su hermano y los abuelos paternos de María);
estrategia de Pedro: hablar lo menos posible, dejarse llevar por María y
rogar que todo termine lo antes posible. El viernes fatal, Pedro llegó a la
casa y como acto inaugural se dirigió a la mesa y se sentó cómodamente
en la cabecera, pero inmediatamente sintió que algo andaba mal. Todavía
nadie se había sentado y María le susurró al oído: es el asiento de mi abuelo,
levántate de ahí que te van a odiar. Pedro se levantó rápidamente mientras el
abuelo y el padre de María le clavaban los ojos, fruncían el ceño y pensaban
“empezaste mal querido”. María le explicaba a Pedro (mientras le señalaba
dónde debía sentarse) que por regla la familia reservaba ese lugar al abuelo.
Esa es una imposición que el viejo Reyna (el tatarabuelo de María, fallecido
hacía unos 50 años) impuso en la familia, y desde esa época el mandato se
respetaba, sosteniendo y aceptando aquella voluntad.

Pasado el mal trago, todos se encontraron sentados. La madre de María


trajo la comida de la cocina, sirvió los platos a cada uno y Pedro, ni lerdo ni
perezoso arremetió con el primer bocado. Automáticamente, experimentó
dos experiencias sensoriales. Una visual: toda la familia volvió a mirarlo poco
amablemente. Otra táctil: María le tiró una patada por debajo de la mesa que
le dio a Pedro en pleno tobillo. En ese instante, la madre con voz ceremo-
niosa adoctrinó a Pedro: “mirá querido, nosotros aquí, antes de comer, ben-
decimos la comida. Para ello primero rezamos en silencio, luego el abuelo
Reyna dice unas palabras y recién allí empezamos a comer”. Pedro, entre
tanto, se debatía en el “trilema” de tragar el pedazo de carne sin masticar,
refregarse el tobillo que le acaba de golpear María o abandonar toda la difícil
tarea y mandarse a mudar. No obstante, respira hondo, traga el bocado y
pretendiendo descomprimir la situación dice “disculpe señora, no sabía que
aquí tenían por hábito bendecir la comida”. La madre no deja pasar la opor-
tunidad y sostiene: “no sólo es un hábito querido, todos consideramos que
es necesario y correcto bendecir la comida y vemos con muy malos ojos el
olvidar agradecer por comer todos los días”.

Felizmente, unos minutos luego, la situación comenzó a “distenderse” y


la familia atropelló con su interrogatorio estilo KGB-CIAINTERPOL contra
Pedro: “...qué hacés, qué estudiás, dónde trabajás, cuántos hermanos tenés,
etc., etc....”, luego de lo cual la cena tomó un tono más amable y Pedro logró
retirarse de la velada en situación de “aprobado”.

El noviazgo continuó su curso normal; Pedro continuó yendo regularmente a


la casa de los Reyna y, poco a poco, comenzó a conocer mejor otra serie de
prácticas propias de la familia. Por ejemplo, era el abuelo Reyna (algo despó-
tico, para qué negarlo) el que decidía qué se comía los domingos. Algunos
días, se le antojaba pasta, entonces se comía pasta; otros asado y se comía
asado. Pedro no dejaba de asombrarse del excesivo poder e importancia
que detentaba en la familia de María su abuelo.

Transcurridos un par de años, la familia Reyna sufrió una desgracia natural


de la vida. El abuelo falleció de viejito, no obstante lo cual, sucedió un efecto
curioso. La familia siguió manteniendo las mismas prácticas. Seguía siendo
obligatorio bendecir la mesa, reservarle un lugar en la cabecera al más viejo
de la familia, y permitirle a éste que decidiera qué comer los domingos (entre

EDUBP | Abogacía | Filosofía y Lógica Jurídica - pag. 47


otras cosas). Otras prácticas, sin embargo, se abandonaron. Por ejemplo,
era bastante usual, mientras el abuelo vivía, que la familia coincidiera en el
living de la casa los sábados a la tarde, frente al televisor y se dedicara a
disfrutar de un video que alguien se encargaba de alquilar el viernes en el
videoclub. Esta había sido una práctica que era generalmente propulsada
por el abuelo, pero nadie veía con malos ojos el faltar a la cita del “video” de
los sábados, o retirarse antes de que finalice la película.

Cuando el abuelo Reyna fallece, entonces, sucede el siguiente fenómeno:


un grupo de prácticas persiste (incluso la impuesta por el tatarabuelo), otras
se delegan o traspasan al mayor de los Reyna y otras simplemente se aban-
donan. Pedro sensatamente, ante la persistencia de determinadas prácticas,
no duda en preguntarle a María: ¿por qué se sigue reservando la cabecera
de la mesa al mayor, siendo que esa imposición fue establecida por tu tatara-
buelo?, ¿cuál es el efecto mágico por el cual ahora tu papá tiene los mismos
derechos que tenía tu abuelo a decidir la comida de los domingos?, ¿por
qué no se hace con estas prácticas lo mismo que se hizo con el hábito de
juntarse a ver video los sábados?, es decir, ¿por qué se abandona el video
y no que tu padre decida qué comer, o quién tiene derecho a sentarse en la
cabecera?

Deficiencias de la noción de hábito de obediencia en la explicación de la


continuidad del poder soberano y de la persistencia del derecho a través
de una sucesión de diferentes legisladores soberanos.

La situación que acabamos de describir reproduce en un modelo-miniatura,


algunas de las perplejidades que nos presenta el derecho o los sistemas
jurídicos.

En primer lugar, si persistiésemos con nuestra idea de reconstruir el sistema


jurídico como un conjunto de órdenes respaldadas por amenazas o simple-
mente mandatos otorgados por un soberano apoyados sobre la amenaza de
sanción, no podríamos explicar dos aspectos centrales del derecho: su persis-
tencia a través de los cambios de legislador soberano y la continuidad del
poder soberano en una sucesión de diferentes legisladores soberanos.

Si la existencia del derecho depende de que alguien me esté amenazando


o de que quien emite la norma aún continúe vivo, ¿por qué razón una vez
que la persona (o conjunto de personas) que emite las órdenes respaldadas
por amenazas ha fallecido, continúo obligado jurídicamente por él? Como
en el caso de la familia Reyna, si ciertos comportamientos obligatorios se
explican porque “lo dijo el abuelo”, dependen de que el abuelo esté presente
para recordarme las órdenes. Entonces, ¿qué mágico proceso hace que la
obligación subsista o persista una vez que el abuelo ha fallecido? En térmi-
nos jurídicos, ¿cómo se explica que yo esté hoy obligado por las normas
del código penal sancionado en 1921, o por ciertas normas constitucionales
sancionadas en 1853? Es posible intentar explicar que la familia Reyna obe-
dece al abuelo o que los súbditos obedecen a los legisladores porque tienen
el hábito de hacerlo. Pero, si bien esta reconstrucción tiene cierta plau-
sibilidad, no es capaz de explicar el problema de la continuidad (suce-
sión) y persistencia de los sistemas jurídicos. Aunque puede ser posible
que en algún sentido los Reyna obedezcan al abuelo (soberano) porque

EDUBP | Abogacía | Filosofía y Lógica Jurídica - pag. 48


están habituados a ello, hay que reconocer que apenas muere el abuelo, y
lo sucede su hijo, no existe ningún hábito de obediencia a su sucesor. Para
que un hábito se constituya como tal, requiere el paso de algún tiempo. No
obstante la inexistencia del hábito, tanto en la familia Reyna como en las
sociedades modernas, no existe una interrupción en la obediencia una vez
que la persona que dictó la norma ya no existe o ha finalizado su mandato.
Un fenómeno de la reconstrucción basada en que se obedece al soberano
por hábito, no puede captar el hecho de que los ciudadanos obedecen las
normas dictadas por los legisladores o soberanos aún cuando éstos hayan
sido designados o nombrados en sus funciones el día anterior al dictado de
su primera norma.

Lo que se pretende mostrar es que la idea de hábito de obediencia, no sirve


para explicar por qué la familia Reyna, el día después del fallecimiento del
abuelo, le permite al padre de María decidir qué se va a comer el domingo, ni
para explicar por qué un conjunto de ciudadanos se siente y está obligado a
responder a un legislador o soberano que acaba de asumir (continuidad del
derecho), ni tampoco por qué se siguen obedeciendo las normas que dictó
el abuelo fallecido o las que dictó el constituyente de 1853 (persistencia del
derecho).

Si reconstruyéramos el sistema jurídico bajo la idea del hábito de obediencia,


habría que asumir que una vez que ha muerto la persona a quien nos había-
mos acostumbrado (habituado) a obedecer, desaparece toda obligación de
continuar obedeciendo la normativa que éste dictó, lo cual claramente no
ocurre, ya que seguimos estando obligados por lo que ha dicho una persona
aún después de haber fallecido o dejado el poder. Sumado a ello, tampoco
podríamos explicar el hecho de que hay gente que sucede en la función de
dar órdenes y dictar leyes a la persona que ya no está más en el cargo y
que las normas que dicta son obedecidas inmediatamente asume el cargo
(sin que medie tiempo alguno que permita hablar de que es obedecido por
hábito). El caso de la familia Reyna muestra también en forma bastante clara
que la existencia de normas dentro de ese ámbito no depende solamente
de que el abuelo o el tatarabuelo hayan dicho que algo era debido, o hayan
amenazado con desheredar a algún miembro de la familia sino cumplía con
su voluntad, sino más bien que los integrantes de la familia acepten que lo
que dice el abuelo detenta algún grado de corrección. La familia no sola-
mente sigue como animalitos lo que el abuelo dice, sino que se involucra
activamente en las pautas de conductas de forma bien variada y sofisti-
cada. Por un lado, reconoce que el abuelo tiene derecho a elegir la comida
los domingos y no, por ejemplo, a decidir qué deben leer los integrantes
de la familia. Además, una vez establecida esa pauta de conducta, los inte-
grantes de la familia ven a cualquier conducta que se aparte de ella como
claramente incorrecta y como un buen motivo para criticar a quien no se
comporta del modo establecido (vgr., es el hombre mayor el que tiene dere-
cho a sentarse en la cabecera de la mesa), ejerciendo presión contra el des-
viado (recordemos el puntapié de María y las miradas asesinas del resto de la
familia cuando Pedro comenzó a comer antes de la bendición de la comida).

EDUBP | Abogacía | Filosofía y Lógica Jurídica - pag. 49


Análogamente, en los sistemas jurídicos, los individuos desarrollan frente a
lo que dice el/ los legislador/es una actitud de aceptación y reconocen en su
autoridad el derecho a decidir determinadas cosas con fuerza obligatoria y
valor jurídico. Ello explica por qué, aún cuando el que dictó la norma ya no
existe, su decisión persiste gravitando sobre la conducta de los individuos
(cientos de años después). Y, a su vez, permite comprender el mágico fenó-
meno de la sucesión de legisladores y funcionarios. No es por la sangre que
detenta el sucesor, o por el hábito de obediencia al soberano fallecido que
continúa o persiste el derecho, sino porque hemos aceptado por adelan-
tado (antes de que asuma el cargo) otorgarle el derecho al nuevo legis-
lador de decir qué está prohibido y permitido y seguir obedeciendo lo que
dijo el fallecido mientras el nuevo no diga otra cosa y estamos dispuestos a
tomar sus órdenes como pautas de conducta, criticar a quien se desvíe
y a ejercer una regular presión social para que esas pautas se respeten.

Pero todavía quedaba en el tintero una pregunta central de Pedro. Si es cierto


que la continuidad y sucesión de las prácticas que existían cuando el abuelo
vivía dependen, en gran parte, de la actitud que desarrollan quienes aceptan
esas pautas, ¿qué explica que la práctica de ver video haya desaparecido?
El ejemplo de la familia Reyna ilustra también la diferencia que existía entre,
por ejemplo, la práctica de dejar la cabecera de la mesa al abuelo, o bendecir
la comida y la de ver video los sábados. Mientras que la desviación de las
dos primeras pautas generaba casi un escándalo familiar, la decisión de no
concurrir al sofá a ver la película no generaba ningún estrépito. Este ejemplo
pretende mostrar una diferencia central:

la diferencia entre hábito de conducta y regla social. Esquemáticamente


puede decirse que en los hábitos (como el de ver video o ir al cine todos
los domingos) los individuos no desarrollan ninguna actitud interna espe-
cial con respecto a la práctica y no están dispuestos a reprochar a otra
persona si no comparte el mismo curso de acción, o no gusta de hacer
lo mismo que uno hace habitualmente. Es decir, en los simples hábitos,
los individuos no ven un curso de acción que debería ser observado por
el resto, ni ven en la ausencia o abandono del hábito una buena razón
para criticar o presionar al resto. Así, que un amigo nuestro no tenga el
hábito de ir al cine, no genera en nosotros (generalmente) la idea de que su
actuar es incorrecto, ni estamos dispuestos a criticarlo por su propensión
a ver fútbol cuando nosotros vamos al cine. Por otra parte, no vemos en la
actitud del resto de los mortales de ir al cine, una buena razón para que noso-
tros vayamos al cine. Cuando convergemos en la entrada del cine con un
montón de gente, ello no es debido a que pensamos, necesariamente, que
el compartir con ellos ese gusto tiene valor alguno. No vemos en ello una
práctica intrínsecamente valiosa que debería ser compartida por todos,
ni vemos en la conducta de los otros (necesariamente) una buena razón
para elegir ir al cine en lugar de ir a la cancha. Con las reglas, en cambio,
la cuestión es diferente: la familia Reyna estaba dispuesta a reprochar a
Pedro por comenzar a comer desesperado porque aceptaba la regla de
la bendición de la comida como una pauta de conducta correcta y era
capaz de desarrollar una cierta presión sobre el comensal. Por último,
el ejemplo de la familia Reyna pretendía rescatar ciertas diferencias entre las
reglas que podemos encontrar en un sistema jurídico.

EDUBP | Abogacía | Filosofía y Lógica Jurídica - pag. 50


Como hemos visto, el abuelo Reyna no podía decidir cualquier cosa. La fami-
lia tenía una serie de criterios más o menos sofisticados para reconocerle
valor (validez) a la decisión del mayor de la familia. Así por ejemplo, el abuelo
podía decidir qué debía comerse los domingos, pero no estaba habilitado
para decidir –afortunadamente– qué debía vestir su nieta María o qué debía
leer su hijo mayor. La familia contaba con un criterio para desechar algu-
nas decisiones del abuelo y tomar a otras como válidas y vinculantes. Por
ejemplo, una decisión de comer asado el domingo era válida y, por lo tanto,
era obligatorio comprar esa comida. Asimismo el abuelo podía ir cambiando
sus decisiones de domingo a domingo (decidiendo locro para los días de
invierno y asadito para los días en que hace calor). No obstante, nadie iba a
tomar como sensata (ni obligatoria) una decisión sobre qué ropa usar para
el cumpleaños de quince de la nena. De este modo, aunque el abuelo era
supremo en sus decisiones, no por ello era ilimitado.

En forma semejante, los sistemas jurídicos modernos poseen formas más


o menos simples de identificar cuál es el derecho existente y, en ese sen-
tido, qué es lo que se debe hacer y también qué cosas están excluidas
de la decisión del soberano. Así, sabemos que la legislatura puede decidir
cuánto debemos pagar en impuestos (el abuelo podía decidir qué se debía
comer los domingos) pero no puede decidir qué debemos pensar ni qué
ropa debemos usar en las fiestas.

Se pretende hacer un paralelismo entre el ejemplo de la familia Reyna y un


sistema constitucional moderno en que el abuelo (legislatura) no puede deci-
dir sobre todas las acciones de los hombres. Así, del mismo modo que nadie
haría caso al abuelo si tratase de imponer qué ropa hay que usar, poseemos
criterios para decidir cuándo la legislatura se ha pasado de la raya y ha deci-
dido sobre un tema vedado a su potestad. Un caso jurídico claro lo consti-
tuye el artículo 19 de la Constitución Nacional.

Por supuesto que en este contexto es posible encontrar un soberano que


tenga tanto poder de hecho que nadie pueda IimitarIo. No obstante eIIo, aquí
nos estamos preocupando más bien por los Iímites jurídicos a un soberano
y no por los Iímites empíricos o físicos. En una sociedad altamente tecnifi-
cada como la actuaI siempre es posible que una persona tenga un poder de
hecho en virtud del cual pueda disponer de Ia vida del resto de Ias personas
sin límite alguno.

A modo de resumen y para concluir este desarrollo: el estado moderno


como una gran familia.
Hasta aquí podemos advertir lo siguiente:
1) los sistemas jurídicos detentan normas y autoridades jurídicas que se
resisten a ser reconstruidas bajo la simple idea de “estamos habituados a
obedecer a este grupo de gente”;
2) los individuos (como los integrantes de la familia Reyna) desarrollan deter-
minadas actitudes internas hacia las reglas dictadas por el soberano, que
no desarrollan hacia otro tipo de conductas convergentes o hábitos;

EDUBP | Abogacía | Filosofía y Lógica Jurídica - pag. 51


3) aunque es posible reconocer que el soberano, o cuerpo legislativo (el
abuelo) es, en algún sentido, un órgano o persona suprema, ello no
implica que no existan herramientas (normas constitucionales) para limi-
tarlo. Del hecho de que una autoridad sea suprema no se sigue que sea
ilimitada jurídicamente;
4) contamos no sólo con reglas que dicen qué conductas están prohibidas
o permitidas (v. gr. está prohibido sentarse en la cabecera de la mesa o
está prohibido matar), sino también con reglas que prescriben qué proce-
dimiento hay que seguir para dictar una norma, quién es la persona habi-
litada para hacerlo y bajo qué condiciones vamos a tomar a esas órdenes
como válidas (v. gr. el abuelo puede decidir qué comer los días domingos,
o la legislatura puede decidir qué conductas son obligatorias o están pro-
hibidas, siempre que no interfiera en el ámbito privado de los hombres).

Hemos analizado en detalle el modelo explicativo del derecho como órdenes


coercitivas generales correspondiente a la versión de John Austin, represen-
tativa de lo que se denomina positivismo clásico. Austin construyó su teoría
del derecho partiendo de la idea básica de hábito de obediencia. En térmi-
nos de esa noción básica define al soberano como supremo, por ser aquel
individuo o cuerpo de individuos cuyas órdenes son habitualmente obede-
cidas, e independiente, porque no obedece habitualmente las órdenes de
nadie. El concepto de soberano, definido en términos de hábito de obedien-
cia, determina una sociedad política independiente y su derecho. El resul-
tado es una teoría reduccionista, pues identifica el derecho con la relación
de dominación característica de todo Estado que detenta el monopolio de la
fuerza coercitiva. En efecto, el derecho está conformado por el conjunto de
órdenes coercitivas generales dadas por el soberano –que detenta el mono-
polio de poder coercitivo- a sus súbditos. Hemos caracterizado también las
semejanzas y diferencias que surgen, dentro de este modelo, entre derecho
y orden de un asaltante.

Hart realiza un análisis crítico de este modelo explicativo del derecho, en


primer lugar por la analogía sostenida entre regla jurídica y orden coercitiva
general. En tal sentido, Hart afirma que muchas de las reglas que forman
parte del derecho positivo difieren de las órdenes coercitivas generales por
su contenido, por su ámbito de aplicación, o por su origen. Hay reglas que
difieren de las órdenes por su contenido, pues no prescriben conductas, sino
que otorgan poderes o habilitaciones. Las reglas potestativas son relevan-
tes pues dan lugar a la caracterización del soberano como autoridad nor-
mativa, a diferencia de la definición de Austin dada en términos de la noción
de hábito de obediencia que no es normativa pues alude a fenómenos
empíricos de simple recurrencia o convergencia efectiva de conductas. Por
supuesto que pueden reproducirse argumentos a favor de la caracterización
de las reglas como órdenes generales coercitivas y, en tal sentido Hart cita
argumentos posibles a favor de una caracterización uniforme de las reglas
en esos términos, tales como el argumento de la nulidad como sanción, el
argumento de las normas potestativas como fragmentos de normas genui-
nas y, en el caso de las costumbres, que son normas que difieren por su
origen de aquellas que surgen de la expresión deliberada de voluntad de un
soberano, también podría asimilárselas a órdenes tácitas del soberano. Hart
proporciona sólidas razones en contra de tales argumentos de uniformidad.

EDUBP | Abogacía | Filosofía y Lógica Jurídica - pag. 52


En segundo lugar, aludimos a las críticas que Hart dirige al análisis de J.
Austin en términos de la noción de hábito de obediencia. Este autor, como
vimos en la segunda parte de los contenidos de este módulo, considera ina-
propiada dicha noción para dar una caracterización satisfactoria del derecho
por tres razones: a. no es adecuada para explicar la continuidad del poder
soberano en una sucesión de legisladores soberanos diferentes. En tal sen-
tido resulta más apropiada una explicación del derecho dada en términos de
reglas –en especial de una regla de sucesión. Ello es así porque el derecho
es una práctica de seguimiento de reglas y no el mero hecho de obedien-
cia habitual a órdenes provenientes de quienes detentan el monopolio de la
fuerza coercitiva. Además, tampoco explica adecuadamente la persistencia
de las normas o reglas a través de los cambios de legisladores soberanos.
En efecto, periódicamente cambian los miembros de un Parlamento, por
ejemplo, y sin embargo, las normas que integran el derecho positivo persis-
ten, siguen siendo válidas a pesar de haber cesado la voluntad real de quien
las dio –un soberano anterior. Por último, la noción de hábito de obediencia
tampoco da una caracterización adecuada de las características de supre-
macía e independencia del poder soberano. Cierto es que un soberano no
puede tener deber jurídico de obedecer a un superior, pues de tenerlo no
sería soberano, pero ello no implica que no pueda tener límites jurídicos a
su potestad soberana. En cambio, si la soberanía es explicada en térmi-
nos de normas o reglas que confieren tal poder, esto es, en términos de
poder normativo, entonces puede comprenderse por qué las normas que
confieren tal poder pueden establecer límites a ese poder bajo la forma
de incompetencias o ausencia de poder. Dicho de otro modo, la norma
–constitucional– que confiere poder soberano, puede limitar ese poder de un
modo formal estableciendo un procedimiento especial para dar las órdenes
generales – leyes – y, puede limitarlo de un modo material excluyendo de su
ejercicio ciertas materias, por ejemplo, establecer que el soberano no puede
sancionar normas que violen ciertos principios constitucionales –tal el caso
de las denominadas cláusulas pétreas que están fuera del alcance del poder
de enmienda que poseen legisladores soberanos.

Para completar el estudio de los temas del Módulo 2, es necesario que


acceda a la siguiente bibliografía: HART, Herbert: El concepto de derecho.
Buenos Aires, Abeledo Perrot, 1992. Capítulos III y IV.

Si está interesado en profundizar sobre la temática del módulo, puede consultar:


NINO, Carlos Santiago: Introducción al Análisis del Derecho. Buenos Aires,
Astrea, segunda edición ampliada y revisada, 1987. Capítulo I.
Isler Soto, Carlos. (2010). ¿ES EL DERECHO UN SISTEMA DE MANDATOS?
LA CRÍTICA DE HART A LA TEORÍA IMPERATIVISTA DEL DERECHO. Revista
de derecho (Coquimbo), 17(1), 147-162. https://bit.ly/301zAXn

Lo invito a realizar las actividades del módulo.

EDUBP | Abogacía | Filosofía y Lógica Jurídica - pag. 53


M2 Actividades
Actividad 1
Un artículo particular
Usted, como futuro jurista preocupado por su formación, se ha suscripto a un
matutino local que se dedica exclusivamente a difundir las “noticias” jurídicas
del día. En el mismo, se incluyen los fallos más relevantes, las declaracio-
nes más estridentes de jueces y abogados. Existe también una sección de
Espectáculos (los “espectáculos” en este matutino tienen por protagonistas
a algunos funcionarios judiciales cuyo desempeño se parece más a “paro-
dias” y sus fallos a “tragedias” que cualquier obra de teatro o película de cine
real). Existe una sección de “chimentos” y, en la última página, una parte
dedicada exclusivamente a chistes, crucigramas y acertijos jurídicos. En la
edición del martes último aparece un entretenimiento llamado Encuentre las
diferencias. Allí puede leerse: “Estimado lector, como todos los martes le
hacemos llegar el desafío de encontrar las diferencias en el texto que se
transcribe a continuación.

Para la resolución de la actividad consulte los contenidos del módulo


2 sobre el tema: el derecho y su relación /diferencia con las órdenes
respaldadas por amenazas y también, el capítulo 2 de El concepto de
derecho de H. L. A. Hart.

AA

Encuentre las diferencias:


En la ciudad de Rosario, a las 10 de la mañana de un día miércoles ocurren
simultáneamente en dos lugares distintos dos hechos muy similares. Ambos
lugares son recintos cerrados, donde la gente va a depositar su dinero, paga
facturas y los clientes son atendidos por empleados sonrientes, empleadas
de trajecito azul y, gerentes siempre dispuestos, los escuchan largas horas sin
darles una solución. A ambos lugares entra una persona escoltada por cuatro
más. En el primer caso, la persona lleva una media en la cabeza, empuña un
arma, se acerca al cajero, lo golpea en la cabeza, se para sobre el mostra-
dor y en forma general grita: “Escúchenme todos, esta es una orden que
todos deben cumplir de inmediato: apaguen los celulares, tírense al suelo
y le entregan todo su efectivo a mis compañeros. Deben permanecer en el
suelo hasta el horario de cierre del local. Más vale que se vayan haciendo a
la idea, no queremos héroes ni vivillos. Es mejor habituarse a que quienes
ordenan acá somos nosotros y quienes obedecen acá son Ustedes”. Un jubi-
lado que se encontraba en el fondo del local y cuya visión y oído no funcionan
del todo bien no escucha ni ve lo ocurrido y mientras todo sucede se dirige
tranquilamente hacia la puerta, momento en que es tomado del brazo por uno
de los asaltantes al momento que le dice: “Oiga abuelo, no importa que Ud.
no escuche ni vea bien, también está obligado a darnos la platita. Desde el
momento mismo que entramos al banco Ud. está bajo nuestras órdenes”.

EDUBP | Abogacía | Filosofía y Lógica Jurídica - pag. 54


En el segundo lugar, también entra una persona comandando a cuatro más.
Manda a cerrar el banco, se dirige al cajero y le espeta: “Clausure las opera-
ciones ya mismo, entrégueme todos los resúmenes de su caja del día de la
fecha y comuníqueme con el gerente”. Acto seguido levanta la voz para que
todos lo escuchen y hablándoles a los clientes presentes dice: “Estimados
clientes, soy el inspector Juan Sabueso y estamos realizando un procedi-
miento de chequeo de las operaciones en esta institución, debido a que exis-
ten innumerables sospechas de transacciones en negro. Por favor, tengan la
amabilidad de esperar sentados en la fila de asientos de la derecha, apaguen
sus celulares y tengan a disposición de mis acompañantes los comprobantes
de las operaciones que acaban de realizar....es importante que retengan que
tengo la potestad de detener o levantar acta de infracción en contra de quien
resista al procedimiento, y que deben hacer todas y cada una de las cosas
que se le solicitan. Una vez terminado el procedimiento es posible que se los
invite a comparecer durante la semana por ante la oficina estatal respectiva.
Muchas gracias”.

AA Actividad 1
asistente académico

Para la realización de la presente actividad usted cuenta con la siguiente


ayuda. Las palabras o diferencias claves son las siguientes: generalidad,
conocimiento de la orden, perduración, hábito de obediencia.

Para orientarse en la búsqueda de las diferencias preste atención a las pala-


bras u oraciones resaltadas en negritas. Para identificar las diferencias con
mayor facilidad, puede repasar los conceptos de normas jurídicas, mandatos
y órdenes. Asimismo son centrales las nociones de variedades de imperati-
vos y las del derecho como órdenes coercitivas.

Actividad 2
Revisando la propia historia
En este último tiempo, usted se ha sentido anímica y sentimentalmente
extraño. No se encuentra a gusto con algunos aspectos de su vida y siente la
necesidad de revisar algunas cuestiones que tienen que ver, particularmente,
con su historia personal y familiar. ¿Quién no ha pasado por esto alguna vez?
Por ello, decide asistir a un profesional en busca de apoyo psicológico.

En la primera entrevista, su analista le pide que rememore su vida familiar y


trate de resumir cuáles eran las prácticas usuales de su familia. En especial,
su analista está interesado en que Ud. le cuente qué prácticas familiares exis-
tían y que no se observan actualmente (o que Ud. ya no respeta u observa),
qué prácticas aún persisten, quién era el principal encargado de poner orden
en la casa, entre otras. Como Ud. está algo nervioso y quiere reflexionar
un poco sobre el tema, le propone a su analista escribir una especie de
memo en donde resumirá algunas de estas cuestiones. El analista acepta y
le sugiere que, en una o dos carillas, le describa:

EDUBP | Abogacía | Filosofía y Lógica Jurídica - pag. 55


i) Algún ejemplo de un hábito de su familia;
ii) algún ejemplo de regla familiar;
iii) qué criterios tiene Ud. para distinguir qué cosa era un hábito familiar de
una regla;
iv) quién o quiénes dictaban en su familia las reglas o imponían obligaciones;
v) si dichas órdenes eran ilimitadas o sólo eran circunscriptas a algunos
aspectos (con quién salir, qué estudiar, qué deporte no hacer, etc.);
vi) qué reglas perduran en su familia desde antaño y cuáles ya se han aban-
donado (aquí puede incluir algún relato familiar sobre cómo se solía coci-
nar o qué se solía usar que ya no se usa, etc.).

AA

AA Actividad 2
asistente académico

Para la realización de la presente actividad no es necesario que Ud. incluya


ejemplos familiares “reales”. Basta con que rememore algún ejemplo
conocido de alguien cercano sobre las diferentes prácticas y costumbres de
una familia y explique cómo haría para diferenciar las prácticas más impor-
tantes de las más irrelevantes, quién era el facultado para dictar las órdenes,
etc. Al momento de resolver la presente actividad, identifique claramente las
diferencias entre los siguientes conceptos:
• hábito y regla,
• órgano supremo y órgano ilimitado,
• persistencia y sucesión de reglas.

Para responder a esta actividad, puede resultar inspirador ver el siguiente


video:

Porque obedecemos la ley


https://www.youtube.com/watch?v=S1frSMlzlyA

EDUBP | Abogacía | Filosofía y Lógica Jurídica - pag. 56


MÓDULO 3

M3 Microobjetivos

• Comprender los aspectos centrales de las obligaciones o reglas sociales


(presión social, crítica al desvío, etc.) a los fines de poder diferenciar los
meros hábitos, de las reglas de una comunidad.
• Distinguir los aspectos internos y externos de las reglas sociales y jurídi-
cas, como clave para comprender la diferencia entre observador externo
y participante de una práctica social.
• Diferenciar las complicaciones que se suscitan en los ámbitos sociales
pequeños con alta cohesión social, de los problemas suscitados en ámbi-
tos de alta complejidad social (como los modernos), con el objeto de
comprender por qué es central a nuestras vidas la existencia del derecho.
• Identificar las razones por las que un conjunto simple de reglas primarias
resulta insuficiente y defectuoso como medio de control social, con el
propósito de comprender la importancia de la institucionalización del sis-
tema jurídico en las sociedades actuales con sus complejas estructuras.

M3 Contenidos

Lo invito a ver el video de presentación del Módulo 3

Una concepción alternativa. El derecho como práctica SOCIAL de un


conjunto de individuos.

Reglas primarias y secundarias. Las claves de nuestras intuiciones jurídicas.

Hemos visto hasta ahora cómo la reconstrucción de un sistema jurídico se


resistía a determinados reduccionismos: al de la orden respaldada por ame-
nazas, a la idea de soberano ilimitado, al hábito de obediencia, al de simples
reglas que imponen deberes y obligaciones, etc.

En ese sentido, caímos en la cuenta que no todas las normas son del tipo
de las respaldadas por amenazas y que inclusive las que prevén algo pare-
cido a una amenaza (sanción), no pueden ser reconstruidas como simples
amenazas; advertíamos también que el derecho es un “juego” en el que
los participantes, a veces, se autoaplican las normas que dictan (juegan sin
árbitro); donde existe un tipo importante de normas que sirven para cambiar
la situación jurídica de los individuos y que en una gran cantidad de casos
(matrimonio, testamento, contrato) las personas actuaban como legisladores
de su propia vida (legisladores privados).

En este contexto, vimos cómo la concepción según la cual el sistema jurídico


podía reducirse a un conjunto de órdenes respaldadas por amenazas dic-
tadas por un soberano habitualmente obedecido, no podía captar nuestras
intuiciones acerca de la persistencia y continuidad de un sistema jurídico y
tampoco era capaz de dar cuenta del aspecto interno o actitud interna que

EDUBP | Abogacía | Filosofía y Lógica Jurídica - pag. 57


desarrollan las personas hacia sus propias instituciones y autoridades. Vimos
cómo la familia Reyna mostraba, frente a las órdenes del abuelo, diferentes
actitudes de crítica, aceptación o indiferencia (consulte los contenidos del
módulo 2 a fin de revisar estos tópicos nuevamente y también, el capítulo 4
del texto de Hart).

A su vez, establecimos una distinción entre autoridad suprema y autoridad ili-


mitada, la cual nos permitía notar que del hecho que una persona sea la com-
petente para decirnos qué hacer, no se sigue que sea completamente libre
o que pueda actuar como se le ocurra (consulte contenidos del módulo 2 y
capítulo 4 de Hart última parte: Limitaciones jurídicas a la potestad legislativa).

Luego de este recorrido, llegamos a un punto clave en nuestra descripción


de un sistema jurídico. La complejidad del objeto que estamos analizando
arrojaba un primer resultado provechoso: podíamos distinguir, al menos,
entre dos tipos de reglas. Aquellas que indicaban a los individuos qué hacer
y qué omitir, lo quieran o no (imponen deberes); llamaremos a este tipo de
reglas, primarias. Y aquellas que establecen que los individuos, haciendo o
diciendo algo (levantando la mano en un recinto llamado Congreso, firmando
al pie de un papel que dice “Contrato de compraventa”, prometiendo frente a
una persona llamada “funcionario del Registro Civil”) pueden crear, extinguir
o modificar las reglas anteriores (otorgan potestades públicas o privadas);
llamaremos a estas reglas, secundarias en razón de que en algún sentido
se refieren a (o versan sobre) las reglas primarias. Y, mientras que las prima-
rias generalmente se refieren a acciones que implican movimientos o
cambios físicos, las del segundo tipo se refieren a actos que implican la
creación o modificación de deberes u obligaciones.

Los ejemplos de ambos tipos de reglas son de muy fácil identificación, veá-
moslos juntos a continuación:

Las reglas del derecho penal, sobre todo de la parte especial, imponen a
los individuos que hagan u omitan algo (no mate, no robe, ayude, sea cui-
dadoso, etc.) y son por ello un caso claro de reglas primarias. Indican una
acción o cambio físico en el mundo que debe llevarse adelante o evitarse.
Las reglas secundarias, por su parte, pululan en los códigos procesales (nos
dicen cómo demandar, cómo pedir prueba, cuáles son los pasos para dictar
una sentencia válida, etc.), en los códigos civiles (cómo se contrata, cómo se
adopta a un menor, etc.), y en la misma constitución (le indican al legislador
cuáles son los pasos para dictar una ley, enmendar una anterior, etc.). Es
decir, en un sentido, las reglas secundarias se ocupan de las reglas primarias
indicándonos cómo crearlas, modificarlas o extinguirlas.

Comprendida esta distinción entre reglas primarias y secundarias, estamos en


condiciones de hacer frente a las visiones reduccionistas respecto al derecho,
analizadas en los módulos anteriores. Sin embargo, es importante no olvidar
que la división entre reglas primarias y secundarias, por sí sola, no es suficiente
para dar cuenta de toda la complejidad del derecho como práctica social.

EDUBP | Abogacía | Filosofía y Lógica Jurídica - pag. 58


Idea de Obligación
Lo invito a reflexionar sobre las siguientes frases: “el derecho sólo impone
obligaciones si amenaza con una sanción” y “el riesgo de que a nuestra
teoría se la lleve el viento”. Las mismas nos ayudan a comprender con cla-
ridad en qué consiste la idea misma de obligación.

Hasta aquí se ha insistido en tratar de mostrar que el derecho no es reducible


a meras órdenes respaldadas por amenazas. También advertimos cómo esta
reducción debía ser desechada en un punto y rescatada en otro. El punto en
que merece ser rescatada consiste en reconocer que, una vez que en nuestra
vida (en nuestra forma de organización social) penetró el derecho, algunas
conductas dejaron de ser optativas. En otras palabras, podemos saber muy
poco sobre el derecho, podemos tener dificultades para distinguirlo del mero
hábito o de las órdenes respaldadas por amenazas, etc., pero sí tenemos
claro algo: cuando hay derecho, al menos un conjunto mínimo de accio-
nes salen de nuestro ámbito de libertad para convertirse en obligatorias.

No obstante, la concepción simple del derecho como órdenes y amenazas


se excedía al pensar que la existencia de una obligación dependía exclusiva-
mente de que una amenaza pendiera sobre nuestras cabezas. Por ejemplo,
este enfoque no podía decir en virtud de qué norma, la persona que daba la
orden respaldada por amenaza (supongamos el legislador) era competente
para hacerlo. En este punto, resulta útil volver a la comparación entre el
asaltante y el agente fiscal que cobra los impuestos.

Estas situaciones eran descriptas por los individuos en forma muy disímil:
algunos sostenían que debían entregar el dinero al asaltante sí y sólo sí
éste los apuntaba, la pistola funcionaba, el ladrón denotaba capacidad
de lastimarlos, etc. Reconocimos que si el asaltante venía con una pistolita
de agua no le hacíamos caso, o que si se retiraba del banco no continuá-
bamos bajo sus órdenes. Resulta que sólo nos sentimos bajo el poder del
asaltante si creemos que es más o menos probable que nos descerraje
un tiro si no le entregamos el reloj o la billetera. La situación con el agente
fiscal era descripta de manera diferente. Hablamos como si tuviéramos la
obligación de entregarle dinero, aún cuando el agente fiscal no nos haya
encontrado en casa el día que vino a reclamarlo, la AFIP tarde en encontrar
mi domicilio para intimarme a pagar o creamos que las armas con las que
cuenta el Estado para cobrarnos son ineficaces.

Resulta claro que existe una diferencia fundamental entre las actitudes
que desarrollan los individuos frente a las órdenes del asaltante y frente
a las órdenes jurídicas. En el primer caso, es central que el individuo crea
que es probable que sobrevenga un mal, mientras que en el segundo la
gente sigue sosteniendo que está obligada, aunque crea (o esté segura) que
podrá evadir el control fiscal y librarse fácilmente de la multa por evasión
de impuestos. Por ello, no sería arbitrario sostener que, frente al asaltante,
el individuo se vio obligado a entregar el dinero ya que creía sensatamente
que lo dañaría si no lo hacía. Sin embargo, sería forzar las cosas sostener
que la víctima del robo tenía la obligación de entregar el dinero ya que, en
general, cuando sostenemos que alguien está obligado nos referimos a una
situación que persiste o se mantiene con independencia de que el individuo

EDUBP | Abogacía | Filosofía y Lógica Jurídica - pag. 59


crea que será dañado o sepa que se encuentra en peligro de ser multado o
sancionado jurídicamente. Lo que la situación del asaltante mostraba es que
en un aspecto importante:

No sostenemos que alguien está obligado frente a un asal-


tante, sino más bien que se vio forzado por las circunstancias.

Mientras que:

Frente al derecho, sostenemos que alguien está obligado (cae


bajo el alcance de alguna norma) incluso aunque la persona
en cuestión no tema a la sanción jurídica o pueda evadirse
del control policial.

En resumen, la afirmación que alguien se vio obligado (forzado) a hacer


algo, es en general una afirmación acerca de los motivos o creencias
que acompañan a una acción (Pedro creía que lo matarían si no entregaba
el dinero).

Resultaría contradictorio sostener: Pedro se vio obligado a entregar el dinero


al asaltante aunque creía que era imposible que le pasase algo. Cualquier
interlocutor sensato podría preguntarse: ¿en qué sentido se vio forzado si
creía que realmente nada pasaría?

En contraste con la situación del asalto, solemos hablar de que alguien tiene
una obligación jurídica aunque crea sensatamente que no le pasará nada si
no cumple con sus deberes jurídicos y cívicos. Por lo tanto, nadie ve una con-
tradicción en la siguiente frase: Pedro está obligado a pagar los impuestos
aunque cree –con razón– que si no paga no sufrirá ninguna consecuencia.

Este punto es de especial importancia porque muestra la metodología o


camino que estamos siguiendo para tratar de elucidar la noción de obliga-
ción jurídica. Lo que intentamos es rescatar la forma en que usualmente nos
referimos a las órdenes del asaltante y a las del juez o el legislador. Bien
miradas las cosas, nos damos cuenta de que las condiciones bajo las cuales
una y otra son usadas en el lenguaje cotidiano son bien diferentes.

Una –la de sentirse obligado – sólo es usada en contextos en que el agente


cree que es altamente probable que, si no obedece la orden, habrá una con-
secuencia desagradable en su contra. Por otro lado, la noción de tener una
obligación se usa cotidianamente para referir a situaciones en las que es
posible que el individuo no se sienta forzado a nada, y ni siquiera desarrolle
una actitud de temor hacia la orden jurídica. Así, estamos dispuestos a decir
que existe una obligación jurídica aún en el caso de que la amenaza sea
mínima y la probabilidad de castigo nula. La idea es que:

Nuestra noción de obligación no incluye como necesarios


hechos psicológicos acerca del agente (como creencias y
motivos).

EDUBP | Abogacía | Filosofía y Lógica Jurídica - pag. 60


El enunciado de que alguien debe ir al servicio militar o pagar los impuestos
sigue siendo verdadero, aún bajo el supuesto de que esta persona no conozca
el enunciado o crea que nada tenía que temer en caso de desobediencia.

Resta por último resaltar una diferencia más en la forma en que usamos
los enunciados de obligación y los de órdenes respaldadas por amenazas.
Mientras que solemos hablar de que alguien tiene una obligación, aún
cuando no cumpla con la conducta prescripta, es bastante raro escu-
char que alguien se vio forzado a hacer algo, y en realidad no lo hizo. En
general, el tipo de enunciados; Juan se vio obligado a hacer X conlleva
que la acción fue efectivamente realizada.

Pero, a pesar de tener en claro la diferencia entre “verse obligado” (forzado)


y “tener una obligación”, todavía no tenemos una noción muy clara de qué
significa estar obligado; sólo sabemos hasta ahora, que no se trata simple-
mente de experimentar una creencia o actitud hacia una amenaza, sino de
algo más complejo e independiente de ello, que no logramos explicar aún.

Una segundo vía

Una opción al intento anterior de describir la noción de obligación en térmi-


nos subjetivos (de lo que el agente cree, teme o desea) ha sido sostener que:

Existe una obligación cuando es posible predecir con proba-


bilidad que un individuo que ha realizado una acción contra-
ria a la ordenada, sufrirá un mal.

Esta concepción sostiene que un enunciado de obligación (“es obligatorio


jurídicamente no matar”) equivale a un enunciado en el cual se predice que
probablemente el autor del hecho será sancionado. Así, el enunciado “Juan
está obligado a no matar” o “es obligatorio no matar” es equivalente al enun-
ciado “es probable que Juan, si mata, reciba una sanción impuesta por un
juez”. Como puede advertirse, esta teoría difiere de la que analizábamos ante-
riormente: mientras que aquella sostenía que existe una obligación cuando
alguien cree que sufrirá un mal (y ya vimos las razones por las cuales esta
afirmación no es correcta), ésta sostiene que afirmamos que alguien está
obligado jurídicamente a algo cuando podemos emitir un juicio de probabi-
lidad sobre el mal que sobrevendrá con motivo de la infracción. La primera
hacía depender nuestra noción de obligación de un componente subjetivo (lo
que el sujeto creía, deseaba o temía); ésta hace depender la obligación de un
juicio predictivo sobre la probabilidad de que se sufra un mal.

Para hacerlo más comprensible, un defensor de esta segunda teoría diría:


alguien tiene una obligación si es altamente factible que de no realizar lo pre-
visto por el derecho, algún juez lo multará o sancionará. Así, podemos decir
que Pedro está obligado jurídicamente a pagar los impuestos si es altamente
probable que, de no pagar los impuestos en un par de meses, se le aparezca
un señor vestido de azul en la puerta y lo lleve detenido o lo notifique de una
multa, o sí es altamente probable que lo lleven ante el Juez y se lo sancione
con prisión.

EDUBP | Abogacía | Filosofía y Lógica Jurídica - pag. 61


En el módulo I, vimos cómo este tipo de concepciones que llamaremos pre-
dictivistas (pues reducen los enunciados jurídicos a predicciones sobre lo
que harán los jueces) terminan transformando las normas jurídicas, las sen-
tencias y cualquier otro enunciado jurídico del tipo “X está obligado a pagar”
en una especie de anuncio de lo que vendrá.

A pesar de las críticas, debemos reconocer que gran parte del metier de los
abogados y jueces es saber el grado de probabilidad con el cual serán aco-
gidas sus demandas o sentencias. No es para nada inusual escuchar en los
pasillos de tribunales: “no presentes el escrito ahí porque seguro que te lo
rebotan”, “espera que pase el turno del juzgado 21 porque ahí esos amparos
suelen ser rechazados” o “ten cuidado porque seguramente si tu defendido
cometió esa evasión en el Juzgado penal económico lo van cocinar”. De igual
manera, un juez de primera instancia (en el fuero civil) o un juez de instruc-
ción (en el fuero penal), saben que algunas de sus sentencias serán bien
acogidas por un órgano superior en caso de ser recurridas, y otras indefec-
tiblemente serán revocadas. Asimismo conoce (o predice) que si el Tribunal
Superior lo pesca en algún tipo de resoluciones, es altamente probable que
le haga llamados de atención, lo aperciba o lo sancione.

En similar tono, si uno quiere dedicarse a la delincuencia con cierto éxito,


es útil que conozca las probabilidades de éxito y de fracaso en la empresa.
Es decir, que conozca qué grado de probabilidad tendrá de ser atrapado,
llevado a juicio y condenado.

Aunque todas estas visiones son plausibles y útiles (siempre es útil saber los
bueyes con los que uno ara en tribunales, comisarías y agencias fiscales)
no sirven para dar cuenta acabadamente de qué significa esto de tener una
obligación jurídica.

Existe un aspecto muy importante, fundamentalmente en las sentencias dicta-


das por los jueces, en los que éstos establecen que una persona está obligada
a pagar algo, a cumplir tantos años de prisión, etc. y, en ese sentido, no están
diciendo lo que harán mañana sino que lo están haciendo ahora. Ya tuvimos
oportunidad de advertir en el módulo I que sostener que el papel de los jueces
se limita a predecir qué harán mañana o la semana que viene, equivale a decir
que ellos cumplen una función semejante a un empleado del servicio meteo-
rológico. Sólo se limitarían a anunciar qué mal o tormenta sobrevendrá sobre
nuestras cabezas la semana que viene. Mientras que para el servicio meteo-
rológico el mal a anunciar sería la tormenta, para un juez sería una sanción,
indemnización o multa. Pero, claramente, el juez cuando dicta una sentencia,
no sienta al imputado y le dice “señor, es posible que continúe sancionando a
los casos de homicidio como el suyo con 20 años de prisión”.

El juez, en la sentencia, determina qué obligación jurídica tiene hoy el impu-


tado. Es, en este sentido, que el juez, cuando está decidiendo en un caso
concreto y señala qué norma es aplicable (cuál es la obligación jurídica de
Pedro), no está anunciando lo que hará el día de mañana sino que está justi-
ficando la decisión de hoy en una norma preexistente. Claro que, como ya se
dijo, lo que hace el juez cuando falla le sirve a una enormidad de personas
para saber qué será de su suerte en ese juzgado en el futuro.

EDUBP | Abogacía | Filosofía y Lógica Jurídica - pag. 62


En otras palabras: la teoría predictivista de la obligación sostiene que tener
una obligación equivale a poder afirmar que es altamente probable que sobre
nuestras cabezas recaerá un mal, si no hacemos lo que el derecho espera de
nosotros. Decir que Pedro está obligado a algo es igual a decir que si no lo
hace, lo aporrearán, maltratarán o sancionarán de algún modo. Pero, a poco
que analizamos algunos fenómenos jurídicos como las sentencias, caemos
en la cuenta de que el juez no sólo está simplemente dando un indicio para el
resto de los abogados y potenciales delincuentes sobre su interpretación de la
norma de homicidio; lo que está haciendo es dictar una norma individual que
sostiene que Pedro está obligado a cumplir 20 años de prisión y justificando
su decisión en una norma del derecho penal (supongamos, el art. 79 del C.P.).

Por ello, una primera objeción a la teoría predictivista consiste en señalar que
ella oscurece la idea según la cual un juez, cuando dicta una sentencia, no
sólo está dando una señal de cómo actuará en el futuro, sino también está
brindando una justificación para su actual resolución (una razón por la que
aplica una sanción o una absolución).

Existe una segunda objeción a esta teoría predictivista. Para desarrollarla


retomaremos la metodología de otorgarle crédito a la forma en que habla-
mos acerca del derecho. Hemos visto cómo, en una sociedad moderna, sole-
mos sostener que alguien tiene una obligación aunque no crea tener una
obligación o aunque no se sienta obligado. Esto reforzaba la idea de que una
obligación jurídica no podía ser equiparada a una orden respaldada por ame-
nazas. Mientras que éstas últimas dependían de las creencias, actitudes y
temores del agente, nuestra noción de obligación jurídica no. Análogamente,
estamos habituados a sostener que alguien tiene una obligación jurídica,
aunque sea altamente improbable que el juez competente lo sancione.

Nuevamente, en este punto, podríamos sentirnos desahuciados. Hemos vis-


lumbrado que las condiciones bajo las cuales decimos que alguien tiene una
obligación jurídica difieren sustancialmente de aquellas en que decimos que
alguien se ve obligado, o de aquellas en las cuales sostenemos un juicio
de predicción y probabilidad, pero no tenemos tan sólo una pista de por
qué ello es así. Sabemos que las situaciones son diferentes, pero no hemos
explicado por qué el significado de estas expresiones difiere. Esto es, no
sabemos exactamente qué es una obligación y menos aún en qué consiste
una obligación jurídica. Para tener éxito en este camino, Hart nos propone
que regresemos a la distinción entre hábito y regla. Hemos analizado, si bien
de manera sucinta, cómo en la familia Reyna había comportamientos, que
si bien eran regulares, no merecían por parte del resto de la familia ninguna
atención especial (recordemos el caso del “video” los sábados). El hábito
requiere, como ya sabemos, un comportamiento regular, pero no exige del
grupo una actitud distintiva de aceptación o aprobación como pauta correcta
de comportamiento. La regla, en cambio, aunque comparte con el hábito
la característica de comportamiento regular, exige o implica que los sujetos
vean en él una pauta de comportamiento que consideran correcta o valiosa.
Para ello basta que los individuos estén dispuestos a declarar como “correc-
tas” “debidas” u “obligatorias” las conductas que se conforman a la regla.
Recordemos el caso de Pedro en la casa de los Reyna. Los integrantes de
la familia tenían una serie de reglas que consideraban valiosas y veían como
“correcto”, “debido” u “obligatorio” su seguimiento. Todo este aparato con-

EDUBP | Abogacía | Filosofía y Lógica Jurídica - pag. 63


ceptual es ya suficiente para caracterizar y diferenciar a las reglas de los
hábitos, pero no lo es para dar cuenta de las condiciones bajo las cuales
existe una obligación jurídica, ni una regla jurídica.

Cuando afirmo que alguien tiene una obligación, mi afirmación


tiene un característico trasfondo normativo pues afirmo que su
caso particular cae bajo el ámbito de aplicación de una regla.

Ya se ha deslizado entre nosotros la idea de que siempre que haya una obli-
gación, habrá una regla, pero de ello no se sigue que siempre que haya una
regla habrá una obligación. Y, consecuentemente, del hecho que exista una
regla tampoco se sigue que haya una obligación jurídica.

Basta recordar que el conjunto de reglas impone pautas de conducta, pero


frente a ellas no estamos dispuestos a decir que el individuo tiene una obliga-
ción, sino un simple deber (v.g. las reglas para cocinar, para jugar al ajedrez,
al fútbol).

En definitiva: toda obligación supone una regla, pero no toda regla impone
obligaciones. Y la pregunta que podría formularse aquí es la siguiente:
¿cuándo una regla impone obligaciones y no tan sólo simples deberes?
O, en el mismo sentido, nos preguntamos ¿por qué percibo de forma dife-
rente a las reglas del backgammon que a las del código penal o a las de la
Biblia?

Una forma de explicar estas diferencias, es sostener que una regla impone
obligaciones, si el grupo que comparte estas reglas desarrolla frente a
ellas:
i) una fuerte e insistente exigencia general a favor de la conformidad
con las pautas de conducta impuestas por la regla;
ii) una regular e importante presión sobre aquellos que se desvían
(infringen la norma) o amenazan con hacerlo.

La ecuación resultante sería:

Obligación = exigencia insistente para seguirla + fuerte presión contra


quienes se desvían.

Esto significa que, conductas que consideramos simplemente debidas


como, por ejemplo, hablar correctamente, serían consideradas obligatorias
si hablar incorrectamente fuera castigado con una pena severa, tal como
cortarle la lengua a quien lo haga.

Retomando el ejemplo de la familia Reyna (analizado en los contenidos del


módulo 2 en oportunidad de distinguir los hábitos de la familia de sus reglas
de convivencia), recordemos que había un conjunto de reglas sobre las
que el grupo presionaba (con comentarios, miradas y actitudes) a favor de
su cumplimiento y también insistía en estas pautas de conducta criticando
a Pedro cuando no las observaba. Este esquema sirve para distinguir las
reglas que imponen obligaciones de las que no lo hacen, pero no nos dice

EDUBP | Abogacía | Filosofía y Lógica Jurídica - pag. 64


cuándo estamos frente a una obligación jurídica. Intentaremos desentrañar
esta cuestión.

Ya sabemos que dentro del conjunto de reglas están aquellas que imponen
obligaciones y aquellas que no lo hacen y lo que diferencia a unas de otras
es la seriedad o importancia con que el grupo ejerce presión social a favor
del cumplimiento de las mismas.

Sin embargo, no cualquier pauta de conducta es acreedora o merecedora de


una presión social seria y sostenida. En general, las pautas de conducta a
favor de las cuales se ejerce este tipo de presión presentan las siguien-
tes características:
a) Las reglas que se eligen para ejercer presión seria, no son cualquier tipo
de reglas sino aquellas que se creen necesarias para la preservación
de la vida social o de algún aspecto de ella al que se atribuye gran
valor (v.g. las que restringen el uso de la fuerza, reclaman honestidad
o veracidad, exigen que cumplamos con nuestras promesas, etc.).
b) En general, se ejerce presión porque lo que exige la norma de deber u
obligación está en contra de lo que el individuo desea hacer. Esto no
es más que decir que los grupos sociales medianamente articulados
no utilizan sus mecanismos de corrección y presión para cualquier
cosa, sino sólo para tipos de conductas que consideran de central
importancia para la continuidad del grupo, aunque vayan en contra
de los intereses de algún miembro del mismo grupo.

Es necesario hacer aquí una breve aclaración: que la noción de obligación


de cualquier índole lleve asociado el aspecto de presión social sobre los
individuos infractores, no debe llevarnos a creer que alguien está obligado
porque se siente presionado o tiene temor a que lo critiquen.

Ya hemos dado razones por las cuáles no debe equipararse la idea de tener
una obligación a la de sentirse obligado. Lo que no quiere decir que en muchas
circunstancias se cumpla con las normas como consecuencia de la fuerte
presión, pero esto nada tiene que ver con la idea de tener una obligación.

Así, el enunciado que sostiene que Pedro está obligado a pagar los impues-
tos o Pedro está obligado a ir a misa los domingos, respetar los sacramentos,
etc. no afirma que Pedro le tiene miedo a lo que pueda predecir que le suce-
derá si no paga los impuestos o no va a misa los domingos, sino que afirma
que la conducta de Pedro cae bajo el alcance de una norma. No preocuparse
demasiado por esto por ahora, ya que será aclarado más adelante.

El aspecto interno y externo de las reglas o el valor filosófico de la expre-


sión criolla “los de afuera son de palo”.

Cualquiera de nosotros, en su vida cotidiana, ha tenido que hacer uso de


la expresión “los de afuera son de palo”. Esta expresión suele ser un eficaz
repelente para el tipo de personas que gusta inmiscuirse en asuntos ajenos,
entrometerse en algún deporte queriendo asumir el papel de árbitro, opinar
sobre alguna estrategia en nuestra partida de truco del día domingo, o sim-
plemente meter la nariz en alguna discusión que no le incumbe. Además

EDUBP | Abogacía | Filosofía y Lógica Jurídica - pag. 65


de cumplir con este efecto paralizador del entrometido, la frase en cuestión
suele expresar y referir a una intuición muy acertada. La intuición, según la
cual quién no ha participado en una cierta discusión o no sabe cuáles son
las reglas del juego que los intervinientes han aceptado, opina sin saber muy
bien de qué se trata.

La idea es que, aquel que es meramente un observador externo de una situa-


ción se pierde una dimensión fundamental de lo que está pasando y, en ese
sentido, corre el riesgo de intervenir o describir la situación erróneamente.
Análogamente ocurre cuando un grupo social tiene diferentes reglas, esto
abre la posibilidad de dos tipos de aserciones. Las aserciones desde el
punto de vista interno y las desde el punto de vista externo.

En las aserciones desde el punto de vista interno, el sujeto


que realiza la acción o sigue la regla está involucrado con la
aceptación de la misma y ve en la regla una razón para actuar.

Las aserciones desde el punto de vista externo no incluyen la aceptación de


la regla. En estos casos, tenemos un sujeto que llega a un determinado lugar
y observa un conjunto de regularidades, pero sin asumir el punto de vista
de los observados, esto es, de quienes siguen reglas. Tomemos el siguiente
ejemplo: en nuestra ciudad, los conductores de automóviles, en general,
detienen el vehículo frente a la luz roja del semáforo. El observador externo,
todo lo que puede decir al describir este comportamiento es que en la ciudad
de Córdoba, en general, los conductores de automóviles detienen la marcha
del vehículo frente a un semáforo en rojo. Este observador, no puede dar
cuenta de cuál es la razón por la que se detienen; no puede saber si lo
hacen porque la luz roja les paraliza el ánimo de seguir conduciendo, o lo
hacen porque tienen la obligación de detenerse frente a la luz roja.

El punto de vista externo es el punto de vista de un observador


que describe los comportamientos uniformes o regulares de
una práctica de seguir reglas, sin asumir el punto de vista de
los observados, esto es, de quienes siguen las reglas.

Tal vez, esto pueda resultar un tanto abstracto, pero podemos llegar a darnos
una idea somera del problema si recordamos lo analizado en el primer
módulo con relación al antropólogo que trataba de describir a su rey un con-
junto de prácticas que ni él ni el rey conocían. El antropólogo tenía una serie
de conjeturas acerca de qué era esto del rock and roll, pero debido a que su
punto de vista era completamente externo (no comprendía lo que hacían) se
limitaba a describir el escenario como un altar, al micrófono como un báculo,
a la pelota de fútbol como una esfera de cuero, al estadio de fútbol como un
templo, etc.

Aunque algunas de las afirmaciones del antropólogo eran acertadas, en un


aspecto central no alcanzaban para dar cuenta de las prácticas descriptas.
Con su aparato rudimentario, el antropólogo no podía describir por qué las
personas concurrían todos los domingos a la cancha, por qué criticaban a
alguien si se adelantaba en la cola de la boletería, etc. Lo único que podía
hacer, más o menos seriamente, era describir regularidades de comportamien-
tos (un montón de gente va a una edificación los domingos,... repite lo que los

EDUBP | Abogacía | Filosofía y Lógica Jurídica - pag. 66


supuestos cantantes dicen frente al báculo,... gritan cuando la esfera de cuero
entra a una zona que tiene tres palos...). Una descripción de regularidades
se limita a describir –en algún sentido – sólo lo que es observable a primera
vista. Los enunciados de tipo externo hacen típicamente esto. Imaginemos al
antropólogo tratando de describir el fenómeno del semáforo. Si realmente él
tiene un punto de vista externo (no conoce la práctica) su descripción debería
limitarse a sostener: cada vez que la luz de un poste de tres luces se pone en
color rojo, los autos tienden a detenerse o es probable que suceda el hecho
de que los conductores aprieten el freno. Aunque esta descripción externa es
adecuada, no sirve para informar a su rey qué es lo que socialmente están
haciendo los individuos cuando se detienen frente a un semáforo, por qué lo
hacen y en virtud de qué consideran que deberían hacerlo.

El tipo de enunciados que se limita a describir regularidades consecutivas al


estilo de “se prende la luz roja, los autos se detienen” o “alguien rompe un
vidrio, vienen cuatro personas vestidas de azul con una gorrita y lo toman del
brazo” se denominan enunciados externos, debido a que no presuponen, ni
utilizan en la descripción, la actitud que los participantes desarrollan hacia
esos hechos o señales. Nótese que en ningún momento la descripción pura-
mente externa del antropólogo puede explicar por qué los automovilistas se
detienen frente al semáforo o cuál es el particular fenómeno por el que la
persona es llevada a la comisaría luego de romper el vidrio. En este sentido,
sus descripciones sólo ven en la regla (detenerse frente al semáforo en rojo,
no romper vidrios, etc.) sólo un signo de lo que vendrá después. Así como
un físico puede ver en la hornalla prendida un signo de que el agua hervirá,
o un astrónomo puede ver en tal o cual disposición estelar un signo de que
se acerca la llegada de tal o cual cometa, el antropólogo sólo podrá ver las
reglas como hechos que indican qué es lo que vendrá después de que se las
infrinja (después de que alguien rompe un vidrio, es probable que lo lleven
preso; después de que siguió de largo frente a la luz roja del semáforo, es
probable que le impongan una multa, etc.) pero no podrá describir en qué
consiste seguir una regla o infringir una regla. Es decir, no nos está diciendo
qué hacen cuando se detienen frente al semáforo o cuando se abstienen de
romper vidrios (ni cuando no se detienen o rompen vidrios). No obstante, el
antropólogo puede optar por tratar de mejorar su capacidad de descripción
de la práctica social en cuestión, adoptando lo que llamaremos un punto de
vista externo moderado. Este punto de vista está en condiciones de explicar
qué hacen los participantes cuando aceptan una regla, se detienen frente
al semáforo en rojo e insultan a quien no respeta esta señal. Cuando el
antropólogo (o cualquiera de nosotros que tratamos de entender qué está
pasando allí) adopta este punto de vista, puede advertir que la actitud de
los individuos no es equiparable al caso del hervor del agua o del paso del
cometa, donde a un hecho simplemente le sigue otro (al hecho del agua
expuesta al calor le sigue el hervor; al hecho de la constelación X le sigue el
paso del cometa Halley). Más bien, el observador logra ahora advertir que
los individuos ven en el semáforo una señal de detenerse (una orden de
detenerse) y, además, encuentran una razón para frenar su vehículo y criticar
a quien no lo hace.

Quién describe una práctica de este tipo, está en condiciones de descri-


bir no solamente hechos brutos (como la luz roja, el freno activado, gente
insultando) sino, además, qué están haciendo socialmente los individuos y

EDUBP | Abogacía | Filosofía y Lógica Jurídica - pag. 67


la razón por la que se detienen, insultan, o siguen una regla. En este sen-
tido, esta descripción capta la perspectiva interna de los individuos hacia las
reglas, según la cual quien maneja un auto no ve en la señal del semáforo en
rojo lo mismo que ve cuando observa cualquier otra luz roja sino una señal
(una orden) para detenerse y una razón para ello.

En este contexto, puede resultar oscura la idea o concepto de razón. Aunque


es fácil entender que los automovilistas cuando se detienen, no sólo ven una
luz de color rojo sino una norma que indica detenerse frente a la luz roja,
no resulta tan fácil entender la cuestión de que están viendo una razón para
detenerse. Para aclarar este punto, diremos que:

Un individuo tiene una razón para hacer o no hacer algo, si


está dispuesto a justificar su conducta o su decisión de actuar
de una determinada manera en base a esa razón.

De este modo, si le preguntásemos al conductor por qué se detuvo (o por


qué insultó a quien pasó en rojo) éste diría: porque el semáforo estaba en
rojo y ello justifica tanto mi detención como indignación frente al infractor.
Cuando alguien, como en este caso, ve en una norma o regla (señal de
tránsito, p. ej.) una razón para actuar, ve en realidad un determinado hecho o
regla sobre el cual justificar su acción frente a sí mismo y frente a otros. En el
mismo sentido, podría decir el conductor:

“Lo hice (me detuve o insulté al que no lo hizo) debido a que existe una norma
de tránsito”. Resulta fácil advertir que, cuando cotidianamente damos razo-
nes sobre por qué hicimos tal o cual cosa (p. ej. le explicamos a un amigo
por qué no fuimos a su cumpleaños) tratamos de justificar nuestro accionar
invocando alguna circunstancia que motivó que actuáramos de esa manera.
Tener una razón para algo es tener un hecho o mecanismo en el cual asentar
nuestra justificación de por qué hicimos lo que hicimos (me detuve porque
el semáforo estaba en rojo, pagué los impuestos porque fui intimado por la
AFIP, etc.). En definitiva, dar una razón o tener una razón para algo es tener
algún soporte justificatorio para actuar como lo hicimos.

Es importante aclarar en este punto que la aceptación de las reglas jurídi-


cas, a diferencia de la aceptación de otras reglas sociales, es un fenómeno
en cierto modo indirecto. En efecto, el grupo social reconoce una regla
que provee a la identificación de las pautas o criterios de conducta jurídica
mediante la referencia a las palabras, oral o escrita, de una persona dada. En
otros términos, se acepta la regla jurídica como pauta de conducta social por
el reconocimiento de la autoridad de su origen o fuente de creación y no por
su contenido.

A modo de síntesis y para redondear nuestro análisis en este punto, debemos


tener en cuenta: que las reglas o prácticas sociales presentan una dimensión
interna. Esta dimensión interna está relacionada con la forma en que los indi-
viduos o participantes (ciudadanos, diríamos en derecho) ven esa práctica y
el conjunto de normas.

EDUBP | Abogacía | Filosofía y Lógica Jurídica - pag. 68


Esas prácticas (cómo los individuos siguen normas) puede ser analizada
desde el punto de vista externo o interno. Desde el punto de vista externo-
extremo, sólo describiremos regularidades que no servirán para explicar qué
es lo que socialmente está pasando cuando alguien se detiene frente a un
semáforo en rojo. Moderando este punto de vista y asumiendo el de los parti-
cipantes (el punto de vista interno del seguimiento de una regla), advertimos
que los individuos ven en las reglas (como las que obligan a detenerse frente
al semáforo en rojo) no sólo un hecho (como sería una nubarrón, una cons-
telación estelar o una llama prendida) sino una señal (orden) de lo que deben
hacer y atribuyen a esta señal no sólo la capacidad de ordenarles qué deben
hacer (detener el auto) sino, además, de justificar qué están haciendo (dar
razones de por qué actuamos de determinada manera). Finalmente, cuando
nosotros aceptamos una determinada norma o regIa y en virtud de ello criti-
camos a otro por Ia infracción cometida, asumimos Iisa y llanamente el punto
de vista interno. Es decir, el de aquel que ve en Ia regIa una razón para actuar
de acuerdo a ello y un buen motivo para criticar a quienes se desvían.

El derecho como unión de reglas primarias y secundarias.

Para comprender la importancia del rol de las reglas secundarias en el dere-


cho analicemos el siguiente ejemplo:

La tribu, la comunidad, la familia y las sociedades complejas. Por qué


todo va bien con los amigos y la cosa se complica cuando aparecen
extraños.

Érase una vez un grupo de cinco amigos (Pedro, Juan, María, Aníbal y Daniela)
quienes deciden emprender un viaje de mochileros hacia el sur de nuestro
país. La idea era bastante simple: cargar las mochilas con carpas, bolsas de
dormir, muchos fideos y arroz y emprender el viaje en el colectivo más barato.
Como es de esperar, entre amigos que se quieren bien, estaba bastante claro
que cada uno respetaría las opiniones del otro (no se trataría de imponer nada
por la fuerza), pedirían prestado cualquier implemento que hiciera falta – v.
gr. jabón o toalla –, (no habría hechos de avasallamiento de la propiedad del
otro) y sobre todo, se cumpliría la promesa inicial de tratar de pasarla bien
y llegar, -vía Puerto Madryn, Bariloche y Esquel – al Glaciar Perito Moreno.
Como recaudo económico, por último, habían formado un pozo común que
sería asignado a la compra de comidas para los campamentos, pago de cam-
ping y demás gastos compartidos.

El viaje anduvo bastante bien hasta que sucedió un pequeño altercado


doméstico. Mientras María, Aníbal y Daniela salieron a recorrer a pie un lago,
Pedro y Juan se quedaron cocinando. Les faltaba un encendedor para pren-
der el calentador a gas por lo que Pedro sugiere que busquen en la mochila
de María, quien solía tener un encendedor ya que era una fumadora empe-
dernida. Una vez revisada la mochila encuentran el encendedor de María
y también su atado de cigarrillos. Nuestros cocineros se tientan y deciden
fumar dos cigarrillos del atado de María mientras terminaban de cocinar, des-
contando que María se enojaría por ello. En el trámite, el encendedor estilo
“50 centavos, transparente, comprado en un kiosco” se rompe (como suele
suceder). María, al regresar, no se siente del todo cómoda con la usurpación
de su propiedad. Sostiene que se ha roto la promesa de no inmiscuirse en la

EDUBP | Abogacía | Filosofía y Lógica Jurídica - pag. 69


propiedad ajena sin antes consultarlo y que, además, había otras reglas de
respeto mutuo que se habían violentado. Pedro y Juan –por su parte– sostie-
nen que la regla que no había que tocar las cosas del resto, no incluía o tenía
en cuenta un caso como el objeto de la discusión, básicamente porque no
habían metido mano en su mochila para “robarle” sino sólo porque era nece-
sario conseguir un encendedor y porque a los cigarrillos los habían “tomado
prestados” y en ningún momento habían pretendido ocultarle el hecho.

La discusión entre los miembros del grupo, bien puede mostrarse como una
batalla entre dos argumentos: el argumento de María que sostiene que esa
actitud era un caso claro de violación de su propiedad y el argumento de
Juan y Pedro que sostiene que lo que ellos habían hecho no podía contar
como un caso de falta de respeto o intromisión en la propiedad de María. Por
otra parte, la discusión no sólo versa sobre cuál es el contenido de las reglas
que el grupo compartía, sino también cuál es el criterio para identificarlas
o reconocerlas. Mientras que María invocaba que las reglas que los regían
eran las que habían consensuado el día que se habían reunido para planear
el viaje, Pedro y Juan alegaban que, además, en otras reuniones se había
acordado que se iba a ser flexible con relación a los préstamos y utilización
de enseres ajenos.

A estas dificultades se sumó una adicional: el encendedor de María se des-


compuso, hubo que comprar uno nuevo y no estaba del todo claro de dónde
debía salir el dinero: si sólo se consideraba como relevante que el encende-
dor era propiedad de María y que ella era la única fumadora del grupo, María
debía cargar con el costo del nuevo encendedor; pero si se tenía en cuenta
que el encendedor se había roto en un empresa o actividad común (la pre-
paración de la comida), el gasto debía ser imputado al pozo común. Unos
días después irrumpió en la armonía del grupo otro hecho inesperado. Pedro
es víctima de la sustracción de su bolsa de dormir. La plata que le restaba
a Pedro era insuficiente para cubrir ese gasto (sólo le quedaba dinero para
pocos gastos personales y el resto estaba en el pozo) y el problema requería
una solución urgente. El dilema que se presentó era obvio: o se usaba plata
del pozo común para ayudar a Pedro, o éste debía emprender el regreso.
Sin embargo, para utilizar el dinero del pozo común, era necesario cambiar
la decisión inicial según la cual el dinero del pozo sólo sería utilizado para
gastos comunes. Claramente, el robo a Pedro no era un caso que debía ser
tratado como un gasto común, pero las circunstancias del viaje y el contexto
habían variado de tal manera que parecía razonable abandonar la norma
anterior sobre el pozo común y adaptarla a este caso. Claro que era posible
adoptar la otra opción, ésta es respetar a rajatabla la regla inicial y tener en
cuenta este tipo de eventualidades para los próximos viajes... (No ayudar a
Pedro y que vuelva a su casa y tener en cuenta esto para la próxima vez que
se salga de viaje en conjunto, lo cual sabe Dios cuándo volvería a ocurrir).

Esta situación hipotética pretende ilustrar situaciones que se suscitan con


respecto a las reglas en un sistema jurídico. Ya hemos distinguido entre las
reglas primarias y secundarias. Las primeras, generalmente se refieren a
acciones o cambios físicos en el mundo, mandan al agente a hacer u omitir
algo (no mates, no mientas, no robes, etc.). En general, una sociedad primi-
tiva o un grupo reducido (como el de nuestros amigos mochileros) puede
funcionar bastante bien con este tipo de reglas. De hecho, el grupo no tuvo

EDUBP | Abogacía | Filosofía y Lógica Jurídica - pag. 70


inconvenientes y pudo arreglárselas bastante bien con un pequeño código
de convivencia que sólo incluía reglas de este tipo. Este código no era muy
complejo y sólo se limitaba a delinear pautas básicas de conducta. Su efecti-
vidad, además, venía garantizada por el afecto y cercanía de los que estaban
regulados o regidos por esas normas. Todos compartían lazos de afecto que
permitían estar, más o menos, de acuerdo sobre las normas de coexistencia
básica que regirían durante el viaje.

Pero resulta claro que, al surgir circunstancias imprevistas (sacar el encen-


dedor de la mochila de María y fumar sus cigarrillos), a partir de las cuales
los protagonistas se preguntaron –en primer lugar– si este caso podía contar
realmente como un caso de intromisión en la propiedad de María, lo que
ocurría era un desacuerdo acerca de la norma primaria, según la cual era
obligatorio respetar las pertenencias del resto. La discusión versaba sobre el
contenido y alcance que debía darse a tal norma de convivencia. Sumado a
ello, Pedro y Juan sostenían que había otras normas básicas (primarias) que
habían sido consensuadas en otros momentos y que no había por qué tomar
como básicas sólo las normas que acordaron en la última reunión antes del
viaje sino aquellas que se habían consensuado durante todas las reunio-
nes. Esto muestra claramente cómo este grupo y cualquier organización
que se rija sólo por un conjunto de normas primarias, necesariamente se
verá enfrentado (en algún momento) a instancias de incertidumbre sobre sus
obligaciones y derechos. Siempre será posible discrepar sobre qué reglas
nos rigen efectivamente y cuál es su alcance. En grupos o sociedades más
o menos complejas, indefectiblemente habrá una regular falta de certeza
sobre cuáles son las reglas y cuál su contenido y alcance.

En nuestro ejemplo, además, podemos advertir de qué manera los problemas


del grupo con relación a su código de convivencia fueron haciéndose cada
vez más complejos. No sólo había una falta de seguridad sobre qué normas
realmente tenían y cuál era su alcance, sino que surgieron problemas como
el del robo a Pedro, que obligaban a decidir si continuar con la regla del “pozo
común” y que Pedro vuelva a su casa, o cambiar la regla en cuestión de allí
para el futuro. Y, como si esto fuera poco, las reglas del código de conviven-
cia –al ser sólo primarias– no preveían ningún procedimiento para su cambio;
no incluían ninguna previsión con respecto a situaciones sorprendentes. Es
decir, o bien seguíamos utilizando la misma regla para un caso que –prima
facie– no estaba previsto, o bien la modificábamos agregando la situación de
imprevistos de fuerza mayor. Es importante notar que con respecto a la regla
del “pozo común” no había un problema de certeza o seguridad. La regla
claramente decía que sólo los gastos comunes, como la comida o los viajes,
contarían como imputables al pozo, pero en ningún momento contaba como
gasto común el caso del robo o infortunio personal.

El ejemplo presentado pretende mostrar cómo en sociedades o grupos que


sólo cuentan con normas primarias, indefectiblemente aparecerán circuns-
tancias que obligarán a repensar las normas consensuadas, careciendo de
reglas que establezcan un procedimiento para adaptar o modificar las normas
que se han mostrado como obsoletas o insuficientes para resolver el con-
flicto. En el caso citado, dado que los individuos no tenían un procedimiento
para cambiar lo previamente consensuado y que el grupo era respetuoso de
las regla preestablecidas, se imponía la obligación de respetar la norma del

EDUBP | Abogacía | Filosofía y Lógica Jurídica - pag. 71


“pozo común”, la cual claramente no contemplaba el caso del robo a Pedro.
De contar con reglas que prevean el procedimiento para cambiarlas, inmedia-
tamente (si son sensatos) incluirían una que contemple que, en caso de robo,
se utilizará dinero del “pozo común” para reponer el objeto robado y evitar la
lamentable consecuencia que Pedro tuviera que regresar. En definitiva, este
tipo de sistemas compuestos por reglas primarias solamente, no sólo ado-
lece de una falta de certeza acerca de cuáles son exactamente las reglas del
grupo, sino que además, sufre de una especie de rigidez que llamaremos
aquí el carácter estático de un sistema jurídico de normas primarias. Por
último, y como consecuencia de todo lo anterior, el caso de los viajeros mues-
tra cómo también se carecía de un procedimiento que determine quién y de
qué manera clausurar la discusión entre María, Pedro y Juan.

En sistemas que sólo poseen normas primarias, no se cuenta con órganos


o personas que precluyan o cierren la discusión. Esto equivale a decir que
determinados conflictos o controversias necesitan ser cerrados o precluidos
con fuerza de autoridad y para ello necesitamos, tanto un procedimiento que
nos diga qué pasos han de seguirse, como quién será el encargado de tomar
una decisión final que clausure la controversia. Esta falencia de los siste-
mas de normas primarias (simples o primitivos) es denominada por Hart el
defecto de la difusa presión social.

El diagnóstico que hemos hecho de grupos sociales que sólo cuentan con
normas primarias (ejemplificado por el grupo de mochileros) arroja sucinta-
mente la siguiente lección: en sociedades simples, en donde el acuerdo y
los lazos familiares y afectivos sean fuertes y estrechos, será más o menos
factible garantizar la convivencia sin problemas. Pero cuando el grupo social
se amplía o las circunstancias cotidianas se sofistican, es necesario contar
con otro tipo de reglas que permitan solucionar los defectos mostrados por
los códigos de convivencia rudimentarios.

Los defectos de un sistema primitivo o de reglas primarias son básicamente:


su carácter estático (falta de reglas que prevean el procedimiento para cam-
biar las reglas), su falta de certeza (no se sabe con exactitud cuáles son las
reglas que obligan al grupo) y su difusa presión social (no prevén quiénes y
cómo se solucionan los conflictos).

El remedio para estos defectos de los sistemas sociales primitivos o simples


consiste en la introducción de las llamadas reglas o normas secundarias.
Estas reglas secundarias son las que nos permiten lidiar y trabajar con los
problemas que acarrean las normas primarias: Estas son la regla de recono-
cimiento, la regla de cambio y la regla de adjudicación.

Para solucionar el problema de la falta de certeza, lo que nuestros mochile-


ros necesitarían es lo que llamaremos aquí una regla de reconocimiento. Es
decir, una regla que indique bajo qué condiciones una regla es una regla de
convivencia obligatoria del grupo. Recordemos que María sostenía que sólo
podían ser reconocidas como reglas del grupo las que habían sido acorda-
das en la última reunión previa al viaje, y Pedro y Juan sostenían que debían
ser reconocidas como reglas del grupo todas las que habían sido discutidas
en el total de las reuniones. Los que participamos en una práctica social
regida por reglas jurídicas tenemos nuestra propia regla de reconocimiento:

EDUBP | Abogacía | Filosofía y Lógica Jurídica - pag. 72


sabemos que lo que está prohibido o permitido en Argentina depende (más
o menos) de lo que dijo el legislador, con tal o cual grado de mayoría en el
Congreso y si ha sido publicado en el código civil, penal, etc. a través del
boletín oficial. En ese sentido, poseemos una regla para reconocer con algún
grado elevado de certeza cuáles son las reglas que nos rigen. Los diferentes
grupos sociales pueden tener diferentes reglas de reconocimiento en dife-
rentes grupos sociales (recordemos que la familia Reyna identificaba qué
debía comer los domingos con sólo conocer qué había decidido el abuelo
el día anterior). Del mismo modo, tenemos criterios para saber o reconocer
cuáles son otros tipos de reglas, como por ejemplo las reglas religiosas (p.
ej. lo que dice la Biblia o el jefe de determinada religión, etc.). Por lo tanto,
podría decirse que al defecto de la falta de certeza se lo subsana con una
regla secundaria de reconocimiento. En el siguiente módulo profundizare-
mos respecto a esta cuestión. El segundo inconveniente que padecía el
sistema de reglas primarias era su carácter estático (carencia de un pro-
cedimiento para modificar las reglas que devienen en obsoletas o que no
prevén nuevos casos). Para solucionar este problema, los sistemas jurídicos
modernos instituyen las llamadas reglas secundarias de cambio. Las reglas
secundarias de cambio son aquellas que permiten introducir, más o menos
rápidamente, un cambio en la situación jurídica de un conjunto de personas
(recordemos las leyes de moratoria, morigeración de pena o amnistía) o de
una persona en particular (por ejemplo la regla en virtud de la cual exonero a
mi deudor del pago de la deuda, o la regla que me permite novar una deuda,
compensarla, etc.).

Al tercer defecto de los sistemas simples lo denominamos aquí la difusa pre-


sión social y consistía en que los individuos carecían de un procedimiento
centralizado que solucione definitivamente los conflictos. Nuestro grupo de
mochileros carecía de un juez que decidiera sus discusiones con algún tipo
de autoridad, de modo que las discusiones sobre los mismos temas no se
repitieran incesantemente. Para esta enfermedad de la discusión infinita,
existe una tercera regla secundaria que denominamos regla de adjudica-
ción, según la cual se instituye un proceso para dirimir controversias y a un
sujeto con la potestad de dirimirlas (se instituye algún tipo de juicio previo y
de juez natural).

Para profundizar en la concepción del derecho como unión


de unión de reglas primarias y secundarias los invitamos a
ver el siguiente video:

https://www.youtube.com/watch?v=UBEaOUFVlwM

Los fundamentos del sistema jurídico:

En el punto anterior, se mostró que un sistema conformado sólo por normas


del tipo primario sólo puede funcionar en contextos simples, dentro de un
grupo social con un número reducido de miembros, con lazos afectivos y cul-
turales estrechos y con un consenso más o menos homogéneo sobre lo que
las normas exigen. Se advirtió también que cuando se trata de grupos socia-
les heterogéneos en cuanto a sus características culturales e ideológicas y

EDUBP | Abogacía | Filosofía y Lógica Jurídica - pag. 73


con un número elevado de miembros, el derecho conformado únicamente
por reglas primarias mostraba tres defectos, a saber: la falta de certeza res-
pecto de cuáles son las reglas jurídicas que integran el derecho positivo, el
carácter estático de las reglas por carencia de un procedimiento para cam-
biar deliberadamente las reglas primarias y, la difusa presión social por no
disponerse de un órgano que centralizara la función de resolver los conflictos
relativos a la transgresión de las reglas y la aplicación de sanciones.

Es una característica de los derechos positivos de las sociedades modernas


que regulan no sólo la conducta de los sujetos de derecho, sino también su
propia creación y aplicación – esto quiere significar que el derecho está insti-
tucionalizado. Se introdujo, para dar cuenta de tal característica, las nociones
de reglas secundarias de reconocimiento, cambio y adjudicación.

Regla de reconocimiento

Profundizaremos ahora el tema relativo a la identificación de las reglas váli-


das que pertenecen a un sistema jurídico y que involucra, como vimos, a la
regla de reconocimiento.

Tal regla, según Hart, indica uno o más criterios de identificación o reconoci-
miento de normas válidas.

Por validez de una regla Hart entiende su pertenencia a un


derecho vigente.

Cuando un abogado reconoce el derecho vigente en materia de contratos


revisando un Código Civil está haciendo uso de una regla de reconocimiento
que le indica que lo que establece un código que posee sanción legislativa
es derecho válido.

Algunos sistemas jurídicos poseen una regla de reconocimiento simple, por


ejemplo que lo que el derecho dispone depende de lo que diga la legislatura.
Otros sistemas, como la mayoría de los sistemas occidentales modernos,
poseen reglas de reconocimiento más complejas, pues incluyen además de
reglas con un origen legislativo, reglas que tienen su origen en la costumbre
social y, también las que surgen de la jurisprudencia – precedentes judiciales.

Para comprender la función y forma en que se usa la regla de reconocimiento


se introducirá la noción de conjunto definido bien por intensión o por exten-
sión y, la distinción entre uso y mención.

En la vida cotidiana, sabemos que cualquier conjunto puede formarse sim-


plemente enumerando sus integrantes -extensión- o señalando alguna pro-
piedad que detenta la clase que forma el conjunto -intensión.

Cuando determinamos un conjunto señalando alguna/s característica/s o


propiedad/es que es/son compartida/s por los elementos del conjunto, divi-
dimos el mundo entre los elementos que tienen la propiedad y aquellos que
no la poseen.

EDUBP | Abogacía | Filosofía y Lógica Jurídica - pag. 74


De ese modo intencional la regla de reconocimiento divide el mundo de las
normas entre aquellas que forman parte del sistema jurídico y aquellas que
están excluidas. Así, sabemos que el conjunto de órdenes proferidas por un
loco parado en un banquito de la plaza no son parte del derecho.

En lo relativo a la distinción uso-mención, la regla de reconocimiento se usa


fundamentalmente para identificar cuál es el derecho aplicable o vigente en
un determinado momento. Pero, en un sentido importante la regla de recono-
cimiento no es sólo un test para identificar lo que los demás consideran dere-
cho, sino también una herramienta para determinar qué es derecho y qué
no lo es. Cuando una persona utiliza una regla de reconocimiento no sólo
describe bajo qué condiciones una regla es considerada jurídica en un deter-
minado territorio, sino que también, usualmente, ejerce un juicio en virtud del
cual acepta esa norma como válida para regir su conducta. De esto se infiere
que una cosa es describir lo que un sujeto reconoce como digno de reco-
nocimiento y, otra cosa es manifestar lo que es obligatorio o prohibido para
mí. La diferencia que existe entre el enunciado de un sujeto llamado Juan:
“la norma ‘x’ es una ley válida y me obliga a ‘p’” y el enunciado proferido por
un tercero: “la norma ‘x’ es una ley y, por ende, Juan la considera válida y
obligatoria”, radica en que, cuando Juan dice lo que dice expresa y usa una
particular actitud hacia la norma ‘x’, actitud que no expresa quien profiere el
segundo enunciado. Mientras Juan está usando el enunciado para expresar
su actitud hacia la norma ‘x’, el tercero menciona o hace referencia al criterio
de Juan para identificar lo que lo obliga.

Lo dicho significa que, frente a la regla de reconocimiento, pueden tomarse


dos posturas. Una, desde el punto de vista interno y otra desde el punto de
vista externo. Para identificar la regla de reconocimiento de un sistema no es
necesario que los aceptantes expliquen o hagan explícito el contenido de
la regla. Basta con observar el particular modo en que ellos asumen deter-
minadas prácticas como obligatorias, correctas y cómo están dispuestos a
criticar su incumplimiento. No hace falta que todos los jueces digan “reco-
nocemos como válida las reglas del congreso en materia contractual” para
darnos cuenta de cuál es la regla de reconocimiento que están utilizando.
Basta con leer las sentencias para advertir qué criterio usan y cuál rechazan.
De este modo, puede decirse que la regla de reconocimiento es un criterio
o conjunto de criterios que los individuos de un grupo usan (expresándolo
abiertamente o no) para identificar sus obligaciones.

En síntesis, el o los criterios de una regla de reconocimiento puede/n ser


usado/s o mencionado/s. Una persona usa una regla de reconocimiento si
acepta el criterio por ella explicitado como válido y ve en la regla así identifi-
cada una pauta de conducta correcta que constituye la razón para criticar a
los que desobedecen tales pautas. Una persona menciona este criterio si la
misma no acepta el criterio como válido para identificar obligaciones, pero
asume que un determinado grupo lo acepta como válido para identificar sus
obligaciones.

Podemos decir de modo esquemático que un enunciado interno es reali-


zado por aquella persona que usa la regla de reconocimiento y la acepta
sin necesidad de manifestar expresamente que la está aplicando o acep-
tando. Cuando un juez sostiene que el caso bajo análisis debe ser resuelto

EDUBP | Abogacía | Filosofía y Lógica Jurídica - pag. 75


con la utilización del artículo 1109 del C.C. está haciendo dos cosas: a)
usando la regla de reconocimiento según la cual “lo que dice el Congreso es
válido jurídicamente” y b) aceptando sin expresar dicha regla. Por su parte,
quien menciona a la regla desde un punto de vista externo, se limita a
describir lo que el derecho prescribe para ese caso, pero no desarrolla
hacia la regla (necesariamente) una actitud de aceptación o aprobación.

Donde existe una regla de reconocimiento, tanto los particulares como los
funcionarios tendrán criterios con autoridad para identificar las reglas pri-
marias de obligación. De este modo, la regla de reconocimiento se expresa
en los diferentes criterios que los individuos de un grupo social utilizan para
identificar sus obligaciones y derechos. Estos criterios pueden asumir de
hecho varias formas: textos revestidos de autoridad, sanción legislativa,
práctica consuetudinaria, declaraciones generales de personas especifica-
das, decisiones judiciales pasadas, dictadas en casos particulares, etc.

En suma, la regla de reconocimiento puede identificar una o varias fuen-


tes sociales como productoras de obligaciones. Quienes aceptan una
regla de reconocimiento compuesta de dos o más criterios o fuentes,
también reconocen cierta relación de prelación –supremacía o subor-
dinación- entre sus criterios. Esta relación se hace evidente en caso de
conflicto entre normas identificadas por diferentes criterios de una misma
regla de reconocimiento pues alguna de ellas, por ejemplo como ocurre en
nuestro sistema jurídico, aquella norma que satisface el criterio legislativo,
prima sobre otras normas que satisfacen algún otro criterio. En consecuen-
cia, lo que otorga validez jurídica a una obligación no es una norma superior
sino la satisfacción de algún requisito –fuente de derecho– de la regla de
reconocimiento del sistema.

Es importante distinguir la validez o existencia de una regla particular de


su ineficacia. Vimos que los enunciados de validez son enunciados desde
un punto de vista interno. Por el contrario, los enunciados sobre la eficacia
de una norma particular son enunciados desde un punto de vista externo que
describen el hecho de que la regla en cuestión es más obedecida que des-
obedecida. En consecuencia, cabe afirmar que en principio los conceptos
de validez y eficacia de una norma particular son lógicamente independien-
tes. El único caso en que la eficacia de una norma puede ser condición de
validez de esa norma se da cuando la regla de reconocimiento del sistema
jurídico establece entre sus criterios la provisión de que ninguna regla será
considerada regla del sistema si durante un tiempo prolongado ha dejado
de ser eficaz.

Para profundizar sobre el concepto de la regla de reconoci-


miento:

https://bit.ly/3H6w6UD

EDUBP | Abogacía | Filosofía y Lógica Jurídica - pag. 76


En síntesis:

La existencia de una regla jurídica particular es su validez o


pertenencia a un sistema jurídico y la validez está dada por la
satisfacción de algún criterio indicado por la regla de reco-
nocimiento.

En cambio, para sostener que una determinada regla de reconocimiento


existe, es necesario un trasfondo normal en el cual los individuos usualmente
la empleen en la identificación de sus obligaciones, de modo que, su exis-
tencia depende de su uso o práctica efectiva que hacia ella desarrollan
funcionarios y ciudadanos en conjunto en la identificación de las reglas
jurídicas válidas. Incluso, puede pensarse en un grupo social donde los
ciudadanos estén disconformes con el/los criterio/s que contiene la regla de
reconocimiento; bastaría con que un grupo relevante de la sociedad (legisla-
dores, jueces, juristas, abogados, etc.) desarrollen una actitud de aceptación
conjunta, para que exista tal regla de reconocimiento.

Por último, es importante recalcar que la regla de reconocimiento, como regla


última del sistema jurídico, no es ni válida ni inválida pues, como dijimos, su
existencia depende de una práctica efectiva (actitud de aceptación al menos
por el sector oficial) empíricamente verificable. Mientras que la noción de vali-
dez designa una relación entre dos reglas (la regla jurídica y la regla de reco-
nocimiento) la noción de existencia de la regla de reconocimiento designa
una relación entre una regla (la de reconocimiento) y un grupo de individuos
(la práctica o uso que ellos hacen de la misma).

Concordantemente, podemos decir que un sistema jurídico existe cuando al


menos el sector oficial –en especial los tribunales– efectivamente aceptan la
regla de reconocimiento y demás reglas secundarias y se da una obediencia
generalizada por parte de los ciudadanos (punto de vista externo).

Teniendo en cuenta las diferentes condiciones de existencia, es posible esta-


blecer las siguientes relaciones entre los conceptos de validez y eficacia:

i) la validez de una regla particular no tiene relación necesaria con su


eficacia;
ii) no se puede predicar validez o invalidez de la regla de reconoci-
miento, sólo su eficacia para identificar reglas particulares válidas;
iii) para que exista un sistema jurídico se requiere que se acepte una
regla de reconocimiento (que permita otorgar validez al resto de las
normas del sistema);
iv) el enunciado efectuado desde el punto de vista interno que expresa,
por ejemplo, que la regla contenida en el art. 1109 del código civil
es obligatoria (válida), presupone la eficacia general del sistema (no
tendría sentido hablar de la validez de una regla particular fuera del
marco de un sistema jurídico existente y, la eficacia general, es una
condición necesaria para la existencia de un sistema jurídico.

EDUBP | Abogacía | Filosofía y Lógica Jurídica - pag. 77


Respecto de la regla de reconocimiento podemos concluir de la siguiente
manera:
• No resulta útil preguntarse si la regla de reconocimiento es o no es dere-
cho (si es o no jurídicamente vinculante).
• Sí resulta sensato preguntarse ¿qué criterios usan los sujetos para iden-
tificar sus obligaciones y decidir contiendas jurídicas?; ¿bajo qué condi-
ciones están dispuestos a decir que algo es obligatorio o permitido jurídi-
camente?; ¿Cuál es el conjunto de requisitos que tiene que cumplir una
orden para ser usada en un tribunal para justificar una decisión?; ¿bajo
qué condiciones un funcionario está dispuesto a desarrollar un punto
de vista interno frente a la regla, aceptando el curso de acción por ella
impuesto como correcto?
• Parece que carece de sentido preguntarse ¿la gente tiene que obede-
cer las normas para sostener que son válidas? Lo que tiene sentido es
conocer y averiguar si existe una práctica más o menos uniforme, en
un territorio dado, de aceptación de algunas conductas como pautas
correctas y si en dicho lugar hay una práctica más o menos uniforme de
desarrollar una crítica hacia quien no sigue esas pautas. Para identificar
un sistema jurídico (como diferente de otro), bastará con identificar un
grupo que acepte una regla de reconocimiento, que especifique los cri-
terios de validez jurídica y que sus reglas de cambio y adjudicación sean
efectivamente aceptadas por sus funcionarios como pautas o modelos
públicos y comunes de conducta oficial. Y por último, habrá que consta-
tar que en general la gente obedece la mayoría de las reglas más de lo
que las desobedece.
• La tendencia a hablar erróneamente de la validez de la regla de recono-
cimiento proviene de la propensión a tratar la regla de reconocimiento
domo una regla jurídica. No tiene sentido preguntarse si la regla de reco-
nocimiento es válida, pues ello equivaldría a preguntarse si la regla de
reconocimiento satisface la regla de reconocimiento. El origen de la con-
fusión puede tener su origen en la decisión de Hart de llamar “regla jurí-
dica” a la regla de reconocimiento. Según el profesor Eugenio Bulygin,
en un artículo titulado “Sobre la regla de reconocimiento”1, la confusión
desaparece si distinguimos entre reglas de conducta y reglas conceptua-
les. Las reglas conceptuales son las que permiten reconocer, identificar o
diagnosticar si determinada actividad está o no llevándose a cabo (jugar
al ajedrez en el caso de las reglas determinativas de ese juego, hablar
castellano si se trata de las reglas gramaticales definitorias de ese len-
guaje). La regla de reconocimiento es de tipo conceptual o definitoria,
pues indica los criterios bajo los cuales puede afirmarse que los indivi-
duos están jugando al “juego del derecho”. Si los miembros de un grupo
social no usan ninguna regla de reconocimiento para identificar sus obli-
gaciones, podemos afirmar que no tienen sistema jurídico alguno –a lo
sumo podrá sostenerse que juegan el juego del arbitrio de los tribunales,
pero no del derecho.

1 Bulygin, Eugenio; “Sobre la regla de reconocimiento”, en Análisis lógico y derecho, ed.Centro de


Estudios Constitucionales, Madrid, 1991, ps. 383-392.

EDUBP | Abogacía | Filosofía y Lógica Jurídica - pag. 78


Para complementar el estudio del presente módulo, es necesario que lea
la siguiente bibliografía: Hart, H. L. A.; El Concepto De Derecho, Abeledo
Perrot, Bs. As., 1968, capítulos 4, 5 y 6.

GUASTINI, Riccardo: Releyendo a Hart. Revista Doxa. Cuadernos de Filosofía


del Derecho, 37 (2014) ISSN: 0214-8676 pp. 99-110 recuperado en:
https://rua.ua.es/dspace/bitstream/10045/53981/1/Doxa_37_06.pdf

Si usted está interesado en profundizar sobre la temática del Módulo 3, puede


consultar: NINO, Carlos Santiago; Introducción al Análisis del Derecho, Buenos
Aires, Editorial Astrea, 2ª edición ampliada y revisada, 1992, capítulo 3.

Lo invito a realizar las actividades de Módulo 3.

M3 Actividades
Actividad 1
Una fotocopia con historia
Ud. ha regresado a su profesión de antropólogo. Recordemos su viaje ante-
rior a comunidades occidentales desconocidas en las que había registrado
ciertas circunstancias asociadas a regularidades de conducta observables
de los miembros de esas comunidades.

De nuevo en ese contexto, se ha vuelto a encontrar con una serie de ele-


mentos que había traído de su último viaje al lugar donde realizó sus inves-
tigaciones: apuntes, objetos varios, notas de entrevistas con los ciudadanos
de ese lugar. Entre todas esas cosas, Ud. encuentra una fotocopia que le
entregaron en un lugar llamado Concejo Deliberante Municipal. La primera
hoja de esa copia tenía el siguiente título “Código de convivencia y urbanidad
de la ciudad”. En el artículo 14 de ese código, Ud. encontró una norma que
dice: “Los habitantes de la ciudad están obligados a limpiar las veredas todos
los domingos, juntar la basura en bolsas de color negro y dejarlas a disposi-
ción de la autoridad municipal antes de las 15 hs. bajo apercibimiento de ser
multados con la suma 200 pesos-ley”.

Asimismo, recuerda que cuando leyó esa norma le causó curiosidad saber
exactamente en qué consistía la obligación estipulada en el artículo 14. Usted
piensa realizar varias entrevistas y espera notar que haya –al menos – tres
tipos de respuestas a la pregunta: “¿En qué consiste el deber impuesto por
el artículo 14?”. Tratando de anticiparse a las mismas, imagina qué respon-
derán a esta pregunta los tres grupos de personas siguientes:
a) los que sostienen que verse obligado está relacionado fundamental-
mente con las creencias, temores y motivaciones del sujeto a quien se
dirige la norma;

EDUBP | Abogacía | Filosofía y Lógica Jurídica - pag. 79


b) los que sostienen que verse obligado está relacionado con un juicio de
probabilidad y predicción.
c) los que sostienen que existe una particular forma en que los individuos
observan las normas y atribuyen valor a la práctica de seguirlas o infringirlas.

AA

AA Actividad 1
asistente académico
Para ubicar las respuestas que corresponden a cada uno de los grupos de
personas seleccionados, tenga en cuenta los diferentes puntos de vista dis-
tinguidos en el contenido de este módulo respecto de la idea de obligación.

Actividad 2

Luego de las entrevistas: ¿en qué consiste el seguimiento de reglas?


Luego de las entrevistas realizadas (relativas al artículo 14 del código de
convivencia y urbanidad) y de analizar las tres concepciones existentes, Ud.
decidió salir a la calle para ver en qué consistía el seguimiento o cumpli-
miento de esta regla sobre la basura y el barrido de la vía pública. Allí pudo
observar (según su parecer):
a) diferentes personas que, según su percepción, cumplían con la norma;
b) otras que las infringían;
c) un grupo que ante la supuesta infracción reaccionaba criticando al des-
obediente, etc.
Mientras Ud. observaba este tipo de actitudes, iba realizando numerosas
notas a partir de las cuáles está en condiciones de realizar o describir la prác-
tica del cumplimiento del artículo 14 del código de convivencia y urbanidad
desde dos puntos de vista: uno externo extremo y otro externo moderado.
Decide, entonces, realizar una descripción de lo que los individuos hacen
cuando tratan de cumplir con el artículo 14 desde un punto de vista externo-
extremo y desde un punto de vista externo-moderado.

Para la realización de esta actividad tenga presente las siguientes diferencias:


• diferencias de actitudes que pueden asumir los miembros de un grupo
social respecto de sus reglas de convivencia y los puntos de vista que
están implicados en esas actitudes.
• la diferencia entre obedecer una regla y cumplirla.
• las diferencias entre punto de vista externo extremo y punto de vista
externo moderado.

AA
AA Actividad 2
asistente académico

Recuerde que todas las distinciones señaladas han sido explicadas en el


contenido del presente módulo. También resulta pertinente la consulta del
texto de Hart, capítulo 4, primera parte.

EDUBP | Abogacía | Filosofía y Lógica Jurídica - pag. 80


MÓDULO 4

M4 Microobjetivos

• Comprender las tesis del escepticismo jurídico, con el propósito de con-


ceptualizar actitudes y puntos de vista de algunos sectores de la socie-
dad con respecto al derecho.
• Desarrollar habilidades que posibiliten crear las herramientas necesarias
para distinguir el discurso escéptico del discurso formalista, a los fines de
encontrar una vía media que permita dar cuenta de las prácticas jurídicas
habituales.
• Reconocer la íntima relación entre derecho, la práctica social, el lenguaje
y la norma, con el propósito de saber conceptualizar dicha relación y de
apropiarse de una nueva forma de llevar a cabo el análisis de los proble-
mas jurídicos.

M4 Contenidos

Acceda al video de presentación del Módulo 4

Formalismo (los jueces son la “boca de la ley”) y escepticismo ante las


reglas (el derecho es lo que los jueces dicen que es).

Formalismo y escepticismo ante las reglas.

En los módulos anteriores estudiamos que un sistema jurídico compuesto


sólo por reglas primarias, genera una serie de defectos que las reglas secun-
darias vienen a solucionar y a marcar, algo así como, el paso de un mundo
pre-jurídico a lo que entendemos hoy por “mundo jurídico moderno”. Los
defectos eran la falta de certeza, la estaticidad y la difusa presión social del
sistema jurídico y los remedios eran, respectivamente, laregla de recono-
cimiento y las reglas de cambio y adjudicación. Por su parte, la regla de
reconocimiento es una regla de tipo especial: define precisamente el juego
del derecho, pero no es ella misma una regla del juego. Por ello, no puede
ser catalogada de válida o inválida y las preguntas a las que da lugar son
bien diferentes a los interrogantes que suscitan las otras reglas del sistema.

Aunque las reglas secundarias atenúan los defectos citados, es posible que
no sean suficientes para erradicarlos. Es altamente posible que el alcance
de una regla primaria esté en discusión y que ninguna de las técnicas que
tenemos disponibles nos permita saber con certeza cuál es su “verdadero”
alcance. Cuando esto sucede, cuando no sabemos exactamente qué es
lo que dice tal o cual regla o artículo de un código y, en ese sentido, qué
conducta es obligatoria, permitida o prohibida, podemos entrar en pánico y
adoptar dos posturas antagónicas y extremas relativas a la naturaleza del
derecho:

EDUBP | Abogacía | Filosofía y Lógica Jurídica - pag. 81


Una de las opciones es sostener que el derecho siempre
está totalmente determinado.

Dicho de otro modo, dada una determinada regla, siempre sabremos qué
tenemos que hacer subsumiendo el caso particular a la regla general (v. gr.
si César mató a Bruto y existe la regla que dice “el que mate a otro debe ir
preso”, entonces César debe ir preso); y:

Otra opción es caer en el escepticismo y sostener que, dado


que algunas reglas generan conflictos de indeterminación
(no se sabe exactamente cuál es su alcance) es imposible
motivar conductas mediante reglas y, en ese sentido, con-
cluir en que el Estado de Derecho es una empresa imposible.

Antes de analizar estas posturas extremas, resultaría conveniente tener en


cuenta en qué consiste – o qué exige– un estado de derecho y el fenómeno
de la legislación.

¿Cómo comprendemos al derecho?


En primer lugar, y de modo muy simplificado, entendemos aquí al derecho
como una técnica de control social, que utiliza el lenguaje como vehículo
para motivar a los individuos a que se comporten de una manera deter-
minada (hagan o dejen de hacer algo). Cuando el legislador emprende
esta empresa se encuentra básicamente frente a la siguiente situación: existe
un cierto propósito o fin que quiere favorecer (v. gr. la igualdad de las per-
sonas, vivienda digna para todos, la paz social, la colaboración con el fisco,
etc.), y el derecho constituye una herramienta para desarrollar esa tarea.

En nuestro ejemplo de la familia Reyna, el abuelo es una especie de legisla-


dor (ha sido designado para determinar qué se debe comer los domingos).
En esta actividad el abuelo se encuentra –al igual que el legislador– inclinado
a favorecer tal o cual propósito con la comida que elige y con la orden que
dará en consecuencia. Imaginemos que el abuelo, al momento de elegir qué
se debe comer, pretende favorecer algún tipo de interés colectivo (familiar).
Supongamos que el abuelo considera que por ser una típica familia argentina,
se han excedido en la ingesta de carne durante la semana. El lunes comieron
milanesa, el miércoles peceto y el sábado asado. Entonces, considerando
valioso promover la ingesta de comida más sana, decide ordenarles a su
hijo y a uno de sus nietos (ambos son quienes se encargan de ejecutar sus
órdenes) que el domingo venidero comerán pastas. Además, les desliza en
el medio de la conversación, algunas de las razones o propósito subyacente
que justifican el dictado de la regla: “es obligatorio comer pastas el próximo
domingo”; siendo el propósito que el abuelo quiere favorecer: “es valioso o
importante favorecer la ingesta de comidas que no tengan carne”.

Llegado el domingo, el hijo y el nieto Reyna concurren a la roticería italiana


de la vuelta de casa. Llegan un poco tarde y, como sucede en toda casa de
pastas los domingos, la variedad se ha reducido notablemente. Sólo quedan
unas tartas rellenas de carne. Existe una opción de comprar tartas que tienen
mucho jamón y algo de queso. Aunque la tarta comparte con la pasta gran

EDUBP | Abogacía | Filosofía y Lógica Jurídica - pag. 82


cantidad de propiedades (las tapas son de harina) generalmente la gente no
la llama pasta. Aunque la gente no suele llamar pasta a este tipo de comida
elaborada, toda casa de pastas vende tapas para tarta. Además, también es
usual encontrar en ese tipo de casas de comidas, tartas como las citadas.
En un sentido importante (su porcentaje de harina, el lugar donde la venden,
etc.) las tartas pertenecen a la familia de las pastas. Sin embargo, a diferen-
cia de los ravioles o los fideos, estamos en la duda de si incluirla en el grupo
citado o no.

Algunos “ingredientes” para analizar este ejemplo.


La tarta ejemplifica un caso curioso de problema lingüístico: el caso dudoso
o incierto de pasta. No estamos seguros si el término en cuestión refiere o
no a ese caso.

El hijo y el nieto de Reyna sostienen una razonable discusión sobre si com-


prar tarta o no sería acorde con lo que el abuelo ordenó: si el concepto de
pasta incluye el de tarta, si la tarta es pasta cuando están las tapas solas,
pero no cuando está rellena, etc.

Este caso muestra, siempre en modelo a escala, que cuando se utiliza una
técnica de motivación semejante a la que utiliza el derecho, siempre es posi-
ble que las personas que tienen que responder a dichas demandas duden
sobre qué es lo que tienen que hacer o dejar de hacer. Parecería ser que:

El “precio” que necesariamente se debe pagar, si se pretende


motivar conductas a través de reglas generales, esto es, que
abarquen un conjunto (clase) de individuos y se refieran a un
tipo general de conductas y circunstancias, es asumir que en
determinados casos marginales no sabremos exactamente
qué debemos hacer, si encuadrarlos dentro de la regla o no.

En este punto, es sumamente importante que usted advierta que el contenido


de toda regla general, más o menos clara, “capturará” un núcleo de casos
a los que claramente se aplica -las pastas a las que se refería el abuelo
incluyen sin lugar a dudas a los tallarines- y una zona periférica, marginal
o de penumbra en los que dudaremos razonablemente si se aplica o no
la regla en cuestión (la tarta, las tapas de tarta, la tarta rellena). Llamamos
casos claros a los primeros y casos difíciles a los segundos.

Lo importante a tener en cuenta aquí es que la dificultad para aplicar la direc-


tiva del abuelo no reside en que él ha sido confuso en algún sentido. Más
bien, él ha sido suficientemente claro, y ha utilizado una terminología enten-
dible por cualquier persona competente en el lenguaje. El problema de la
directiva del abuelo no está tan relacionada con su torpeza o pericia al dictar
la regla (él ha sido bastante claro y prolijo al dictarla) sino en un defecto
inerradicable del lenguaje cotidiano: la textura abierta –o vaguedad
potencial– de los términos que utilizamos. Siempre será posible que, en
la utilización de términos simples y claros, encontremos casos de penumbra
de ese término, al que no sabremos si aplicárselo o no. Todo término lin-
güístico necesariamente tendrá un conjunto de casos que integran el núcleo

EDUBP | Abogacía | Filosofía y Lógica Jurídica - pag. 83


de significado claro, esto es, casos a los que el término se aplica sin que
tengamos duda sobre ellos (ñoquis, tallarines, etc.) y casos que, aunque
comparten con aquellos algunas características centrales, nos hacen dudar
razonablemente sobre la aplicación de la palabra al estado de cosas que
se trate. Los casos que se encuentran en la zona de penumbra comparten
algunas propiedades con los casos centrales (ser de harina, que se vendan
en una casa de pastas, etc.), y esto es lo que nos arroja a una duda razona-
ble sobre cuál es el alcance o denotación del término. En este sentido, una
palabra puede compararse con el haz de luz de una linterna. Existe un grupo
de objetos que se encuentran claramente iluminados. Generalmente son los
que se encuentran en el centro del haz de luz. Para identificarlos no tenemos
mayores problemas. Pero existen otros que se encuentran en la zona de
periferia del haz de luz. Esos casos aparecen borrosos, a veces nos parece
estar seguros qué son, y otras veces, no.

El lenguaje natural o, más precisamente, los términos de


clase indefectiblemente presentan una cierta textura abierta
que nos impide saber, para determinados casos, si tal o cual
cosa cuentan como miembro o no de dicha clase. Un tér-
mino de clase indefectiblemente presentará casos centrales
o paradigmáticos a los que no dudaremos en aplicar el tér-
mino, y casos que subsisten en la zona de penumbra a los
que no sabremos si aplicarle dicho término.

La técnica de motivación o de publicidad de sus normas que utilizó el abuelo,


consiste en emplear términos generales que se aplican a clase de cosas,
clase de casos y clase de personas. Este tipo de técnica legislativa tiene
una cierta ventaja económica. Le permite al abuelo (y obviamente al Estado
de Derecho) no tener que dar las órdenes cara a cara. Basta con que diga
de modo general, que es obligatorio para toda la familia comprar pasta el
domingo, para que los individuos del grupo se den por notificados. Por otra
parte, permite que los individuos puedan desarrollar su propio plan de vida
durante la semana, prevenidos de lo que deben hacer con respecto a la
comida del domingo. Las leyes, entendidas como reglas generales, diri-
gidas a un conjunto de individuos, persiguen evitar el engorro de tener que
dar las órdenes cara a cara, y favorecen que los individuos se autoapliquen
las mismas, sin necesidad de tener que estar consultando diariamente a la
autoridad sobre cuál es el curso de acción a llevar a cabo.

Sin embargo, el abuelo podría haber adoptado otra técnica de comunica-


ción de directivas generales. En este sentido, podría haber ido él mismo a
la casa de pastas y señalarle a su hijo y nieto un plato en particular y decir, al
mismo tiempo: este es el tipo de comidas que debemos ingerir de ahora en
adelante los domingos. Este método más que ser una orden general, dirigida
abstractamente, es una especie de escenificación de lo que los familiares
deberían hacer. El abuelo, para reducir el riesgo de que se compre algo inde-
seado, decide mostrarles él mismo la comida que preferiría.

EDUBP | Abogacía | Filosofía y Lógica Jurídica - pag. 84


Otra técnica de comunicación de directivas generales,
además de la legislación, es por el método de ejemplifica-
ción – técnica de los precedentes o del derecho jurispruden-
cial. Respecto de ella también cabe afirmar que padece de
textura abierta o indeterminación. En efecto, indicar un caso
semejante o análogo – no idéntico – puede generar dudas
respecto de si las semejanzas entre los casos son más o
menos relevantes que las diferencias-.
El método de indicar qué se debe hacer para decidir un caso
mediante el uso de un ejemplo dotado de autoridad deja
indeterminados aspectos que sólo podrán ser decididos por
quien tiene que tomar la decisión en el caso concreto.

Sin embargo, este método de ejemplificación con el que venimos trabajando,


también padece de un cierto grado de indeterminación. Supongamos que el
abuelo dijo “esta es la comida que debemos comer los domingos” y señaló
claramente los ravioles. ¿Quiere decir que debe comerse ravioles todos los
domingos?, ¿quiso señalar sólo los ravioles o se refirió a todas las pastas
que estaban en el exhibidor (donde también había ñoquis, tallarines, etc.)?
La duda aquí radica básicamente en cuáles han sido los aspectos relevantes
que quiso señalar el abuelo al extender su dedo hacia el exhibidor. Incluso
puede aparecer una duda adicional. Se puede vacilar con respecto a si el caso
en cuestión (el de este domingo, cuando nos enfrentamos solos a este exhi-
bidor) se parece en los aspectos relevantes al anterior y, además, en cuáles.
Al igual que el método de la legislación, el método de señalar qué se debe
hacer mediante el uso de un ejemplo (dotado de la autoridad que le imprime
el abuelo), deja indeterminados una serie de aspectos que sólo podrán ser
decididos por quien tiene que tomar la decisión en el caso concreto.

Los ejemplos citados pretenden mostrar diferentes cosas. Para señalar


algunas de ellas podríamos considerar que el lenguaje natural, o que ordi-
nariamente usamos para comunicarnos, es un medio para la formulación de
reglas generales de conducta; que como consecuencia del defecto de tex-
tura abierta de ese lenguaje las reglas resultan indeterminadas en su alcance
o aplicabilidad a ciertos casos; que el legislador puede decidir sin conocer
algunas circunstancias que pueden suscitarse en el futuro respecto de la apli-
cación de la regla general y que técnicas alternativas como ordenar mediante
el ejemplo (precedente, jurisprudencia, etc.), no tienen mejor suerte.

Ocurre que, tanto el abuelo como el legislador, cuando dan una directiva
general o dictan una ley, deciden, indefectiblemente, en condiciones subóp-
timas de conocimiento. Es imposible que un ser humano conozca todos los
hechos relevantes del futuro y que, además, sepa cuál sería su propósito,
intención o deseo con respecto a ese estado de cosas futuro. En el ejemplo,
el abuelo no podía prever ni la escasez de pasta estándar, ni la complicación
relativa a si la tarta contaba o no como pasta. Él conocía perfectamente cuál
era su parecer frente a un tipo de comida, pero frente a la tarta puede muy
bien no haber tenido una opinión formada.

EDUBP | Abogacía | Filosofía y Lógica Jurídica - pag. 85


En un aspecto importante, estos problemas no son sólo relativos al derecho,
sino a la esfera más general del lenguaje, las decisiones sobre el futuro y la
especificación de cualquier tipo de regla, ya sea mediante el ejemplo o la
elección de un enunciado general. Así podemos decir que en el derecho:

i) siempre habrá casos que –debido a la textura abierta del lenguaje–


parecen no ser claramente designados por el término de clase. Habrá
una razonable duda sobre si el término tal o cual se aplica al caso en
cuestión o no;
ii) siempre puede subsistir una razonable duda acerca de si el caso en
cuestión se parece en aspectos relevantes al señalado en el momento
de la ejemplificación;
iii) siempre se tomarán decisiones sobre el futuro en condiciones de
relativa indeterminación de propósitos y relativo desconocimiento
del futuro;
iv) siempre habrá que resolver si el caso en cuestión se asemeja en grado
suficiente o no al caso familiar o claro de aplicación de la regla.

Dicho esto, es útil regresar al caso del abuelo y su directiva de comer pasta,
con el propósito de cuidar la salud de la familia. Haremos hincapié en lo
citado en “iii”. Supongamos que el abuelo, para evitar el problema al que
dio lugar su orden y mejorar la posibilidad de que se cumpla su propósito,
dicta otra regla, pero de carácter interpretativo. Esta regla pretende aclarar
el significado de la regla anterior. Así, el abuelo aclara: cuando dije pasta
quise referirme a cualquier tipo de comida que no tenga carne, de modo de
ingerir comida que no nos genere problemas de salud. Con esta regla adicio-
nal, aparentemente se ha subsanado el problema. El abuelo ha dictado otra
norma, para entender mejor la anterior. Por lo tanto, Reyna hijo y nieto van
a la casa de pastas y se encuentran frente a la siguiente complicación: sólo
hay ravioles de pollo o ñoquis hechos con harina de soja marca Sojénica
y papas de la granja que lleva el mismo nombre. Es vox populi por esos
días que la marca Sojénica realiza sus harinas con soja transgénica y que
las papas también tienen el mismo origen, lo cual pone en serias dudas la
calidad del producto, su salubridad y ecología. Una vez más los Reyna se
encuentran ante problemas de textura abierta e indeterminación de propósi-
tos ¿cuenta el pollo de los ravioles como carne?, ¿no se refería el abuelo tan
sólo a carne vacuna?, ¿cuenta la soja como una comida salubre? Pareciera
que sí, pero ¿y la transgénica?

La enseñanza adicional que persigue este ejemplo es que la textura abierta


del lenguaje, el relativo desconocimiento del futuro y la indeterminación de
propósitos al momento de dictar una decisión son defectos inerradicables
(en cierto punto) de la regla. Lo que hace el abuelo Reyna es querer subsa-
nar el defecto intrínseco de toda regla (siempre puede dar lugar a dudas en
los casos de penumbra) con otra regla. Esto, indefectiblemente, necesitará
otra regla para indicar qué se quiso decir con carne, la que a su vez nece-
sitará otra regla y así hasta el infinito. Por ello, es útil retener que existe un
ámbito irreducible en que toda regla generará una instancia en la que quien
la aplica al caso concreto, debe decidir si ella comprende el caso concreto.

EDUBP | Abogacía | Filosofía y Lógica Jurídica - pag. 86


Esta instancia, no puede ser solucionada de ninguna manera dictando una
regla y otra regla y otra regla. Siempre existirá la necesidad de que quien
tiene que juzgar (comprar las pastas o condenar a un imputado) decida en
el caso en cuestión cuál es la mejor manera de conjugar los propósitos en
juego, las intenciones del legislador, los resultados a que puede dar lugar la
aplicación de la regla, etc. Siempre será necesario, al momento de aplicar
una regla un ejercicio adicional de elección.

La apelación a una regla de carácter interpretativo no resuelve


el problema de indeterminación de la regla a interpretar, en
casos dudosos siempre será necesario un ejercicio adicional
de elección sobre la aplicación o no de la regla en cuestión.

Veremos en el próximo apartado, que hay quienes pretenden negar estos


males o el déficit de la legislación y el lenguaje. Para entender bien su planteo,
hay que distinguir claramente los diferentes problemas planteados hasta aquí.

Un problema es el relativo a la textura abierta del lenguaje: siempre habrá


casos que caerán en la zona de penumbra del término y sobre los cuales ten-
dremos dudas si se les aplica o no la regla general. Un segundo problema,
ya no relativo al lenguaje común, se relaciona con los defectos de conoci-
miento del agente. El agente (legislador o abuelo) decidirá siempre en con-
diciones subóptimas, es decir, sin saber exactamente qué deparará el futuro
y, por lo tanto, sin tener formada una opinión al respecto. Como ya se dijo,
el legislador, en tanto ser humano, decidirá con un relativo desconocimiento
del futuro y una relativa indeterminación de propósitos.

Como también se intentó mostrar, cuando se legisla se intenta conseguir


un cierto estado de cosas ideal, esto es, promover determinados valores,
bienes, fines, etc. Para ello el legislador puede optar por dos vías o técnicas
de motivación de conductas:
a) Legislar mediante enunciados bastante precisos y circunscriptos a
los que llamamos reglas. Por ejemplo, prohibiendo o permitiendo deter-
minada acción (como comer carne). Este tipo de técnica legislativa consi-
dera que es posible, prohibiendo un número cerrado de acciones (como
no comer carne, no tomar bebidas sintéticas, etc.) promover el bien de
la salud. El defecto de legislar mediante enunciados claros, precisos y
circunscriptos de este tipo, es que siempre será posible que la acción
prohibida o permitida no promueva, en el caso concreto, el valor que se
quería promover. Nuestro ejemplo de la soja pretendió mostrar ello de
modo bastante claro. El abuelo quería proteger la salud, pero en vez de
dictar una norma que dijese “es obligatorio proteger la salud” o “comprar
comida sana”, seleccionó un grupo de acciones que él pensó contribui-
rían regularmente a lograr ese propósito, como es la no ingesta de carne.
Incluso podría haber agregado que era obligatorio la ingesta de soja,
lo cual nos dejaría en el mismo lugar donde estamos. Llegado el caso,
siempre es posible que la regla elegida, lejos de favorecer o promover el
propósito subyacente, tienda a debilitarlo. Aunque generalmente la soja
ayuda a la salud, parece claro que en el caso concreto la soja transgé-
nica no lo hace. Iguales observaciones podrían hacerse con la pasta, o
con los ravioles rellenos con pollo si se descubriese que la harina tiene

EDUBP | Abogacía | Filosofía y Lógica Jurídica - pag. 87


tal o cual componente perjudicial para la salud. El punto que se quiere
mostrar claramente es que, cuando el legislador elige prohibir un con-
junto de acciones determinadas, con el fin de favorecer cierto propósito,
siempre existirá la posibilidad de que la acción elegida, lejos de ayudar a
la promoción del valor, contribuya a su debilitamiento. Llamaremos a este
problema la tensión entre la regla y su propósito subyacente.

b) El legislador puede optar por una técnica de legislación diferente


explicitando su propósito, v. gr. diciendo “es obligatorio proteger la
salud”. El problema de esta técnica de legislación es la enorme indeter-
minación a la que da lugar. La utilización de fórmulas tan abiertas genera
una mayor inseguridad sobre el alcance de la norma, el tipo de conduc-
tas que se pretende prohibir, etc. Llamaremos a este método de legisla-
ción, la legislación mediante principios.

Una síntesis de lo trabajado hasta aquí…


Todo término de clase presenta una textura abierta; siempre es posible que
se presenten situaciones que el legislador no ha previsto; es posible que
la acción seleccionada por el legislador genere el efecto contrario al que
se quería favorecer (tensión entre regla y principios subyacentes); puede
optarse por legislar mediante fórmulas más abiertas, pero ello generará en
los jueces y ciudadanos una inseguridad mayor que atenta contra la previsi-
bilidad, siendo éste un valor fundamental del Estado de Derecho.

Dos posiciones contrapuestas frente al problema de la textura abierta del


lenguaje y la indeterminación del derecho: Formalismo vs. Escepticismo
ante las reglas.

Reyna hijo: lo que se dice un optimista-formalista.

Reyna nieto: lo que se dice un anarquista, desfachatado y escéptico.

Aunque hemos mostrado que siempre es posible que una regla obligue a
ejercer una elección adicional por parte de quien la aplica, existen corrientes
teóricas que sostienen lo contrario. Por un lado, están los llamados forma-
listas -dogmáticos- que niegan la necesidad de ejercer elección entre
diferentes cursos de acción. Para esta corriente, lo que hay que hacer viene
absolutamente determinado por el lenguaje y contenido de la norma, y su
aplicación no tiene por qué dar lugar a controversias. En el otro extremo, la
variante escéptica ante las reglas, sostiene que en ningún caso es posi-
ble seguir una regla, ya que los ámbitos de actuación que contiene están
siempre indeterminados, y que lo que hacemos los ciudadanos y los jueces,
cuando aplican una regla, es fingir que siguen una regla, pero en realidad la
decisión acerca de lo que se debe hacer es siempre fruto de una elección.

Una vez más utilizaremos a la familia Reyna para comprender mejor de qué
hablamos. Recordemos que el abuelo había ordenado comprar pasta. Esto
trajo inconvenientes con la textura abierta del término pasta. Luego el abuelo
aclaró que no quería que se compre comida que tuviera carne, y además,
aclaró que su propósito era promover la salud de la familia. Así las cosas, hijo
y nieto Reyna se encuentran en la casa de pastas.

EDUBP | Abogacía | Filosofía y Lógica Jurídica - pag. 88


Para el hijo, lo que debe hacer es bastante simple y razona de la siguiente
manera:
i) está prohibido comprar carne;
ii) sólo hay ravioles de pollo o ñoquis Sojénica
iii) el pollo es una especie de carne;
iv) la soja no es carne;
v) conclusión: deben comprarse ñoquis Sojénica.

El formalista niega el problema de la textura abierta del len-


guaje y, por lo tanto, niega que haya casos de penumbra o
duda sobre la aplicación de la regla.

En primer lugar, es importante reconocer que, aunque esta teoría pretende


negar el problema de la textura abierta del lenguaje y los casos de penum-
bra, parece bastante razonable dudar si en este contexto “carne” significó-
toda carne o sólo carne vacuna.

Pero más allá de esta discusión, lo que hace el hijo de Reyna al razonar como
lo haría un formalista, es negar la suerte de fenómeno que se ha suscitado en
el caso en cuestión. En primer lugar, el abuelo no había previsto semejante
sofisticación de los hechos (pocas opciones de comida, una que contiene
carne, otra que no tiene carne pero puede ser perjudicial para la salud), por
lo que se podría decir que la situación acaecida se parece bastante poco a
la que el abuelo tuvo en mente cuando los mandó a comprar pastas. En este
sentido, resulta algo artificial sostener sin más ni más que este es el tipo de
situación en la que el abuelo estaba pensando al momento de dictar la norma.
En segundo lugar, la visión formalista de Reyna hijo no da cuenta de un con-
flicto que suele darse en la vida cotidiana entre dos valores: por un lado, el
valor que tiene conocer de antemano qué se debe hacer (comprar) porque
ello nos permite organizarnos en la vida, unido al valor que tiene respetar la
voluntad de la persona a quien (por diferentes razones) hemos elegido como
autoridad para regir nuestro destinos; y, por el otro, valores adicionales como
el de realizar conductas que no nos perjudiquen en nuestra salud, nuestra
integridad o nuestra moral.

En términos del propio Hart (1968) el formalista no da cuenta del conflicto


entre el valor certeza (que los abogados llamamos seguridad jurídica)
y la necesidad (también valiosa) de adecuar las decisiones al caso en
concreto (valor justicia en el caso concreto, razonabilidad de la deci-
sión, etc.). Siempre es posible que ambos valores (la certeza y la adecua-
ción al caso concreto), se encuentren en conflicto. Y hacer de cuenta que tal
conflicto no existe, presupone que los seres humanos somos algo así como
autómatas no racionales. El caso de la familia Reyna que usamos para com-
prender estos problemas, por lo liviano, tal vez oculte la real gravedad del
conflicto en cuestión y las consecuencias que acarrean las teorías que (como
la formalista) actúan como si dicho conflicto no existiera. El real problema
de la propuesta formalista puede verse más claro si reemplazamos la duda
sobre la calidad de los alimentos transgénicos por la certeza de que dicho
alimento nos generará algún malestar o malformación. Si fuésemos forma-

EDUBP | Abogacía | Filosofía y Lógica Jurídica - pag. 89


listas coherentes, deberíamos llevar nuestra concepción hasta las últimas
consecuencias, comprando la comida que nos puede perjudicar por el sólo
hecho de que ella puede ser subsumida en el concepto de pasta contenido
en la orden del abuelo, priorizando sin más el valor certeza por sobre el valor
adecuación al caso concreto.

Las aplicaciones formalistas del derecho, por un lado, no toman en cuenta


el problema estructural del lenguaje (su vaguedad potencial o textura
abierta) y, por el otro, el problema de aplicación de una regla a un caso
concreto. Siempre es posible que la aplicación de la regla a un caso concreto
implique una solución que está completamente en contra del propósito que la
ley o regla quería promover. En el caso de Reyna, la aplicación formalista de
la regla dictada por el abuelo, podía llegar a un resultado totalmente opuesto
al propósito que la regla quería promover (el de la salud de los familiares).
Análogamente existen, en el ámbito de la teoría jurídica, visiones que pre-
tenden sostener que el juez debe aplicar ciegamente la ley, operando por
medio de un razonamiento subsuntivo. Sin embargo, esa pretensión no sólo
es falsa, sino indeseable. Es falsa porque obvia el vicio de la textura abierta
de los lenguajes naturales en el que se expresan las normas; y resulta inde-
seable porque impediría resolver casos en que el contexto sorprende de una
manera tal que, al aplicar la regla mecánicamente, se puede perjudicar o
violentar el propósito o fin que se quería promover.

En definitiva, una visión formalista del derecho niega el carácter necesario


(para los casos de textura abierta) y deseable (para el caso de conflictos
entre propósito y resultado) del ejercicio adicional de elección.

Veamos ahora la actitud que adopta el nieto de Reyna, el que posee una
actitud contestataria, anarquista y escéptica.

Para este sucesor del abuelo, los problemas de textura abierta y de tensión
entre adecuación y certeza demuestran que siempre es posible hacer con
las reglas lo que uno quiere, o lo que es lo mismo, no hay regla alguna, ya
que siempre se puede invocar alguna circunstancia que justifica no tener
en cuenta la orden del abuelo. El nieto de los Reyna observa que la textura
abierta de la regla del abuelo, generalmente le permite a él y a su padre com-
prar pastas que contienen carne y, en ese sentido, es como si la regla del
abuelo no existiera. Esta concepción sostiene que, si bien existen las reglas,
siempre es posible violarlas amparándose en alguna argucia del lenguaje, lo
cual termina equiparando la situación al caso en que no hay regla alguna.
Si por definición, tener una regla es hacer lo que otro quiere y no lo que uno
quiere, y el nieto de Reyna tiene la percepción de que siempre puede com-
prar lo que quiere (amparándose en lo dudoso de la palabra carne) enton-
ces, a sus ojos, tener una regla equivale a no tenerla.

El escéptico extremo ante las reglas, amparándose en la tex-


tura abierta del lenguaje y la indeterminación de las reglas,
niega que haya reglas que restrinjan la decisión del órgano
aplicación –en especial los jueces o tribunales. En definitiva,
piensan que el derecho no es otra cosa que el producto del
arbitrio del tribunal.

EDUBP | Abogacía | Filosofía y Lógica Jurídica - pag. 90


Esta forma extrema de escepticismo ante las reglas, centra su posición
fundamentalmente en el problema de la textura abierta del lenguaje. Sin
embargo, parece bastante sensato pensar (Hart: 1968) que, aunque existen
casos que pueden ser dudosos (como el del canelón de carne y verdura).
Todos coincidiríamos en que un kilo de jamón crudo no cuenta como un
caso de pasta y el abuelo estaría bien justificado en rechazarlo si el nieto pre-
tendiera hacerlo pasar por pasta amparándose en la textura abierta del len-
guaje. Este ejemplo muestra que, aunque un término pueda tener algunos
casos de penumbra, en los cuales no sabemos si realmente se aplica a
un determinado objeto, de ello no se sigue que para todo caso pueda
aplicarse cualquier cosa. La reacción del abuelo de rechazar el jamón
crudo al nieto, muestra que todo término tiene casos paradigmáticos de
aplicación. Un raviol es claramente pasta y una vaca no es un caso de pasta.

Si cualquier término pudiera ser utilizado para cualquier caso, no sólo no sería
posible el derecho concebido como técnica de motivación de conductas,
sino que tampoco sería posible la comunicación entre los seres humanos
en cualquier ámbito. Tener un código de comunicación es tener instancias
paradigmáticas de comunicación compartidas, donde cuando se diga la
palabra pasta nadie piense en una usina atómica, aunque se pueda quizá
pensar en ñoquis cuando yo estaba pensando en ravioles.

En el ámbito jurídico, esto muestra que, aunque es posible que en algunos


casos los jueces tengan cierta libertad o discreción con respecto a qué cuenta
como impuesto, ganancia, lesiones culposas, de ello no se sigue que tengan
total libertad. Es posible, para un número grande de casos, identificar cla-
ramente cuándo están aplicando la ley y cuándo la están desobedeciendo.

Hart (1968) adopta frente al problema de textura abierta e inde-


terminación del derecho una posición intermedia: aunque la
regla general puede tener algunos casos de penumbra de
significado, hay un gran número de casos que constituyen
instancias claras o paradigmáticas de aplicación de la regla.

Existe una variante interesante del escepticismo del nieto Reyna, que habría
que tomársela en serio y que consiste en lo siguiente: dado que el abuelo les
ordena comprar pasta y deja librado a su decisión qué cuenta como pasta,
el contenido de la orden del abuelo (lo que ellos deben hacer) depende en
última instancia de lo que él o su padre deciden al momento de llegar a la casa
de pastas. En ese sentido, sostiene esta variante que lo que el abuelo decide o
lo que el abuelo ordena es, en última instancia, lo que el hijo o el nieto Reyna
deciden qué debe ser. No hay tal cosa como decisión del abuelo, sino que
lo que se debe comer o no comer el domingo depende de quién decide en
última instancia y esa actividad no es ejercida por el abuelo, sino por él o su
padre. En otras palabras, tiene buen sentido decir que, dado que el contenido
de lo que dijo el abuelo queda librado a lo que decida hijo o nieto (pense-
mos en una decisión judicial de última instancia), lo que se debe hacer es lo
que decide el que toma esta última decisión (sea lo que fuere que decida).
Trasladado el ejemplo al ámbito del derecho sería equivalente a afirmar que lo
que se debe hacer u omitir depende no del legislador, sino de lo que dis-
ponga una decisión judicial de última instancia, esto es, definitiva o final.

EDUBP | Abogacía | Filosofía y Lógica Jurídica - pag. 91


Esto último no es más que una ejemplificación de lo que ya hemos analizado
someramente en los contenidos del módulo 1(Hart: 1968; cap.1) como la
concepción realista del derecho, según la cual (en una de sus muchas
variantes) el derecho es lo que los jueces dicen que es.

Otra forma de expresión del escepticismo es sostener la tesis


según la cual lo que los jueces dicen que es derecho cuando
la decisión es final e irrevisable, es lo que cuenta como dere-
cho en cuanto a sus efectos prácticos.

Pero si bien algunos argumentos del escepticismo ante las reglas resultan
atractivos, tengamos en cuenta que, si el realismo tuviera razón, en el caso
de los Reyna quedaría sin explicar una parte central del fenómeno o práctica
familiar. Si es cierto que las reglas son lo que hijo o nietos deciden que son
¿por qué razón son ellos los habilitados o facultados a decidir qué pasta traer
los domingos?, ¿por qué razón se consideran como válidas sus elecciones
y no las que quisiera hacer María?, ¿qué es lo que justifica que toda la fami-
lia los vea a ellos como quienes pueden elegir la pasta del domingo y no
María? Claramente, la condición de ser los facultados para elegir la comida
los domingos debe estar basada en una regla que diga “los facultados para
ir a la casa de pastas son Reyna hijo y nieto” y esa regla (que los constituye
a ellos en decisores) no es la que ellos mismos dictan cuando deciden qué
comer. Aunque el contenido de qué comer sí depende de ellos, no depende
de ellos el resto de las reglas familiares, ni mucho menos las que les otorga
la potestad de decisión.

Análogamente, en un sentido, parte del derecho puede depender de lo


que determinados jueces dicen que es, pero de ello no se sigue que no
exista ninguna regla o que todo el derecho sea lo que ellos dicen que es,
porque para que sus resoluciones sean vistas como revestidas de autori-
dad, debe existir al menos una regla que los designe jueces y le otorgue
a sus veredictos valor de autoridad. Y esa regla que les otorga autoridad
no puede ser dictada por ellos mismos, porque en ese caso ¿qué auto-
ridad tendrían para dictar una regla que dice que ellos tienen autoridad?

Lo que el caso de Reyna pretende mostrar en este punto, es que la familia


ve de diferente forma las decisiones del hijo y nieto de Reyna con respecto
a la pasta, de la forma en que ve a la decisión que tomaría María. Hijo y
nieto tienen autoridad para decidir en ese punto, mientras que María no. Del
mismo modo, en el Estado de Derecho, el conjunto de reglas que otor-
gan o reconocen autoridad a determinado cuerpo de funcionarios, nos
permite distinguir la decisión de un particular parado en un banquito de
la plaza de la decisión de un juez o una legislatura. Ambos dicen lo mismo
(p. ej. prohibido envenenar los ríos), pero las consecuencias que se siguen
de lo que dicen y la particular forma en que los individuos las observan difie-
ren radicalmente.

También es interesante tener en cuenta que del hecho que lo que diga una
autoridad de última instancia sea definitivo (en el sentido que debe ser
lo que ésta decida), no se sigue que su decisión sea infalible. Existe una
tendencia a pensar que por el sólo hecho de que una decisión es la definitiva,

EDUBP | Abogacía | Filosofía y Lógica Jurídica - pag. 92


es por ello la correcta, sustentado particularmente en la cuestión de que en
última instancia, por más errónea que la decisión haya sido, nuestra situación
no variará. En el caso del nieto y el jamón crudo, uno podría sostener que
lo que hay que hacer (lo que se debe hacer) es en última instancia comer el
jamón crudo, y debido a que no hay posibilidad práctica de volver atrás (ya
no hay posibilidad de volver a comprar pasta), la decisión última del nieto
equivale a la decisión correcta. Pero, como muestra claramente el caso, esto
no es así. Nadie puede discutir que, en un sentido importante, la decisión
última del nieto sobre la comida no puede ser revertida y, por lo tanto, nues-
tra vida no cambiará sustancialmente aunque sostengamos que lo correcto
hubiese sido que comprara pastas. Pero:

Del hecho de que una decisión sea final no se sigue que sea
correcta. Hay un criterio externo a la propia decisión para
evaluar su corrección y, ese criterio nos viene dado por el
núcleo de certeza o significado claro de la regla general.

En el derecho ocurre lo mismo. Aunque una resolución de la Corte Suprema


sea última e irrecurrible (y, en ese sentido, es la Corte la que determina el
contenido de lo que se debe hacer), no es necesariamente infalible. Tenemos
criterios para decidir si la Corte se ha equivocado (como para el caso de la
compra de jamón crudo). Uno de los hechos que nos inclina a pensar que
lo que la Corte dice es derecho, o a sostener que la Corte no se puede equi-
vocar, es la idea según la cual aunque sea cierto que se equivocó, nuestro
mundo no cambiará, pues si fuimos condenados a pagar, tendremos que
pagar, y si fuimos absueltos, podremos ampararnos en la cosa juzgada. Sin
embargo, nadie ve como un sinsentido la expresión “la Corte aplicó mal el
rebalanceo tarifario” o “la Corte se equivocó revocando la condena de Juan
Pérez”. Ello es así porque tenemos criterios independientes de la decisión
de la Corte para evaluar lo que ella ha fallado. Y los criterios para evaluar
la corrección o incorrección de una decisión judicial (aunque ésta fuese
de última instancia), no son otros que lo que efectivamente dijo el abuelo
para el caso de la familia Reyna y lo que efectivamente dijo el legislador
para el caso del derecho; o sea la existencia de reglas.

Por último, el nieto Reyna, persistiendo en su argumentación escéptica,


podría sostener que la existencia de la regla atinente a las pastas, para
lo único que sirve es para predecir, con cierto grado de certeza, que
cada domingo probablemente en su casa se comerán pastas y no carne.
Sin embargo, no parece que esta idea reconstruya la particular forma en que
los participantes de la familia (o de un sistema jurídico) perciben sus propias
prácticas. Los integrantes de la familia no sólo ven la regla del abuelo como
una predicción de lo que sucederá el domingo, sino que desarrollan hacia
ella una actitud de aceptación, están dispuestos a ir a la casa de pastas, criti-
car si se desoye la directiva de no comprar carne, etc. No sólo hay en la regla
que dicta el abuelo un juicio de probabilidad, sino que existe la expresión
de toda una dimensión interna del sujeto o los sujetos hacia un deter-
minado tipo de conducta. Que el abuelo haya dicho lo que dijo no es una
predicción, sino una buena razón para hacer lo que dijo y criticar a quien
se desvió de lo que dijo.

EDUBP | Abogacía | Filosofía y Lógica Jurídica - pag. 93


Todavía existe una última variante de escepticismo que sostiene que, en
realidad, los sujetos (jueces) encubren sus decisiones discrecionales
atrás de una regla. Según esta concepción, los jueces no decidirían en
base a reglas, sino en base a instinto, deseos, pareceres, sentires. Una
vez tomada la decisión (sin tener en cuenta regla alguna) elegirían una norma
para camuflar sus decisiones, de modo que parezca que han decidido de
acuerdo a reglas.

Otra variante de escepticismo está constituida por una tesis


“moderada” que admite que hay reglas, pero que ellas sólo
son fuente de derecho y no derecho. En otros términos, el
derecho es lo que los jueces dicen que es pero, usan reglas
–del mismo modo que apelan a la doctrina o jurisprudencia
–para justificar o dar razones de su decisión.

Con relación a esta variante, conviene distinguir entre la cuestión fáctica


y la justificatoria, ya que es perfectamente factible que el juez, al momento
de dictar sentencia, decida de acuerdo a la intuición, tirando la moneda o
consultando el horóscopo; también es posible que luego de haber decidido
mediante este método, busque una norma con la cual intentar encubrir el
método de decisión. Sin embargo, aunque ello ocurra de hecho, no quiere
decir que no se cuenten con mecanismos para controlar si la decisión judicial
está justificada o no en derecho. Y en realidad, lo que interesa es la justifica-
ción. Para una mejor explicación de ello, recordemos la noción de razón ya
explicitada anteriormente: algo puede contar como una razón en un razona-
miento práctico, si es posible utilizar razón para justificar un razonamiento.
En determinados contextos, como el jurídico, no es tan importante saber
cómo llegó, de hecho, el juez a su veredicto (si tirando la moneda o pregun-
tándole su opinión a una amiga por teléfono), sino lo que interesa es saber
si la decisión que ha tomado mediante ese u otro procedimiento, está
apoyada o puede justificarse de acuerdo a razones jurídicas. El momento
y la forma en que el juez toma la decisión de condenar a alguien, es irre-
levante a los fines jurídicos. Él puede decidir incluso mirando las estrellas,
o mientras canta en la ducha, pero lo relevante en derecho, para saber si
el juez ha actuado dentro de los márgenes de la ley, será corroborar si su
decisión puede sustentarse en alguna razón o norma jurídica. El derecho no
le exige al juez que su veredicto sea tomado experimentando determinadas
sensaciones o procesos mentales, lo único que el derecho exige es que la
solución jurídica dada a un caso se presente justificada, esto es, apoyada en
una norma. Así, si un juez decide condenar a Juan tirando la moneda y luego
trata de encubrir su forma de decidir citando prueba que no es concluyente,
testigos dubitativos o documentación apócrifa, su resolución carecerá de jus-
tificación jurídica y tendremos un método para corroborar si la resolución es
legítima o no, con independencia del proceso mental del juez. Si no es posi-
ble reconstruir una decisión judicial como apoyada por normas, la resolución
podrá ser revocada; pero si es posible hacerlo (aunque el juez llegue a su
decisión por métodos mágicos) será suficiente para sostener que la sentencia
es válida. Es por ello que el control que se ejerce de las resoluciones judicia-
les no incluye preguntarle al juez la forma o modo en que llegó a pensar lo que
pensó, sino qué razones de hecho y de derecho justifican lo que dijo.

EDUBP | Abogacía | Filosofía y Lógica Jurídica - pag. 94


Incertidumbre o textura abierta de la Regla de Reconocimiento
Como vimos hasta ahora la textura abierta es un problema que afecta a la
determinación del significado y alcance –aplicabilidad– de una regla válida
del sistema jurídico. Pero, el problema de la textura abierta tiene un alcance
aún más general que afecta a la propia regla de reconocimiento. En efecto,
sabemos que la regla de reconocimiento es el medio de identificación de
las reglas de un sistema jurídico, pues indica las condiciones de validez,
pertenencia o existencia de dichas reglas. El problema es que en ciertos
casos puede haber certeza respecto de la aplicación de una regla a un caso
concreto, pero puede dudarse de la validez o pertenencia de esa regla al
sistema jurídico.

La textura abierta también puede generar incertidumbre respecto de la


regla de reconocimiento y, por ende, del criterio último usado por los
tribunales al identificar reglas válidas de derecho.

Dicho de otro modo, hay casos en los que la duda de un juez no versa sobre
la aplicación o no aplicación de una regla particular, sino sobre la validez de
una regla. La aplicación de una ley en un caso particular puede ser obvia, sin
embargo, puede ocurrir que haya dudas sobre si la legislatura tenía poder
para legislar de ese modo. Cuando la duda versa sobre la competencia
jurídica de la propia legislatura suprema, la cuestión se refiere a los límites
y alcance del poder soberano. En tales casos –casos que versan sobre la
constitucionalidad de las leyes– el tribunal debe resolver qué ha de enten-
derse por soberanía. Como quedó reflejado en la controversia planteada
entre Austin y Hart en torno al concepto de soberanía (ver contenidos de los
módulos 2 y 3 y Hart (1968) capítulo 4, última parte), el término “soberanía”
es ambiguo. El poder soberano puede ser interpretado como una ilimitada
omnipotencia auto - comprensiva, o como una omnipotencia continuada,
según la cual, los límites al poder del soberano actual están dados por la
preservación de igual poder para los futuros soberanos, de modo que el
Parlamento no puede, por ejemplo, poner sus leyes a cubierto de la deroga-
ción pues ello implicaría limitar el poder para los soberanos futuros. En tales
casos de incertidumbre los tribunales tienen que decidir el contenido mismo
de la regla última por la cual se identifica el derecho válido. Puede parecer
paradójico que la constitución confiera poder discrecional a las Cortes para
determinar lo que establece la constitución, pero la paradoja se disuelve
cuando se observa que, en una amplia área de casos, los criterios de validez
no originan dudas, aunque sí puedan surgir casos de incertidumbre sobre su
dominio preciso y su alcance (Hart: 1968; cap.7).

Con el fin de complementar el estudio de los temas presentados, acceda


a la siguiente bibliografía: Hart, H. L. A.; El Concepto de Derecho. Bs. As.,
Abeledo Perrot, 1968, capítulo 7.

EDUBP | Abogacía | Filosofía y Lógica Jurídica - pag. 95


Si usted está interesado en profundizar sobre estos temas puede consultar:

• Nino, Carlos Santiago; Introducción al Análisis del Derecho. Buenos


Aires. Ed. Astrea, 2ª edición ampliada y revisada, 1992, capítulo 5

Para profundizar en la historia de las escuelas formalistas y escépticas en el


pensamiento jurídico norteamericano ver:

• Hernandez, José Lopéz. El formalismo en la teoría jurídica estadou-


nidense Anuario de filosofía del derecho, ISSN 0518-0872, Nº 18,
2001, págs. 267-300. Recuperado en: https://dialnet.unirioja.es/servlet/
articulo?codigo=257666

Lo invitamos a realizar las actividades del módulo 4, como forma de transferir


los conocimientos a situaciones concretas.

Al finalizar el estudio del módulo 4, usted está en condiciones de realizar las


consignas de la Primera Parte de la evaluación parcial.

M4 Actividades
Actividad 1
Analizando un caso controvertido
Hace algunos días, Usted ha recibido en su estudio jurídico la última publica-
ción de la revista jurídica Serás lo que debas ser y sino serás abogado. Allí,
entre ofertas de códigos, libros y cursos de capacitación, se encuentra la
publicación del Caso “Saguir y Dib”. El mismo tiene las siguientes particula-
ridades: Juan Isaac Saguir y Dib padecía insuficiencia renal y necesitaba con
urgencia un transplante de riñón.

Su hermana, de diecisiete años, era la única persona que podía brindar el


órgano. De acuerdo a la ley de transplantes vigente en ese momento (21.541),
para poder donar órganos se debía cumplir con dos requisitos básicos: tener
un cierto vínculo sanguíneo (lo cual era satisfecho por la hermana), y deten-
tar más de 18 años de edad. La hermana no llenaba este último requisito.

Al pedir la autorización de la donación de órganos ante el órgano jurisdic-


cional, éste denegó el transplante. Su decisión sostenía que hasta que no
se cumpliese con la edad prevista por la ley 21.541, no podía autorizarse
la donación. Igual posición adoptó la Asesora de Menores. La familia recu-
rrió la resolución de primera instancia y la Cámara Nacional de Apelaciones
también denegó la autorización, basándose en las mismas razones de la
primera instancia. La familia interpuso un recurso extraordinario, en virtud
del cual llegó a la Corte Suprema. Allí existieron dos posturas. Por un lado, el

EDUBP | Abogacía | Filosofía y Lógica Jurídica - pag. 96


Procurador de La Nación emitió un dictamen en contra de la ablación, apo-
yándose en los mismos argumentos de primera y segunda instancia. Por el
otro, la Corte, autorizó la donación sosteniendo:
I) que la solución de la norma era notoriamente disvaliosa e injusta;
II) que no puede optarse en estos casos por una interpretación meramente
teórica, literal y rígida de la ley;
III) que al momento de solucionar un caso en particular, hay que tener en cuenta
otros principios jurídicos como los contenidos en la Constitución Nacional;

Desde esta perspectiva, Ud. se propone tratar de encontrar respuesta ante


el interrogante mediante el que se indaga si la regla de reconocimiento que
están utilizando la Corte Suprema, por un lado y el Procurador y Juzgados
inferiores, por el otro, es la misma o difieren en algún aspecto.
AA

AA Actividad 1
asistente académico

Para la realización de la presente actividad, sería conveniente que tuviese en


cuenta las formas de analizar la reglas de reconocimiento, expuestas en los
contenidos del módulo y en el propio libro de Hart: 1968, capítulo 7, primera
parte, críticas al formalismo o conceptualismo. De allí, Ud. podrá colegir qué
hechos hay que tener en cuenta para poder descubrir qué regla de recono-
cimiento está utilizando un determinado grupo social.

Actividad 2

“Una oportunidad para elaborar argumentos en contra del escepticismo


radical”
Usted camina tranquilamente un martes de abril por los pasillos de Tribunales.
Se encuentra allí con un amigo que fue alguna vez compañero de estudio y
que se dedica al derecho civil. Su ex-compañero de estudio, ahora abogado,
de nombre Juan Mufoni, suele desarrollar una actitud de crítica hacia las
resoluciones judiciales en general. Particularmente, lo acaban de notificar
de una resolución en la que el Juez, no teniendo en cuenta un artículo del
código civil, condenó al defendido de Mufoni a pagar una suma importante
de dinero. Según Mufoni, de haber aplicado el Juez lo que dijo el legislador
ordinario, su defendido habría salido airoso. Mufoni sostiene que esto es una
muestra clara de que en realidad lo que dice el legislador no vale para nada,
que en realidad no hay tal cosa como un sistema jurídico estable en el que
los jueces deben respetar el mandato del legislador, que todo es un gran
“corso jurídico”. En fin, sostiene que dada la ineficacia (en esta causa en par-
ticular) de la regla que obliga a los jueces a obedecer al poder legislativo, es
posible sostener que dicha regla es inválida y, además, que no hay tal cosa
como sistema jurídico.

EDUBP | Abogacía | Filosofía y Lógica Jurídica - pag. 97


Teniendo en cuenta el marco conceptual desarrollado en los contenidos
del presente módulo y los argumentos a favor de una posición ecléctica o
intermedia esgrimidos por Hart (1968) capítulo 7, ensaye una respuesta en
contra de Mufoni.

Esta deberá dirigirse a mostrar que los hechos que él invoca no son sufi-
cientes para mostrar ni que la regla en cuestión es inválida, ni que el sistema
jurídico es una ficción.

Actividad 3

Determinando casos de textura abierta en la aplicación de una regla.


Usted ha obtenido una pasantía rentada de 6 meses en la Legislatura Provincial.
Allí se está discutiendo un nuevo código de tránsito para la Provincia.

En uno de los artículos, se ha incluido la prohibición de estacionar vehículos


sobre la vereda. El artículo en cuestión incorpora el siguiente texto: “se impon-
drá multa de 100 a 1000 pesos a quien estacione su vehículo sobre la vereda”.

Ud. deberá imaginar casos difíciles o de textura abierta (penumbra) a los que
puede dar lugar la norma referida. En especial, en cuanto a los términos de
clase “vehículo” y “vereda”.

Redacte, al menos, dos ejemplos de casos difíciles respecto de la aplicación


de la norma mencionada, cuya dificultad radique en la textura abierta de
cada uno de estos términos “vehículo” y “vereda”.
AA

AA Actividad 3
asistente académico

Para sacar mayor provecho a la resolución de la presente actividad, le sugeri-


mos consulte la primera parte de los contenidos de este módulo y el capítulo
7, primera parte, del texto de Hart: 1968.

EDUBP | Abogacía | Filosofía y Lógica Jurídica - pag. 98


MÓDULO 5

M5 Microobjetivos

• Comprender las diferencias conceptuales entre lo que se suele llamar el


terreno de la moral y lo que se refiere al derecho, a los fines de contar
con herramientas teóricas para dividir cuestiones de la ciencia jurídica,
de cuestiones de otros ámbitos teóricos.
• Conocer y comprender la diferencia entre relaciones conceptuales y rela-
ciones contingentes entre el derecho y la moral, con el fin de distinguir
cuándo la moral es relevante para el derecho debido al devenir histórico
y cuándo lo es en virtud del contenido o significado mismo de la palabra
”derecho”.

M5 Contenidos

Acceda al video de presentación del Módulo 5

Lo bueno y lo correcto. Una distinción entre Sistemas Jurídicos y su valor.

Introducción
A simple vista, parecería ser que lo que caracteriza a una determinada
sociedad tiene que ver con las normas que regulan el comportamiento de
sus miembros y a las que éstos muestran su adhesión en mayor o menor
extensión. No obstante, las sociedades están regidas por distintos tipos de
normas, de modo que es importante saber cómo podemos diferenciar a las
normas jurídicas del resto de normas. En su caracterización del derecho
como técnica social, Hart incluyó elementos que no pueden extraerse de
una explicación del fenómeno jurídico, dada exclusivamente en términos de
relaciones de dominación o fuerza.

Una noción particularmente importante, incorporada por el autor, es la del


punto de vista interno de las reglas, el cual se manifiesta en el uso de las
mismas como pautas de conducta orientadoras y críticas. Así también, incor-
pora la noción de reglas secundarias de reconocimiento, cambio y adju-
dicación (Tal como usted recordará, en el módulo 3 han comenzado ya a
desarrollarse muchas de estas cuestiones).

Dichas reglas secundarias permiten individualizar las normas jurídicas, dis-


tinguirlas de las demás normas por su fuente u origen y, además, porque la
aplicación de sus sanciones está institucionalizada. De tal modo, podemos
distinguir la norma jurídica que nos prohíbe robar, por ejemplo, de la norma
que nos impone el deber de saludar a un vecino o de aquella que nos obliga
a prestar ayuda.

EDUBP | Abogacía | Filosofía y Lógica Jurídica - pag. 99


Ahora bien, el derecho como técnica de control social de la conducta ¿es
necesario? y, si lo es, ¿para qué fin o propósito es necesario? Como una
cuestión de hecho puede decirse que siempre que ha existido una sociedad,
ha existido derecho. De ello puede inferirse que el derecho cumple una serie
de funciones que toda sociedad bien ordenada requiere.

Por supuesto, el rol social que se otorgue al derecho depende de nuestra


consideración de cómo debe ser una sociedad bien ordenada y ésta es una
cuestión valorativa. Al respecto Hart sostiene que la existencia de la sociedad
y el derecho surgen de las limitaciones o imperfecciones de la naturaleza
humana, y del propósito que la mayoría de los hombres tienen de sobrevivir.
La importancia que la supervivencia tiene, para la mayoría de los hombres,
se refleja, según el autor, en estructuras completas de pensamiento y len-
guaje, en los modos que los hombres tienen de describir y valorar conduc-
tas, objetos y eventos del mundo, por su contribución a la supervivencia. Al
ser la supervivencia una meta o propósito común de los seres humanos, el
derecho, como medio de garantizar esa meta, tendrá un rol compartido por
las distintas sociedades. Al referirse a la moral y al derecho Hart afirma: “nos
ocupamos de medidas sociales para la existencia continuada, no de reglas
para un club de suicidas” (Hart, 1961:238). De tal modo cuando se discute
el modo adecuado de regular la conducta social hay que tener presente que
sus miembros, en términos generales, tienen el propósito de convivir.

Si la asociación de los hombres en torno a reglas morales y jurídicas -socie-


dad- es un medio para lograr el propósito de la supervivencia, entonces
hay ciertas normas -base normativa común- que cualquier sociedad debe
contener para ser viable. A esa base normativa común Hart la denomina
“contenidos mínimos de derecho natural”, e incluye prohibiciones al libre
uso de la violencia contra personas o cosas, la obligación de cumplir las
promesas o la palabra empeñada, el deber de no mentir y de ser hones-
tos –presupuestos básicos del derecho criminal y contractual. Aquí resulta
interesante preguntarnos: ¿por qué se imponen tales contenidos normativos
mínimos? Precisamente por la conexión racional que los mismos tienen con
las mencionadas limitaciones o imperfecciones características de la natura-
leza humana. En efecto, los hombres por naturaleza:
1. son vulnerables a las agresiones, cualquier hombre puede dañar o ser
dañado por otro;
2. son aproximadamente iguales, de modo que, ninguno de ellos puede
dominar al resto si no cuenta con algún tipo de cooperación.
3. poseen un altruismo limitado, no son ni ángeles ni demonios. De 1, 2 y
3 ya se infiere la necesidad de normas que limiten sus acciones. Si fueran
ángeles las reglas de conducta serían superfluas y, si fueran demonios,
serían ineficaces.
4. tienen recursos limitados para cubrir las necesidades básicas que la
supervivencia requiere y, la obtención de esos recursos demanda la
intervención del hombre en la naturaleza. Esta circunstancia impone
alguna forma de institución de propiedad, de reglas que distribuyan los
recursos, regulen su uso, intercambio y disfrute, creando en relación a
ellos derechos y obligaciones. Esta circunstancia hace necesaria la coo-
peración entre los hombres y la existencia de normas que aseguren el
reconocimiento de la promesa como fuente de obligaciones.

EDUBP | Abogacía | Filosofía y Lógica Jurídica - pag.100


5. tienen comprensión y fuerza de voluntad limitadas. No todos los hom-
bres entienden del mismo modo sus intereses a largo plazo, ni poseen
la voluntad suficiente para sacrificar sus intereses inmediatos en pos de
mejores beneficios a largo plazo. Ello impone la necesidad de normas que
prohíban o restrinjan ciertas acciones e impongan sanciones a quienes no
las cumplan voluntariamente. Estas normas también garantizan la necesa-
ria cooperación entre los hombres contra los gorrones o free riders, quie-
nes pretenden sacar ventajas de los beneficios de la cooperación de los
demás, sin cumplir con las cargas u obligaciones que les corresponden.

Tales razones, en consecuencia, justifican para Hart la existencia de la socie-


dad, su moral y su derecho.

Tomando en consideración la meta o el fin de la supervivencia como criterio


evaluativo de lo que ha de considerarse una estructura social bien ordenada
con su moral y su derecho, cabe distinguir dos tipos de moral: la moral posi-
tiva o conjunto de estándares o criterios que los miembros de un grupo social
creen que son los correctos para evaluar los comportamientos e instituciones
humanas; de la moral crítica o conjunto de criterios o estándares objetivos
aptos para evaluar la corrección de las acciones o instituciones humanas.

Para la moral crítica, toda estructura social, incluida su moral aceptada y su


derecho, tiene que satisfacer dos condiciones formales:
1. Racionalidad: las estructuras sociales no deben descansar en creencias
cuyo error pueda demostrarse, por ejemplo, sería erróneo y carente de
sustento racional creer que las aptitudes de las personas para votar o rea-
lizar contratos dependen de su sexo o color de piel. No sería erróneo, en
cambio, considerar que los insanos o menores de edad carecen de apti-
tudes para votar o contratar, en relación a las que posee un adulto normal.
2. Generalidad: las protecciones frente al daño que la moral y el derecho
acuerdan a través de las acciones y omisiones que requieren, deben
extenderse por igual a todos los hombres que puedan y quieran aceptar
tales restricciones. La protección social requerida para el aseguramiento
de la supervivencia debe extenderse a todos los miembros que volunta-
riamente cumplan con las cargas y obligaciones que imponga la coope-
ración o convivencia social.

La justicia como un segmento de la moral


Los términos más empleados para ponderar o reprobar el derecho o su
administración son los términos “justo” o “injusto”. La palabra “justicia” tiene
un ámbito de aplicación más restringido que la palabra “moral”. En efecto,
usamos conceptos morales cuando evaluamos conductas aisladas y, por
ejemplo, decimos que “es malo mentir o robar” y, “es bueno prestar ayuda
al hambriento o al enfermo”; sin embargo, no sería apropiado referirse a la
calificación moral de esas conductas en términos de justicia. Hablamos de
“justicia” más específicamente para evaluar dos situaciones típicas:

EDUBP | Abogacía | Filosofía y Lógica Jurídica - pag.101


a. Situaciones en que juzgamos la distribución de cargas o beneficios
dentro de una clase o grupo de individuos –justicia distributiva.
b. Situaciones en las que se ha causado algún daño y se evalúa moral-
mente la compensación o indemnización reclamada por la/s víctima/s
–justicia compensatoria.

En la primera de las situaciones mencionadas, la idea de justicia se corres-


ponde con la de equidad distributiva expresada en la fórmula: “tratar los
casos semejantes de modo semejante y los diferentes de modo diferente”.
Esta idea de justicia puede implementarse en la evaluación de dos aspec-
tos del derecho:

i. la aplicación del derecho o administración de justicia. En tal sentido


se dirá que la aplicación del derecho es justa cuando casos semejantes
son resueltos de modo semejante. En tales casos, el criterio para decidir
qué casos son semejantes y qué casos no lo son es proporcionado por
el propio derecho positivo. Por ejemplo, si una norma jurídica prohíbe
a las mujeres votar, entonces una aplicación justa de esa norma debe
prohibir el voto a todas las mujeres por igual. Resulta evidente que una
evaluación en términos de la justicia restringida sólo a este ámbito del
derecho sería insuficiente, pues, nuestra intuición común indica que el
caso mencionado sería un caso de aplicación justa de una ley que es, en
sí misma, injusta.

ii. Resulta importante, en consecuencia, emplear el principio de justicia dis-


tributiva para evaluar también los contenidos del propio derecho. El
problema que surge en este punto es el de determinar un criterio que
nos indique qué semejanzas y qué diferencias entre los individuos son
relevantes, o bien, en función de qué fines o propósitos puede justifi-
carse racionalmente que los individuos sean tratados de modo desigual.
Tal criterio, aplicable a la evaluación en términos de justicia de las pro-
pias normas jurídicas, debe ser extraído de la moral. Ahora bien, hemos
distinguido dos tipos de moralidad: social o convencional y crítica. En
cuanto a la primera, podemos decir que las creencias vigentes en una
sociedad sobre lo que es justo o no lo es pueden ser erradas –pensemos
en sociedades que distinguen a los individuos por su raza, religión o por
el color de su piel. En consecuencia, Hart considera que el criterio para
determinar cuándo un trato desigual o discriminatorio es justo y cuándo
no lo es debe ser extraído de la moral crítica. Conforme a las condicio-
nes de racionalidad y generalidad del juicio moral bien informado, todo
trato desigual que descanse sobre creencias que no pueden ser justifi-
cadas racionalmente constituye una discriminación injusta. De tal modo,
sería injusto hacer depender la capacidad jurídica para efectuar contra-
tos válidos del sexo, religión o raza, pero no consideramos injusto que
se prohíba, por ejemplo, a los menores o insanos contratar, pues puede
probarse que sus capacidades o aptitudes para contratar son diferentes
a las de un adulto normal. En estos casos pensamos que el trato desigual
redunda en beneficio o protección del menor o del insano.

EDUBP | Abogacía | Filosofía y Lógica Jurídica - pag.102


Las situaciones de compensación o indemnización por daños que se esti-
man injustas son aquellas en que las normas jurídicas no confieren ese
derecho a nadie, en casos de daño que la moral considera deberían ser
compensados. Los casos de indemnizaciones por alguna clase de daño que
el derecho acuerda a algunos individuos y a otros no, constituyen casos de
justicia distributiva, y no casos de justicia compensatoria.

Obligación moral y jurídica


Como hemos sostenido anteriormente, la justicia constituye una parte de la
moral que no se aplica al juicio evaluativo de la conducta individual, sino al
modo en que son tratadas clases de individuos. Pero las normas jurídicas
también pueden prescribir como obligatorias acciones particulares que la
moral prohíbe, o exigir abstenciones de acciones consideradas moralmente
obligatorias y, en tal sentido, las normas pueden ser calificadas de moral-
mente malas. En consecuencia, resulta pertinente caracterizar las obligacio-
nes impuestas por normas morales y, distinguirlas conceptualmente de obli-
gaciones impuestas por las normas jurídicas.

Para Hart, al aludir a la noción de obligación moral (y no al aludir a la de obli-


gación jurídica) es central considerar las siguientes cuatro características:
i) Importancia.
ii) Inmunidad al cambio deliberado.
iii) Carácter voluntario de las transgresiones morales.
iv) Forma en que se ejerce la presión social.

Veamos a continuación las implicancias de cada una de ellas.

La importancia de las reglas morales es algo que asociamos directamente


con el status de las mismas, cuya ausencia nos lleva directamente a con-
cluir que no estamos verdaderamente ante reglas morales. En general, las
reglas morales implican sacrificar intereses egoístas. Además, se considera
que la falta de reconocimiento extendido de sus pautas acarrearía graves
consecuencias para el grupo social –piénsese en la incidencia social de una
falta de aceptación general y continuada de la prohibición del libre uso de la
violencia contra personas o cosas. Dicha razón se hace evidente en la seria
presión social que se ejerce para asegurar su cumplimiento.

En el derecho, en cambio, se observa que existen algunas obligaciones que


no revisten este carácter de seriedad, gravedad o importancia y que, sin
embargo, son casos evidentes de reglas jurídicas.

En relación a la inmunidad al cambio deliberado, la idea de que podemos


cambiar por actos deliberados de voluntad de una autoridad o grupo social
una norma moral es inadmisible, mientras que nadie se asombra si a partir
del primero de febrero se despenaliza tal o cual conducta. Un aspecto central
relacionado con la inmunidad al cambio deliberado, consiste en la ausencia
en el terreno moral de autoridades normativas. No existen legisladores del
derecho natural o de la moral (salvo que se presuponga que Dios es una

EDUBP | Abogacía | Filosofía y Lógica Jurídica - pag.103


especie de legislador, pero en todo caso no concebimos a Dios de la forma
en que concebimos la autoridad humana). Parece claro que nuestro aparato
conceptual se resiste a utilizar expresiones como derogación y anulación en
el terreno moral. Sobre este aspecto se centra, en realidad, el grueso de la
discusión entre iusnaturalistas y iuspositivistas, pues estos últimos sostienen
que la fuente de nuestras obligaciones pueden ser sociales (humanas) y que,
por tanto, es completamente comprensible que nuestras obligaciones cam-
bien de un día para el otro; mientras que los iusnaturalistas sostienen que
ninguna obligación última y definitiva puede depender de una fuente social
y, en consecuencia, el derecho no impone obligaciones, sino en tanto y en
cuanto se corresponda con alguna concepción del bien o corrección.

La tercera diferencia, el carácter voluntario de las transgresiones morales,


pretende resaltar la idea de que en la moral no existen ejemplos de respon-
sabilidad objetiva (responsabilidad de un sujeto por haber hecho algo sin
intención), mientras que es muy común en el derecho que se reprochen
acciones o se impongan responsabilidades en ausencia de intencionalidad
o elemento subjetivo alguno (v. g. el Art. 1.113 del CC). Dicho en otros tér-
minos, un individuo que transgredió una norma moral y puede probar que lo
hizo de modo no intencional es excusado moralmente de responsabilidad,
pues la moral considera que si no hubo voluntad o intención no hubo acción.
La justificación en el derecho, en cambio, se da en casos de conductas que
el sistema jurídico no trata de impedir y hasta puede alentar o permitir, tal el
caso de matar en legítima defensa, aún cuando se trate de una excepción a
una prohibición general de matar.

Por último, la forma típica de presión moral es exhortar al respeto hacia las
reglas mediante el reproche, la crítica o el aislamiento, a fin de generar en los
que se desvían de sus obligaciones morales un sentimiento de vergüenza,
culpa o remordimiento que es en lo que consiste la verdadera sanción moral,
esto significa que la sanción moral tiene un carácter voluntario. En el caso
del derecho, la presión apela a la amenaza de castigo físico o consecuencias
desagradables, la cual se hace efectiva, en caso de incumplimiento de la
obligación, independientemente de la voluntad del individuo obligado.

Derecho natural y positivismo jurídico.


La reflexión iusfilosófica ha estado gobernada históricamente por concepcio-
nes iusnaturalistas, pero a partir del siglo XIX dichas concepciones han resig-
nado su lugar preponderante a favor de concepciones iuspositivistas y, hasta
nuestros días, el debate entre ambas permanece abierto. Para esclarecer
las diversas cuestiones que se entretejen en ese debate y, poder realizar un
análisis crítico de los argumentos que se esgrimen en defensa de una y otra
concepción, distinguiremos las variadas tesis que se han sostenido sobre
la relación conceptual entre derecho y moral. Por moral entenderemos en
lo que sigue la que hemos denominado, siguiendo a Hart, moralidad crítica
(consultar la introducción y el glosario del presente módulo). No obstante,
es oportuno hacer dos aclaraciones: por un lado, no debe entenderse que
los conceptos de moralidad positiva y moralidad crítica son independientes,
pues, algunas de las creencias socialmente compartidas sobre lo que es
correcto se ajustan a los requerimientos de moralidad crítica y, por otro lado,

EDUBP | Abogacía | Filosofía y Lógica Jurídica - pag.104


la moralidad crítica no es una abstracción sino una práctica humana relacio-
nada con el conjunto de actitudes y valoraciones que conforman un grupo
social, y contribuye a precisar su alcance. De ese modo, la moral crítica es
una herramienta útil para determinar si una creencia sobre lo que es correcto
es errónea o carece de fundamento racional.

En cuanto a la relación entre el derecho y la moral, la tesis que define a las


concepciones iusnaturalistas es la tesis de la conexión necesaria entre
el derecho y la moral que puede expresarse como sigue:

(1) La determinación de lo que es Derecho depende de su adecuación a


la moralidad.

Esta tesis establece una condición necesaria, pero no suficiente, para


una definición precisa del derecho positivo. En este sentido decimos que,
además de la adecuación con la moral, la existencia del derecho depende
de la presencia de ciertos hechos sociales, por ejemplo, de la promulgación
de normas por parte de autoridades legislativas, pues es necesario conocer
previamente qué normas pertenecen al sistema jurídico para poder, luego,
determinar su adecuación o falta de adecuación moral.

La tesis, tal como fue sostenida por la concepción teológica del Derecho
Natural, ha sido expresada en términos de Tomás de Aquino del siguiente
modo: la ley humana que no deriva del Derecho natural no es ley, sino corrup-
ción de ley. El Derecho Natural, a su vez, es expresión de un conjunto de prin-
cipios correspondientes al orden humano natural como parte de la ley divina
que rige el universo. En otras versiones iusnaturalistas el origen del Derecho
Natural es atribuido a la naturaleza –Aristóteles– o a la razón humana -John
Locke, Kant-.

Tomás de Aquino distingue dos modos en que el derecho positivo se adecua


al Derecho Natural, a saber:
i. por deducción lógica, por ejemplo las condiciones jurídicas para la cele-
bración de un contrato son consecuencia lógica del principio de derecho
natural “se deben cumplir las promesas”;
ii. por determinación o especificación de principios de derecho natural, por
ejemplo la norma jurídica que impone un límite de velocidad a los auto-
movilistas determina o especifica un modo de cumplir con el principio “se
debe disminuir la probabilidad de ocasionar daños a terceros”.

Según versiones contemporáneas del iusnaturalismo, la conexión nece-


saria entre derecho y moral debe ser entendida en el sentido de que la exis-
tencia del derecho positivo tiene necesariamente valor moral y, no en el
sentido tradicional de que normas injustas no son normas jurídicas válidas.

A los fines de complementar la información acerca de este tema, le sugerimos


consultar el siguiente artículo. Garzón Valdés, Ernesto, “Algo más acerca
de la relación entre derecho y moral”, Doxa 8, 1990, ps. 111-130 .

EDUBP | Abogacía | Filosofía y Lógica Jurídica - pag.105


Esta tesis puede ser compatible con el iuspositivismo si es comprendida al
modo de Hart. En efecto, este autor afirma que las protecciones mínimas
que el derecho y la moral positiva acuerdan pueden tener distintos grados de
extensión en diversas sociedades. En efecto, puede ocurrir que el derecho y
la moral aceptados de una sociedad no hagan extensivas dichas proteccio-
nes mínimas a todos los sujetos comprendidos por sus normas. En dichas
sociedades el poder coercitivo del estado, a través del derecho, es usado de
dos modos:
a. para sancionar a los transgresores, esto es, a quienes reciben la protec-
ción de las normas, pero las violan para satisfacer intereses egoístas y,
b. para sojuzgar y someter a un grupo, grande o pequeño que, a pesar de
cumplir con las obligaciones y cargas que implica vivir en sociedad, se les
priva de los beneficios y protecciones que debe garantizar la sociedad y,
de ese modo, resultan víctimas del sistema. Es probable que lo que motive
el comportamiento de éstos últimos no sea la aceptación de las normas
ni el respeto a la autoridad, sino el temor a sufrir las sanciones coercitivas
previstas en el sistema. El sistema jurídico que no satisface de modo equi-
tativo los intereses vitales de aquellos a los que exige obediencia existe
pero, cuánto mayor es su grado de injusticia, más represivo e inestable es,
lo cual quiere decir que, para Hart, la mayor o menor adecuación moral del
derecho no es condición de su existencia, sino de su estabilidad.

Pero hay otro modo de entender la tesis iusnaturalista formulada en su ver-


sión contemporánea y, es la siguiente: Si la existencia de cualquier derecho
positivo tiene algún valor moral entonces, el derecho merece por parte de
sus destinatarios un deber prima facie de obediencia. En este último sentido,
resulta discutible que el derecho instancie algún valor moral cuando otorga
las protecciones mínimas (vida, salud, seguridad, etc.) a un sector muy redu-
cido de la sociedad, y mantenga en el sometimiento más injusto a la mayoría
de los sujetos.

En cuanto a las tesis del iuspositivismo pueden distinguirse una tesis nega-
tiva y una tesis positiva. La tesis negativa puede formularse como sigue:

(2) La determinación de aquello que el Derecho es, no depende de su


adecuación a la moralidad.

Esta tesis es una negación de la tesis iusnaturalista.

La tesis positiva de la concepción iuspositivista es:

(3) la existencia del derecho depende de un conjunto de hechos sociales


–tesis de las fuentes sociales– y esta tesis, a su vez, no es negada por el
iusnaturalismo sólo que, además del origen en algún hecho social, los
iusnaturalistas exigen la adecuación moral del derecho.

A su vez, la tesis negativa del iuspositivismo expresada en (2) puede ser inter-
pretada de tres diferentes modos y cada uno de ellos da lugar a una forma de
positivismo jurídico contemporáneo:

EDUBP | Abogacía | Filosofía y Lógica Jurídica - pag.106


• Es necesariamente el caso que la legalidad de una norma jurídica no
depende de su mérito moral.

Esta interpretación da lugar a la forma de positivismo denominada “positi-


vismo exclusivo” que constituye la versión más fuerte de la tesis (3) de las
fuentes sociales, pues afirma que la determinación del contenido del derecho
depende únicamente de su origen en ciertos hechos sociales sin referencia
a argumentación moral alguna. El problema de esta versión del positivismo
es que algunas normas jurídicas incorporan conceptos morales y, la identifi-
cación de las mismas, en consecuencia, requiere de argumentación moral.
Por ejemplo, el artículo 18 de la Constitución Nacional prohíbe la tortura y
determinar qué ha de entenderse por tortura, requiere argumentación moral.

• No es necesariamente el caso que la legalidad de una norma jurídica


dependa de su mérito moral.

La validez jurídica de las normas sólo puede depender de su validez moral


de un modo contingente. En efecto, sólo en el caso de normas jurídicas
que incorporen en su formulación términos morales y por ende requieran
para su determinación y alcance de argumentación moral, su validez depen-
derá de su adecuación moral. Esta versión es expresión del denominado
“positivismo inclusivo” o “incorporacionismo”. Como observaremos en el
módulo siguiente, esta forma de positivismo constituye la respuesta de la
concepción positivista a las importantes críticas que fueran formuladas en su
contra por Ronald Dworkin. El problema del positivismo inclusivo es deter-
minar si hay criterios objetivos para determinar la verdad o falsedad de los
juicios morales. Si sostenemos que no hay criterios objetivos de corrección
de los argumentos morales, entonces no sólo tendremos que rechazar la
existencia de un Derecho Natural como conjunto de principios y estándares
universalmente válidos y considerar que el iusnaturalismo carece de sentido,
sino también el positivismo inclusivo mismo, pues el razonamiento moral
exigido por normas que contienen conceptos morales sólo constituirá una
remisión al poder discrecional de los órganos de aplicación –en cuanto a la
noción de discrecionalidad judicial ver contenidos del módulo 4 y capítulo 7
de El concepto de derecho de Hart. Si el objetivismo moral es falso –no hay
criterio objetivo de corrección de un juicio moral– entonces el positivismo
exclusivo resulta una concepción adecuada del derecho.

Pero, ante tal conclusión, una última reflexión es necesaria. Si el escepti-


cismo moral, o bien la idea de que los juicios morales no son ni verdaderos ni
falsos, es correcto, entonces no estaremos en condiciones de fundamentar
racionalmente ningún juicio moral, por ejemplo, no estaremos en condicio-
nes de afirmar con certeza que las prácticas corruptas dentro del gobierno
son prácticas inmorales y, esta conclusión parece no corresponderse con
nuestras intuiciones respecto de las valoraciones morales. La búsqueda filo-
sófica en materia ética se centra en encontrar un punto medio de reflexión
racional que impida el despotismo moral y deje espacio a la tolerancia y al
respeto de diferentes concepciones y planes de vida.

EDUBP | Abogacía | Filosofía y Lógica Jurídica - pag.107


• La legalidad de una norma no debe depender de su mérito moral.

Esta versión da lugar a la forma de positivismo denominada “positivismo


normativo” y, presupone que la determinación del contenido del derecho
puede depender de argumentos morales. No obstante, sostiene que el dere-
cho no debe depender de argumentos morales por tres razones:
a. hay desacuerdo moral respecto de lo que constituye acción moralmente
correcta;
b. por respeto a la autonomía moral de los sujetos, el derecho debe gober-
narse por reglas claras que proporcionen certeza respecto de qué con-
ductas están jurídicamente prohibidas;
c. a y b implican que, si hay que identificar lo prohibido jurídicamente por
argumentos morales, entonces habrá desacuerdo, se vulnerará la cer-
teza y, con ella, la autonomía moral de los sujetos.

Por tanto, concluye:


d. el derecho debe identificarse sin acudir a la moralidad y, por ende, no
deben incorporarse conceptos morales en la normativa jurídica.

Todas las versiones contemporáneas del positivismo reconocen que el


debate en el ámbito de la ética acerca de lo moralmente correcto es amplio
y que es importante contar con leyes claras y con una separación estricta
entre creación del derecho y su aplicación que garantice a los sujetos que
están gobernados por leyes y no por hombres. No obstante, tratan de refor-
mular las premisas del argumento sostenido por el positivismo normativo, de
modo de poder demostrar que ellas no habilitan a inferir la conclusión que
esta forma de positivismo sostiene. De tal modo, estas versiones sostienen:

a’ Si bien hay un grado importante de desacuerdos en cuestiones morales, el


desacuerdo acerca de lo que es correcto no es total y absoluto.

b’ No siempre la certeza del derecho protege la autonomía de los sujetos.


Hay casos en que el respeto a la autonomía de los sujetos requiere hacer
posible a los sujetos dar razones morales para explicar sus conductas. Por
ejemplo, las causas de justificación, como la legítima defensa o el estado
de necesidad, restan certeza al derecho penal, pero están previstas en pro-
tección de la autonomía personal. Por tanto, la incorporación de conceptos
morales en el derecho puede restar certeza pero, cuando funcionan como
cláusulas de excepción de prohibiciones jurídicas generales, pueden contri-
buir a un mayor respeto de la autonomía moral de los sujetos.

a’ y b’ no implican c. sino su versión débil:

c’ Si determinar las conductas jurídicamente prohibidas requiere argumentos


morales, entonces, habrá menos certeza y algún grado de desacuerdo, pero
esto contribuirá a garantizar la autonomía moral de los sujetos.

EDUBP | Abogacía | Filosofía y Lógica Jurídica - pag.108


Consideraciones finales…
De lo expuesto y basándonos nuevamente en los aportes de Hart, podría-
mos concluir diciendo que una sociedad superviene a la práctica de reglas. A
su vez, el seguimiento de reglas, es un fenómeno que una teoría del derecho
no puede soslayar.

A las actitudes y creencias de quienes muestran, en mayor o menor exten-


sión, adhesión a las normas, constituye el punto de vista interno -o de los
participantes-, y es un ingrediente primordial de su teoría. Tales ingredientes
no pueden extraerse de una explicación del fenómeno jurídico, dada exclu-
sivamente en términos de relaciones de dominación o fuerza. De ahí, sus
críticas a las teorías del soberano y del escepticismo o realismo jurídico.

Además, es importante resaltar que, para una descripción adecuada del dere-
cho en sociedades modernas, Hart incorpora la noción de reglas secunda-
rias de reconocimiento, cambio y adjudicación. Dichas reglas secundarias
permiten distinguir las normas jurídicas de las demás normas sociales por su
fuente u origen, e identificar la autoridad normativa u órgano de creación y
aplicación de normas jurídicas.

La relevancia teórica del punto de vista interno surge de las funciones carac-
terísticas del derecho en la conformación de toda sociedad bien ordenada.
La cuestión de lo que ha de entenderse por sociedad bien ordenada es una
cuestión valorativa a la que Hart responde en términos de las falencias propias
de la naturaleza humana y del propósito o fin que la mayoría de los hombres
tienen de sobrevivir. La moral y el derecho son medios de asociación humana
para la consecución de ese fin o propósito. De tal modo, cuando se discute la
forma adecuada de regular la conducta social hay que tener presente que sus
miembros, en términos generales, tienen el propósito común de sobrevivir.
El punto de vista interno es el termómetro que indica en qué medida el dere-
cho contribuye al logro del propósito para el cual es implementado. El grado
de adhesión a las normas que confieren autoridad, depende, en gran medida
del reconocimiento de normas constitutivas de la base normativa común, tam-
bién denominada por Hart “contenidos mínimos de derecho natural”.

Un relato para continuar la reflexión….

Acceda al siguiente material de la lectura para complementar su estudio: Hart,


H. L. A.; El Concepto de Derecho, Bs. As. Abeledo Perrot, 1968, capítulos 8 y 9.

A los fines de advertir la importancia del análisis del punto de vista interno
y su relación con la cooperación, cohesión y estabilidad social, se sugiere la
lectura reflexiva de La Isla de Llewellyn Smith.

Si está interesado en profundizar estos temas puede leer: Garzón Valdés,


Ernesto, “Algo más acerca de la relación entre derecho y moral”, Doxa 8,
1990. Págs. 111-130.

Lo invito a realizar las actividades del módulo.

EDUBP | Abogacía | Filosofía y Lógica Jurídica - pag.109


M5 Actividades
Actividad 1
Positivismo e iusnaturalismo
En la actividad 1 del módulo 4, Ud. tuvo oportunidad de interiorizarse de
los pormenores de un fallo de la Corte Suprema, en el que se hacía lugar a
un transplante de órganos a pesar de que, en el caso, la ley de transplante
vigente expresamente negaba la posibilidad del mismo. Según dicha ley, los
menores de 18 años no pueden disponer de sus órganos y el caso se refería
precisamente a un menor de 18 años que quería disponer de un riñón para
donárselo a su hermano. En la solución del caso, la Corte Suprema sostuvo
que ninguna ley podía tener un contenido tan injusto como la ley en cuestión.
De aplicarse ésta a rajatabla –sostuvo la Corte– se condenaría al requirente
del órgano a la muerte, ya que la única posibilidad que éste tenía de salvar su
vida era recibir la donación de su hermana (menor de 18 años). Entre otros
argumentos, la Corte sostuvo que no era posible tomar una ley de semejante
injusticia o inequidad como una ley en absoluto, ya que una ley que conlleve
a las personas regidas por ella a la muerte no puede considerarse tal en
ningún aspecto, y éste era el caso.

Considerando lo estudiado en este módulo, en relación con el debate entre


positivismo e iusnaturalismo:
i) ¿A qué escuela iusfilosófica pertenece el argumento de la Corte?
ii) En su caso, exprese las razones en las cuales apoya su conclusión.
iii) Si Ud. ha considerado que el argumento de la Corte es positivista, redacte
un argumento iusnaturalista en respuesta a aquél. Si Ud. ha considerado
que el argumento de la Corte es iusnaturalista redacte un argumento,
desde la óptica positivista, que responda y pretenda rebatir el primero.
AA

AA Actividad 1
asistente académico

A la hora de elaborar los argumentos, le sugerimos escribir textos breves que


abarquen aproximadamente 15 renglones.

EDUBP | Abogacía | Filosofía y Lógica Jurídica - pag.110


MÓDULO 6

M6 Microobjetivos

• Profundizar los conocimientos sobre el positivismo de Herbert Hart,


desarrollados en los módulos anteriores, de modo de poder dar cuenta
de esta doctrina en forma sucinta y contraponerla a otra doctrina iusfilo-
sófica contemporánea.
• Saber analizar el grado de desacuerdo teórico existente entre las con-
cepciones contemporáneas del iuspositivismo e iusnaturalismo, con el
propósito de evaluar si su incompatibilidad es más aparente que real.
• Comprender en qué términos se ha trabado modernamente la discusión
entre iusnaturalistas y iuspositivistas, con el fin de contar con una herra-
mienta conceptual para analizar resoluciones judiciales y realizar aborda-
jes teóricos en el plano del derecho.

M6 Contenidos

Lo invito a ver el video de presentación del Módulo 6

Discusiones centrales en la teoría del derecho

Introducción
En los módulos anteriores, hemos analizado la teoría general del derecho de
H. L. A. Hart y, más específicamente, en el módulo 5 se hizo referencia a la
incidencia de los análisis de Hart en el debate contemporáneo, no sólo entre
iusnaturalismo e iuspositivismo, sino en el seno mismo de la teoría positivista
del derecho. En efecto, distinguimos las diferentes formas de positivismo
que se confrontan hoy en relación con la cuestión de la naturaleza del vínculo
entre derecho y moral (Positivismo exclusivo vs. Positivismo inclusivo). Los
desacuerdos existentes en el positivismo jurídico contemporáneo son pro-
ducto de una controversia más radical que Hart sostuvo con Ronald Dworkin,
quien fue uno de sus discípulos más prominentes. En efecto, Dworkin realizó
un análisis crítico de las tesis centrales de la teoría de Hart, a saber: su tesis
de las fuentes sociales y su tesis de la discrecionalidad judicial1.

Por su parte, las críticas de Dworkin tuvieron como finalidad proponer una
nueva teoría del derecho, denominada “interpretativa” o “coherentista”, con-
siderada como la versión contemporánea de la concepción iusnaturalista del
derecho, según la cual, el derecho resulta identificado cuando se descubren
las razones que justifican moralmente el derecho basado en fuentes sociales.

En el presente módulo, trataremos de reproducir concisamente los argumen-


tos de Ronald Dworkin pero, previamente, con el propósito de esclarecer el
origen de la confusión, dentro del positivismo, en relación a la naturaleza del

1 Ronald Dworkin ha desarrollado sus críticas a Hart y su propuesta alternativa de teoría del dere-
cho en Los Derechos en Serio y en El imperio del Derecho, además de numerosos artículos y obras
posteriores.

EDUBP | Abogacía | Filosofía y Lógica Jurídica - pag.111


derecho y de su relación con la moral, distinguiremos, siguiendo a Norberto
Bobbio2, tres sentidos de la acepción “ius positivismo”. Esta distinción resul-
tará también útil para entender mejor la concepción positivista del propio
Hart y, también, las críticas que le dirige Dworkin.

Los tres sentidos de la expresión positivismo jurídico son:


i) Positivismo jurídico como método, esto es, como un modo de abor-
dar el estudio del derecho, según el cual debe distinguirse la cuestión
de cómo debe ser el derecho de lo que el derecho de hecho es. Desde
esta perspectiva, la teoría positivista propone estudiar el derecho desde
un enfoque valorativamente neutral. En consecuencia, se considera que
no existe una conexión necesaria entre derecho y moral. Este sentido
de “positivismo” es denominado “positivismo jurídico como approach” o
actitud positivista y afirma que es posible una ciencia del derecho, pues
el derecho es una práctica susceptible de ser observada como cualquier
fenómeno social y, además, que puede ser descripta y caracterizada
con independencia de conceptos morales tales como justicia, equidad,
corrección, bondad, etc. De ello se infiere que no es en absoluto incompa-
tible afirmar, por ejemplo, “el derecho existente en Argentina es injusto”.
ii) Positivismo jurídico como teoría, este sentido está relacionado con
la idea de derecho introducida por el liberalismo político del siglo XIX
que Max Weber denominara dominación burocrática racional y que se
caracteriza por los siguientes rasgos: monopolio de la fuerza coercitiva,
imperativismo, supremacía de la ley respecto de otras fuentes de dere-
cho, consistencia y completitud del derecho, separación de funciones
o poderes de creación de derecho y de aplicación, concepción de la
aplicación de derecho como actividad puramente deductiva. Todos los
rasgos mencionados caracterizan a lo que conocemos como “forma-
lismo jurídico” o “dogmática jurídica”. Una versión epistemológica y
refinada de la dogmática será objeto de análisis en el último módulo del
programa de la asignatura.
iii) Positivismo jurídico como ideología, sostiene que el derecho positivo
es justo y, en consecuencia, debe ser obedecido. Afirmar que el derecho
justo es una tautología, y sostener que es injusto es una contradicción en
los términos pues, por definición si algo es derecho, no puede ser injusto.

H. L. A. Hart.
Con respecto a H. L. A. Hart, cabe afirmar que es un positivista del primer
tipo. Es decir, un positivista en el sentido que adopta una actitud positivista
o un approach positivista. No correspondería calificarlo de positivista ni en el
segundo ni en el tercer sentido. En efecto, su rechazo del segundo sentido
(positivismo como teoría) surge de la tesis de textura abierta del derecho que
opone al postulado, sustentado por el formalismo jurídico, de precisión o cer-
teza del derecho. Con respecto al tercer sentido (positivismo ideológico) se
infiere claramente de su análisis sobre la relación derecho-moral, desarrollado
en el módulo 5, que rechaza esta idea de la naturaleza intrínsecamente justa
del derecho y, del consiguiente deber de obedecerlo. Sabemos que Hart sos-
tiene que el derecho se identifica por su origen, en algún hecho social al que
2 Bobbio, Norberto; Giusnaturalismo e positivismo giuridico, Milano, Comunità, 1965.

EDUBP | Abogacía | Filosofía y Lógica Jurídica - pag.112


los participantes de la práctica jurídica reconocen autoridad generadora de
normas válidas (regla de reconocimiento) y, que el derecho así reconocido es
válido independientemente de su adecuación moral. La adopción del primer
sentido y el rechazo de los otros dos, habilita el análisis crítico de los orde-
namientos jurídicos. Si un jurista o teórico quiere evaluar si un determinado
ordenamiento jurídico es justo o injusto, debe contar primero con las herra-
mientas necesarias para poder identificarlo y, una vez hecho esto, considerar
si lo que ha identificado como derecho es incorrecto, injusto, etc.

Habíamos visto con anterioridad que la regla de reconocimiento resuelve


en parte, aunque no totalmente, la falta de certeza del régimen de reglas
primarias -recordemos que los criterios de la regla de reconocimiento tam-
bién poseen textura abierta-. Esta regla de reconocimiento existe como una
práctica convergente del grupo social de reconocer como válidas jurídica-
mente, reglas que satisfacían alguna fuente expresada por la regla de reco-
nocimiento. Las leyes emanadas del Congreso, son derecho en Argentina
porque satisfacen un criterio de origen (sanción legislativa) al que recono-
cemos autoridad generadora de normas válidas y, en razón de ello, forman
parte del contenido de la regla de reconocimiento del sistema jurídico argen-
tino. Dicho brevemente, es condición de existencia de una regla de recono-
cimiento que un grupo de personas -en especial los jueces- desarrollen una
cierta actitud convergente hacia las reglas.

De hecho todos los participantes usamos una regla de reconocimiento para


identificar lo que es derecho en Argentina. Sabemos cuando formalizamos
un contrato que el mismo ha de valer jurídicamente si satisface las condicio-
nes establecidas en el Código Civil, pues dicho código constituye una ley
válida en nuestro sistema por haber sido sancionado por el Congreso.

A grandes rasgos, entonces, puede decirse que la tesis de Hart asume tres
ideas fundamentales:

Tesis Uno (Tesis de la Separación): no existe una conexión necesaria entre


el derecho y la moral. El derecho es un fenómeno social cuya identificación
no necesita apelar a las nociones de “justicia”, “bondad” o “corrección” (a
este respecto puede consultarse el material del módulo anterior). Esta tesis
podría resumirse en la frase de John Austin: “el derecho es una cosa, su
mérito o demérito otra”.

Tesis Dos (Tesis de las fuentes sociales): ésta sostiene que es posible iden-
tificar lo que el derecho indica, atendiendo a lo que dice un grupo de per-
sonas (legisladores, jueces) o la forma en que actúa un grupo considerable
de personas. En definitiva, se afirma que los últimos tests de validez jurídica
descansan en una práctica social.

Tesis Tres (Tesis de la discrecionalidad judicial): esta tesis sostiene que


indefectiblemente habrá casos imprevistos y no regulados legalmente, es
decir, casos para los cuales el derecho es parcialmente indeterminado e
incompleto y en los cuales los jueces tendrán la posibilidad de ejercer discre-
ción (esto es, deberán crear nuevo derecho en un ámbito de libertad restrin-
gida por el núcleo de significado claro de las reglas jurídicas -consultar con-
tenidos del módulo anterior y capítulo 7 del Concepto de derecho de Hart-).

EDUBP | Abogacía | Filosofía y Lógica Jurídica - pag.113


Un problema interesante se suscita cuando la regla de reconocimiento no
permite identificar una respuesta jurídica para un caso concreto -casos de
incertidumbre de la regla de reconocimiento-. Supongamos que nuestra
regla de reconocimiento sostiene que el derecho depende de lo que ordena
el Congreso (éste ha sancionado una ley que viola alguna limitación formal
(procedimiento de sanción de las leyes) o alguna restricción sustancial (algún
derecho o garantía constitucional) y esta situación da lugar a un desacuerdo
entre los participantes –en especial los jueces– respecto del alcance y límites
del criterio legislativo de la regla de reconocimiento por lo que algunos sos-
tienen que es derecho lo que dice el Congreso y otros lo que establece la
Constitución). En tales casos dudamos de la validez o pertenencia de la ley en
cuestión por la falta de acuerdo o consenso en su identificación y, en conse-
cuencia, el derecho resulta indeterminado. Al respecto Ronald Dworkin afirma
que el problema de indeterminación no es un problema del derecho, sino del
criterio empleado por Hart -regla de reconocimiento- para su identificación.

El debate Hart – Dworkin.


La crítica de Dworkin al positivismo va dirigida a la tres tesis características
de la concepción de Hart:
1. En contra de la tesis de la separación conceptual derecho–moral; pro-
cura demostrar que en la práctica jurídica de los tribunales la diferencia
entre derecho y moral no es tan clara. No obstante, niega la existencia
de un derecho natural constituido por un conjunto de principios univer-
sales e inmutables. Propone un modelo de razonamiento reconstructivo
del material jurídico a partir de principios que justifican y dan sentido a
nuestras intuiciones básicas respecto de lo que el derecho dispone.
2. En cuanto a la tesis de discrecionalidad de los jueces, entendida como
poder legislativo excepcional que los jueces poseen para crear derecho
nuevo ex post facto, afirma que la misma es consecuencia de una inde-
terminación conceptual de la teoría de Hart. Dworkin opone a esta tesis
la tesis de la respuesta correcta, según la cual el derecho da una solu-
ción normativa a todos los casos, aún los casos difíciles generados por
la textura abierta de términos contenidos en la formulación de las reglas.
Ello es así, porque el derecho, además de reglas “derecho explícito”,
contiene principios que configuran el substrato o conjunto de razones
justificatorias de las reglas, al que denomina “derecho implícito”. El con-
junto de reglas y principios, según el autor, constituye material jurídico
suficiente para proporcionar respuesta correcta a todos los casos.
3. Con respecto a la tesis de las fuentes sostenida por Hart a través de
su regla de reconocimiento, Dworkin sostiene que el criterio de la fuente
social u origen no confiere fuerza autoritativa a los principios, pues éstos,
debido a su carácter inespecífico y no concluyente, no pueden identi-
ficarse por criterios convencionales. En efecto, los principios surgen por
un sentido de conveniencia, se suprimen por desgaste y se identifican
por su mayor peso o fuerza argumentativa en la confrontación o com-
petencia con otros principios en un caso en mano. Además, esta nueva
concepción del derecho como conjunto de normas y principios da lugar
a una idea de obligación diferente de la sostenida por Hart. En efecto, los
principios son los que determinan en definitiva cuando hay una obliga-

EDUBP | Abogacía | Filosofía y Lógica Jurídica - pag.114


ción jurídica. Las normas obligan sólo cuando los principios que justifi-
can su aplicación son más fuertes o importantes que los principios que
orientan la decisión en sentido contrario.

Es un hecho obvio que los sujetos involucrados en una práctica jurídica suelen
disentir sobre qué cosas impone el derecho. No por ello concluyen que no
tienen un parámetro en común para dirimir sus cuestiones. En los casos de
desacuerdo (v.g. cuando algunos sostienen que el derecho depende de lo
que dice el Congreso y otros de lo que dice la Constitución) la teoría de Hart
se ve obligada a decir que ese grupo no tiene una regla común y que, por
tanto, no tiene derecho (para ese caso, o en general). Sin embargo –sostiene
Dworkin– es un hecho bastante obvio que cuando esto sucede los juristas
siguen pensando que tienen derecho o parámetros jurídicos para solucionar
la controversia. Cualquier jurista sabe que si no se cuenta con reglas clara-
mente aplicables para solucionar un caso, puede echar mano a los principios
y con ellos tratar de resolver el caso en cuestión.

Según Dworkin, por tanto, el derecho no sólo tiene reglas, como supone
Hart, sino también principios. Son de estos últimos elementos de los que no
podría dar cuenta la teoría de Hart. Si una teoría quiere preciarse de tal, tiene
que tratar de describir todos los fenómenos que analiza y es un fenómeno
comprobable que los juristas usan principios y no sólo reglas y, que cuando
desacuerdan sobre las exigencias de la regla de reconocimiento (unos dicen
que es válido lo que dice el Congreso, otros la Constitución) siguen acor-
dando en que hay una forma de solucionar el caso. Siguen creyendo que
hay derecho a pesar del desacuerdo sobre reglas. En definitiva, la teoría de
Hart sólo sería útil para explicar cuando hay reglas y nos fuerza a sostener
que cuando no se puede identificar qué reglas tenemos (si las del Congreso
o las de la Constitución o las de la Biblia, etc.) no tenemos derecho. Es una
teoría deficitaria porque basa su análisis en los casos de acuerdo, cuando es
casi un hecho obvio que en el derecho suele haber desacuerdos sin que ello
implique creer que no tenemos derecho. En conclusión, diría Dworkin, es un
hecho innegable que:
i) los juristas disienten sobre qué dice el derecho, pero siguen pensando
que existe una respuesta jurídica para solucionar el caso;
ii) los juristas utilizan unos parámetros distintos a las reglas para solucionar
una infinidad de casos, i. e. utilizan principios. La tesis de Hart, en este
sentido, sólo serviría para identificar las reglas pero no los principios.
Como ya hemos visto, una de las tesis del positivismo de Hart es la de
las fuentes sociales. Según ella, para identificar qué es derecho en una
determinada sociedad debo observar cuáles son las prácticas conver-
gentes. Si una sociedad de hecho usa como criterio el tener por válidas
las reglas emanadas del Congreso, yo puedo saber si una regla es válida,
fijándome de donde proviene. Es decir, fijándome si ha sido producida
por el Congreso o por otro órgano. Esto ha sido llamado por Dworkin el
test del pedigrí, es decir un test que permite reconocer si una regla es
válida fijándome en su procedencia, fuente u origen. Por definición, este
test de pedigrí u origen no puede ser utilizado para reconocer los princi-
pios, pues se supone que los principios aparecen cuando, precisamente,
no existe una regla de reconocimiento o una práctica convergente sobre
qué es derecho en una sociedad. Necesitamos de los principios precisa-

EDUBP | Abogacía | Filosofía y Lógica Jurídica - pag.115


mente cuando el pedigrí (la fuente, el origen) de la regla no es una condi-
ción suficiente para concluir que la regla en cuestión es válida.

Para entender este punto, es necesario retomar el caso Saguir y Dib desa-
rrollado en el módulo 5 (En esta instancia nos referimos específicamente a la
actividad 1 del módulo 5).

Tal como usted recordará, en ese caso, los juristas disienten sobre cuál es el
pedigrí u origen que haría válida la regla sobre la donación de órganos. Por
su parte, el procurador sostenía que si la ley emanaba del Congreso, esto era
suficiente para concluir que la donación debía ser denegada.

Su regla de reconocimiento tenía el siguiente contenido: “es válido en Argentina


lo que dice el Congreso en materia de donación de órganos”. Toda ley válida
–diría el procurador – tiene que detentar un cierto origen, esto es provenir de
lo que dijo el Poder Legislativo. Sin embargo, la Corte suprema sostenía otro
criterio. El criterio según el cual las reglas son válidas en tanto y en cuanto no
violen la constitución. Si nos atuviésemos al criterio de las fuentes sociales no
podríamos determinar cuál es la regla que rige el caso porque, como muestra
el ejemplo, no hay acuerdo alguno sobre qué es lo que impone el derecho
en ese caso. Las fuentes no son unánimes. En el caso Saguir y Dib, la Corte
debió echar mano al principio de respeto a la vida, que no estaba consagrado
expresamente en la Constitución al momento del fallo. Este caso muestra
cómo existe desacuerdo sobre las reglas pero, no obstante ello, los juristas
tienen un “plan B” para solucionar ciertos vacíos. “Plan B” que no depende ni
del acuerdo, ni de la noción de regla concebida por Hart.

Ante el desacuerdo –explica Dworkin– los juristas suelen utilizar principios


para exponer sus diferencias, los cuales no pueden ser identificados a través
de este test de origen. Es precisamente porque el test no funciona que los
juristas echan mano a los principios. Por tanto, si es cierto que el derecho
consiste en reglas y principios, Hart nos ha dado un test que permite identifi-
car a las primeras pero no a los segundos. La tesis de las fuentes sociales, la
regla de reconocimiento y el test de origen no permiten identificar principios
ni dar cuenta del desacuerdo entre juristas. Esta crítica implica otra conse-
cuencia directa sobre la teoría de Hart. Esta no sólo sostenía la tesis de las
fuentes sociales, sino que adosaba a ella la tesis de la discrecionalidad (Tesis
Tres) según la cual, cuando no había reglas, el juez debía decidir dentro de
un margen de libertad o discrecionalidad. Al sostener que en el derecho hay
más cosas que reglas, Dworkin no sólo socava la tesis de las fuentes socia-
les, sino también la idea misma de discrecionalidad sostenida por la Tesis
Tres de Hart (discrecionalidad). La tesis de las fuentes se ve socavada porque
Dworkin llegaría a mostrar que el mundo jurídico posee elementos cuya exis-
tencia no depende de un acuerdo o fuente social. Es decir, que el test y el
approach propuesto por Hart no permitirían dar cuenta de un grupo impor-
tante de fenómenos jurídicos como son los principios. Como consecuencia
de esto, la tesis sobre la discrecionalidad se vería seriamente dañada ya que
la ausencia de reglas no daría lugar directamente a la discrecionalidad, sino
a otro tipo de análisis jurídico: el basado en principios. Mientras que para
Hart la ausencia de reglas jurídicas equivale a ausencia de solución correcta
y por tanto abre la discrecionalidad, para Dworkin la ausencia de reglas jurí-
dicas abre paso a la utilización de principios, los que de una u otra manera

EDUBP | Abogacía | Filosofía y Lógica Jurídica - pag.116


permitirán resolver al juez la discusión basándose en derecho. Para decirlo
de forma diferente, Hart sostendría que el derecho es un conjunto de reglas
que dependen de cierta convergencia o práctica social. Si no hay acuerdo
o convergencia social regular (los juristas desacuerdan), no hay reglas; si
no hay reglas, el juez debe decidir por su cuenta y no tiene material jurídico
en qué apoyar su decisión. Dworkin, en cambio, sostiene que el derecho es
más cosas que reglas. Si se acaban las reglas habrá principios, y ellos nos
permitirán solucionar un caso con arreglo al derecho.

Para profundizar en la discusión entre Dworkin y Hart, los


invitamos a ver el siguiente video:
Video Corto sobre Crítica de Dworkin a Hart: (20 minutos)
https://www.youtube.com/watch?v=MfSRaXY4xh8

Normas y Principios
Los principios son definidos por Dworkin en un sentido amplio como están-
dares que no son reglas. Cabe distinguir dentro del dominio de los principios
dos clases diferentes, a saber: a. principios en sentido estricto o están-
dares que son exigencias de justicia, equidad o alguna otra dimensión
moral –pensemos en principios como “debido cuidado”, “debido proceso”,
“nadie puede enriquecerse por su propio delito”, “derecho a la autonomía
personal, “derecho a la vida”, “derecho a la libertad de expresión”, “dere-
cho a ser tratado con igual consideración y respeto”, etc. y, b. directrices
o estándares que proponen un objetivo colectivo político, económico o
social -por ejemplo, defensa nacional, bienestar social, promoción de la
salud o la educación, etc-. De los ejemplos se desprende que los principios
son fines o propósitos que justifican decisiones institucionales.

Las normas y los principios se asemejan en que ambos apuntan a decisio-


nes particulares relativas a un orden jurídico. Pero hay una diferencia con-
ceptual o lógica, según Dworkin, entre normas y principios, pues:
• Las normas tienen una estructura disyuntiva, en el sentido de que son
o no son funcionalmente importantes según sean válidas o no, se identi-
fican por su origen y, es posible precisar y enumerar las excepciones
a su aplicación.
• Los principios no son concluyentes, son muy generales e inespecíficos
y compiten entre sí orientando en diferentes sentidos o direcciones
la decisión en un caso, no determinan ni las condiciones, ni las con-
secuencias de su aplicación, las excepciones a su aplicación no son
susceptibles de enumeración y, por último, se identifican por su peso
argumentativo o importancia relativa al competir con otros principios
en un caso en mano y, no por su origen. Además, no especifican el
contenido de los derechos ni los deberes correlativos a esos derechos.

Sobre este último punto será uno de los puntos más trascendentales de la
obra de Dworkin para entender los estados de derechos constitucionales
modernos. Efectivamente llama la atención concretamente el conflicto nor-
mativo que puede existir entre principios jurídicos. Sobre el cual volveremos
al final de este módulo.

EDUBP | Abogacía | Filosofía y Lógica Jurídica - pag.117


Dworkin distingue dos modos de tratamiento teórico de los principios: por
un lado, pueden ser analizados como vinculantes jurídicamente para los
jueces, con lo cual, los principios constituyen criterios que se imponen al
tribunal para decidir en un sentido o en otro en cumplimiento de derechos
y obligaciones jurídicamente preexistentes, aún en casos difíciles. Este es el
modo de analizarlos que Dworkin considera correcto. Por otro lado, pueden
ser tratados, como Hart lo hace según este autor, como no vinculantes jurí-
dicamente para los jueces, de modo que se los utiliza en casos difíciles
como parte de la elección discrecional del juez cuando decide un caso difícil
creando ex post facto derechos y obligaciones nuevos.

Respecto del concepto de discrecionalidad, Dworkin distingue tres sentidos


diferentes en que puede ser entendido, a saber: a.- un sentido débil como
discernimiento que el órgano aplicador de la regla tiene que ejercer debido a
que, en ciertos casos, su aplicación mecánica resulta imposible; b.- un sen-
tido débil en relación a la autoridad de aplicación, la dificultad que plantean
ciertos casos y el carácter definitivo de las decisiones en cierta instancia; c.-
un sentido fuerte, que es el que atribuye a Hart, según el cual, por discre-
cionalidad se describe la situación del juez como una situación en la que no
se encuentra vinculado por estándares o principios por considerar que los
principios, por problemas de alcance y aplicabilidad, no determinan de modo
concluyente un resultado jurídico, pues su autoridad y peso son discutibles.

Por lo tanto, si los principios son derecho hay que abandonar, según Dworkin,
los tres dogmas del positivismo, a saber:
• La tesis de las fuentes, según la cual, las normas son válidas por su pedi-
grí u origen.
• La tesis de la discrecionalidad judicial en casos difíciles.
• La idea de obligación jurídica proveniente de normas válidas por su origen.

Ronald Dworkin opone a las tesis del positivismo la tesis de los derechos,
según la cual:
• El deber del juez es descubrir qué derechos tienen las partes aún en los
casos difíciles.
• Las decisiones judiciales siempre imponen derechos jurídicamente existentes.
• Las decisiones judiciales siempre son y deben ser generadas por princi-
pios- derechos.
• La decisión judicial correcta se enmarca dentro de una teoría política.
• La decisión judicial correcta debe ser coherente con decisiones anterio-
res e hipotéticas futuras.
• La coherencia de la decisión judicial con el resto del material jurídico
se debilita si juegan en su justificación directrices, pues éstas no exigen
igualdad y distribución equitativa de derechos individuales, sino que
apuntan a la realización de fines colectivos.

EDUBP | Abogacía | Filosofía y Lógica Jurídica - pag.118


Dworkin también realiza críticas a la concepción tradicional de la función judi-
cial, sostenida por Hart. Las críticas pueden sintetizarse del siguiente modo:
• Subordina la función judicial a la legislación porque ignora la diferencia
entre argumento de principio y argumento político. La delegación excep-
cional de facultades legislativas a los jueces en casos difíciles –discre-
cionalidad– es una explicación inadecuada de cómo deben resolver los
jueces casos de incertidumbre.
• Hay una diferencia relevante entre legislación y jurisdicción. Los legislado-
res son elegidos por la mayoría y la representan para decidir cuestiones
colectivas, en consecuencia, pueden justificar sus decisiones valiéndose
tanto de argumentos que apuntan a la realización de objetivos colectivos,
como de argumentos que atienden a los derechos de los individuos. Los
jueces, en cambio, no son responsables frente a electorado alguno, sus
cargos no son electivos precisamente porque la finalidad de su función
es garantizar los derechos de las personas; en consecuencia, no pueden
actuar como si fuesen legisladores, sólo pueden justificar sus decisiones
en argumentos de principio –derechos. En otros términos, considerando
que los principios en general son finalidades que justifican decisiones
y que teniendo en cuenta la finalidad hemos distinguido dos clases de
principios -principios en sentido estricto y directrices políticas-; cabe, en
consecuencia, distinguir dos clases de argumentos justificativos de la
decisiones institucionales, a saber: argumentos políticos que justifican
decisiones demostrando que favorece algún objetivo social y, argumen-
tos de principio que justifican decisiones institucionales demostrando
que satisfacen derechos individuales. La justificación de un programa
legislativo puede darse en función de ambos tipos de argumentos, en
cambio, la justificación de una decisión judicial sólo puede darse por
medio de argumentos de principio.
• Las dos objeciones más relevantes al poder creador de los jueces en
casos difíciles son:
a.- La sociedad está gobernada por hombres elegidos por la mayoría
–legisladores- y;
b.- que el juez crea derechos nuevos implica conceder la creación retroac-
tiva de derecho, esto es, creación de derechos y obligaciones después
del hecho sometido a juzgamiento.

Para repasar la estructura de las críticas de Dworkin en “el


modelo de las Normas 1” los invitamos a ver el siguiente video:
https://www.youtube.com/watch?v=nksXj9Z3Mtw

EDUBP | Abogacía | Filosofía y Lógica Jurídica - pag.119


Hart y la defensa del positivismo jurídico conceptual o metodológico.
Según Hart3, una teoría del derecho tiene que ser general y descriptiva en
el sentido que debe describir la forma y estructura general de una práctica
gobernada por reglas, de un modo valorativamente neutral y sin pretensio-
nes justificatorias. La teoría del derecho propuesta por Ronald Dworkin, en
cambio, es normativa –justificatoria– y particular, pues evalúa un derecho
positivo en particular; el derecho anglo americano. En efecto, la teoría de
Dworkin es en realidad una teoría de la adjudicación, en términos del propio
Hart un “prólogo de la decisión judicial”. Si una teoría del derecho es parte
de la adjudicación judicial esto implica tratar la teoría como parte de la prác-
tica del derecho, visto desde el punto de vista interno de los jueces. En otros
términos, funde la teoría en la práctica que se pretende explicar.

Una teoría general del derecho, según Hart, puede describir el punto de vista
interno de los participantes, sus actitudes y valoraciones, desde un punto de
vista externo. Dicho de otro modo, la teoría del derecho es descriptiva de las
creencias de los participantes respecto de la existencia de obligaciones y razo-
nes para actuar, sin que ello implique asumir teóricamente la existencia obje-
tiva de tales obligaciones y razones. Si bien el rasgo de dar razones y estable-
cer deberes constituye un carácter normativo distintivo de las reglas jurídicas,
describir las creencias de los participantes sobre la existencia de razones y
obligaciones no compromete a afirmar la existencia objetiva de hechos norma-
tivos. Si hay o no hechos normativos es un debate abierto y, la teoría jurídica
no puede reposar en tal presupuesto ontológico controvertido. Además, no
todos los deberes jurídicos reposan en razones morales o tienen fuerza moral.

Por otro lado, interpretar el derecho para determinar las condiciones de su


aplicación presupone su identificación previa por apelación a hechos socia-
les –fuentes-.

En cuanto a la diferencia entre normas y principios Hart afirma que no hay


una distinción conceptual o lógica entre reglas y principios, la diferencia
entre ambos es sólo una cuestión de grado. En efecto, las normas o reglas
son más específicas o determinadas que los principios, pero tanto las reglas
como los principios poseen textura abierta.

La regla de reconocimiento proporciona criterios que dan mayor certeza res-


pecto de lo que ha de identificarse por derecho, pero no certeza absoluta,
pues sus criterios adolecen de imprecisión o textura abierta. Ahora bien, la
interpretación de principios sugerida por Dworkin como medio de identifica-
ción del derecho aplicable tampoco resuelve el problema de falta de certeza
respecto de lo que es derecho en ciertos casos, pues los principios también
poseen textura abierta. Hemos visto que la textura abierta es un problema
imposible de erradicar totalmente del lenguaje que empleamos. Por medio
de la interpretación se resuelve la aplicación o no de una regla o principio
en un caso dado, pero esa regla o principio seguirá siendo indeterminado o
vago en el futuro.

Los principios, según Dworkin, sólo pueden ser identificados por interpre-
tación reconstructiva de todo el material jurídico existente -única interpreta-
3 H. L. A. Hart; “Postscriptum”, en La Decisión Judicial: El debate Hart – Dwokin, Siglo del Hombre
Editores, Universidad de los Andes, 5ª reimpresión, Bogotá, Colombia, 2005.

EDUBP | Abogacía | Filosofía y Lógica Jurídica - pag.120


ción correcta-, pues muestra al derecho aplicable en su mejor perspectiva.
Pero este tipo de interpretación sólo puede ser llevada a cabo por un juez
ideal, esto es, conocedor de todo el derecho explícito e implícito existente al
momento de su decisión. Por otra parte, el que accede al conocimiento del
derecho es un participante de la práctica particularmente calificado, el juez y
no un teórico. Es por esto que decimos que Dworkin disuelve la teoría en la
práctica judicial del derecho.

Por otra parte, según Hart, no hay dos etapas diferentes en el proceso de
decisión judicial: una en la que el juez descubre que el derecho no da solu-
ción y otra en la que crea derecho ex post. El recurso a la analogía en casos
difíciles (búsqueda de casos anteriores ya resueltos similares al que está
sujeto a resolución) no elimina la creación, pues, pueden presentarse dife-
rentes principios que sostienen analogías compitiendo y, hay que hacer una
elección, pues según Dworkin no hay principios ordenados en un grado de
primacía o subordinación jerárquica.

Por último, Hart sostiene que cuando un derecho positivo es injusto o contiene
normas moralmente malas un juez, apelando a los principios subyacentes,
tendrá que basar su decisión priorizando los principios menos malos y, las
consecuencias que de esto se infieren no se distinguen de las del positivismo.

En suma la cuestión que estos autores debaten es si en casos difíciles el juez


descubre o crea derecho ex post. Responder a esta cuestión es importante
porque de la respuesta dependerá que contemos con un criterio para distin-
guir una decisión justificada jurídicamente de una decisión judicial arbitraria.

Los desacuerdos teóricos abarcan diferencias relativas a los aspectos que


deben considerarse relevantes en el análisis del derecho. Los adherentes
al positivismo exclusivo -Hart- sostienen que el aspecto central del derecho
radica en el carácter autoritativo de los estándares que generan razones para
la acción, mientras que para aquellos que postulan el carácter interpretativo
de la teoría -tesis de la coherencia- la característica relevante del derecho es
la adjudicación, esto es, los modos en que los jueces deciden casos y lo que
ellos piensan que es el derecho.

Los sostenedores de la tesis social podrían conceder que las conclusiones


extraídas por los coherentistas respecto a cómo deben decidir los jueces
son adecuadas, pero desacuerdan con la idea de que ese sea el propósito
asignable a una teoría del derecho. Por el contrario, piensan que el análisis
impone describir lo que de hecho hacen los jueces cuando deciden casos,
pues, si la finalidad de una directiva jurídica de regular la conducta humana
dependiera de su derivación de algún estándar moral, el derecho se disolve-
ría en juicios morales de los decidores.

No obstante, a pesar de la variedad de desacuerdos existentes, se intenta


examinar la posibilidad de argumentos que intenten conciliar todas las tesis
en discusión, eliminando incompatibilidades y, permitiendo integrarlas en una
teoría general del derecho unitaria y que abarque una teoría de la adjudicación.

EDUBP | Abogacía | Filosofía y Lógica Jurídica - pag.121


Algunos autores han sugerido alternativas de superación de las dicotomías
planteadas a partir de la idea de que es función de una teoría explicar los
diferentes puntos de vista sobre el derecho sostenidos por actores sociales
que integran diferentes sectores, por un lado los individuos que integran el
sector privado, esto es, los ciudadanos y por otro, aquellos que forman parte
del sector público u oficial, en especial, los jueces. Suele afirmarse que las
concepciones características de los miembros de cada uno de estos sectores
no coinciden. Estos autores piensan que la tesis social representa adecua-
damente la función que el derecho tiene en la relación estado-ciudadano, en
tanto que la tesis de la coherencia proporcionaría una explicación adecuada
del rol institucional del derecho en la aplicación a la resolución de casos.

De la disolución de los desacuerdos depende la posibilidad de diferenciar


una resolución judicial válida jurídicamente de una decisión arbitraria. Las
diferentes partes en el debate unánimemente aceptarían la siguiente máxima:
una decisión judicial justificada jurídicamente sería aquella que maximiza la
racionalidad y minimiza la arbitrariedad.

El tema relativo a las condiciones de justificación racional o lógica de las


decisiones judiciales será motivo de tratamiento en el último módulo del pre-
sente programa.

La ponderación de principios:
Volvamos a Dworkin, para poder entender la relevancia de esta discusión
para la práctica jurídica y la forma en que se debe argumentar en derecho
frente a los casos difíciles que involucran principios.

La atención brindada por Dworkin a la existencia y funcionamiento de los prin-


cipios, permiten comprender algunos aspectos fundamentales de los Estados
Constitucionales de Derecho y de la existencia de derechos fundamentales
que generalmente están expresados en forma de principios jurídicos.

Es decir, en todo ordenamiento coexisten las reglas y los principios, un ejem-


plo sería el nuestro, donde, por ejemplo, tenemos reglas y principios dis-
puestas en los diferentes Códigos jurídicos, y también principios contenidos
en el bloque constitucional (Constitución Nacional y Tratados de Derechos
Humanos en los que la Nación es parte), lo cual genera ciertos conflictos.

Dworkin (1989) indica que “los principios tienen una dimensión que falta en
las normas: la dimensión del peso o importancia” (pag.77).

En este sentido (colisión reglas-principios), Bayón (1996) sostiene:


en un derecho de principios y reglas la solución prevista por la regla
goza de una presunción prima facie de aplicabilidad que solo puede
ser desvirtuada en un caso concreto mediante una argumentación
basada en principios (esto es: vale >>Salvo principio en contrario>>)
lo cual dota a las reglas de una fuerza en la justificación de decisiones,
sino irrebatible, tampoco despreciable (pág. 48).

EDUBP | Abogacía | Filosofía y Lógica Jurídica - pag.122


Sin embargo, Dworkin (1989) señala que detrás de toda regla se puede
encontrar un principio que la justifica; por ende, y en cuestiones prácticas
dentro de un sistema de reglas y principios, la decisión en concreto del caso
particular difícil no sería un conflicto de reglas y principios; sino finalmente un
conflicto de principios

De esta forma, es donde la ponderación tiene su mayor fundamentación


práctica. Es decir, en un sistema de reglas y principios, donde las primeras
a su vez pueden estar fundamentadas o ser el reflejo de la optimización de
ciertos principios, en los llamados casos difíciles donde principios colisionan
en un caso concreto, el juez debe ponderar. En este sentido, Dworkin (1989)
indica que “cuando los principios se interfieren quien debe resolver el con-
flicto tiene que tener en cuenta el peso relativo de cada uno” (pág. 77).

En los casos de conflictos fundamentales será principalmente la Corte Suprema


quien defina en última instancia la cuestión de la ponderación de los dere-
chos. Sin embargo, hay quienes se oponen dentro de la filosofía del derecho,
sobre a quienes les corresponde en última instancia resolver los conflictos de
derechos fundamentales; hay quienes, como Habermas, piensan que éstos se
deben resolver en una democracia por los legisladores que son los represen-
tantes de la ciudadanía a través de debates públicos. Éstos son los que deben
establecer los límites que optimicen el respecto de los derechos fundamenta-
les a través de la regulación de los casos generales. (Villanueva, 2019)

Remitiremos a la clara exposición de Moreso (2006) sobre los diversos mode-


los de ponderación existentes y la presentación de su propio modelo.

Moreso, J. (2010) Conflicto de derechos constitucionales y como resolverlos


∙ Extraído del siguiente sitio: https://bit.ly/3wGbdee

Allí se hace una distinción muy relevante entre modelos ponderativos, pro-
pone y aporta una suerte de modelo propio en base a ellos. Dicho autor
hace una introducción base donde plantea diversos casos reales, como por
ejemplo el presente en la jurisprudencia española, llamado “El caso del Niño
Marcos”. Sin posibilidad de transcribir todo el caso por razones de obviedad
de económica de la lectura, se remite a los puntos más importantes y que a
los fines explicativos sirva. Marcos A.V de 13 años sufre un accidente el cual
en prescripción médica se insinúa la necesidad imperiosa de hacer trans-
fusión sanguínea; la cuestión fáctica problemática está en que tanto el niño
como sus padres (tutores decisores) eran de una religión que no permite
dicha actividad de transfusión (testigos de jehová). De esta manera, el niño
termina por no recibir dicha transfusión y fallece a consecuencia de ello. Los
padres de Marcos fueron acusados del delito de homicidio. La sentencia de
la Audiencia Provincial de Huesca los absuelve, el Tribunal Supremo caso
y anulo dicha absolución dictando sentencia condenatoria por el delito de
homicidio con la circunstancia atenuante, y el Tribunal Constitucional resolvió
concederé al amparo, declarar vulnerado el derecho fundamental a la liber-
Lectura tad religiosa y, en consecuencia, anular la decisión del Tribunal Supremo.
(desde página 177) Como se desprende de este resumen de un caso jurisprudencial real, “apa-
rentemente el derecho a la vida de Marcos y el derecho a la libertad religiosa
de Marcos y de sus padres, están en conflicto”

EDUBP | Abogacía | Filosofía y Lógica Jurídica - pag.123


Moreso (2006) distingue dos modos contrarios y no complementarios tratar
a los conflictos de principios o derechos fundamentales. Por un lado, la que
denomina “Concepción Subsuntiva”:
Se trata de reconstruir adecuadamente las pautas que han de guiar
nuestra conducta y una vez realizada dicha operación de delimitación
de nuestras pautas, aplicarla al caso concreto. La apelación a la pauta
moral adecuada, junto con la afirmación de que el supuesto en cues-
tión es subsumible a ella, permite justificar la decisión (pág. 15).

Este modelo parte de una premisa básica fundamental: “delimitar adecua-


damente el contorno de los comportamientos incluidos en cada una de las
normas que establecen derechos fundamentales de forma que encajen per-
fectamente entre sí” (Moreso, 2006 , pág. 17); de esta forma, se sostiene,
los derechos fundamentales, al estar demarcados en cada uno una zona de
aplicación, no colisionarían nunca porque no tendrían zonas de conflictos, es
decir, zonas comunes. Además de esta tarea de delimitación, sería necesario
realizar una jerarquización de los derechos fundamentales, con ello, se tendría
de antemano la cadena de valor de los mismos. Bajo este modelo, el juez solo
tendría que individualizar el derecho fundamental aplicable que estaría perfec-
tamente individualizado y jerarquizado, no suponiendo mayores problemas.

Por otro lado, la “Concepción Particularista” prescribe que: no hay tal cosa
como pautas universales en las cuales anclar la corrección moral de las
acciones, hay que atender siempre al modo en que, en el caso individual,
se presentan entrelazadas las circunstancias y ponderarlas con el objeto de
percibir la solución correcta para dicho caso (Moreso, 2006 , pág. 15).

Lo básicamente contrario entre estas concepciones esta dado porque el par-


ticularismo no acepta la idea de que lo aplicable a un caso, también sea
aplicable a otro, tal como plantea la concepción Subsuntiva. Es decir, se le
da importancia, desde el particularismo, a las circunstancias extraordinaria-
mente particulares de cada caso, lo cual imposibilita la aplicación igualitaria
de los derechos en todos momentos. Por ello, “la corrección de una acción,
para la concepción particularista, no depende de su capacidad de ser subsu-
mida a una pauta general, sino que depende de cómo estén moldeadas en
ese caso concreto las circunstancias” (Moreso, 2006 , pág. 18).

Ante el desafío particularista, surgen como mecanismo o modelo de pon-


deración ponderación de principios “proporcionalista”, que tiene entre sus
máximos exponentes a a Alexy, este indica:

Cuando dos principios entran en colisión (…) uno de los dos ha de


ceder frente al otro. Pero esto no significa que uno de los dos princi-
pios sea invalido, ni que en el principio desplazado haya que introducir
alguna excepción. Lo que sucede es que, en determinadas circuns-
tancias, un principio precede al otro. Es por esta razón que se afirma
que, en los casos concretos, los principios tienen diferente peso y el
conflicto ha de resolverse según la dimensión de peso y no según la
dimensión de validez (pág. 17).

Y propone una fórmula de peso para resolver esta clase de conflictos a las
remitimos en el texto.

EDUBP | Abogacía | Filosofía y Lógica Jurídica - pag.124


Por su parte, Moreso (2006) propone lo que el determina como una vía inter-
media,
una vía según la cual la ponderación es únicamente la operación que
permite pasar de las normas que establecen derechos fundamentales,
que tienen la estructura de principios –pautas con las condiciones de
aplicación abiertas- a reglas –pautas con las condiciones de aplicación
clausuradas- con las cuales es posible llevar a cabo la subsunción, en
el ámbito de un problema normativo determinado (pág. 22).

Este modelo propuesto por Moreso (2006) al que lo nombre como el “modelo
especificacionista” y que consta de cinco etapas:
1. Consiste en la delimitación del problema normativo del universo del dis-
curso;
2. Identificación de las pautas prima facie aplicables a este ámbito de acciones;
3. La consideración de determinados casos paradigmáticos, reales o hipoté-
ticos, del ámbito normativo previamente seleccionado en la primera etapa;
4. Establecer las propiedades relevantes de ese universo del discurso;
5. Formulación de las reglas que resuelven de modo univoco todos los
casos del universo del discurso.

Básicamente, este modelo, que intenta Moreso al momento de fusionar dos


concepciones antagónicas sobre la aplicación conflictual de derechos fun-
damentales, puede ser entendido como un proceso, donde primero se debe
ponderar las circunstancias particulares del caso y demás casos parecidos
o que compartan propiedades relevantes, y luego crear una regla, regla que
será aplicable a casos donde las circunstancias particulares del mismo sean
de la misma cualidad que las tenidas en cuenta para la resolución del ante-
rior. Es decir, una regla que sirve para que sea aplicable en la posterioridad
con el método subsuntivo.

El sistema de Moreso, con estas cinco etapas pretende ser un modo de


concebir la ponderación que lo hace compatible con la subsunción y con
una limitada generalidad. Las reglas surgidas de una reconstrucción como
la propuesta se aplican de modo subsuntivo y permiten articular y otorgar
coherencia a la función judicial.

A los fines de completar el estudio de los temas del módulo diríjase a las
siguientes lecturas:
- Hart, H. L. A.; El Concepto de Derecho. Buenos Aires, Abeledo Perrot.1968,
capítulo 7.
- Calsalmiglia, Albert; “Por qué es importante Dworkin? en Doxa Revista
de Filosofía del Derecho, 1965, Nº 2, págs.159-165. (A este material lo
encontrará en las actividades de este módulo).

Hart, H.L.A.; “Poscriptum”, en La Decisión Judicial: El debate Hart – Dworkin,


Bogotá, Colombia. Siglo del Hombre Editores, Universidad de los Andes, 5ª
reimpresión, 2005.

EDUBP | Abogacía | Filosofía y Lógica Jurídica - pag.125


Lo invito a realizar las actividades del módulo, a fin de poner en práctica los
conceptos estudiados.

Al finalizar el estudio del módulo 6, usted está en condiciones de realizar las


consignas de la Segunda Parte de la evaluación.

M6 Actividades
Actividad 1

El “Caso García” (STC 105/ 1990). Caracterizado de un fallo desde la


perspectiva de Hart.
El periodista José María García fue condenado por un delito de desacato
(delito que se comete al insultar a una autoridad en el ejercicio de sus fun-
ciones o con ocasión de éstas) contra José Luis Roca, Presidente de la
Asociación Española de Fútbol y Diputado de las Cortes de Aragón. García
difundió la información de que Roca había cobrado determinadas dietas por
supuestos desplazamientos a Zaragoza que, sin embargo, no se habían pro-
ducido. El Tribunal Constitucional recordó que, según sus propios fallos, el
derecho a la información goza de máxima protección cuando la información
se refiere a una personalidad pública, se vincula con la formación de una
opinión pública libre y quien la difunde es un profesional de la información en
el ejercicio de su función. En el caso considerado se daban todos los requi-
sitos, además del de veracidad (entendida como información comprobada
según los cánones de la profesión informativa), pero el Tribunal no amparó a
García porque éste había emitido, según dice el fallo, apelativos formalmente
injuriosos en cualquier contexto e innecesarios para la labor informativa o de
formación de la opinión; la Constitución, señalaba el Tribunal, “no reconoce
un pretendido derecho al insulto”.

A. Explique si el caso García constituye o no un caso difícil y exponga las


razones.
B. Indique si la decisión del Tribunal Constitucional en el caso García puede
ser considerada una decisión discrecional en el sentido de Hart y haga
referencia a las críticas que Dworkin formularía a tal descripción de la
decisión del Tribunal Constitucional.
AA

AA Actividad 1
asistente académico

Para la resolución de esta actividad, le sugerimos consultar los conteni-


dos desarrollados en el presente módulo y el capítulo 7 de El Concepto de
Derecho de H. L. A. Hart.

EDUBP | Abogacía | Filosofía y Lógica Jurídica - pag.126


Actividad 2

Caracterización del Caso García desde la perspectiva de Dworkin


En el estudio donde usted se desempeña a diario, es habitual enfrentarse a
nuevos desafíos. El Caso García puede analizarse desde diferentes perspec-
tivas, es por ello que en esta oportunidad usted quiere dar un paso más y
continuar analizándolo.

A los fines de iniciarse en su tarea, un colega le propone continuar traba-


jando el caso en cuestión realizando las siguientes actividades:
A. Indicar cuáles son los principios – derechos – en competencia en el caso
planteado.
B. Exponer el procedimiento o método que conduce a la decisión correcta
según Dworkin y, analizar si el fallo del Tribunal Constitucional satisface las
condiciones requeridas para ser considerado correcto en el caso García.
AA

AA Actividad 2
asistente académico

Para la resolución de esta actividad consulte los contenidos del presente


módulo y el artículo de Albert Calsalmiglia ¿“Por qué es importante Dworkin?
∙ , Doxa, 1985, Nº 2.

Actividad 3

Distinguiendo normas y principios


Su colega queda sumamente conforme con su trabajo de análisis, no obs-
tante le realiza algunas preguntas asociadas a las siguientes cuestiones:
¿Cuáles son los principios invocados en el fallo del caso García? ¿Cómo
se diferencian de las normas aplicadas en el mismo -leyes que regulan los
deberes inherentes al correcto desempeño de la función pública-?
AA

AA Actividad 3
asistente académico

Recuerde consultar los contenidos desarrollados en el presente módulo y el


artículo de Albert Casalmiglia ¿“Por qué es importante Dworkin”? Doxa, 1985,
Nº 2, brindado en la actividad 2 del presente módulo.

Lectura
(desde página 178)

EDUBP | Abogacía | Filosofía y Lógica Jurídica - pag.127


Actividad 4

Distinguiendo entre juridicidad y arbitrariedad de las decisiones judiciales.

En relación a la controversia planteada entre la Cámara Federal de Paraná


y la C.S.J.N en el caso “Bustos, Alberto Roque y otros c/ Estado Nacional y
otros s/amparo” (Consultar extracto de fallo ∙ ), aplique el método de
interpretación de principios sugerido por Ronald Dworkin y:
• Distinga los diferentes tipos de principios que compiten orientando las
decisiones opuestas del caso controvertido.
• Considere cuál de las decisiones judiciales en juego, si la sostenida por
la Cámara Federal de Paraná o la de la C.S.J.N, es correcta desde la
perspectiva de Dworkin y fundamente su conclusión.
• Analice el caso tomando en consideración la cuestión de la incertidum-
bre de la regla de reconocimiento de H. L .A. Hart.

Consulte para el desarrollo de la actividad los contenidos del presente módulo


y el capítulo 7, último punto del Concepto de Derecho de H. L. A. Hart.

Lectura
(desde página 179)

EDUBP | Abogacía | Filosofía y Lógica Jurídica - pag.128


MÓDULO 7

M7 Microobjetivos

• Conocer el manejo de las herramientas proporcionadas por la lógica, para


reconstruir, del modo más racional posible, los conceptos normativos.
• Comprender los criterios para producir mejoras en las capacidades argu-
mentativas requeridas para sobrellevar los debates y litigios, caracterís-
ticos de nuestra práctica profesional.
• Reconocer la utilidad de la lógica deóntica para proceder en la reformula-
ción precisa de las normas jurídicas y sus relaciones recíprocas.

M7 Contenidos

Acceda al video de presentación del Módulo 7

1. Lógica y lenguaje del derecho


En los módulos anteriores hemos analizados las cuestiones que suelen deno-
minarse lógico filosóficas de la teoría general del derecho. Nos hemos
interiorizado de la visión de la teoría del derecho expuesta por Hart que
constituye uno de los desarrollos más sofisticados del positivismo jurídico.
Tuvimos oportunidad de analizar las críticas que Hart dirige a otros modelos
de teoría del derecho, fundamentalmente al modelo del positivismo clásico
de J. Austin y al modelo del escepticismo o realismo jurídico. Lo importante
es esclarecer por qué denominamos a las cuestiones tratadas hasta aquí
lógico- filosóficas y la razón es la siguiente: son análisis que reconstruyen o
explican el fenómeno jurídico a partir del estudio del significado en uso de
los términos y expresiones jurídicas haciendo explícitos los problemas sin-
tácticos y semánticos de ambigüedad y textura abierta del lenguaje jurídico;
problemas que tienen su origen en la imprecisión de las reglas que guían el
uso ordinario de dicho lenguaje.

En lo que sigue analizaremos la implementación o aplicación de las técnicas de


formalización y reglas de inferencia estrictamente lógicas al lenguaje jurídico.

La relación estrecha entre lógica y lenguaje es indiscutible, en particular cuando


nos ocupamos del análisis del significado. En efecto, usamos el lenguaje en
las argumentaciones presentes en nuestras conversaciones y debates.

Ahora bien, posiblemente usted se pregunte: ¿cuál es el propósito de la


lógica? Este último consiste en elaborar una teoría general que suministre
criterios que permitan distinguir los buenos argumentos de aquellos que no
lo son. La práctica de la argumentación consiste en sostener un enunciado
a partir de otro y, para entender lo que hacemos cuando argumentamos,
necesitamos hacer explícitos los criterios que constituyen esa práctica y para
esto necesitamos la lógica. Si observamos la práctica jurídica veremos que la
misma es una práctica esencialmente argumentativa, tanto en el: 1) ámbito de
la creación del derecho, por ejemplo cuando los legisladores discuten sobre

EDUBP | Abogacía | Filosofía y Lógica Jurídica - pag.129


cómo regular una determinada realidad social. 2) En el ámbito de la ciencia
jurídica, por ejemplo, cual el doctrinario argumenta sobre cómo solucionar
un caso difícil en la identificación del derecho aplicable. 3) En el ámbito de la
aplicación del derecho, cuando los litigantes alegan o los jueces fundamen-
tan sus decisiones. La competencia básica de cualquier abogado es saber
argumentar correctamente. En términos generales, como afirma Weston,
argumentar es dar razones a favor de una determina conclusión o tesis.

Ahora bien, la estructura de la argumentación es el razonamiento que posee


dos partes principales premisas y conclusión. La segunda es la afirmación
que se deriva de las primeras. Los razonamientos se dividen en razonamien-
tos deductivo y no deductivos, o inductivos en sentido general.

Los razonamientos no deductivos o inductivos en sentido general son la


inducción por generalización incompleta, la analogía y la abducción.

Nosotros en esta materia nos dedicamos principalmente a los razonamientos


deductivos y a las estrategias de formalización del lenguaje y la prueba de
validez a partir de las tablas de verdad.

En el presente módulo, comenzaremos presentando los razonamientos


deductivos principales.

Principales razonamientos deductivos válidos:

Los principales razonamientos deductivos válidos son importantes porque


nos permiten comprender las principales reglas de inferencias.

Denominamos reglas de inferencia a los principios o reglas que la lógica


elabora y, deducción al proceso de obtención de un enunciado a partir
de otros.

Hay infinitos modos de construir razonamientos deductivos y en muchos


casos serán válidos por su estructura, tendremos que cumplen la condición
de que las premisas sean verdaderas y que no de un resultado o una con-
clusión falsa.

Los razonamientos deductivos que lo que necesariamente hacen es pasar


de premisas verdaderas a conclusión verdadera, y no podía darse el caso de
que tuviéramos premisas verdaderas y conclusión falsa. Si el esquema si la
estructura del razonamiento, nos permite que las premisas sean verdaderas
y la conclusión falsa estamos frente a un razonamiento inválido.

Las principales formas de razonamiento deductivos válidos, son muy utiliza-


dos en el ámbito jurídico, es decir, la forma en la que hacemos las construc-
ciones lógicas cuando tenemos que fundamentar un escrito, o un juez tiene
que fundamentar una sentencia; habitualmente se utiliza una gran estruc-
tura lógica, es decir, estos razonamientos deductivos para poder hacer sus
demostraciones.

Podemos hacer una distinción entre razonamientos categóricos y compues-


tos, y dentro de ellos los silogismos disyuntivos excluyentes e incluyentes,

EDUBP | Abogacía | Filosofía y Lógica Jurídica - pag.130


silogismo hipótetico puro, modus ponens, modus tollens y dilemas, tanto
simples como compuestos.

Se conoce como silogismo a un argumento compuesto por tres proposicio-


nes; de ellas, la última es la que se deduce siempre de las anteriores.

El primero de estos razonamientos es el silogismo categórico.

Hay que subrayar que el padre del silogismo es el gran pensador y filósofo
griego Aristóteles.

Puede considerarse como una modalidad de razonamiento deductivo, donde


dos de las proposiciones categóricas son premisas y la restante proposición
categórica funciona como conclusión.

Las proposiciones categóricas o simples son aquellas que están compues-


tas por un cuantificador (todos, ninguno, algunos, algunos no), un término
sujeto, un verbo copula (traducible a el verbo en infinitivo ser) y un término
predicado.

Por ejemplo,

“Todos los juicios ejecutivos son procesos breves.”


“Todos” es el cuantificador.
“juicios ejecutivos” es el término sujeto.
“son” es el verbo copula.
“procesos breves” es el término predicado.

Por su cantidad y el cualidad las proposiciones categóricas se dividen:


• En universales afirmativas “Todos los juicios ejecutivos son procesos breves”
• En Universales negativos “Ningún juicio ejecutivo constituye procesos breves”
• En particulares afirmativos “Algunos juicios ejecutivos son procesos breves”
• En particulares negativos “Algunos juicios ejecutivos no son juicios breves”

Entonces, un silogismo categórico es un silogismo cuyas premisas y conclu-


siones están compuestas por proposiciones categóricas.

Por ejemplo:
• Todos los juicios ejecutivos son procesos breves.
• Todos los juicios por cobro de un pagaré son juicios ejecutivos.
• Luego, todos los juicios por cobro de un pagaré son procesos breves.

El silogismo es una argumentación en la que, en base a un antecedente que


compara dos conceptos con un tercero, permite inferir o deducir un conse-
cuente.

EDUBP | Abogacía | Filosofía y Lógica Jurídica - pag.131


En otras palabras, se puede decir que el modelo de silogismo está com-
puesto por tres proposiciones que incluyen un término medio (el cual es
común a las dos premisas y se descarta en la conclusión) y dos extremos, el
termino mayor y menor.

El término medio en este caso es “juicios Ejecutivos”

Hay que tener en cuenta que un silogismo no siempre impulsa conclusiones


verdaderas, más allá de que siga una forma válida de razonamiento. Por
ejemplo:
“Todos los perros fueron cachorros”
“Snoopy es un perro”
“Por lo tanto, Snoopy fue cachorro”

Aunque el silogismo es válido desde un punto de vista lógico, la conclusión


tiene un carácter falso, ya que Snoopy es un perro ficticio y nunca fue cachorro.

Como vemos son muchas las combinaciones de tipo de premisas que pode-
mos utilizar para construir silogismos pero solo algunos serán válidos, para
ellos se deberán respectar determinadas reglas respecto a los términos y a
las proposiciones. No profundizaremos en ellas y no se pedirán ejercicios de
determinación de su validez, pero es útil conocerlas:
1. Los términos deben ser tres y emplearse con el mismo significado en
cada proposición.
2. El término medio debe tomarse en toda su extensión al menos en una
premisa.
3. El término medio debe tomarse en toda su extensión al menos en una
premisa.
4. El término medio debe tomarse en toda su extensión al menos en una
premisa.
5. El término medio debe tomarse en toda su extensión al menos en una
premisa.
6. De dos premisas negativas no se infiere nada.
7. De dos premisas particulares no se infiere nada.
8. De dos premisas afirmativas no se puede inferir una conclusión negativa.
9. La conclusión debe seguir la parte más débil: Si una premisa es universal
y otra particular, la conclusión es particular. Si una premisa es afirmativa
y la segunda negativa, la conclusión debe ser negativa.

Los razonamientos compuestos, se constituyen por al menos una premisa que


no es categórica o simple sino que son proposiciones que se conectan pro-
posiciones a través de conectivas lógicas como la disyunción o el condicional.

El silogismo disyuntivo incluyente tiene dos proposiciones como premisas;


donde la primera premisa es una fórmula que enlaza dos proposiciones (ató-
micas o moleculares) mediante una conectiva de disyunción incluyente. Una
disyunción que presenta una alternativa pero que permite que ambas alter-

EDUBP | Abogacía | Filosofía y Lógica Jurídica - pag.132


nativas sean verdaderas. Por ejemplo: “Me recibiré de abogado o de médico”
si no lo analizamos no se excluyen esas posibilidades es posible ambas por
eso, es como si esta tercera opción estuviera incluida “Me recibiré de abo-
gado o de médico, (o ambas)”. Por lo tanto, una disyunción incluyente, pre-
senta una proposición cuya conectiva es una disyunción incluyente (lo que
implica, que será verdadero en el caso que se diera la primera proposición,
que se de la segunda proposición, o ambas proposiciones sean verdaderas,
lo que no podía pasar es que fuera falso A y B, es decir, que no se dieran
ninguna de las opciones).

La segunda premisa es uno de los disyuntivos negados (–A v -B )

La conclusión es la formulación del otro disyunto afirmado. Ejemplo:


“El asesino fue el mayordomo o el chofer, (o ambos)”
“No fue el mayordomo”
Entonces, fue el chofer.

Así, si sabemos que fue el mayordomo o el chofer; o ambos; si luego deci-


mos que no fue el chofer, necesariamente estamos afirmando que ha sido el
mayordomo.

Estas construcciones las hacemos todo el tiempo, sabemos que puede ser una
cosa u otra o ambas a la vez, pero no puede ser que no sea alguna de ellas. Si
negamos una de ella, sabemos que necesariamente se da la otra opción.

A v B (A v B ) –A => B
-A
________
B

El segundo es El silogismo disyuntivo excluyente que otra versión de la


disyunción, que en este caso no permite que sean ambas a la vez. Es A o B,
una u otra. Traduciendo en lenguaje natural sería, por ejemplo, “el imputado
es culpable o inocente”, las categorías se excluyen entre sí. (La mayoría de
las clasificaciones son excluyentes).

Acá, la diferencia con el primer tipo de silogismo es que como la primera pre-
misa es un silogismo disyuntivo excluyente, también puedo afirmar, es decir,
una de ellas me da la negación de la otra, por ejemplo “el reo era culpable o
inocente, es inocente, luego no es culpable”

La primera premisa es una fórmula que enlaza dos proposiciones (atómicas


o moleculares) unidas mediante un conector disyuntivo excluyente.

La segunda de las premisas es uno de los disyuntos afirmado o negado.

La conclusión es la formulación del otro disyunto afirmado o negado:


(A v B) – A = > B
o
(A v B) B = > - A

EDUBP | Abogacía | Filosofía y Lógica Jurídica - pag.133


Ejemplo:
Este es un contrato gratuito u oneroso.
Es un contrato gratuito
Por lo tanto, no es oneroso.

La tercera forma de razonamiento válido es el silogismo hipotético puro


que se presenta bajo la siguiente fórmula:
A  B (A => B) (B => C) => (A => C)
BC
______
AC

Esto lo habíamos visto como una de las reglas de la implicación; tenemos dos
proposiciones TODAS con un condicional, por eso es hipotético puro, porque
las dos premisas iniciales son condicionales; estas dos premisas están enla-
zadas con una condicional: A enlaza a B y B enlaza a C, por lo tanto, la con-
junción es la primera con la tercera. Ejemplo: Si estudiamos (A) aprobaremos
(B) el parcial, y si aprobamos (C) el parcial regularizaremos la materia.

Silogismo hipotético Mixtos – Modus Ponens y Modus Tollens


Se llama hipotético mixto al silogismo que está formado por una premisa
mayor hipotética, una premisa menor categórica y una conclusión también
categórica.

Tiene dos modos principales: modus ponens (afirmativo) y modus tollens


(negativo).

En el modus ponens la primera fórmula enlaza una premisa o proposición


(atómica o molecular) mediante una condición Si A entonces B

La segunda premisa es la afirmación del antecedente.

La conclusión es la formulación del consecuente afirmado.

Esta situación se ve mucho en la formulación de la sentencia judicial, es


decir, si alguien comente un delito, entonces será castigado; alguien cometió
un delito entonces es castigado.

El hecho que se diera A es lo que provoca que se deba concluir en B

La estructura formal del “modus ponens” es la siguiente:

EDUBP | Abogacía | Filosofía y Lógica Jurídica - pag.134


Si es A, B es A=> B (A =>B) . A =>B
Ahora bien, A es A____
Luego, es B B

(( p => q ) . p ) => q

Si tenemos una implicación y sabemos que el primer término es verdadero,


entonces podemos concluir el segundo término.
“el río tiene agua, entonces me mojo si entro en él”
Si entro en el río me mojo

El modus tollens tiene una estructura diferente se utiliza más en ciencias


La estructura formal del “modus tollens” es la siguiente:
Si A es, B es A=>B (A=>B). –B => -A
Ahora bien, B no es - B____
Luego, A no es. -A

(( p => q ) . –q ) => -p
Si sabemos que una implicación es verdadera, y sabemos que su segundo
término o conclusión es falso, entonces podemos concluir la negación del
primero.

La primera premisa es una fórmula que enlaza dos premisas o proposiciones


(Atómicas o moleculares) unidas mediante un conector condicional.

La segunda premisa es la negación del consecuente

La conclusión es la formulación del antecedente negado.

Ejemplo: “Si la teoría de la evolución es correcta, entonces descendemos


de los primates. Se observo que no descendemos de los primates. Luego la
teoría de la evolución es incorrecta.”

La forma inválida de este silogismo es la falacia de la negación del antecedente

Es decir, no es lo mismo negar el consecuente que negar el antecedente; en


el caso que tengamos una fórmula de negar el consecuente tenemos una
fórmula válida.

Dijimos que el modus tollens se utiliza mucho en ciencia ¿por qué?, porque
importantes epistemológos como Popper, dicen que realmente es la forma
en la cual debe funcionar la ciencia, debe formular sus hipótesis, si es cierta
la teoría tal debe darse la consecuencia observacional B, no se da la conse-
cuencia observacional B, en consecuencia, es incorrecta la teoría. A esto, en
epistemología se lo denomina falsacionismo, y el falsacionista cree que debe
usarse un silogismo hipotético que es el modus tollens.

EDUBP | Abogacía | Filosofía y Lógica Jurídica - pag.135


Una última forma de razonamiento válido son los dilemas:
El Dilema constructivo complejo:
Este tipo de razonamiento era muy famoso en la antigüedad, era una retórica
muy importante; una forma de convencer muy persuasiva siempre era la for-
mulación de un dilema. Nosotros decimos en nuestro lenguaje corriente, que
estamos frente a un dilema, cuando estamos atrapados frente a alternativas,
que habitualmente, son negativas. Obvio que esto no es necesario que sea
así siempre, pero sí, que estemos frente a situaciones donde tenemos dife-
rentes alternativas.

En un dilema tendremos la siguiente estructura:

La primera premisa se trata de dos proposiciones (atómicas o moleculares)


unidas mediante un conector disyuntivo, es decir, A o B (A v B).

La segunda premisa son dos proposiciones (atómicas o moleculares) unidas


mediante una condición. El antecedente es uno de los disyuntos de la pri-
mera proposición al cual se le agrega una consecuencia. Entonces tenemos
A=>C.

La tercera premisa son dos proposiciones unidas mediante un condicional.


El antecedente es el otro de los disyuntos de la primera premisa. Tenemos la
proposición que es B => D.

La conclusión es la formulación de las disyunciones de los antecedentes de


la segunda y tercera premisa. Es decir, es la formulación de la disyunción de
ambas consecuencias, podemos decir:

“O estudio en enero o me voy de vacaciones. Si estudio en enero me recibiré


en febrero, si me voy de vacaciones me recibiré en marzo. Entonces la con-
clusión es que me recibo en enero o me recibo en marzo.”

Estas son las consecuencias que se siguen, lo que se dice en un sistema


constructivo complejo es que al elegir dos alternativas, las cuales tienen con-
secuencias distintas, estoy eligiendo entre dos consecuencias distintas C v D.
AvB
A => C
B => D
CvD

El Dilema constructivo simple:


El dilema constructivo simple:
La primera premisa se trata de dos proposiciones (atómicas o moleculares)
unidas mediante un conector disyuntivo, es decir, A o B (A v B).

La segunda premisa son dos proposiciones (atómicas o moleculares) unidas


mediante una condición. El antecedente es uno de los disyuntos de la pri-
mera proposición al cual se le agrega una consecuencia. Entonces tenemos
A=>C.

EDUBP | Abogacía | Filosofía y Lógica Jurídica - pag.136


La tercera premisa es otro condicional que enlaza el otro disyunto, con la
misma consecuencia enlazada en segunda premisa. Tenemos la proposición
que es B => C.

La conclusión es la formulación de las disyunciones de los antecedentes de


la segunda y tercera premisa. Graficado sería:
AvB
A => D
B => D
________
D

1.1 Lenguaje artificial: variables, conectivos y tablas de verdad


Como ya hemos planteado en la presentación a esta asignatura, los criterios
que guían nuestras argumentaciones están relacionados con el significado
de expresiones que conectan nuestros enunciados. Las expresiones espe-
ciales que tienen ese rol son denominadas operadores o conectivos lógicos.
La lógica no puede dar una enumeración completa de dichas expresiones,
pero hay ciertos acuerdos donde las negaciones, conjunciones, disyuncio-
nes, condicionales, equivalencias, cuantificadores y modalidades, son algu-
nos de los signos cuyo significado debe ser investigado por la lógica para
poder reconstruir, de modo aceptable, las razones a favor o en contra de los
enunciados implicados en una disputa o desacuerdo.

Analizar el modo sistemático en que usamos la conjunción y la disyunción,


por ejemplo, nos puede proporcionar razones por las que no resulta racional
rechazar que “el derecho argentino es un conjunto de normas” si aceptamos
que “el derecho es un conjunto de normas y está parcialmente codificado”,
ni podemos dejar de admitir que “o el derecho es un conjunto de normas o
no lo es”. Para hacer explícitas las condiciones de corrección y de las infe-
rencias que involucran a tales expresiones, la lógica elabora un conjunto de
principios o reglas elementales.

A su vez, para evitar distraer nuestra atención en cuestiones extralingüísticas,


desglosaremos los aspectos formales del proceso de inferencia, eliminando
todo aquello relacionado con las condiciones de verdad de los enunciados
implicados en dicho proceso.

Ésta es la razón por la cual la lógica utiliza un lenguaje artificial o formal como
una representación esquemática y, si se quiere, idealizada, del lenguaje que
hablamos.

Para ciertos fines especiales, el lenguaje ordinario puede resultar inadecuado


debido a la imprecisión y ambigüedad del vocabulario, a la multiplicidad de
reglas de formación, etc.

Los lenguajes naturales, si bien son suficientes para la comunicación normal,


resultan insuficientes para comunicaciones que exigen mayor precisión. Por
ello, la ciencia apela a artificios consistentes tanto en la incorporación de
términos técnicos al lenguaje natural, como en la elaboración de lengua-

EDUBP | Abogacía | Filosofía y Lógica Jurídica - pag.137


jes formales para transmitir en forma más precisa conocimientos sobre
temas específicos.

Los términos técnicos que incorpora a sus discursos cada disciplina cien-
tífica, son abreviaturas de expresiones más complejas. El discurso jurídico,
por ejemplo, introduce términos como “propiedad”, “hipoteca”, “prenda” y
tantos otros, para hacer más operativa, económica y precisa la comunica-
ción entre los miembros de la comunidad jurídica.

Los lenguajes formales son los que poseen mayor grado de artificialidad y,
sirven para exhibir con mayor claridad relaciones abstractas entre enuncia-
dos. Un lenguaje formalizado como el de la ecuación: (a + b)² = a² + 2ab +
b², sirve para exhibir las propiedades del cuadrado de una suma, sin prestar
atención a cuáles son los números sumados. Con esa misma finalidad, la
lógica apela al uso del lenguaje formal.

En párrafos anteriores decíamos que un enunciado debe constituir una ora-


ción gramaticalmente bien formulada para tener algún sentido en el lenguaje
en que se elabora. El enunciado “El cielo está nublado” satisface dicha
condición y hace referencia a un hecho, un estado del mundo y, en conse-
cuencia, tiene un sentido informativo o declarativo.

La lógica abrevia la referencia a enunciados mediante el uso de letras deno-


minadas variables proposicionales tales como p,q,r,s. Dichas letras son
símbolos o representaciones de enunciados o proposiciones, no de pala-
bras. También tiene sentido la negación de dicho enunciado, esto es “El cielo
no está nublado” que se simboliza con – p (léase “no es el caso que p). Se
afirma que ‘p’ es una proposición atómica y que ‘– p’ es una proposición
compuesta o molecular formada a partir de p, mediante la operación lógica
que se denomina negación. Para negar una proposición simbolizada por la
letra p, anteponemos al símbolo proposicional p el signo ‘–‘. La negación es
una operación lógica funcional veritativa, esto es, permite inferir falsedad de
verdad y viceversa. De ese modo, si ‘p’ representa una proposición atómica
verdadera, ‘– p’ es falsa y, viceversa.

La verdad de una proposición atómica señala la correspondencia entre


dicha proposición y el estado de cosas al cual hace referencia.

La combinación de proposiciones atómicas, verdaderas o falsas, producen


proposiciones compuestas o moleculares que también poseen sentido y, la
verdad o falsedad de estas, depende tanto de la verdad o falsedad de las
proposiciones simples que la componen como de las relaciones que se esta-
blecen entre ellas (negación, conjunción, disyunción, etc.).

Recapitulando lo que hemos venido desarrollando, diremos que la lógica


proposicional es un sistema simple cuyos operadores lógicos son la nega-
ción que, como vimos, opera sobre un único enunciado –atómico o molecu-
lar- y los conectivos que vinculan dos enunciados -atómicos o moleculares-
para formar un nuevo enunciado molecular -conjunción, disyunción, con-
dicional y bicondicional-. El principio de composicionalidad del significado

EDUBP | Abogacía | Filosofía y Lógica Jurídica - pag.138


requiere que el valor de verdad de un enunciado compuesto dependa del
valor de verdad de los enunciados que lo componen. A modo de ilustración
consideremos los siguientes enunciados:

(1) María tiene un fuerte dolor y está llorando.


(2) María está llorando porque tiene un fuerte dolor.
(3) María está llorando.
(4) María tiene un fuerte dolor.

Si suponemos que de hecho María tiene un fuerte dolor y que, efectivamente,


está llorando, entonces (1) resulta verdadero. En lugar de (3) podría haber
escrito cualquier otra oración verdadera, por ejemplo “afuera está lloviendo”
que en este momento en que la escribo es verdadera, o cualquier otra ora-
ción verdadera. La oración resultante de la sustitución “María tiene un fuerte
dolor y afuera está lloviendo” también sería verdadera. En el caso (2), en
cambio, puede ser que de hecho María tenga un fuerte dolor y efectivamente
esté llorando, con lo cual (2) sería verdadera, pero no es necesario que lo sea
–tal vez está llorando porque su novio la dejó. Por el contrario, si (2) es ver-
dadera, entonces “María tiene un fuerte dolor porque afuera está lloviendo”
es falsa, aunque efectivamente afuera esté lloviendo. El ejemplo muestra
claramente la diferencia entre el comportamiento de ‘y’ y el de ‘porque’.
El enunciado (1) es verdadero si (3) y (4) son ambas verdaderas, y falsa
si al menos una de ellas es falsa. El valor de verdad de un enunciado con
‘y’ depende sólo del valor de verdad de los enunciados que lo componen.
Pero esto no se aplica al enunciado (2) cuyo valor de verdad no depende
únicamente de la verdad de los enunciados (3) y (4) que la componen, sino
también de cómo son los hechos. De los operadores que dan lugar a enun-
ciados compuestos o moleculares se dice que son veritativo-funcionales.
Del ejemplo se extrae que ‘y’ es un conectivo veritativo funcional en tanto que
‘porque’ no lo es.

A continuación, especificaremos los símbolos correspondientes a los conec-


tivos, los enunciados compuestos a que dan lugar y sus significados, en la
siguiente tabla:

Símbolo Oración compuesta Significado


– (negación) – p (negación de p) No es el caso que p

∙ (conjunción) (p ∙ q) (conjunción de p y q) pyq

٧ (disyunción) (p ٧ q) (disyunción de p y p) poq

‫( כ‬condicional) (p ‫ כ‬q) (condicional de p y q) si p entonces q

≡ (bicondicional) (p ≡ q) (bicondicional de p y q) p si y sólo si q

EDUBP | Abogacía | Filosofía y Lógica Jurídica - pag.139


Los miembros p y q de la conjunción se denominan conjuntivos. La misma
denominación se aplica a los miembros de los condicionales, disyunciones
y bicondicionales, aunque al primer y segundo miembro de la implicación o
condicional se les denomina también antecedente y consecuente, respecti-
vamente. A su vez, a los miembros de la disyunción se denominan disyunti-
vos. La notación de los conectivos también suele variar, por ejemplo, como
signo de la negación suele emplearse también ‘¬’ entre otros; para el condi-
cional ‘→’ y para el bicondicional ‘↔’, entre otros.

Cada variable o letra proposicional se denomina fórmula atómica. Las reglas


sintácticas permiten vincular, mediante conectivos y el uso de paréntesis, no
sólo fórmulas atómicas, sino también moleculares.

Los paréntesis permiten eliminar ambigüedades. Por ejemplo enuncia-


dos como p ٧ q ∙ r podrían tener significados distintos. Podría ser, como
se expresa abajo en lenguaje natural en (5): (p ٧ q) ∙ r - conjunción de la
disyunción de p y q, por un lado y r por otro, o como expresamos en (6): p
٧ (q ∙ r) - la disyunción de p, por un lado y la conjunción de q y r por el otro.
¿Cómo podríamos ver con mayor claridad lo que venimos planteando?

Interpretando las fórmulas puede notarse que tienen diferentes significados:

(5) Pérez está ausente o García está presente y la clase fue interesante.
(6) Pérez está ausente, o García está presente y la clase fue interesante.

¿Puede advertir lo que hemos realizado? Que las fórmulas representativas de


los enunciados (5) y (6) expresan diferentes significados, puede comprobarse
elaborando las correspondientes tablas de verdad. Al hacerlo se comprueba
que ambos enunciados poseen diferentes condiciones de verdad. La justifica-
ción de lo afirmado respecto de (5) y (6) nos conduce en consecuencia a las
tablas de verdad que determinan los significados de los conectivos lógicos.

Significados de los conectivos


Lo que interesa resaltar, es la forma en que el valor de verdad de una ora-
ción compuesta, depende del valor de verdad de las oraciones simples o
atómicas que la componen y del conectivo empleado. El significado de cada
conectivo está determinado por una tabla de verdad correspondiente, que
prescribe sus condiciones de verdad. Pero lo que es necesario para una
comprensión precisa de la negación, la conjunción y la disyunción inclu-
yente viene determinado por los siguientes principios:

‘– p’ es verdadero sí, y solamente si, ‘p’ es falso.

‘p ∙ q ∙…. ∙ s’ es verdadero sí, y solamente si ‘p’ y ‘q’ y….y ‘s’ son


todos verdaderos.

‘p ٧ q ٧….٧ s’ es verdadero sí, y solamente si ‘p’ y ‘q’ y… y ‘s’ no son


todos falsos.

EDUBP | Abogacía | Filosofía y Lógica Jurídica - pag.140


Esto se confirma consultando las correspondientes tablas de verdad para
cada uno de estos conectivos en Lógica, proposición y norma.

En cuanto a la conjunción, aunque a simple vista puede parecer que el com-


portamiento lógico de ‘∙’ es concordante con el de ‘y’ del lenguaje natural,
éste no es siempre el caso, por ejemplo:

(7) María se casó y tuvo un hijo.

(8) María tuvo un hijo y se casó.

El hecho de colocar una oración después de la otra puede sugerir que ese
fue el orden en que sucedieron los hechos descriptos. En cambio, la con-
junción ‘∙’ tiene la propiedad de ser asociativa, pues la agrupación interna
carece de efecto y, además, es conmutativa porque el orden en que figuren
sus conjuntivos es irrelevante no habiendo, en consecuencia, necesidad de
distinguir entre, por ejemplo, ‘p ∙ q’ y ‘q ∙ p’.

El único caso en que una disyunción, por ejemplo ‘p ٧ q’, es falsa es aquel
en el que tanto ‘p’ como ‘q’ son falsos. Casi todas las funciones veritativas
son traducibles a la notación de la negación y de la conjunción, esto se ilus-
tra si observamos que p ٧ q es falso cuando – p ∙ – q es verdadero. De ese
modo, en lugar de escribir ‘p ٧ q’, podemos simplemente negar ‘– p ∙ – q’
escribiendo ‘– (– p ∙ – q)’. La equivalencia entre ambas fórmulas puede ser
demostrada mediante la construcción de una tabla de verdad.

¿Qué deberíamos considerar a la hora de elaborar una Tabla de Verdad?


La elaboración de dicha tabla requiere descomponer todos sus elementos,
desde los más simples a los más complejos, y establecer sus valores de
verdad posibles. Las combinaciones posibles de valores de verdad se deter-
minan elevando las dos posibilidades de valor de verdad, verdad o falsedad,
al número de variables proposicionales que intervienen en las fórmulas cuya
equivalencia se desea demostrar. En este caso, las variables son dos: ‘p’ y ‘q’,
por tanto, las combinaciones posibles serán 2² = 4, que se ordenan en colum-
nas y alternando verdad o falsedad como se observa en el siguiente gráfico:

Las columnas demarcadas con una elipse demuestran que las dos fórmulas
involucradas son equivalentes porque poseen idénticos valores de verdad
en todas sus combinaciones, o en todas las filas.

En cuanto al condicional material, cabe señalar que a una aserción de la


forma ‘si p entonces q’ se la interpreta como la aserción condicionada de
su consecuente, en el sentido que si el antecedente resulta ser verdadero,

EDUBP | Abogacía | Filosofía y Lógica Jurídica - pag.141


entonces nos veríamos en la obligación de considerar verdadero el conse-
cuente, y habría que reconocer que estábamos equivocados si el conse-
cuente fuera falso. Si el antecedente fuera falso es como si no hubiésemos
llevado a cabo nuestra aserción condicionada. Cuando el antecedente es
verdadero, el valor de verdad de todo el condicional es el valor de verdad del
consecuente, por tanto, un condicional con un antecedente verdadero y un
consecuente verdadero es verdadero, y un condicional con un antecedente
verdadero y un consecuente falso es falso. Cuando el antecedente es falso el
condicional es siempre verdadero. En función de las condiciones de verdad
del condicional material se dice que, la verdad el consecuente es condición
necesaria de verdad del antecedente y, la verdad del antecedente es sólo
condición suficiente de verdad del consecuente.

En síntesis, se considera que ‘p ‫ כ‬q’ es verdadero cuando ‘p’ y ‘q’ son ver-
daderos, cuando ‘p’ es falso y ‘q’ es verdadero y también cuando tanto ‘p’
como ‘q’ son falsos; y se lo considera falso en el único caso en que ‘p’ es
verdadero y ‘q’ es falso. Respecto de la verdad del condicional cabe afirmar
en términos de negación y conjunción que ‘no es el caso que ‘p’ sea verda-
dero y ‘q’ falso, según la notación lógica: – (p ∙ – q).

Por último, el bicondicional material ‘p sí, y solamente si, q’ equivale a la con-


junción de estos dos condicionales: ‘si p, entonces q’ y ‘si q, entonces p’. En
consecuencia, corresponde considerar a ‘p ≡ q’ como una abreviatura de ‘(p
‫ כ‬q) ∙ (q ‫ כ‬p). De esto se obtiene que el bicondicional será falso en dos casos:
en el caso de que ‘p’ sea verdadero y ‘q’ sea falso, y en el caso que ‘q’ sea
verdadero y ‘p’ sea falso. Un bicondicional material es verdadero si sus com-
ponentes son idénticos en cuanto a su valor de verdad, y será falso si sus com-
ponentes difieren en su valor de verdad. De ello se infiere que cada término del
bicondicional es condición necesaria y suficiente de verdad del otro.

A su vez, se dice que un esquema veritativo funcional es consistente o con-


tingente cuando es verdadero para alguna interpretación de sus variables
proposicionales, en el caso contrario se dice que es inconsistente. Además,
se denomina válido o tautológico todo esquema veritativo funcional cuando
es verdadero para toda interpretación de sus variables o letras proposicio-
nales. Por ejemplo, el esquema ‘p ∙ – q’ es consistente, porque resulta ver-
dadero si interpretamos ‘p’ como verdadero y ‘q’ como falso; pero dicho
esquema no es válido, porque hay otras interpretaciones de ‘p’ y ‘q’ bajo las
cuales resulta ser falso.

Implicación y equivalencia.
Habiendo desarrollado algunas cuestiones centrales de este módulo,
veamos ahora cuál es el objetivo principal de la lógica en su aplicación a la
ciencia y al discurso ordinario. La lógica se ocupa principalmente de inda-
gar las técnicas que demuestren que un enunciado se ‘sigue lógicamente’,
o no se sigue, de otro enunciado. El enunciado ‘Casio no está arruinado y
hambriento’ se sigue lógicamente del enunciado ‘Casio no está hambriento’.
Las formas lógicas respectivas de estos enunciados son ‘– (p ∙ q)’ y ‘– q’. (En
lugar de ‘p’ y de ‘q’ tenemos los enunciados ‘Casio está arruinado’ y ‘Casio
está hambriento’). Puede demostrarse por la tabla de verdad respectiva que

EDUBP | Abogacía | Filosofía y Lógica Jurídica - pag.142


no hay enunciado puesto en los lugares de ‘p’ y de ‘q’ que hagan verdadero
‘– q’ y falso ‘– (p ∙ q)’.

Mediante la elaboración de la correspondiente tabla de verdad se demuestra


que:

‘– q’ implica ‘– (p ∙ q):

Se dice que un esquema veritativo funcional implica otro, si no hay forma de


interpretar las variables o letras proposicionales de ambos de modo tal que
el primer esquema sea verdadero y el segundo sea falso. De otro modo, no
hay ninguna interpretación que haga verdadero al primer esquema y falso al
segundo.

Si aplicamos los métodos –tablas de verdad– para probar la presencia o


ausencia de implicación, validez y consistencia, se comprueba la vigencia
de cuatro leyes generales:
1. Cualquier esquema se implica a sí mismo.
2. Si un esquema implica un segundo esquema, y éste un tercero, entonces
el primero implica el tercero.
3. Un esquema inconsistente implica todo esquema y, a su vez, sólo lo
implican esquemas que sean inconsistentes.
4. Cualquier esquema implica un esquema válido, y cualquiera de este tipo
implica únicamente esquemas válidos.

La implicación se halla íntimamente relacionada con el condicional. La impli-


cación es la validez del condicional como queda demostrado en el gráfico
precedente que demuestra la relación de implicación.

La equivalencia, a su vez, es la validez del bicondicional. Si dos enuncia-


dos tienen siempre el mismo valor de verdad se dice que son equiva-
lentes en el sentido de que con ellos puede formarse un bicondicional
tautológico, esto es, válido. Dicho de otro modo, dos esquemas veritativo-
funcionales son equivalentes si coinciden entre sí en sus valores de verdad
bajo toda interpretación de sus variables proposicionales; o si concuerdan
entre sí caso por caso cuando son analizados por medio de tablas de verdad.
De su definición y comprobación se sigue:
1. La equivalencia es implicación recíproca.
2. Cualquier esquema es equivalente a sí mismo.

EDUBP | Abogacía | Filosofía y Lógica Jurídica - pag.143


3. Si un esquema es equivalente a un segundo y éste lo es a un tercero,
entonces el primero es equivalente al tercero.
4. Si un esquema es equivalente a un segundo, entonces éste es equiva-
lente al primero (nótese que esto no ocurre con la implicación).
5. Los esquemas válidos son únicamente equivalentes entre sí; lo mismo
vale para los esquemas inconsistentes.

Por ejemplo, la equivalencia demostrada en la primera tabla de verdad, ela-


borada precedentemente, puede ser representada en la tabla de verdad
correspondiente al bicondicional tautológico resultante, tomado la tabla del
siguiente gráfico como una continuación de aquella:

Aplicación de la lógica al lenguaje ordinario


Mientras que el nexo entre premisas y conclusión se basa en la lógica,
con frecuencia las premisas y conclusión en sí mismas no se fundan en ella
y, en esta circunstancia reside:

La aplicación de la lógica a dominios distintos de sí misma,


por ejemplo, su aplicación al lenguaje ordinario.

Las premisas y la conclusión pueden versar sobre cualquier tema y se


ejemplifican mejor en lenguaje ordinario o natural que en lenguaje formal.
Introduciendo letras esquemáticas distorsionamos un tanto los enuncia-
dos para resaltar sus estructuras. La paráfrasis a lenguaje formal de
enunciados en lenguaje natural reduce giros lingüísticos a símbolos unifor-
mes, pero cierta correspondencia cabe establecer.

La conjunción, en lenguaje natural no sólo se expresa mediante ‘y’, sino


también por medio de ‘pero’, de ‘aunque`, de signos de puntuación, etc. La
diferencia entre el uso de ‘pero’ y de ‘aunque’ hace explícita la distinción
entre los aspectos lógicos y los aspectos retóricos del lenguaje. Pero las
diferencias retóricas no bastan para señalar diferencias entre la verdad
o falsedad de la conjunción. En el caso del condicional hay también
variantes en el lenguaje natural para el uso de ‘si p, entonces q’: ‘p sólo
si q’, ‘q supuesto que p’, ‘q en caso de que p’ ‘no p a menos que q’.
Nótese que el antecedente de un condicional ‘p’, no es siempre en el
habla ordinaria la parte del condicional que ocupa el primer lugar en la
oración emitida. Por ejemplo, ‘si’ es signo del antecedente y, si decimos
‘p si q’ en lenguaje formal se transforma en ‘qvp’. El ‘sólo’ en cambio es
signo del consecuente y, por ende, ‘p sólo si q’ significa ‘si p, entonces

EDUBP | Abogacía | Filosofía y Lógica Jurídica - pag.144


q’. Por ejemplo: ‘se licenciará sólo si paga su matrícula’ significa ‘si se
licencia, entonces ha pagado su matrícula’. Hay que tener claro que ‘sólo
si’ no tiene el significado de ‘º’, el cual es ‘si, y solamente sí’ (‘si y sólo si’).
Con respecto a la disyunción ‘a menos que’ es uno de los significados
de ‘o’. La labor de paráfrasis requiere también, en ocasiones, reformular
las oraciones componentes a fin de salvaguardarlas de alteraciones en
su significado. Esta necesidad se hace evidente en el siguiente ejemplo:
(1) Fue a Villaconejos y yo lo acompañé,
(2) Fue a Vilamarranos, pero yo no lo acompañé.

Estas dos conjunciones pueden ser ambas verdaderas simultáneamente; no


obstante, si las representamos como ‘p. q’ y ‘r. –q’ el compuesto resulta
incon- sistente: ‘p. q. r. –q’. Pero debe notarse que el ‘yo le acompañé’ de
(1) no es el de (2), pues en (1) es ‘yo le acompañé a Villaconejos” y en (2)
‘no le acompañé a Villamarranos’. Por lo tanto, lo correcto es reemplazarlos
en lenguaje formal por variables diferentes: ‘p. q’ y ‘r. –s’, con lo cual la apa-
rente inconsistencia del compuesto desaparece. Es importante en el análi-
sis formal no asignar diferentes interpretaciones a una misma expresión en
el curso del mismo argumento. La violación de este principio se denomina
‘falacia de equivocidad’. En relación a la corrección lógica no importa cómo
interpretemos los enunciados que componen un argumento, lo que importa
es que la interpretación de expresiones ambiguas sea uniforme a lo largo de
todo el argumento.

Por último, cuando un enunciado es complejo, es conveniente comenzar por


la estructura más externa y parafrasear hacia adentro, paso por paso. Por
ejemplo:

Si el contrato se celebra es país perteneciente al Mercosur entonces la capa-


cidad de los contratantes y los efectos se regirán por la ley nacional. Si la
capacidad y los efectos se rigen por la ley nacional entonces o le conviene
al deudor o al acreedor. Si el contrato se celebra en país perteneciente al
Mercosur o le conviene al deudor o al acreedor.

Primero cabe localizar el conector que divide la estructura más externa del
enunciado:

Si el contrato se celebra es país perteneciente al Mercosur enton-


ces la capacidad de los contratantes y los efectos se regirán por
la ley nacional.

Como vemos, la primera premisa es un condicional pero su con-


secuente también tiene una conectiva en este caso una conjun-
ción. Queda: P => (q.r)

La segunda premisa que comienza después del punto dice:

Si la capacidad y los efectos se rigen por la ley nacional enton-


ces o le conviene al deudor o al acreedor.

EDUBP | Abogacía | Filosofía y Lógica Jurídica - pag.145


Es otra afirmación condicional cuyo antecedente en su interior
tiene una conjunción y su consecuente tiene una disyunción.

(q.r) = > (s v t)

Como se trata de un razonamiento, uno las premisas entre sí con


una conjunción y lo enlazo al consecuente con un condicional.
Para diferenciar las estructuras de las premisas y la conclusión
agrego una llave.

[{P = > (q.r)}. {(q.r) = > (s v t)}]

Posteriormente solo debo agregarle la conclusión.

Si el contrato se celebra en país perteneciente al Mercosur o le


conviene al deudor o al acreedor. P = > (s v t)

La fórmula final me queda:

[[{P = > (q.r)}. {(q.r) = > (s v t)}] => P => (s v t)]

1.2 Modalidades aléticas y deónticas


El lenguaje, como sistema de comunicación, es un instrumento al servicio
de diversas funciones. Hemos visto que los enunciados declarativos, o pro-
posiciones, tienen una función informativa, pues afirman o niegan eventos o
estados de cosas del mundo.

Pero, por medio del uso del lenguaje, se pueden satisfacer otras finalidades,
por ejemplo cuando escribimos un poema lo que hacemos es expresar o
intentar provocar una cierta actitud, por ejemplo, estética.

Por medio de la emisión de una orden, por ejemplo, no se informa nada sobre
el mundo sino, por el contrario, se intenta modificarlo en un determinado
aspecto en función de lo que se considera que debe ser el mundo. Este es
el uso prescriptivo del lenguaje que nos interesa analizar por su estrecha
relación con el derecho como técnica de motivación de la conducta social.

Es importante distinguir entre el enunciado “La ventana está abierta” y, el


enunciado imperativo ¡Cierre la ventana! Este último no es informativo y, por
ende, no es ni verdadero ni falso, pues primariamente no hace referencia a
un hecho, sino que ordena una determinada acción, tiene una función moti-
vadora de la conducta.

En el mismo sentido, las normas que conforman el derecho positivo de un


país, por ejemplo la que expresa que “el propietario debe pagar el impuesto
a la renta”, son enunciados prescriptivos y, por tanto, ni verdaderos ni falsos.
Afirma que la acción del propietario de pagar el impuesto es obligatoria en
ciertas circunstancias.

EDUBP | Abogacía | Filosofía y Lógica Jurídica - pag.146


En consecuencia, una norma enuncia que cierta clase de acción es obli-
gatoria para ciertos sujetos en ciertas circunstancias. La norma indica qué
es lo que corresponde hacer en ciertas circunstancias, si se desea actuar
como la sociedad espera que la gente actúe. No informa sobre la situa-
ción de hecho, no informa si los propietarios efectivamente pagaron el
impuesto, o no lo hicieron.

La finalidad de todo enunciado prescriptivo es lograr que las personas que


lo entienden actúen de cierta manera, y no de otra. Si el enunciado prescrip-
tivo es un enunciado con sentido, entonces cabe sostener que una norma,
secundariamente, nos informa sobre el modo en que es estimada social-
mente una clase de acción.

En relación a las modalidades, ya desde la lógica aristotélica se advirtió que


no todos los enunciados de la ciencia son simplemente verdaderos. Con
frecuencia, la ciencia, en lugar de sostener la verdad de un enunciado que
informa sobre la existencia de algún hecho, se limita a indicar que su exis-
tencia es posible. Además, de la verdad de un enunciado que describe la
existencia de un hecho, puede inferirse la verdad del enunciado que afirma
que ese hecho es posible. En conclusión, cabe afirmar que si algo es ver-
dadero, entonces es posible. Y también, si algo es necesario, por ejemplo
un enunciado tautológico, podemos correctamente afirmar que, en conse-
cuencia, es verdadero.

En síntesis, puede afirmarse que de la necesidad se infiere la verdad, y


de la verdad, la posibilidad. Necesidad y posibilidad son modalidades de
los enunciados: un enunciado necesariamente verdadero es algo más que
un enunciado verdadero, y un enunciado posiblemente verdadero es algo
menos que el enunciado verdadero.

Estos modos, como se ha tratado de mostrar, están en estrecha relación


lógica con la verdad. Dichas modalidades son denominadas aléticas (es
decir: de la verdad). Estas modalidades lógicas aléticas no valen sin reservas
cuando tratamos con enunciados prescriptivos. Fue Hume quien indicó que
del hecho que una proposición sea verdadera, necesaria o posible, nada
puede inferirse con respecto a su obligatoriedad.

Inversamente, de que una norma prescriba la obligatoriedad de una con-


ducta, nada puede inferirse sobre la verdad, posibilidad o necesidad alética
del hecho de que se dé efectivamente esa conducta – el sujeto puede trans-
gredir su obligación u omitir hacer lo que debe hacer.

No obstante, una norma puede ser también analizada como un enunciado


modalizado, es decir, como un enunciado que establece que una conducta
es obligatoria, prohibida o permitida. Estas modalidades se denominan
modalidades deónticas: modos de obligación o deber.

La estructura de una norma puede pensarse como la combinación de un


enunciado declarativo -como el enunciado: “el propietario paga el impuesto
a la renta”- y un giro lingüístico que exhibe la modalidad deóntica: “es obli-
gatorio que el propietario pague el impuesto a la renta”. Simbolizando la
modalidad “es obligatorio que…” con la letra O, una norma puede represen-

EDUBP | Abogacía | Filosofía y Lógica Jurídica - pag.147


tarse formalmente como “Op” – el enunciado representado por la variable ‘p’
es descriptivo del contenido de la norma, o bien, de la clase de acción pres-
cripta. El modalizador deóntico O se antepone a una variable proposicio-
nal para formar una norma o enunciado normativo. Op se lee: “es obliga-
torio que p”. Estos giros modalizadores carecen de sentido independiente
y operan modificando el sentido de los enunciados a los que se anteponen.

Suponiendo que ‘p’ es interpretado como el enunciado “el propietario paga


el impuesto a la renta”. Anteponiendo a dicho enunciado un operador modal
alético, obtenemos un enunciado modalizado aléticamente: “Es posible que
el propietario pague el impuesto a la renta”. Si le anteponemos un operador
modal deóntico obtenemos un enunciado modal deóntico: “Es obligatorio
que el propietario pague el impuesto a la renta”.

La regla de formación de una norma consiste en la aplicación de un opera-


dor modal deóntico a un enunciado; el enunciado resultante es una norma.
Esta regla de formación corresponde sólo al análisis lógico de las normas.
En su expresión lingüística corriente, los enunciados normativos pueden
asumir diferentes formas. No hay característica gramatical en un len-
guaje ordinario, por ejemplo el castellano, que permita distinguir un
enunciado normativo de uno que no lo es; las normas pueden ser expre-
sadas en forma imperativa o en oraciones indicativas, etc. Cuando el jefe
se dirige a su empleado diciendo: “Ud. irá al correo mañana a primera hora y
enviará esta correspondencia” no está haciendo una profecía de lo que hará
su empleado, sino que está emitiendo una directiva a su empleado.

Interdefinibilidad de operadores deónticos


Los operadores deónticos usuales son, además del operador O: es obliga-
torio que…, los operadores Ph: está prohibido que… y P: está permitido
que…

Supongamos que ‘p’ sea el enunciado “Pedro fuma”:

Op sería la norma: “es obligatorio que Pedro fume”, pero supongamos que
lo obligatorio no sea la acción indicada, fumar, sino su omisión, o sea, no
fumar. La norma correspondiente diría: “es obligatorio que Pedro no fume”.
Pero si “Pedro fuma” es simbolizado por ‘p’, “Pedro no fuma” será “– p” y,
la norma que obliga a Pedro a no fumar será: O – p. Pero obligar a Pedro a
omitir fumar es lo mismo que prohibirle fumar. Prohibido es otro operador
modal deóntico que se simboliza habitualmente con las letras Ph, y se lee:
“está prohibido que…”. De ese modo obtenemos la equivalencia:

(1) Php = O–p

A su vez, decir que una acción está prohibida: Php, significa que no está
permitido realizarla. Así, “está prohibido que Pedro fume” es equivalente a
“no está permitido que…”, con la letra P simbolizamos el operador modal
deóntico “está permitido que…” y, obtenemos:

EDUBP | Abogacía | Filosofía y Lógica Jurídica - pag.148


(2) Php = – Pp
A la inversa, puede decirse que lo que no está prohibido, –Php, está
permitido:
Pp.
(3) –Php = Pp
Si negamos la equivalencia mencionada en (1 arriba) se obtiene:
(4) – Php = – O–p
De lo dicho surge la siguiente ecuación:
(5) Pp = –Php = –O–p

Decir que un acto p es permitido equivale a afirmar que no está prohibido,


o que no es obligatorio omitirlo. Dicha ecuación señala que los operadores
deónticos pueden ser interdefinidos por medio del uso de la negación.

A su vez, de (2) y (1) se obtiene la equivalencia:

(6) –Pp = O–p que puede leerse como: si no está permitido el


acto p, es obligatorio no hacerlo.

Si en ambos lados del signo = reemplazamos el acto enunciado por p, por


su negación: –p, tendremos una nueva equivalencia:

(1) –P–p = O– –p

Como las dobles negaciones se anulan (Ver Lógica, proposición y normas,


ley lógica de la doble negación) la equivalencia se simplifica:

(2) –P–p = Op

Si no está permitido que se omita un acto, ese acto es obligatorio.

(Cotejar con tabla de interdefinición de operadores deónticos que figura en


Lógica, proposición y normas).

Hay otro modalizador deóntico de uso habitual en ciencias jurídicas: facul-


tad o libre permisión. Un acto es facultativo o libre cuando no es obligatorio,
ni está prohibido, sino que está permitida no sólo su realización, sino tam-
bién su omisión. Este operador compuesto se simboliza con la letra F. Un
ejemplo de actos facultativos sería el determinado por normas que confieren
potestades privadas para contraer matrimonio, efectuar contratos o testar.

(3) Fp = (Pp ∙ P–p)


(4) Fp = –Op ∙ –O–p

EDUBP | Abogacía | Filosofía y Lógica Jurídica - pag.149


Algunos juristas utilizan como operadores deónticos básicos O, Ph y F
siendo, en este caso, las interdefiniciones recíprocas las siguientes:

(5) Op = –Fp ∙ –Php

(6) Fp = –Op ∙ –Php

(7) Php = –Op ∙ –Fp

De (10), (11) y (12) se obtiene el siguiente esquema de modalidades mutua-


mente excluyentes:

Lo permitido será, en este análisis, o bien lo que es obligatorio (si un acto es


obligatorio, entonces está permitido realizarlo), o bien lo que es facultativo
(pues lo facultativo siempre está permitido). En lenguaje formal:

(8) Pp = Fp ٧ Op
(9) se lee: Todo acto obligatorio o facultativo es permitido.

Cabe distinguir dos sentidos en el uso del término permisión: permisión


fuerte para hacer referencia a la conducta facultativa u obligatoria, según una
norma jurídica positiva y, permisión débil que describe una conducta que no
está regulada jurídicamente (permisión como ausencia de prohibición).

Luego del análisis precedente es oportuno explicar clasificaciones relevantes


de las normas jurídicas, a saber: normas positivas y negativas; normas cate-
góricas e hipotéticas; y normas generales y particulares.

Las normas se clasifican en positivas y negativas por su contenido, esto


es, por la conducta sujeta a regulación. Este contenido, o enunciado descrip-
tivo de una clase de acción, es el que simbolizamos con una letra correspon-
diente a una variable proposicional: p, q, r, s, etc. Por acción entendemos
tanto un acto que produce un cambio en el mundo, como una omisión que
impide un cambio en el mundo. La omisión, en tanto modo de acción, no
constituye un simple no hacer, sino un no hacer un acto cuya realización es
posible para un sujeto (por ejemplo: no auxiliar a quien se está ahogando
sabiendo nadar). Norma positiva es la que regula actos y, que se representa
simbólicamente como Op, Php, etc. Norma negativa es la que regula omi-
siones y se representan negando la variable afectada por el operador deón-
tico (negación interna), por ejemplo, O–p, Ph–p, etc. La negación externa
es la que afecta al modalizador deóntico y no a la variable proposicional.

EDUBP | Abogacía | Filosofía y Lógica Jurídica - pag.150


En síntesis, la acción deónticamente modalizada es la que denominamos el
contenido de la norma. En cuanto a la división de las normas en categóricas
e hipotéticas, se sostiene que una norma es categórica cuando regula un
cierto contenido incondicionalmente: Op, Pp, Php serían normas categóricas.

Se sostiene que la mayoría de las normas jurídicas son hipotéticas o con-


dicionadas, pues establecen que una cierta acción es obligatoria, está
prohibida o es permitida dada ciertas condiciones.

Según V, Wright (Norma y Acción: Una investigación lógica) toda norma tiene
ciertas condiciones de aplicación: si es obligatorio cerrar la ventana, la cir-
cunstancia de que la ventana esté abierta es una condición de aplicación
de la norma. Pero si las condiciones de aplicación son aquellas condiciones
implícitas -no mencionadas expresamente en las normas-por ser aquellas
condiciones que tienen que darse para que sea posible la realización del
contenido de la norma, y no se hace explícita ninguna otra condición, se dice
que la norma es categórica.

Por el contrario, una norma es hipotética o condicional cuando la acción


u omisión regulada por la norma es O, P o Ph bajo la condición de que se
produzca algún otro hecho. Por ejemplo, “si se encuentra dentro de la institu-
ción, entonces está prohibido fumar”. Se representan simbólicamente bajo la
forma de un condicional: Si se da p – está dentro de la institución-, entonces
prohibido q –prohibido fumar-, en símbolos: p ‫ כ‬Phq.

Leyes lógicas normativas


Las leyes lógicas son enunciados, en general moleculares, que tienen la
característica de ser verdaderos en todos los casos, esto es, nunca pueden
ser falsos. Se suelen denominar tautologías. Un enunciado contingente es
aquel cuya verdad depende de cómo sea el mundo en un determinado res-
pecto, por ejemplo: “El cielo está nublado”. Pero, en cambio, el enunciado
disyuntivo: “El cielo está nublado o el cielo no está nublado” es verdadero
en todos los casos posibles, esto es, no puede ser falso. Sabemos que la
verdad de uno de los enunciados atómicos hace verdadera la disyunción y,
en el caso que nos ocupa, o el primer enunciado atómico es verdadero, o lo
será el segundo, con lo cual la disyunción siempre será verdadera.

Esta tautología de lógica proposicional es conocida tradicionalmente como:

1.- Principio de tercero excluido: p ٧ –p.

En rigor, una tautología no afirma nada sobre el mundo – en este caso el


cielo; en tal sentido se dice que es siempre verdadera, pero vacía o formal.

Otros ejemplos de leyes lógicas bien conocidas son los siguientes:

2.- El principio de no contradicción: p ∙ –p.


3.- El principio de identidad: p ∙ p.
4.- La ley del modus ponens: ((p ‫ כ‬q) ∙ p) ‫ כ‬q.

EDUBP | Abogacía | Filosofía y Lógica Jurídica - pag.151


Por ejemplo: “Si la temperatura se eleva, entonces el volumen del gas
aumenta, la temperatura efectivamente se elevó, entonces también es verdad
que el volumen del gas aumenta”.

Si sustituimos las variables proposicionales por variables precedidas por un


operador deóntico descubrimos que las leyes lógicas valen también para
las normas. Así, por ejemplo, Pp ٧ – Pp, todo acto está permitido o no lo
está. Reemplazando –Pp por su equivalente Php, obtenemos: Pp ٧ Php, que
podemos traducir como: “todo acto está permitido o está prohibido”.

El principio de Hume indica que no se puede inferir un enunciado normativo


de un enunciado declarativo, y viceversa. De tal modo, que un hecho p se
dé efectivamente no podemos inferir que p está permitido, o es obligatorio.
Dicho principio no se extiende a las leyes lógicas, como hemos corroborado
en el caso de la ley de tercero excluido, porque las tautologías son vacuas.
Así como una tautología nada afirma, la norma derivada de ella nada regula
(por ejemplo: Pp ٧ Php no regula conducta alguna).

La ley de contradicción deóntica también es derivable de la correspon-


diente ley de lógica proposicional, sustituyendo las variables proposicionales
por variables precedidas por un operador deóntico, obteniéndose: Pp ∙ –Pp
o, reemplazando –Pp por su equivalente Php:

Pp ∙ Php. Es una contradicción deóntica que un acto sea prohibido y permi-


tido simultáneamente, sea obligatorio y esté permitida su omisión, o sea obli-
gatorio y prohibido de consuno. No es posible determinar el carácter deón-
tico de una acción regulada por un enunciado normativo contradictorio.

Una ley lógica deóntica considerada axiomática es la denominada ley de


subalternación deóntica que se simboliza como: Op ‫ כ‬Pp y dice que “si una
acción es obligatoria, entonces es permitida”. Dicha ley parece una verdad
de lógica deóntica indiscutible, pues sería absurdo pensar que si un acto es
obligatorio entonces está prohibido. En cambio, sería incorrecto afirmar que
si un acto está permitido, entonces es obligatorio. Ello así porque puede ocu-
rrir que ese acto sea facultativo y no obligatorio. En consecuencia, se afirma
que la relación conversa de la implicación citada en (4) no resulta válida.
Tampoco es posible, por esa misma razón, aceptar como fórmula deóntica
válida una norma de la forma: –Php‫כ‬Op.

Lógica de normas y lógica de proposiciones normativas.


Los enunciados normativos, aquellos que contienen palabras típicamente
normativas como obligatorio, prohibido o permitido, además de tener una
función prescriptiva, como técnica de motivación de la conducta, tienen
una función informativa cuando son usados para describir cómo es apre-
ciada socialmente una conducta, o informar cuál es la conducta socialmente
debida en ciertas circunstancias.

En el primer caso, es decir cuando se usan para prescribir conductas, los


enunciados normativos expresan normas y, en el segundo caso, decimos
que expresan proposiciones normativas. Pero si se acepta que las normas
no son enunciados ni verdaderos ni falsos, no se puede admitir que haya

EDUBP | Abogacía | Filosofía y Lógica Jurídica - pag.152


relaciones lógicas entre normas, por ejemplo, que haya normas contradicto-
rias, subalternas, etc., pues las relaciones lógicas sólo se darían entre enun-
ciados verdaderos o falsos.

Algunos autores, por esa razón, niegan que haya una lógica de normas y
estiman que las leyes lógico normativas sólo rigen para las proposiciones
normativas, no para las normas. Claro está que admitir que hay una lógica
de proposiciones normativas presupone una lógica de normas. Las conduc-
tas no son obligatorias, permitidas o prohibidas independientemente de una
norma que les atribuya esa propiedad.

Otros autores, por ejemplo Von Wright, indican que el lenguaje prescriptivo
o normativo tiene un sentido racional o lógico. Ello así, pues sería irracional
que un legislador formulara simultáneamente normas contradictorias con
una pretensión motivadora pues, ¿qué sentido puede tener obligar a un
sujeto a hacer algo y, simultáneamente, prohibirle que lo haga?

Lo cierto es que tanto las normas, como las proposiciones normativas,


pueden ser expresadas por los mismos enunciados, pero sus propieda-
des son visiblemente diferentes. Las normas no son ni verdaderas ni falsas,
aunque pueden ser válidas o inválidas, obedecidas o desobedecidas, justas
o injustas, etc. Las proposiciones normativas, en cambio, son verdaderas o
falsas, pero no pueden ser justas o injustas, obedecidas o desobedecidas, y
carece de sentido afirmar que son válidas. Una proposición normativa es ver-
dadera con independencia de quien la formule, pues su verdad depende del
hecho que describe, esto es, de que exista una norma válida que prescriba
tal o cual obligación, permisión etc. Por el contrario, la validez de una norma
puede depender de quien sea la persona que la formula -si es o no una
autoridad normativa competente para dar la norma-. Hay proposiciones de
diferentes tipos, algunas describen la validez de una norma, otras su eficacia
o su aplicabilidad. Las proposiciones normativas que producen un interés
especial son aquellas que no se refieren a normas, sino a acciones, o esta-
dos de cosas resultantes de acciones, y describen la propiedad que poseen
de ser obligatorias, permitidas o prohibidas. Por supuesto que, como vimos,
no se trata de propiedades que puedan predicarse con independencia de
las normas. Estas proposiciones, cuya expresión lingüística suele ser muy
parecida a la de formulación de una norma, pueden ser analizadas como
proposiciones relativas a la existencia de normas. Afirmar que ‘p’ es obligato-
ria, equivale a decir que existe una norma que obliga a hacer ‘p’.

A los fines de completar el estudio de los temas del módulo diríjase a la


siguiente lectura:
• GUIBOURG, Ricardo y Otros; Lógica, proposición y norma. Bs. As. Ed.
Astrea, 1980.

EDUBP | Abogacía | Filosofía y Lógica Jurídica - pag.153


Si usted desea ampliar acerca de conceptos vistos en el módulo puede pro-
fundizar leyendo los siguientes materiales:
• RODRÍGUEZ, Jorge L.; “Naturaleza y lógica de las proposiciones nor-
mativas” ∙ , Doxa 26, 2003.
• VERNENGO, Roberto; Curso de Teoría General del Derecho, Ed. Depalma,
1ª reimpresión de la 2ª edición, 1985, capítulo 1.

Lo invito a realizar las actividades del módulo, a fin de poner en práctica los
conceptos estudiados.

Al finalizar el estudio del Módulo 7, usted está en condiciones de realizar la


Tercera Parte de la evaluación parcial.

M7 Actividades
Actividad 1

Descubriendo las implicaciones de nuestros argumentos


a) La enfermedad de Hobson
Los científicos han demostrado que la enfermedad de Hobson es causada
por la presencia de una elevada cantidad de mercurio en el agua potable, y
se encuentra asociada a ciertos tipos de polución industrial. Ahora bien, los
habitantes de la isla “Paraíso” presentan un nivel de incidencia de la enfer-
medad de Hobson extraordinariamente alto, a pesar de que dicha isla posee
una economía basada exclusivamente en la agricultura, sin industrias ni polu-
ción de ninguna clase.

Habiendo leído el fragmento anterior, le proponemos indagar lo siguiente:

¿Cuál de los siguientes enunciados se puede deducir del pasaje anterior?


¿Por qué?
(a) La presencia de mercurio en el agua potable es totalmente inofensiva.
(b) La presencia de mercurio en el agua potable debe provenir de otras fuen-
tes además de la polución industrial.
(c) La enfermedad de Hobson debe tener otras causas además de la pre-
sencia de mercurio en el agua potable.
(d) O bien la presencia de mercurio en el agua potable proviene de otras fuen-
tes además de la polución industrial o bien la enfermedad de Hobson tiene
Lectura otras causas además de la presencia de mercurio en el agua potable.
(desde página 180)
(e) O bien la presencia de mercurio en el agua potable es totalmente inofen-
siva o bien la enfermedad de Hobson tiene otras causas además de la
presencia de mercurio en el agua potable.

EDUBP | Abogacía | Filosofía y Lógica Jurídica - pag.154


b) El uso de cinturón de seguridad en debate
Quienes se oponen a las leyes que hacen obligatorio el uso de cinturones de
seguridad para los conductores, arguyen que no usar cinturones de segu-
ridad no causa perjuicio a terceros. Sobre esta base, concluyen que cada
persona debería ser libre de decidir si usa o no usa cinturón de seguridad.

¿Cuál de los siguientes enunciados, de ser verdadero, debilitaría más seria-


mente la conclusión anterior? ¿Por qué?
(a) Muchos automóviles nuevos incluyen en su fabricación mecanismos que
abrochan automáticamente el cinturón de seguridad cuando alguien se
sienta en el asiento delantero.
(b) El crecimiento de los costos que deben afrontar las compañías de segu-
ros, como consecuencia de la mayor gravedad de los accidentes sufridos
por personas que no usan cinturones de seguridad, ha hecho aumentar
el costo de los seguros para todos los propietarios de automóviles.
(c) A los pasajeros de aviones siempre se les obliga a usar cinturón de segu-
ridad durante el despegue y el aterrizaje.
(d) La proporción de accidentes automovilísticos fatales en los estados que
no hacen obligatorio el uso de cinturón de seguridad es mayor que la de
los estados que sí lo hacen.
(e) En los accidentes automovilísticos, la proporción de pasajeros que
sufren lesiones es mayor entre quienes no usan cinturón de seguridad
que entre quienes lo usan.

Actividad 2

Traduciendo a lenguaje formal nuestros argumentos cotidianos.

A.- Determine si el segundo de los enunciados es consecuencia deductiva


del primero. Exponga su conclusión por escrito.

1.- Si García está enfermo o Pérez está ausente, entonces ni se concluirá


el asunto “Argus” ni se reunirán los directores y convocarán a asamblea, a
menos que Fernández se haga cargo de la situación.

2.- Si Pérez está ausente y Fernández no se hace cargo de la situación,


entonces no se concluirá el asunto “Argus”.

B.- Determine si alguno de los siguientes enunciados es consecuencia


deductiva del otro:

1.- La compañía es responsable si y sólo si el aparato era un Interplex y


estaba instalado desde enero.

2.- Si el aparato era un Interplex, entonces estaba instalado desde enero y la


compañía es responsable.
AA

EDUBP | Abogacía | Filosofía y Lógica Jurídica - pag.155


AA Actividad 2
asistente académico

Para la traducción a lenguaje formal de enunciados en lenguaje ordinario o


natural tenga presente las recomendaciones establecidas en:

Aplicación de la lógica al lenguaje ordinario ∙

Clases de silogismo ∙

La lectura de los textos indicados tiene como finalidad proporcionar algunas


reglas intuitivas para traducir giros de lenguaje natural a conectivas veritativo-
funcionales de lenguaje formal y, a distinguir clases de silogismo válido y de
silogismo inválido que pueden contribuir, junto con las condiciones de verdad
de los conectivos lógicos, a detectar implicaciones verdaderas y falsas.

Actividad 3

Distinguiendo las condiciones de verdad lógica


1.- ¿Por qué son verdaderas las siguientes afirmaciones?
a) Las tautologías sólo implican tautologías.
b) Las contradicciones implican cualquier proposición.
c) Si A es una tautología, –A es una contradicción.
d) Si A‫ כ‬B es una contradicción, A es una tautología y B es una contradicción.
e) Si A es una contradicción, –A es una tautología.
f) A. B es una tautología si y sólo si tanto A como B son tautologías.
g) A es contingente si y sólo si –A es contingente.
h) Si de un conjunto de proposiciones se sigue una contradicción, no pueden
ser verdaderas todas las proposiciones del conjunto.

2.- Aplique las condiciones de verdad de conectivos lógicos en la clasi-


ficación lógica de enunciados compuestos:
Sea A una tautología, B una contradicción y C una oración contingente.
A.- El derecho es justo o no lo es.
B.- Hoy hace frío y no hace frío.
C.- Si María está enferma, entonces no irá al cine con Juan.
¿Cuáles de las siguientes oraciones son tautológicas, cuáles son con-
tradictorias y cuáles son contingentes?

Lecturas
(desde página 181)

EDUBP | Abogacía | Filosofía y Lógica Jurídica - pag.156


Actividad 4

Análisis de enunciados normativos y determinación de sus relaciones


según cuadrado de oposiciones deónticas y tabla de interdefinibilidad.

Considere el siguiente argumento:


1.- La ley divina prohíbe matar a los hijos.
2.- Miriam, la hija de Jefté, es el primer ser que saldrá al encuentro de Jefté
cuando él regrese a casa.
3.- Es obligatorio que Jefté inmole al primer ser que salga a su encuentro
cuando el regrese a casa porque así lo prometió a Jehová.

Teniendo en cuenta las premisas normativas, establezca las relaciones que


surgen entre ellas considerando las relaciones posibles entre enunciados
normativos que se especifican en el cuadrado de oposiciones deónticas.

Elabore un cuadrado de oposiciones deónticas formulado en términos de


“obligación” (O), como único operador deóntico, por medio de la aplicación
de la tabla de interdefinibilidad de operadores deónticos.

Actividad 5

Analizando lógicamente los argumentos justificatorios de un fallo.


Retomando el extracto de fallo de la C.S.J.N. del 26 de octubre de 2004 en el
caso: “Bustos, Alberto Roque y otros c/ Estado Nacional y otros s/amparo”,
ya trabajado en la actividad 4 del módulo 6:
• Analicemos formalmente los argumentos de la Corte.
• Consideremos si las premisas son coherentes entre sí y si permiten
deducir lógicamente la conclusión de la Corte.

Para la resolución de esta actividad consulte los contenidos del módulo 7 y


Lógica, proposición y norma de Guibourg, Ricardo y otros.

EDUBP | Abogacía | Filosofía y Lógica Jurídica - pag.157


MÓDULO 8

M8 Microobjetivos

• Conocer los modos de implementar el método de reconstrucción de pro-


blemas jurídicos, para poder determinarlo con mayor precisión.
• Desarrollar habilidades en el uso del método de delimitación de los
casos posibles que pueden darse en un problema determinado, para
saber cómo proceder en una caracterización más precisa de algún caso
individual real o hipotético.
• Conocer y comprender en qué consiste la sistematización lógica del con-
junto de disposiciones jurídicas relativas a un problema jurídico deter-
minado, para detectar las soluciones normativas establecidas por dichas
disposiciones.
• Desarrollar habilidades en la aplicación del método de sistematización
de normas relativas a un determinado problema jurídico, con el propósito
de identificar problemas formales tales como lagunas, inconsistencias y
redundancias.

M8 Contenidos

Acceda al video de presentación del Módulo 8

Introducción
Conforme vimos en el módulo anterior, los lógico-deónticos analizaron los
sistemas normativos sólo como sistemas puramente formales (lógicos). En
este módulo expondremos un intento de aplicación de la lógica deóntica al
lenguaje de la ciencia jurídica, esto es, a sistemas interpretados.

La propuesta teórica objeto del presente análisis constituye un


intento de aplicación de las técnicas de análisis formal o
lógico a problemas jurídicos concretos, esto es, a conjuntos
determinados de normas jurídicas.

La teoría general del derecho, tal como la abordamos en la presente asig-


natura, se origina en problemas conceptuales que surgen en el interior de
la experiencia jurídica. Su interés preponderante consiste en analizar cues-
tiones relevantes de la ciencia jurídica, en un nivel de mayor abstracción.

La actividad del teórico del derecho puede ser descripta como una práctica
filosófica. Comprendemos esa práctica por referencia a la idea de la filosofía
analítica y entendemos la labor filosófica como un análisis lógico del len-
guaje. En otros términos y desde la perspectiva expuesta, el propósito de la
filosofía no es el conocimiento del mundo –el cual es objeto de la ciencia-
sino el esclarecimiento lógico de los conceptos.

EDUBP | Abogacía | Filosofía y Lógica Jurídica - pag.158


En concordancia con dicha caracterización de la filosofía decimos, desde un
punto de vista lógico, que la teoría del derecho está ubicada en un nivel supe-
rior al de la ciencia. Según esta concepción, la teoría del derecho es tan sólo
un método, e implica un análisis lógico del lenguaje de la ciencia jurídica.

La teoría del derecho, al igual que la ciencia o dogmática jurídica, tiene por
objeto el análisis del derecho. No obstante, aquella se distingue de ésta, en
primer lugar, porque su análisis es general, además su propósito no reside
en el estudio de los contenidos normativos de un derecho positivo en parti-
cular, sino en indagar las propiedades formales y estructurales del derecho
en general. El producto de su actividad configura un metadiscurso en rela-
ción con el estudio dogmático del derecho.

El análisis formal presupone la identificación del derecho como conjunto de


comunicaciones lingüísticas prescriptivas (lenguaje del legislador, en sentido
amplio) y la distinción, en el seno del discurso prescriptivo, de un “conte-
nido” y de una “forma”. La labor de la dogmática es estudiar el contenido del
derecho y el objeto de la teoría del derecho es el estudio de su forma.

Determinar el contenido del derecho es especificar qué es lo que está


prescripto, a quien se dirige y bajo qué circunstancias. Tal especifica-
ción, supone la interpretación del discurso legislativo. La forma es lo que
permanece una vez abstraído el contenido, es decir, la actividad de pres-
cribir o calificar normativamente el comportamiento (obligatorio, prohibido
o permitido). Por lo tanto, el análisis “formal” del derecho no es más que el
análisis del lenguaje prescriptivo en general -la lógica deóntica-. La teoría del
derecho es una mixtura de una teoría de las fuentes del derecho, como vimos
en la unidad 4 del módulo 3: los fundamentos del derecho y una teoría del
lenguaje prescriptivo. Cabe observar, por un lado, que la teoría de las fuentes
es, al mismo tiempo, una parte de la ciencia jurídica y por el otro, que la teoría
del lenguaje prescriptivo no es una teoría específicamente jurídica, puesto
que el uso del lenguaje prescriptivo forma parte también de otros ámbitos,
además del derecho, por ejemplo, de la moral.

La actividad de los juristas dogmáticos es, típicamente, una actividad de


interpretación y sistematización del discurso legislativo. La labor del teórico,
en cambio, es una reflexión crítica acerca del discurso de los juristas, como
se hará evidente en el tratamiento de los temas comprendidos en lo que
resta del programa. Por ello es atinado afirmar que la teoría del derecho es
una meta jurisprudencia o filosofía de la ciencia jurídica.

La metodología de análisis conceptual del discurso de la ciencia jurídica


resulta relevante en la sistematización formal de las disposiciones jurídicas.
Los resultados de esa labor de sistematización son, a su vez, útiles no sólo
para el proceso de codificación de las disposiciones, sino también, para
precisar las tesis fundamentales sobre la naturaleza normativa del derecho
sostenidas por la dogmática jurídica –derecho concebido como conjunto
de normas legisladas que poseen las propiedades formales de completitud,
coherencia e independencia.

EDUBP | Abogacía | Filosofía y Lógica Jurídica - pag.159


En el marco del presente módulo, la Unidad 10 propone una técnica lógica de
determinación del Problema Normativo y plantea el estudio de sus aspectos
fácticos y normativos. La comprensión respecto a lo que es un problema jurí-
dico y sus aspectos facticos y normativos, es muy importante en el desarrollo
tanto de la comprensión de los problemas centrales de la ciencia jurídica
como la presencia de casos difíciles por indeterminación de las propiedades
relevantes o las consecuencias jurídicas, como así también para el desarrollo
de la práctica de la litigación en la aplicación del derecho.

Quien puede comprender la estructura de los sistemas normativos y la forma


en que regulan la conducta, puede entender, por ejemplo, qué es jurídica-
mente relevante en tal circunstancia, porque el uso del lenguaje utilizado
por el legislador para la descripción de estas propiedades relevantes puede
dar lugar a casos difíciles por la ambigüedad o vaguedad de las palabras.
Asimismo, estas propiedades pueden colisionar con un principio o valor jurí-
dico, como también la forma en que están regulados los casos que se con-
forman con la conexión de esas propiedades pueden dar lugar a contradic-
ciones, lagunas o redundancias, entre otros problemas. La sistematización
y las dificultades que presentan es algo que tiene que enfrentar cualquiera
que se enfrente a la tarea de aplicar el derecho, principalmente los jueces y
los abogados litigantes. Pensemos por un momento qué es lo que debemos
probar en un juicio, sobre lo que tenemos que alegar, y comenzaremos a
vislumbrar que estamos tratando con estas propiedades, estos casos que
comprenden el ámbito fáctico del problema normativo, como así también
con las consecuencias, las soluciones que conforman el aspecto normativo.
El tema es tratado en esta en esta sección, con un nivel abstracto con las
herramientas que Alchourron y Bulygin desarrollaron. El análisis formal abor-
dado nos permite entender la estructura del derecho y a través de ello, poder
utilizarlo mejor.

El Problema normativo: los aspectos fácticos y normativos


La utilización de la lógica deóntica resulta más provechosa en el ámbito del
derecho que en otros ámbitos normativos, por ejemplo, en el de la moral. Ello
se debe, por un lado, a que el derecho cuenta con una plataforma inicial más
claramente identificable pues, en gran parte, las normas jurídicas son pro-
ducto de actos de creación de los legisladores y son expresadas en forma
escrita. Por otro lado, la dogmática jurídica tradicionalmente se ha ocupado
de describir y sistematizar las normas jurídicas. En consecuencia, tanto el
producto de la labor de la ciencia jurídica, como su objeto de estudio, las
normas jurídicas, constituyen una base satisfactoria para la aplicación de las
herramientas suministradas por la lógica deóntica.

La ausencia de un empleo riguroso de las herramientas lógicas en el ámbito


de la ciencia jurídica ha provocado resultados insatisfactorios en el trata-
miento de los problemas centrales del derecho. Una revisión del conoci-
miento obtenido por esta disciplina a la luz de esta perspectiva metodológica
da lugar a resultados esclarecedores.

Tal el caso de la noción de sistema normativo, cuyo análisis es particular-


mente propicio para el objetivo propuesto. La lógica deóntica se caracte-

EDUBP | Abogacía | Filosofía y Lógica Jurídica - pag.160


rizó por el estudio de los sistemas normativos como sistemas deductivos
puramente formales, es decir, como sistemas puramente sintácticos. No obs-
tante, resulta provechosa la aplicación de los cálculos lógicos a determina-
das normas jurídicas como sistemas interpretados.

Si el derecho configura un sistema de normas debe adecuarse a ciertos cri-


terios de racionalidad, pues las normas no pueden ser auto contradictorias
y deben mantener entre sí relaciones de coherencia mutua. La ausencia de
laguna, o la completitud del derecho, también es uno de los aspectos del
derecho que ningún jurista o teórico deja de reconocer como deseable. Por
último, la ausencia de normas redundantes, o la independencia de las dis-
posiciones jurídicas, es un objetivo tanto de legisladores como de científicos
del derecho.

En consecuencia, tanto la noción de sistema normativo, como las ideas de


coherencia, completitud e independencia dan un fundamento sólido al tra-
tamiento lógico de dichos conceptos. Los profesores Carlos Alchourrón y
Eugenio Bulygin, en su Introducción a la Metodología de las Ciencias Jurídicas
y Sociales elaboran, partiendo de un problema determinado, un modelo res-
tringido de sistema jurídico con el propósito de exhibir el comportamiento
lógico del concepto de laguna en la dogmática jurídica a fin de dar una
definición más precisa de ese término. Dicha definición pondrá de manifiesto
el carácter esencialmente relacional de la noción de laguna. En efecto,
esta noción relaciona tres elementos, un conjunto de normas –sistema
normativo, un conjunto de circunstancias fácticas o casos elementales
lógicamente posibles –universo de casos, y un conjunto de soluciones
normativas –universo de soluciones. Un análisis del problema de las lagu-
nas en el derecho impone, como tarea primordial, precisar el dominio de
cada uno de los elementos de esa relación. La explicación de un concepto
es el método por el cual se transforma un concepto impreciso o vago en
un concepto más exacto o preciso. El método de explicación consta de
dos etapas: por un lado, tenemos que clarificar, sea por medio de ejem-
plos o descripciones del uso vigente, el alcance del concepto que se
quiere explicar, al que denominamos explicandum y, por el otro, construir
el nuevo concepto, explicatum, que sustituirá al anterior. El nuevo concepto
que resulte de la explicación debe ser:
1.- más exacto: su regla de uso debe estar formulada explícitamente a
través, por ejemplo, de una definición.
2.- más fecundo: más usado en la formulación de enunciados o principios
generales de la teoría científica.
3.- similar: al concepto que se quiere sustituir, de modo que pueda ser
empleado en las mismas ocasiones en que se usaba el anterior.
4.- preferentemente más simple.

Para la explicación del concepto de sistema normativo se parte, en primer


lugar, de la construcción de un modelo que reproduzca un problema real
tomado del derecho. Esta primera etapa en la reconstrucción conceptual
da lugar a definiciones provisionales de todos los conceptos jurídicos funda-
mentales que se quieren explicar, a saber: sistema normativo, completitud,
coherencia e independencia. Dichas definiciones admiten ser generaliza-

EDUBP | Abogacía | Filosofía y Lógica Jurídica - pag.161


das y precisadas en su aplicación no sólo a los sistemas jurídicos, sino a
sistemas normativos de cualquier tipo.

El problema normativo tomado como punto de partida del análisis es el


problema de la reivindicación de cosas inmuebles contra terceros posee-
dores. Este problema surge toda vez que un poseedor de un inmueble, que
no es de su propiedad, lo transfiere a un tercero. La cuestión jurídica emer-
gente es la de determinar bajo qué circunstancias el propietario del inmue-
ble puede revindicarlo accionando judicialmente contra el tercero poseedor.
Desde la perspectiva del tercero poseedor, la misma cuestión puede formu-
larse en términos de la determinación de las circunstancias bajo las cuáles
estaría obligado a restituir el inmueble a su propietario y, también, de las
circunstancias en que el derecho le permite retenerlo.

Lo relevante para responder al problema planteado es determinar el estatus


normativo – obligatorio, prohibido o permitido – de la acción consistente en
la restitución del inmueble.

La acción de restitución puede tener lugar dentro de un conjunto de situacio-


nes a las que se denomina Universo del Discurso (UD). Cada situación indi-
vidual que dé lugar a esta cuestión jurídica será un elemento de ese conjunto
que denominamos universo del discurso. Todos los elementos del discurso
tienen en común una propiedad que es definitoria del conjunto de ellos,
esto es, del universo del discurso.

La propiedad definitoria del conjunto de situaciones -UD, que da lugar al


problema normativo planteado es la enajenación o transferencia de un
inmueble que pertenece a un tercero.

Toda acción, cuyo estatus normativo se quiere determinar, puede presen-


tarse de un modo elemental o básico. El conjunto de acciones básicas o ele-
mentales, lógicamente posibles constituye lo que se denomina Universo de
Acciones (UA). La acción es básica si no forma parte de un compuesto verita-
tivo funcional consistente en una conjunción, disyunción o condicional de des-
cripciones de acciones de restitución y algunas otras. En el problema que nos
ocupa, la acción de restitución es la única acción básica perteneciente a UA.

Tanto el dominio o universo de discurso como el universo de acciones deli-


mitan el ámbito del problema normativo. Si cambiamos el UD o el UA se pro-
duce un cambio de problema. En efecto, el tipo de situación -UD- puede ser
el mismo pero podemos cuestionarnos sobre el estatus de otras acciones,
por ejemplo, el pago del impuesto inmobiliario, la apropiación de los frutos
producidos por el inmueble, etc. Por el contrario, puede permanecer la cues-
tión sobre el estatus de la acción de restitución –UA-, pero variar el tipo de
situación –UD-, por ejemplo, porque la cosa enajenada es una cosa mueble.
En cualquiera de los dos casos cambia el problema normativo.

La respuesta relativa al estatus normativo de una acción, como la de resti-


tución en el problema que analizamos, depende de la valoración de las cir-
cunstancias que se consideren relevantes en el tipo de situación planteada.
Las circunstancias o propiedades que se estimen relevantes son hechos
contingentes, esto es, pueden variar de un contexto a otro. De ese modo,

EDUBP | Abogacía | Filosofía y Lógica Jurídica - pag.162


propiedades que en ciertos contextos pueden considerarse irrelevantes,
como el color de la piel del propietario, en otros pueden valorarse como rele-
vantes a los fines de la calificación normativa de la acción. La selección de
propiedades relevantes, en consecuencia, depende del consenso valorativo
del grupo social. En el problema que nos ocupa, las propiedades relevantes
son tres: buena fe del adquirente, buena fe del enajenante y título oneroso,
que serán designadas como BFA, BFE y TO respectivamente. La ausencia
de estas propiedades, o bien, la mala fe de adquirente o enajenante y el título
gratuito, serán designadas anteponiendo el signo de la negación a cada uno
de esos símbolos según corresponda, esto es, –BFA, –BFE y –TO. El signi-
ficado que se estipula para “buena fe” es desconocimiento del hecho de que
el dominio del inmueble es de un tercero.

Las propiedades mencionadas son entonces propiedades de los elementos


del universo de discurso. Cada una de estas propiedades divide los elemen-
tos del UD en dos clases: la clase de aquellos elementos en los que la pro-
piedad está presente y la clase de los elementos en los que la propiedad está
ausente y en consecuencia, se da su propiedad complementaria (designada
anteponiéndole ‘–‘ ). El conjunto total de propiedades presentes o ausentes
en los elementos de un UD es denominado Universo de Propiedades - UP.

Toda propiedad de un UP y todo compuesto veritativo funcional de tales pro-


piedades -que no sea tautológico o contradictorio- define un caso posible.

El caso elemental es aquel cuya propiedad definitoria es una conjunción


que contiene todas las propiedades del UP o sus negaciones (pero no
ambas). A su vez, llamaremos Universo de Casos a todos los casos ele-
mentales del UP.

El número de casos elementales lógicamente posibles se determina en fun-


ción del número de propiedades del UP. Si n es el número de propiedades, 2ⁿ
es el número de los casos elementales, o casos del UC. El procedimiento es
una aplicación del método de tablas de verdad estudiado en el módulo 7 en
oportunidad de determinar el significado de los conectivos proposicionales.
El número 2 surge de las dos posibilidades que cada propiedad tiene de estar
presente o ausente en cada situación particular del UD. En el problema jurí-
dico analizado las propiedades relevantes son tres, de modo que el número
de casos elementales, o casos del UC será 2³ = 8. La siguiente tabla repre-
senta los casos elementales, en los que la presencia de una propiedad se
representa con el símbolo ‘+’ y la ausencia de la propiedad con el símbolo ‘–‘:

TABLA 1:
BFE BFA TO
1 + + +
2 – + +
3 + – +
4 – – +
5 + + –
6 – + –
7 + – –
8 – – –

EDUBP | Abogacía | Filosofía y Lógica Jurídica - pag.163


El Ámbito Fáctico del Modelo queda determinado por todos los casos ele-
mentales posibles indicados en la tabla precedente.

Los casos elementales especificados, son casos simples en el sentido de


que no pueden subdividirse en otros casos. Los casos complejos, en
cambio, son analizables en términos de los casos elementales y, equiva-
len a la disyunción de dos o más casos elementales. Además toda situa-
ción individual que sea un elemento del UD tiene que pertenecer a un caso
elemental del UD y no más que a uno.

Cada uno de los casos especificados son casos en los que surge la cuestión
de determinar el estatus normativo de la acción de restitución del inmue-
ble. El paso siguiente consiste en especificar el carácter deóntico que posee
la acción en cada uno de esos casos elementales. El dominio de todas las
respuestas o soluciones posibles a esta cuestión normativa es lo que se
denomina el ámbito normativo del problema.

Dijimos anteriormente que el UA es unitario pues está determinado por una


única clase de acción, a saber, la acción de restitución del inmueble, por lo
cual, el UA tendrá únicamente dos elementos, a saber: restitución (R), o no
restitución (–R). En cuanto al posible estatus deóntico de la acción R, se dirá
que las posibles calificaciones deónticas de R son las siguientes:
1. OR = PR ∙ –P–R.
2. PhR = –PR ∙ P–R
3. FR = PR ∙ P–R

Cada una de las expresiones precedentes (PR, OR, etc.) serán llamadas
soluciones y cada solución determina deónticamente algún contenido nor-
mativo –acción elemental. Cuando la solución determina todos los conteni-
dos que corresponden a los elementos del UA decimos que es una solución
maximal. El conjunto de todas las soluciones maximales relativas a un UA es
el Universo de Soluciones Maximales (USmax) que contiene el conjunto
de todas las respuestas posibles a la pregunta sobre el carácter normativo
de la acción de restitución del inmueble, dentro del dominio de discurso
determinado por la delimitación del problema. Si la solución del universo de
soluciones no fuera maximal, ésta no sería completa porque habría alguna
acción elemental que carecería de determinación normativa o deóntica.
Habría, pues, una laguna normativa respecto del / los casos elementales
carentes de estatus normativo. Por ejemplo, si uno de los casos elemen-
tales tiene la solución PR y, –R carece de determinación deóntica, nada
puede inferirse respecto del estatus de –R a partir de PR, pues –R puede
estar o bien Ph, o bien P. El universo de soluciones que no sea maximal
será denominado parcial y será definible en términos de una disyunción de
dos o más soluciones maximales. En el caso del ejemplo precedente el
estatus deóntico de –R será equivalente a: F–R ٧ Ph –R.

Habiendo delimitado los conceptos de caso y solución cabe definir norma


como todo enunciado que correlaciona un caso con una solución. Un
ejemplo sería: “si el adquirente es de mala fe, entonces está obligado a
restituir el inmueble al propietario”, esta norma puede representarse por la
expresión ‘OR/–BFA”.

EDUBP | Abogacía | Filosofía y Lógica Jurídica - pag.164


Todo conjunto de normas constituye un sistema normativo, pero la
inversa no vale, pues un sistema normativo además de normas puede
contener otros enunciados, por ejemplo definiciones.

Como ejemplo de reconstrucción de un sistema normativo restringido


a un problema, en este caso al problema usado para delinear el modelo,
se compara la regulación tanto del Código Civil argentino, artículos 2777 y
2778, como la del proyecto de Freitas usado como fuente de nuestra regula-
ción y constituido por los artículos 3877, 3878, 3882.

Según surge de la interpretación de las disposiciones del proyecto de


Freitas, su sistema al que se denomina ‘S1’, estaría integrado por las siguien-
tes normas:
S1 = {N1N2N3N4}
N1: OR/–BFE; abarca los casos 2, 4, 6 y 8 de Tabla 1
N2: OR/–BFA; abarca los casos 3, 4, 7 y 8 de Tabla 1
N3: OR/–TO; abarca los casos 5, 6, 7 y 8 de Tabla 1
N4: FR/BFA∙BFE∙TO; FR para el caso 1 de Tabla 1 único que reúne las
tres propiedades.

Uniendo en un solo gráfico, los casos elementales y las normas que correla-
cionan cada uno de ellos con una solución normativa tomando como base
el S1, obtenemos lo que se denomina una matriz que se reproduce en la
siguiente tabla:

TABLA 2:

N1 N2 N3 N4
1 BFE BFA TO FR
2 –BFE BFA TO OR
3 BFE –BFA TO OR
4 –BFE –BFA TO OR OR
5 BFE BFA –TO OR
6 –BFE BFA –TO OR OR
7 BFE –BFA –TO OR OR
8 –BFE –BFA –TO OR OR OR

Para el modelo propuesto se formulan las siguientes definiciones:


1.- cuando en la línea correspondiente al menos a un caso no aparece nin-
guna solución se dirá que ese caso constituye una laguna (normativa) y,
el sistema es incompleto.
2.- cuando en la línea correspondiente al menos a un caso figuran dos o
más soluciones incompatibles, diremos que el sistema es incoherente.
3.- cuando en la línea correspondiente al menos a un caso figura la misma
solución más de una vez, diremos que el sistema es redundante.

EDUBP | Abogacía | Filosofía y Lógica Jurídica - pag.165


De las definiciones precedentes surge que el sistema S1 es completo, cohe-
rente, pero redundante. Para eliminar la redundancia habría que sustituir las
normas N1 y N2 por las normas:
N5: OR/–BFE∙BFA∙TO
y
N6: OR/–BFA∙TO
S2 = {N3,N4,N5,N6}

El sistema resultante de la sustitución es denominado S2 y, si bien propor-


ciona las mismas soluciones que S1, del mismo puede decirse que a diferen-
cia de S1, es completo, coherente e independiente, lo cual puede ser corro-
borado mediante la construcción de la correspondiente matriz.

La construcción de matrices permite detectar problemas formales de los


sistemas normativos que por la simple lectura de las normas que lo com-
ponen pueden pasar desapercibidos.

La diferencia entre los casos de laguna y los casos de incoherencia es que


en los primeros hay ausencia de norma y, en los segundos superabundancia
de normas. Las consecuencias prácticas que surgen de ambos problemas
suelen ser similares, no obstante, resulta importante distinguirlos.

En cuanto al sistema que surge de las dos disposiciones de nuestro código


civil, a saber:

Art. 2777: “Compete también (la reivindicación) contra el actual poseedor


de buena fe que por título oneroso la hubiere obtenido de un enajenante de
mala fe…”

Art. 2778: “Sea cosa mueble o inmueble, la reivindicación compete… contra


el actual poseedor, aunque de buena fe, si la hubo por título gratuito…”

De la interpretación de esas disposiciones surge el siguiente sistema S3 =


{N5,N3} del sistema S2. Las matrices correspondientes a los tres sistemas
analizados quedan representadas en el siguiente cuadro:

Casos S1: S2: S3:


N1, N2, N3, N5, N6, N3, N 5 ,
N4 N4 N6
1 BFE BFA TO FR FR
2 –BFE BFA TO OR OR OR
3 BFE –BFA TO OR OR
4 –BFE –BFA TO OR OR OR
5 BFE BFA –TO OR OR OR
6 –BFE BFA –TO OR OR OR OR
7 BFE –BFA –TO OR OR OR OR
8 –BFE –BFA –TO OR OR OR OR OR

La matriz del sistema S3, correspondiente a la regulación de Vélez Sársfield


de nuestro código civil, muestra la existencia de tres lagunas: en los casos 1,
3 y 4. El sistema es incompleto, coherente e independiente.

EDUBP | Abogacía | Filosofía y Lógica Jurídica - pag.166


Se ha tratado de demostrar la relevancia práctica que puede tener la imple-
mentación de las técnicas de lógica deóntica en la reconstrucción racional de
un problema jurídico, por medio de la elaboración de matrices que sistemati-
zan un conjunto de normas positivas que proporcionen una solución a todas
las situaciones particulares involucradas en el problema. La construcción de
dichas matrices puede guiar racionalmente tanto el tratamiento legislativo de
un problema, como la subsunción de una cuestión jurídica particular en una
norma general a los fines de su resolución judicial.

Para terminar y con el propósito de acceder a una definición más precisa


de los conceptos de completitud–laguna y coherencia–inconsistencia, se
plantea la necesidad de hacer algunas aclaraciones respecto de la noción
de caso.

Concepto de Caso.
Si lo que importa es determinar la completitud de un sistema normativo en el
sentido que soluciona todos los casos posibles, esto implica que los únicos
casos que integrará el universo de casos son los casos contingentes. Se
excluyen, en consecuencia, los casos necesarios y los imposibles pues
ambos tipos de casos hacen a las normas inoperantes, imposibles de aplicar
por razones que no dependen de su aceptación o rechazo. En efecto, las
normas que establecen condiciones de aplicación que no pueden darse,
por ejemplo, una norma que estableciera una pena especial para el aborto
cometido en el decimosegundo mes de embarazo, sería inoperante o inapli-
cable. También lo sería una norma que prescribiera una conducta de realiza-
ción imposible, por ejemplo, la norma que prohíbe a los médicos resucitar a
los muertos, también es inaplicable, pues es imposible realizar esa conducta.
Asimismo, la norma que obliga a los médicos a abstenerse de resucitar muer-
tos es superflua, pues obliga a la realización de una acción -abstención- que
es de realización necesaria.

Los casos, como hemos visto anteriormente, se determinan por una combi-
nación de propiedades y esas propiedades deben satisfacer características
que garanticen que sus combinaciones sean contingentes (ni necesarias, ni
imposibles). Tales requisitos de las propiedades son:

1.- deben ser lógicamente independientes entre sí y, lo son cuando la pre-


sencia de cualquiera de esas propiedades en una cosa o suceso es com-
patible con la presencia o ausencia de las otras.

2.- las propiedades que definen los casos posibles deben ser independien-
tes de las características que definen la/s acción/es que conforma/n el
universo de acciones.

3.- debe haber correspondencia entre el universo del discurso y el universo


de propiedades en el sentido de que cada elemento del universo del
discurso, puede poseer cada una de las propiedades del universo de
propiedades.

EDUBP | Abogacía | Filosofía y Lógica Jurídica - pag.167


Universo de Casos:
El concepto de caso se define en términos de las propiedades de UP del
siguiente modo:
1.- Si Pi es una propiedad de UP, entonces Pi define un caso.
2.- Si Pi es un caso, entonces la negación de Pi (–Pi) es un caso.
3.- Si Pi y Pj son casos, entonces Pi . Pj y Pi v Pj son casos.

Dicho en términos más sencillos, cada propiedad o su ausencia definen


un caso posible.

Si UP= { P1,P2 } (conjunto de elementos de UP)

2² = 2 posibilidades: que propiedades estén presentes o ausentes, elevado


al número, en este caso 2, de propiedades del UP da como resultado el
número de casos elementales lógicamente posibles. ‘+’ indica presencia de
propiedad y, ‘–‘ indica su ausencia.

Tabla T1:

P1 P2 Casos Elementales Propiedades definitorias de cada caso


+ + C1 P1 . P2
– + C2 – P1 . P2
+ – C3 P1 . – P2
– – C4 – P1 . – P2

Entonces, UC = {C1, C2, C3, C4} (conjunto de casos elementales del UC)

Cada caso se define por la conjunción de cada una de las propiedades, o su


complementaria

(–P). Esto supone que tanto los elementos del UP como los elementos del
UC son finitos.

El conjunto de propiedades del UP dividen los casos sí y sólo si satisfa-


cen tres condiciones:
(a) que sean lógicamente disyuntos, pues todo elemento del UD tiene
alguna de las propiedades del conjunto.
(b) Cada par de propiedades distintas – propiedad y su complementaria -
son mutuamente excluyentes.
(c) Ninguna propiedad es lógicamente vacía o lógicamente imposible.

De este modo, los casos determinados por un UP son lógicamente disyuntos


y mutuamente excluyentes. En el UC definido por la tabla T1.

EDUBP | Abogacía | Filosofía y Lógica Jurídica - pag.168


P1. P2 –P1. P2
UC UC = {(P1. P2) v (–P1. P2) v (P1.–P2) v ( –
P1.–P2)}
P1. –P2 –P1. –2

También es posible un UC constituido por casos caracterizados por un valor


numérico. Por ejemplo, una ley que establece el impuesto a las ganancias y
divide a los contribuyentes por su renta mensual en diferentes categorías, por
ejemplo, aquellos con un ingreso mayor a $4.000 y menor a $6.000 pagarán,
en concepto de impuesto, la suma de $200, los que posean ingreso mayor
a $6.000 y menor a $8.000, pagarán $300 y los que ganen más de $8.000
pagarán $500. De este modo se crean tres categorías de contribuyentes y se
determinan, en consecuencia, tres casos posibles.

La infinitud de casos posibles no implica imposibilidad de dar una solución


para todos ellos. Un modo de solucionar todos los casos de un UC infinito
puede consistir en indicar porcentajes de contribución sobre, por ejemplo,
valores patrimoniales, en el caso del impuesto al patrimonio o sobre el valor
de la sucesión, en el caso del impuesto sucesorio, etc.

Casos genéricos y casos individuales.


El término caso es ambiguo pues suele usarse tanto para hacer referencia a
los elementos del UD como a los elementos del UC (cotejar con lo analizado
en la primera unidad de este módulo). De tal modo, suele hablarse del divor-
cio de Juan y María y del instituto de divorcio por presentación conjunta, etc.
Para eliminar la ambigüedad se distinguen las nociones de caso individual y
de caso genérico.

Se denominan:

Casos individuales: a los elementos del UD. Dichos casos son situaciones
o acontecimientos que se dan en una ocasión temporal y espacial determi-
nada. El UD es el conjunto de casos individuales o particulares que compar-
ten una cierta característica o propiedad común que los define como miem-
bros del UD,

Casos genéricos: a toda propiedad que define un subconjunto del UD (caso


de enajenación de un inmueble que es propiedad de un tercero, caso de
divorcio por presentación conjunta, etc.).

Los casos genéricos pueden ser ejemplificados por un número infinito de


casos individuales, por ejemplo, el divorcio por presentación conjunta de
Juan y María.

El conjunto de todos los casos de un UC es exhaustivo del UD porque


abarca todos los elementos del UD y, además, los casos genéricos de un
UC son mutuamente excluyentes en el sentido de que cada elemento del
UD se corresponde con uno y sólo un elemento del UC, por eso decíamos
que son lógicamente disyuntos.

EDUBP | Abogacía | Filosofía y Lógica Jurídica - pag.169


El carácter exhaustivo del UC garantiza la completitud y el carácter
mutuamente excluyente de los casos del UC garantiza la coherencia.

Por tanto, la solución de todos los casos genéricos del UC proporciona solu-
ción a todos los casos individuales del UD.

Toda norma general, que correlaciona un caso genérico con una solución,
proporciona indirectamente una solución a todos los casos individuales que
pertenecen a ese caso genérico.

El legislador puede solucionar un número infinito de casos individuales pro-


mulgando un número finito de normas generales. Para dar solución a todos
los casos individuales de un UD tiene que clasificar el UD mediante un UC
y solucionar todos los casos elementales del UC. Si omite solucionar algún
caso genérico del UC su ordenamiento tendrá una laguna normativa. Por
tanto, el problema de las lagunas se da a nivel de casos genéricos y no
de casos individuales. El legislador no necesita prever todos los casos
individuales que puedan darse en el futuro, pues su función consiste en
solucionar casos genéricos mediante la creación de normas generales.
(Cotejar este punto con lo analizado en el módulo 4, Unidad 5. En dicho
tema se analiza la crítica de Hart al formalismo – dogmática jurídica – basada
en la tesis de la textura abierta de términos que figuran en la formulación de
leyes o normas generales. Obsérvese que conforme a la clasificación de los
casos, examinada en este punto, se infiere que Hart hace referencia a los
casos individuales).

Problemas de aplicación de normas generales a casos individuales:


lagunas de conocimiento y lagunas de reconocimiento.
Es importante distinguir problemas conceptuales que pueden surgir en la
determinación de los casos genéricos y las normas generales y, los proble-
mas asociados con la aplicación de las normas generales a casos indi-
viduales.

El problema de aplicación es un problema de clasificación del caso indi-


vidual, esto es, su ubicación dentro de algún caso genérico -problema de
subsunción- puede tener su origen en dos fuentes:
a) falta de información relativa al hecho concreto que deseamos clasi-
ficar porque desconocemos si se da o no un rasgo importante para su
clasificación, por ejemplo, si la enajenación que Pedro hizo de su casa
a Juan fue a título oneroso o gratuito, pues ignoramos si se pagó o no
un precio. Este tipo de problemas es resuelto por los juristas por medio
del recurso a las presunciones legales. Este recurso permite a lo jueces
decidir ‘como si’ conocieran todos los hechos relevantes del caso. Así,
por ejemplo, rige el principio de la carga de la prueba, según el cual,
todo el que alega un hecho debe probarlo y si no se lo ha acreditado
debidamente, se lo tiene como no acaecido. Sabemos que la “existencia
jurídica” de los hechos no siempre coincide con su “existencia real”.
b) indeterminación semántica, vaguedad o textura abierta de los concep-
tos generales, pues podemos saber que Juan pagó un precio a Pedro,
pero ignoramos si la suma abonada por Juan constituía o no un precio

EDUBP | Abogacía | Filosofía y Lógica Jurídica - pag.170


en sentido técnico, por ejemplo, porque la suma abonada era inferior al
valor estimativo de la casa vendida. Esta dificultad es más seria porque
no puede ser resuelta totalmente, sino sólo mitigada por la introducción
de términos técnicos mediante definiciones explícitas que estipulen con
mayor precisión sus reglas de aplicación. Pero, por más precisas que
sean esas definiciones, siempre surgirán eventos o hechos atípicos que
escapan a la aplicación de esas reglas. En el ejemplo citado, la duda que
versa sobre si la suma abonada constituyó o no un precio, implica duda
sobre la naturaleza del contrato celebrado por Pedro y Juan, esto es, no
se sabe con certeza si entre ellos tuvo lugar un contrato de compraventa
o una donación. Esta dificultad surge de la vaguedad que poseen tanto
los conceptos jurídicos como los empíricos y, como dijimos, puede ser
mitigada pero no puede ser eliminada totalmente.

Los problemas conceptuales y los inherentes a la aplicación de normas gene-


rales a casos individuales suelen confundirse agrupándolos bajo la denomi-
nación común de “problemas de interpretación”.

La confusión que se genera hace explícita la dificultad que surge cuando se


aplican herramientas lógicas a sistemas interpretados, esto es, al lenguaje
de las ciencias, como es el caso del modelo que estamos analizando.

En general, se habla de ‘lagunas’ para hacer referencia tanto a los proble-


mas de conocimiento como a los problemas de indeterminación, pero resulta
conveniente distinguir los dos tipos de problemas del siguiente modo:

1.- Se denominarán lagunas de conocimiento a los problemas de subsun-


ción o clasificación de los casos individuales que surgen por falta de
conocimiento de algún rasgo o característica del hecho.

2.- Se denominarán lagunas de reconocimiento a los problemas de clasifica-


ción de casos individuales que surgen por la indeterminación semántica de
los conceptos que caracterizan a un caso genérico (por ejemplo, indeter-
minación semántica de las propiedades “título oneroso” y “título gratuito”).

Para evitar las aludidas confusiones, es pertinente distinguir tres clases de


lagunas:

Lagunas normativas: son lagunas de índole conceptual o lógico. Cuando en


una matriz no aparece correlacionado algún caso elemental de un UC con
una solución normativa decimos que el sistema normativo es incompleto. Por
lo tanto, este concepto de laguna está relacionado con la completitud o falta
de completitud del sistema normativo.

Lagunas de conocimiento y lagunas de reconocimiento: que aparecen en


el nivel de aplicación de las normas generales a casos individuales y tienen
su origen en problemas empírico-semánticos. Estos problemas surgen con
independencia del hecho de que el sistema normativo sea o no completo.
Son problemas de duda en la clasificación de un caso individual. Podemos
saber que el caso tiene una solución normativa, pero no saber cuál de las
soluciones aplicar. Los casos de laguna de reconocimiento son denomina-

EDUBP | Abogacía | Filosofía y Lógica Jurídica - pag.171


dos por H. L. A Hart “casos de penumbra”1 y es importante recalcar que
esos casos no tienen su origen en un defecto formal del sistema, sino que se
deben a ciertas características del lenguaje empleado en la formulación de
las normas generales. Se suele emplear el término de “umbral” para determi-
nar el problema conceptual como un problema que da origen a una alternativa
gobernada por el principio lógico del tercero excluido, por ejemplo, en el
caso citado de Pedro y Juan, sabemos que la enajenación fue a título oneroso
o a título gratuito, sólo que dudamos a cuál de los dos casos genéricos perte-
nece el caso particular citado. Pero a nivel conceptual sabemos que, o bien
hay compraventa o hay donación, lo que ignoramos es a cuál de las catego-
rías conceptuales corresponde la enajenación de Pedro. También sabemos
que, por el carácter excluyente que tienen los casos elementales de un mismo
UC, el caso de Pedro no puede ser compraventa y donación a la vez.

Con el propósito de afinar aún más los conceptos de completitud y coheren-


cia de un sistema normativo procedimos a dar un concepto de “caso” más
preciso. Vimos que cada propiedad o su ausencia definen un caso posible
del universo de casos y que los casos determinados por un UP son lógica-
mente disyuntos y mutuamente excluyentes. La especificación del carácter
exhaustivo del UC garantiza la completitud y el carácter mutuamente exclu-
yente de los casos del UC garantiza la coherencia.

También se esclareció una confusión generada en el seno de la teoría del


derecho como consecuencia de la ambigüedad del término caso. La distin-
ción efectuada entre dominio o universo del discurso y dominio o universo
de casos deslinda los dos sentidos diversos en que empleamos el término
“caso”. En efecto, denominamos “casos individuales” a los elementos que
integran el universo de discurso y “casos genéricos” a los elementos del
universo de casos.

La ambigüedad del término “caso” da lugar a diferentes sentidos en que


suele usarse el término “laguna”. Uno de los sentidos hace referencia al pro-
blema que se da a nivel de casos genéricos y normas generales y tiene
lugar cuando el legislador omite dar solución a alguno de los casos elemen-
tales que conforman el universo de casos. A dicho sentido se lo denominó
“laguna normativa”.

Pero también se habla dentro de la teoría del derecho de “lagunas” para


aludir a problemas asociados con la aplicación o subsunción de las normas
generales a casos individuales. Tales problemas pueden surgir como conse-
cuencia de falta de información relativa a los hechos concretos que un juez
debe juzgar –laguna de conocimiento–, o de la indeterminación semántica
o textura abierta de conceptos empleados por el legislador al formular la
norma general –laguna de reconocimiento– que genera problemas de cla-
sificación de casos individuales.

La distinción resulta importante a la hora de evaluar la importancia de una


teoría del derecho que proporcione un método de reconstrucción racional
del material jurídico que sirva de marco general para la determinación lógica
1 Hart, H. L. A, El Concepto de Derecho, Abeledo Perrot, Buenos Aires, 1998. En la primera parte
del capítulo 7 el autor hace un análisis minucioso de la vaguedad o textura abierta de los términos
generales y de los casos de penumbra que surgen de la aplicación de estos términos. Ver también
contenidos del módulo 4, unidad 5.

EDUBP | Abogacía | Filosofía y Lógica Jurídica - pag.172


o conceptual de casos y soluciones lógicamente posibles, pese a la dificul-
tad que puede generarse al momento de su aplicación a hechos concre-
tos pues, como es sabido, cualquier teoría o modelo conceptual se vuelven
imprecisos al ser aplicados.

A los fines de completar el estudio de los temas del módulo diríjase a la


siguiente lectura:
• Alchourrón, Carlos y Bulygin, Eugenio; Introducción a la Metodología de
las Ciencias Jurídicas y Sociales. Bs. As., Ed. Astrea, 1974, capítulos 1 y 2.

Si usted desea ampliar acerca de los conceptos vistos en el módulo, puede


profundizar leyendo los siguientes materiales complementarios:
• Alchourrón, Carlos y Bulygin, Eugenio; “Lógica de normas y lógica de
proposiciones normativas” en Análisis Lógico y Derecho, Centro de
Estudios Constitucionales. Madrid, 1991, págs. 25-50.
• Alchourrón, Carlos y Bulygin, Eugenio; “Normas, proposiciones normati-
vas y enunciados Jurídicos”, en Análisis Lógico y Derecho, ps. 169-194.
• Alchourrón, Carlos y Bulygin, Eugenio; “Sentencia judicial y creación de
derecho”, en Análisis Lógico y Derecho, ps. 355-370.

Lo invito a realizar las actividades del módulo, a fin de poner en práctica los
conceptos estudiados.

Al finalizar el estudio del Módulo 8, usted está en condiciones de realizar la


Cuarta Parte de la evaluación parcial.

Acceda al video de cierre

EDUBP | Abogacía | Filosofía y Lógica Jurídica - pag.173


M8 Actividades
Actividad 1

Determinando el problema normativo y correlacionando los casos ele-


mentales con las soluciones deónticas.

Problema normativo: Un simple tenedor de un inmueble ha sido destinata-


rio de una acción de reivindicación.

El ámbito fáctico del problema está constituido por dos circunstancias: es


poseedor a nombre de otro (PO), declaró el nombre y residencia de la per-
sona a cuyo nombre posee (D).

Ámbito normativo: Determinado por la legislación conformada por los artí-


culos 2462, 2464, 2467 y 2782 del Código Civil.

¿En qué casos está obligado a responder por la acción?

Determine los casos elementales -universo de casos-, y construya la matriz


que establezca la correlación entre los casos elementales y las soluciones
normativas -universo de soluciones- tomando en consideración las disposi-
ciones citadas.

Actividad 2

Sistematicemos un problema normativo


Si nuestro problema es determinar en qué casos corresponde la ejecución
judicial de un contrato y las propiedades relevantes –ámbito fáctico del pro-
blema- es que haya o no error o que haya o no violencia, ¿qué casos ele-
mentales se configurarían?

Las disposiciones jurídicas de un sistema normativo imaginario serían:

N1: El tribunal podrá ordenar la ejecución judicial de todo contrato que haya
sido celebrado con el consentimiento de las partes y en los que no haya
mediado error ni violencia.

N2: En los casos en que haya mediado error el tribunal no podrá ordenar la
ejecución judicial del contrato.

N3: Si se trata de un contrato que ha sido celebrado mediante error y violen-


cia, el tribunal no podrá ordenar su ejecución judicial.

¿El sistema normativo es completo o posee alguna laguna normativa?


Determine la matriz que correlacione los casos elementales con las solucio-
nes normativas para responder a esta última cuestión planteada.

EDUBP | Abogacía | Filosofía y Lógica Jurídica - pag.174


Actividad 3

Deslindando problemas normativos de problemas de conocimiento, de


reconocimiento y valorativos.

En relación a la controversia planteada entre la Cámara Federal de Paraná


y la C.S.J.N. en el caso “Bustos, Alberto Roque y otros c/ Estado Nacional y
otros s/amparo”, ya trabajado en la actividad 4 del módulo 6:

• Exponga tomando en consideración los argumentos en debate, si resul-


tan aplicables al análisis alguna/s de las nociones de laguna distinguidas
por Alchourrón y Bulygin.

Consulte para el desarrollo de la actividad, los contenidos del módulo 8 rela-


tivos al tema en cuestión.

Actividad 4

Distingamos normas de enunciados que no son normas.


Teniendo en cuenta la noción de norma como enunciado que correlaciona
casos con soluciones y, además la idea de que todo conjunto de normas
constituye un sistema normativo pero que no todo sistema normativo con-
tiene únicamente normas sino que puede contener otro tipo de enunciados:
distinga cuáles de los siguientes enunciados son normas y cuáles no:
1.- Las convenciones particulares no pueden dejar sin efecto las leyes en
cuya observancia estén interesados el orden público y las buenas cos-
tumbres.
2.- Se declaran incapaces por demencia a las personas que por causa
de enfermedades mentales no tengan aptitud para dirigir su persona o
administrar sus bienes.
3.- Será reprimido con… el funcionario público que impusiese a los presos
que guarden severidades, vejaciones o apremios ilegales.
4.- Será reprimido con… el que publicare, fabricare o reprodujere libros,
escritos, imágenes u objetos obscenos.
5.- El que pague en término el impuesto a la renta será beneficiado con un
descuento del 10% y el que incurra en mora en el pago del impuesto a la
renta sufrirá un recargo del 20%.
6.- El presidente de la Nación deberá haber nacido en el territorio nacional.
7.- El que se apoderare ilegítimamente de una cosa ajena sin hacer uso de
intimidación o violencia será reprimido con 6 años de prisión.
8.- El que mediante el empleo de armas se apoderare ilegítimamente de una
cosa que no le pertenece sufrirá 4 años de prisión.

¿Ha podido identificar las normas entre el conjunto de enunciados enu-


merados?
Fundamente su decisión.

EDUBP | Abogacía | Filosofía y Lógica Jurídica - pag.175


ANEXO
Lecturas
FLJ · n o v e n o c u at r i m e s t r e

EDUBP | Abogacía | Filosofía y Lógica Jurídica - pag.176


módulo 6
Lectura
referencia página 123

Moreso, J. (2010) Conflicto de derechos


constitucionales y como resolverlos

EDUBP | Abogacía | Filosofía y Lógica Jurídica - pag.177


ARBOR Ciencia, Pensamiento y Cultura
CLXXXVI 745   s eptiembre-octubre (2010)   8 21-832   I SSN: 0210-1963
doi: 10.3989/arbor.2010.745n1233

CONFLICTOS ENTRE DERECHOS CONFLICTS AMONG


CONSTITUCIONALES Y MANERAS CONSTITUTIONAL RIGHTS
DE RESOLVERLOS AND WAYS TO SOLVE THEM

José Juan Moreso


(Universitat Pompeu Fabra, Barcelona)

ABSTRACT: This paper deals with the question of the conflict of RESUMEN: En este trabajo se analiza la cuestión de la colisión
constitutional rights which acknowledge basic rights. It is shown entre principios constitucionales que reconocen derechos básicos.
that there are two extreme accounts: the subsumptive approach Se muestra que hay dos posiciones extremas: el enfoque subsuntivo
and the particularist approach that embody two main conceptions y el enfoque particularista, que encarnan dos grandes concepciones
of practical rationality. Between both approaches there is room for de la racionalidad práctica. Entre ambos enfoques hay lugar para un
a range of options. Two of them are taken into account: the propor- amplio espectro de opciones. Se toman dos en cuenta: el enfoque
tionalist approach, which conserves the scope of rights restricting proporcionalista, que conserva el alcance de los derechos restrin-
their stringency, and it is argued in favour of the specificationist giendo su fuerza y el enfoque especificacionista, a favor del cual
approach, which preserve the stringency of rights restricting their se argumenta, que conserva la fuerza de los derechos restringiendo
scope. su alcance.

KEY WORDS: Constitutional theory; conflicts of rights; subsumptive PALABRAS CLAVE: Teoría constitucional; conflictos de derechos;
rationality; moral particularism. racionalidad subsuntiva; particularismo moral.

1. Introducción: Tres casos constitucionales zar dicha acción. Si hubiera dos principios en conflicto,
que requiriesen acciones incompatibles, este modo de
En este trabajo me propongo prestar especial atención concebir la racionalidad práctica sería insuficiente. Por
a los problemas de aplicación de las constituciones que esta razón, lo que podemos denominar el enfoque sub-
contienen declaraciones de derechos, dado que las pau- suntivo es atractivo, se trata de la razón que llevaba a
tas que establecen derechos pueden entrar en conflicto Kant (Kant, 1989, 31) a sostener que obligaciones non
entre sí. collinduntur.

En este sentido, se trata de un fenómeno muy conocido en La aplicación del derecho, tal y como usualmente la com-
la filosofía práctica en general y en la filosofía moral en prendemos, es una instancia de esta concepción general
particular. Supongamos que una persona sugiere que hay de la racionalidad práctica. Aplicar el derecho consiste en
un principio moral que establece “Se deben cumplir las determinar la norma individual que establece una cierta
promesas”, entonces otra persona trata de mostrarle que el consecuencia normativa para un caso individual deter-
hecho de haber prometido hacer A no siempre constituye minado. Para tal fin, se trata de mostrar que dicho caso
una razón para hacer A, puesto que puede chocar con individual es una instancia de un caso genérico al que
otro principio moral, así ¿es una razón para matar a una una norma aplicable correlaciona con esa consecuencia
persona el hecho de haberlo prometido a cambio de un normativa. Dicha operación se conoce con el nombre de
millón de euros?, parece que aquí vence el principio moral subsunción.
con arreglo al cual no se debe matar.
Ahora bien, hay supuestos en los que el enfoque subsuntivo
Sin embargo, un modo aparentemente muy plausible de parece plantear problemas. Traigo aquí a colación tres de
concebir la racionalidad práctica consiste en justificar los que aparecen en los diversos ensayos de la tercera
una determinada acción aludiendo a un principio sub- parte: el caso Titanic, el caso del cura de Hío y el caso del
yacente que, en aquellas circunstancias, requiere reali- niño Marcos.
El caso Titanic1 es un caso de colisión entre la libertad de tinentes y ante el alto riesgo de fuertes hemorragias, los
expresión y el derecho al honor en el cual la revista satírica médicos prescribieron una transfusión de sangre. En ese
Nº 745
Titanic había llamado “asesino nato” y, en otra edición pos- momento los padres del menor manifestaron educadamen-
terior, “tullido” a un oficial de la reserva que era parapléjico te que su religión, profesaban la religión de los testigos de
y que había logrado ser convocado de nuevo para llevar Jehová, les prohibía el uso de transfusiones y preguntaron
CONFLICTOS ENTRE DERECHOS CONSTITUCIONALES Y MANERAS DE RESOLVERLOS

a cabo unos ejercicios militares. El Tribunal Superior de si había algún tratamiento alternativo. Los médicos afir-
Dusseldorf condenó a la revista a pagar una indemnización maron que ellos no conocían ningún tratamiento alternati-
de 12.000 marcos alemanes al oficial. La revista interpuso vo, se negaron a conceder el alta voluntaria que los padres
un recurso de amparo y el Tribunal Constitucional alemán solicitaban, y se dirigieron al Juzgado de Guardia para que
consideró que mientras llamar al oficial “asesino nato” autorizara la transfusión. Los padres, a pesar de no com-
era una interferencia moderada o leve en su derecho al partir dicha decisión, la acataron y no hicieron nada para
honor, porque este tipo de apelativos eran usuales en su impedir la transfusión, que fue autorizada por el Juzgado.
estilo satírico, y en cambio la interferencia en la libertad de Marcos, que profesaba la misma religión que sus padres,
expresión se considera grave; llamar “tullido” a un paraplé- rechazó con auténtico terror la intervención, reaccionando
jico se considera una interferencia gravísima que derrota la con gran excitación y violencia que los médicos estimaron
interferencia grave en la libertad de expresión de la revista. muy peligrosas, pues podían precipitar una hemorragia
O sea que por este segundo apelativo únicamente estimó cerebral. Fracasados sus intentos de convencer al menor,
el Tribunal el recurso de amparo. el personal sanitario del Hospital solicitó a los padres
que trataran de convencer a Marcos, a lo que los padres
El caso del cura de Hío es, en cambio, un supuesto de la se negaron arguyendo que ellos, como Marcos, pensaban
jurisprudencia del Tribunal Constitucional español2, que en que Dios no autorizaba las transfusiones de sangre ni en
1992 resolvió un recurso de amparo en el cual el cura de los casos de peligro para la vida. Desestimado el uso de
Hío demandaba al diario El País que en 1984 había publi- algún procedimiento anestésico por razones médicas, al
cado una noticia según la cual el cura de dicha parroquia final el Hospital accedió a otorgar el alta voluntaria para
lideraba, garrote en mano, una manifestación de vecinos que los padres buscaran algún tratamiento alternativo en
contra algunos ciudadanos que practicaban el nudismo otro lugar. Regresaron a su domicilio y tres días después
en determinadas playas gallegas. Resultó que, a pesar de lo trasladaron a varios hospitales de Barcelona, donde les
la diligencia del redactor de la noticia al comprobar los reiteraron la necesidad de la transfusión, pero ninguno de
datos, el cura de Hío no estaba involucrado en dicha cir- dichos hospitales, dada la expresa oposición de Marcos y
cunstancia. ¿Cómo resolver el conflicto entre las razones sus padres, tomó la decisión de intentar llevarla a cabo.
constitucionales que favorecen la libertad de información Regresaron a su domicilio, en el cual el menor contó so-
y aquellas que favorecen el derecho al honor? El Tribunal lamente con la asistencia del médico titular de Ballobar,
Constitucional español se inclinó, esta vez, por la libertad hasta que el día 14 de septiembre el Juzgado de Instruc-
de información, con el argumento según el cual los infor- ción de Fraga (Huesca) dispuso mediante Auto, a la vista
madores están justificados no sólo con la verdad de las de que el menor empeoraba progresivamente por anemia
noticias que publican, sino también con su veracidad, para aguda posthemorrágica, la autorización para la entrada en
la que basta que la noticia, si falsa, esté diligentemente el domicilio del menor con el fin de que le fueran realiza-
contrastada. dos los tratamientos médicos que precisaba, es decir, una
transfusión de sangre. Marcos fue trasladado en ambulan-
Y, por último, el caso del niño Marcos: el día 3 de septiem- cia primero al Hospital de Barbastro y después al Hospital
bre de 1994, Marcos, A. V., un niño de 13 años que vivía Miguel Servet de Zaragoza al que llegó con signos clínicos
en Ballobar (Huesca) con sus padres, tuvo una caída con su de descerebración por hemorragia cerebral. Marcos falleció
bicicleta a la que no dieron mayor importancia. Pocos días el día 15 de septiembre en el Hospital Miguel Servet.
después, el 8 de septiembre, al haber sangrado por la nariz
en varias ocasiones, fue llevado por su madre al médico Los padres de Marcos fueron acusados del delito de ho-
que aconsejó su ingreso en el Hospital Arnau de Vilanova micidio, del que fueron absueltos por sentencia de la Au-
de Lleida. En dicho hospital, después de las pruebas per- diencia Provincial de Huesca de 20 de noviembre de 1996.

822 ARBOR  CLXXXVI  745  septiembre-octubre [2010]  821-832  ISSN: 0210-1963 doi: 10.3989/arbor.2010.745n1233
El Tribunal Supremo casó y anuló dicha sentencia y les para qué no incluya estos supuestos, por ejemplo, “se de-
condenó, por sentencia de 27 de junio de 1997, por el ben cumplir las promesas que no conllevan la realización
delito de homicidio con la circunstancia atenuante, muy de comportamientos inmorales”. Ahora el particularista o
cualificada, de obcecación o estado pasional, a la pena de defensor del enfoque subsuntivo vuelve a la carga: ¿qué
dos años y seis meses de prisión, y al pago de las costas ocurre si la promesa ha sido obtenida bajo la amenaza de

José Juan Moreso


correspondientes. Los condenados interpusieron recurso de secuestrar a la hija del que ha prometido? Y el universalista
amparo contra dicha sentencia condenatoria ante el Tribu- replica con una nueva formulación: “se deben cumplir las
nal Constitucional que, en STC 154/2002, de 18 de julio, promesas que no han sido otorgadas bajo amenaza ni con-
resolvió conceder el amparo, declarar vulnerado el derecho llevan la realización de actos inmorales”. El particularista
fundamental a la libertad religiosa y, en consecuencia, todavía puede desafiar diciendo que si arrancamos de un
anular la decisión del Tribunal Supremo. torturador, bajo amenaza, la promesa de dejar de torturar,
entonces debe dejar de torturar y el universalista puede
Aquí vemos, en cambio, una colisión entre dos derechos añadir el adjetivo “injustas” al sustantivo “amenazas”. Y así
diversos: el derecho a la vida y el derecho a la libertad sucesivamente. Los que cuestionan el enfoque subsuntivo
religiosa. de la moralidad sostienen que no hay modo plausible de
poner fin a este sucesivamente. Sostienen que los con-
textos se comportan de modos muy diversos y que, por lo
tanto, no hay manera de atrapar un principio moral válido
2. El desafío particularista al enfoque en todos los contextos de manera invariable.
subsuntivo
Esta conclusión produce cierto desazón cuando se traslada
La variedad de nuestro paisaje moral hace comprensible el a la teoría del derecho, puesto que conllevaría el corolario
desafío al enfoque subsuntivo por parte de las concepcio- de que en la aplicación de las pautas que establecen los
nes particularistas3. Según dichas concepciones, el hecho derechos constitucionales, la solución dependería siempre
de que una propiedad de una situación determinada sea del contexto, dejando a los destinatarios de las normas al
relevante en relación con su corrección, no presupone que arbitrio de los aplicadores5.
lo sea en cualquier otra circunstancia. Digámoslo con un
ejemplo de aroma kantiano, el hecho de que una persona
posea una información verdadera es una razón para afir-
marla cuando se la piden4. Ahora bien, si lo que solicitan 3. Interludio: un amplio espectro de opciones
a alguien que está en posesión de la información es el intermedias
escondite de una persona inocente a la que quieren ase-
sinar, entonces el hecho de poseer dicha información no En realidad, tanto el enfoque subsuntivo como el enfo-
debe conducir a relevarla, porque –por así decirlo– vence que particularista constituyen dos extremos, que iluminan
el principio que obliga a proteger a las personas inocen- algunos aspectos de la cuestión a costa de distorsionar
tes cuando está en nuestra mano. Pénsemos en un buen de manera excesiva otros aspectos. Entre estos extremos,
profesor universitario prusiano durante la segunda guerra cabe un espectro de posiciones intermedias6.
mundial que, sabedor porque él mismo se lo proporcionó,
del escondite de una mujer de origen judío, que había sido Dicho en palabras de Russ Shafer-Landau7:
una de sus alumnas, revela el lugar como respuesta a una
pregunta de un oficial de las SS. Tal vez por esta razón Cuando nuestros derechos parecen entrar en conflicto con
David Ross (Ross, 1930) sostuvo que los deberes morales no otras consideraciones morales, incluyendo otros derechos
son absolutos, sino prima facie. Ahora bien, una vez hace- morales, podemos resolver la tensión reduciendo el alcance
mos esta concesión, ¿dónde podemos detenernos? Regre- del derecho o bien reduciendo su fuerza.
semos a nuestro ejemplo referido al cumplimiento de las
promesas. Supóngamos que, ante la promesa de asesinar a Reducir la fuerza de los principios comporta considerar
alguien, se replica reformulando la norma moral propuesta que los principios propiamente cubren los casos en los

doi: 10.3989/arbor.2010.745n1233 ARBOR  CLXXXVI  745  septiembre-octubre [2010]  821-832  ISSN: 0210-1963
823
cuales varios de ellos pueden entrar en conflicto, pero en adecuado para preservar otro derecho o un bien cons-
dichos supuestos uno de los principios resulta vencedor, titucionalmente protegido, b) el principio de necesidad,
Nº 745
conserva su fuerza, mientras el otro –el vencido– pierde esto es, que el sacrificio impuesto sea necesario –que no
su aplicabilidad al caso concreto. Me referiré aquí a uno exista otro menos lesivo– para preservar otro derecho o
de los enfoques posibles de esta concepción, el enfoque un bien constitucionalmente protegido11, y c) el princi-
CONFLICTOS ENTRE DERECHOS CONSTITUCIONALES Y MANERAS DE RESOLVERLOS

proporcionalista, tal como ha sido presentado por Robert pio de proporcionalidad en sentido estricto, en donde la
Alexy, cuyas ideas acerca de la denominada ponderación ponderación propiamente dicha ocupa su lugar, que se
han sido y son la posición más articulada y sugerente de afecte al ejercicio del derecho en el menor grado posible
la que disponemos en la teoría constitucional del pre- compatible con la mayor satisfacción en el ejercicio del
sente8. otro derecho.

Reducir el alcance de los principios conservando su fuer- Según Alexy, la ponderación puede ser dividida en tres
za representa considerar la formulación de los principios etapas. La primera etapa comporta el establecimiento del
como incompleta y expandirla9 de modo que, conservando grado de no satisfacción del primer principio. La segunda
el alcance, los principios debidamente reformulados no en- etapa consiste en establecer el grado de satisfacción del
tren ya en conflicto. Se trata de la posición que subyace a principio en pugna con el primero. Finalmente, la tercera
todos los ensayos de esta parte y que podemos denominar etapa evalúa si la importancia de la satisfacción del segun-
enfoque especificacionista10. do principio justifica la falta de satisfacción del primero.
Podemos dividir el grado de afección a un derecho deter-
minado en tres rangos: leve, medio y grave. Como es obvio,
estos grados de afección son relativos al contexto esta-
4. El enfoque proporcionalista blecido por el caso concreto. De ello resulta lo siguiente:
las vulneraciones leves de un derecho fundamental ceden
Veamos, en primer lugar, la concepción proporcionalista ante la protección media y la grave de otro derecho fun-
tal como la concibe Robert Alexy. Para dicha concepción, damental, y las medias ceden ante las graves. Quedan tres
mientras las colisiones entre normas son un síntoma de casos de empate, en donde –si entiendo bien la propuesta
que, o bien alguna de las normas es inválida, o bien una de Alexy– el legislador goza de discreción para afectar uno
puede considerarse una excepción de la otra; las colisiones u otro derecho, lo que equivale a decir que, en los casos
entre principios han de ser resueltas de modo distinto. de empate, las restricciones legislativas al ejercicio de un
Cuando dos principios entran en colisión (por ejemplo, derecho fundamental están justificadas.
porque el primero establece que una conducta determi-
nada está prohibida y el segundo que está permitida) uno Entonces, en la fórmula más simple, el peso concreto de
de los dos ha de ceder frente al otro. Pero esto no significa un principio Pi que colisiona con otro principio Pj, es decir,
que uno de los dos principios sea inválido, ni que en el Wi, j, es el cociente que resulta de dividir la intensidad de
principio desplazado haya que introducir alguna excepción. la interferencia en el primer principio (Ii) por la intensidad
Lo que sucede es que, en determinadas circunstancias, un de la hipotética interferencia sobre el segundo principio
principio precede al otro. Es por esta razón que se afirma bajo el supuesto de que se omitiera la interferencia con el
que, en los casos concretos, los principios tienen diferente primero (Ij). Esta es, entonces, la fórmula:
peso y el conflicto ha de resolverse según la dimensión de
peso y no según la dimensión de validez. Wi, j = Ii / Ij

La dimensión de peso, entonces, configura el núcleo de En los casos en los que el valor es mayor que 1 el principio
la ponderación. Dicha operación forma parte de lo que Pi precede a Pj, en los casos en que es menor que 1, Pj
es requerido por un principio más comprensivo: el prin- precede a Pi. El empate se da cuando la división es igual
cipio de proporcionalidad. Este principio comprende tres a 1. Los valores numéricos pueden asignarse suponiendo
subprincipios: a) el principio de adecuación, es decir, que que las interferencias leves equivalen a 20, las moderadas
el sacrificio impuesto en el ejercicio de un derecho sea a 21 y las graves a 22.

824 ARBOR  CLXXXVI  745  septiembre-octubre [2010]  821-832  ISSN: 0210-1963 doi: 10.3989/arbor.2010.745n1233
Insisto en que se trata del peso concreto, es decir, de semejante al test del rayado, ni siquiera somos capaces de
la precedencia de un principio sobre otro para un caso delimitar con claridad de qué propiedad estamos hablando
individual. Como Alexy afirma (Alexy, 2003b, 440): “Las frente a la cual la interferencia pueda ser calificada de
interferencias son siempre interferencias concretas”. Es leve, moderada o grave. Y como hay varias propiedades que
posible añadir en la fórmula lo que Alexy denomina el son candidatas plausibles a representar ese papel, podemos

José Juan Moreso


peso abstracto de los principios, es decir la importancia generar varias escalas, distintas entre sí.
en abstracto de un principio sobre otro, al margen de las
circunstancias del caso concreto. Esto da como resultado La tercera y última duda que quiero plantear está relacio-
la fórmula siguiente12: nada con la insistencia de Alexy en que la operación de
ponderación se refiere siempre a un caso individual. Lo que
Wi, j = Ii. Wi / Ij. Wj conlleva una concepción que ha sido denominada una con-
cepción ad hoc de la ponderación. Mientras la ponderación
Según Alexy, esta reconstrucción de la ponderación hace en abstracto es una ponderación definicional –es decir, una
de ella una actividad controlable racionalmente. Mientras asignación de peso independiente de las circunstancias–,
la subsunción es un esquema que trabaja con arreglo a las pero inconcluyente, puesto que de la fórmula de Alexy
reglas de la lógica, la ponderación trabaja de acuerdo a las se deriva que un principio con mayor peso en abstracto
reglas de la aritmética. puede ser derrotado en concreto por otro con menor peso
en abstracto; la ponderación en concreto es siempre ad
Veamos, a continuación, tres posibles objeciones a este hoc y ello conlleva que “una sola característica peculiar
enfoque alexiano. La primera duda que se me plantea es la puede justificar una solución diversa de aquella que se
siguiente: dado que los pesos abstractos de los principios ha atribuido a un caso anterior”13. De este modo, se veda
son independientes de cualquier circunstancia concreta, uno de los modos de control racional de las decisiones
deberíamos tener a nuestra disposición una asignación judiciales: aquel basado en la articulación de dicho tipo
de peso abstracto para cada principio que establece un de decisiones. Es más, se sustituye un modelo generalista
derecho constitucional, deberíamos tener una escala de de toma de decisiones, por otro de carácter particularista,
ordenación abstracta de los derechos. No conozco ninguna una sola propiedad diferente (y, como ha de resultar obvio,
escala de este tipo que pueda ser aceptada razonablemen- si dos casos individuales son diferentes entonces tienen
te. En mi opinión no es claro qué es lo que debería medir al menos una propiedad diferente) puede comportar una
la supuesta escala abstracta de ordenación de los derechos solución diversa para el caso.
constitucionales: ¿debería medir estados de cosas en los
que un derecho es máximamente optimizado mientras el En mi opinión, estos tres problemas socavan gravemente
otro no lo es en absoluto? el modelo proporcionalista de Alexy, puesto que no per-
miten ni prever lo que los órganos de aplicación del de-
El segundo punto al que quiero referirme guarda relación recho decidirán en los casos de conflictos entre principios
con la distinción de Alexy entre interferencias leves, mo- constitucionales que establecen derechos fundamentales,
deradas y graves en los principios constitucionales. Aquí ni menos aún someter a crítica racional y articulada sus
estamos frente a una escala, pero ¿de qué depende la asig- decisiones.
nación de estos tres conceptos en un caso concreto? Sólo
podemos construir escalas ordinales o cardinales cuando A menudo se añade a esta crítica la idea según la cual el
estamos en presencia de una propiedad claramente defi- juicio que subyace a la asignación de peso a los diversos
nida, como sucede con el test del rayado para la escala de principios es un juicio totalmente subjetivo y, como tal, no
los minerales: un mineral es más duro que otro si y sólo si controlable racionalmente14. Baste decir aquí, aunque no
el primero puede rayar al segundo y el segundo no puede puedo argumentarlo, que no considero que la evaluación
rayar al primero. La dureza de los minerales nos permite de de la aplicabilidad de los principios sea una tarea plena-
este modo construir una escala ordinal. No veo cómo po- mente subjetiva y, en este sentido, comparto el propósito
demos hacer lo mismo con la interferencia en los derechos de Alexy de buscar un mecanismo de control racional de
constitucionales, dado que no estamos en posesión de nada esta tarea.

doi: 10.3989/arbor.2010.745n1233 ARBOR  CLXXXVI  745  septiembre-octubre [2010]  821-832  ISSN: 0210-1963
825
5. El enfoque especificacionista la subsunción, en el ámbito de un problema normativo
determinado. Intentaré mostrar cuáles son las etapas de
Nº 745
Merece la pena, entonces, explorar el enfoque especifica- una operación de este tipo, tomando como supuesto el
cionista, consistente en reducir el alcance de los principios caso del Titanic.
pero conservar su fuerza. Uno de los puntos centrales en
CONFLICTOS ENTRE DERECHOS CONSTITUCIONALES Y MANERAS DE RESOLVERLOS

los que insisten los particularistas es que las razones (mo- La primera etapa consiste en la delimitación del problema
rales, jurídicas) no se comportan de modo atomista, sino normativo, de lo que Alchourrón y Bulygin han llamado el
holista; pues bien como ha sido sugerido (Väyrynen, 2006, universo del discurso17. Delimitar claramente el ámbito del
710) la estrategia que voy a presentar trata de secuestrar problema normativo que nos ocupa permite, principalmen-
al holismo y acomodarlo en un marco generalista15. te, hacer el problema manejable: ya no nos referimos al
conjunto de todas las acciones humanas posibles, sino sólo
Lo haré valiéndome del caso Titanic propuesto tantas ve- a un conjunto mucho más reducido de acciones humanas.
ces por Alexy. En este caso se ponen de manifiesto los En el caso del Titanic, el universo del discurso podría ser
problemas que he señalado, creo, en la concepción de el siguiente: acciones de información en los medios de
Alexy. En primer lugar, ¿cuál de los principios, la libertad comunicación sobre asuntos que afectan a las personas.
de expresión y el derecho al honor, tiene mayor peso en
abstracto? Nada nos dice al respecto Alexy y, de ello, tal La segunda etapa consiste en la identificación de las pau-
vez haya que concluir que Alexy considera que tienen igual tas prima facie aplicables a este ámbito de acciones. Aquí
peso. Sin embargo, sólo una teoría plenamente articulada obviamente son aplicables el principio que establece la
de los derechos nos permitiría alcanzar dicha conclusión libertad de expresión e información y el principio que
y una teoría así está aún por construir. En segundo lugar, protege el derecho al honor de las personas.
¿por qué denominar al oficial “asesino nato” es una in-
terferencia moderada o leve (y, es más, debería decirse si La tercera etapa consiste en la consideración de deter-
es moderada o es leve), mientras tildarle de “tullido” es minados casos paradigmáticos, reales o hipotéticos, del
gravísima (que, por cierto, no es una categoría presente en ámbito normativo previamente seleccionado en la primera
la clasificación de Alexy)? Y ¿por qué imponer una indem- etapa. Los casos paradigmáticos tienen la función de cons-
nización no muy alta, como en este caso, a los editores de treñir el ámbito de reconstrucciones admisibles: sólo son
la revista constituye una interferencia grave en la libertad admisibles aquellas reconstrucciones que reconstruyen los
de expresión? Alguien podría argüir, con perfecto sentido, casos paradigmáticos adecuadamente18. Los casos paradig-
que este tipo de expresiones puede ser evitado sin merma máticos constituyen el trasfondo, a menudo inarticulado,
significativa de la libertad de expresión ni de la libertad de en el cual el razonamiento práctico tiene lugar. En el pro-
información. En tercer lugar, esta sentencia también mues- blema normativo delimitado en la primera etapa a modo
tra claramente la dificultad de establecer criterios gene- de ejemplo, podríamos considerar como paradigmáticos
rales con este método: en el caso de la expresión “asesino casos como los siguientes: a) publicar la noticia falsa, sin
nato” la libertad de expresión precede al derecho al honor, comprobación alguna de su veracidad, de que el arzobispo
en el caso de “tullido” ocurre lo contrario. ¿Qué sucederá, de Barcelona está implicado en una trama de prostitución
entonces, en otro supuesto de expresión denigratoria en el infantil, es un ejemplo de un supuesto en el cual la libertad
futuro? ¿Puede alguien decirlo con seguridad? de información cede ante el derecho al honor y, b) publi-
car la noticia verdadera de que, por ejemplo, un ministro
Por estas razones, creo que es mejor pensar en un modo de del gobierno ha cobrado diez millones de euros de cierta
configurar la ponderación que la considera un paso previo empresa a cambio de la concesión para construir una au-
a la subsunción. Una vía según la cual la ponderación es topista es un supuesto en el que la libertad de información
la operación que permite pasar de las normas que esta- desplaza al derecho al honor.
blecen derechos fundamentales, que tienen la estructura
de principios –pautas con las condiciones de aplicación En la cuarta etapa se establecen las propiedades relevantes
abiertas–16, a reglas –pautas con las condiciones de apli- de ese universo del discurso. El establecimiento de las pro-
cación clausuradas–, con las cuales es posible llevar a cabo piedades relevantes ha de hacer posible la determinación

826 ARBOR  CLXXXVI  745  septiembre-octubre [2010]  821-832  ISSN: 0210-1963 doi: 10.3989/arbor.2010.745n1233
de las soluciones normativas. En nuestro supuesto son Es obvio que en muchos casos individuales la solución
claramente relevantes las siguientes propiedades: la rele- ofrecida por una y otra reconstrucción será la misma.
vancia pública de la noticia, que la noticia sea veraz (tal No obstante, insistiré en aquello que las distingue. La
como ello es entendido por muchos altos Tribunales, que concepción aquí defendida requiere que seamos capaces
sea verdadera o, si falsa, diligentemente contrastada)19, y de establecer cuáles son las propiedades seleccionadas

José Juan Moreso


que la noticia no sea injuriosa. relevantes. Guiar el comportamiento es, en realidad, se-
leccionar propiedades a cuya presencia o ausencia se co-
La quinta y última etapa consiste en la formulación de las rrelacionan diversas calificaciones deónticas. Obviamente
reglas que resuelven de modo unívoco todos los casos del que esta selección de propiedades no está en disposición
universo del discurso. Una regla, me parece que indiscuti- de resolver todos los casos individuales, principalmente
da, sería la siguiente: porque los conceptos que describen las propiedades son
vagos y siempre quedarán casos de duda: ¿Es determina-
R1: Las informaciones de relevancia pública, veraces y no da expresión un insulto o no, como por ejemplo llamar al
injuriosas están permitidas. oficial de la reserva “asesino nato” en el contexto satírico
de la revista Titanic? Esta podría ser una forma de recons-
También indiscutida, creo, tendríamos una segunda regla: truir la decisión del Tribunal Constitucional alemán con el
esquema presentado. La ventaja, en mi opinión, es que el
R2: Las informaciones que no son de relevancia pública Tribunal en el futuro sólo ha de plantearse si determinada
o carecen de veracidad o son injuriosas están prohibidas expresión es o no injuriosa y no el grado de interferencia
y, en el caso que se produzcan, generan un derecho a ser de dicha expresión en la libertad de información.
indemnizado.
Por otra parte, en la concepción especificacionista no hace
Obviamente que las tres últimas etapas están íntimamente falta la ponderación en abstracto, si por ella entendemos
relacionadas entre sí. El establecimiento de las reglas debe el peso de cada principio al margen de cualquier circuns-
ser controlado de acuerdo con su capacidad de reconstruir tancia. El derecho a la vida que parece un candidato a
los casos paradigmáticos. La selección de las propiedades tener mayor peso, también está sujeto a especificación: en
relevantes debe refinarse en la medida en que este objetivo primer lugar, porque algunas veces el derecho a la vida de
no sea alcanzado y, a partir de una nueva selección, debe una persona entra en conflicto con el derecho a la vida de
procederse a la formulación del conjunto de reglas que otra persona, pero también porque aunque el derecho a la
disciplinan dicho problema normativo. vida genera un deber especial de, por ejemplo, los médicos
para asistir a aquellos cuya vida corre peligro, dicho deber
Estas cinco etapas constituyen un modo de concebir la es exceptuado cuando la persona cuya vida corre peligro
ponderación que lo hace compatible con la subsunción y ha expresado seria y libremente su rechazo a determinado
con una limitada generalidad. Las reglas surgidas de una tratamiento (así las transfusiones de sangre) en virtud de
reconstrucción como la propuesta se aplican de modo sus convicciones religiosas.
subsuntivo y permiten articular y otorgar coherencia a la
función judicial, aunque reducen el alcance de los prin- Por último y más importante: el modelo de Alexy está
cipios. Cuando un órgano jurisdiccional resuelve un caso abocado al particularismo, en el sentido de que una pro-
individual de dicho ámbito normativo, resuelve con su piedad diferente puede hacer que un nuevo caso tenga
reconstrucción todos los casos individuales de dicho ám- una solución distinta. El modelo especificacionista no está
bito. Y lo hace de manera compatible con que en otro caso necesariamente vinculado con el particularismo, en un
individual perteneciente a ese ámbito, o bien debe seguir ámbito determinado y con determinados principios en liza,
la reconstrucción llevada a cabo o debe mostrar una pro- el modelo presentado es generalista y con él se resuelven
piedad relevante, no apreciada previamente, que le permita todos los casos previamente delimitados.
resolver algunos casos individuales de un modo diverso20.
Creo que de este modo es posible el control racional de la Considero la estrategia especificacionista razonable. Sin
función aplicadora del derecho. embargo, para que dicha estrategia sea convincente debe

doi: 10.3989/arbor.2010.745n1233 ARBOR  CLXXXVI  745  septiembre-octubre [2010]  821-832  ISSN: 0210-1963
827
ser sometida a dos limitaciones importantes21. Por una tigado, en principio, por el código penal; pero dicha acción
parte, dicha especificación contendrá siempre, entre las no es punible si ha sido realizada en legítima defensa o en
Nº 745
circunstancias que permiten revocar la obligación moral estado de necesidad o cualquiera del resto de causas de
a la que en principio llevan determinadas propiedades justificación, o ha sido realizada en presencia de alguna de
–llamémosle defeaters–22, conceptos morales. Suponga- las causas de exculpación (trastorno mental, por ejemplo).
CONFLICTOS ENTRE DERECHOS CONSTITUCIONALES Y MANERAS DE RESOLVERLOS

mos que yo he prometido ir esta noche a cenar a casa de Las causas de justificación y las excusas son los defeaters
un amigo. Si mi amigo me llama para decirme que si no en la codificación de las normas penales. Dichos defeaters
termino el artículo que estoy escribiendo puedo no ir a están de hecho formulados de tal modo que incorporan
su casa, este hecho socava la razón para cumplir la pro- conceptos evaluativos (en el caso de la legítima defensa,
mesa, se trata de un underminer, una causa de supensión. que la agresión a la que se responde sea ilegítima, que
Ahora bien, si mi amigo me llama para decirme que está no mediare provocación suficiente, etc.) Si alguien realiza
muy cansado y que no se siente con fuerzas para cocinar un contrato de compraventa, obligándose a la entrega de
esta noche, entonces este hecho cancela mi obligación un coche a cambio de un precio, tal contrato es válido
de un modo diverso al anterior: no sólo la socava sino siempre que no medien, por ejemplo, algunos vicios del
que me ofrece una razón para no ir a su casa, se trata de consentimiento. Es decir las obligaciones en el derecho
un reverser, una causa de inversión. Y si yo sufro mareos privado están también sujetas a defaters: los contratos son
y no me encuentro bien para ir a su casa, entonces mi nulos si son contrarios a las leyes, a la moral o al orden
responsabilidad es anulada mediante una excusa. Por otra público, son también nulos si se otorgan con algún vi-
parte, si es mi mujer la que me llama desde el hospital en cio del consentimiento (error, coacción, etc.). Como puede
el que ha sido ingresada, entonces el deber de acudir al apreciarse, también en este caso los defeaters contienen
hospital revoca el deber de cenar con mi amigo, se trata conceptos evaluativos.
de un overrider, una causa de anulación. Como dice Walter
Sinnot-Armstrong, al que se deben estas ideas y esta ter- No veo razones para rechazar este punto de vista para
minología: “Overriders, underminers, reversers, and excuses la moral, ni desde el punto de vista conceptual, ni desde
are, then, all reason defaters” (anulaciones, suspensiones, el punto de vista justificativo. Ni siquiera Jonathan Dan-
inversiones y excusas son, entonces, todas ellas causas de cy rechaza esta posibilidad26. Ahora bien, Dancy replica
revocación de las razones)23. diciendo que de este modo no conseguimos todavía dar
forma (shape) a las propiedades naturales sobre las que las
Entonces, los principios morales deben ser formulados propiedades morales supervienen, puesto que ahora queda
incluyendo sus defaters. Ahora bien, los defeaters siem- por averiguar sobre qué propiedades naturales supervienen
pre contendrán conceptos morales. Sólo en este sentido los defaters. Es decir, esta forma de universalismo no es
es posible codificar los principios morales. Es claro que, más que un particularismo disfrazado de universalismo, un
al menos, por razones epistémicas no podemos codificar lobo con piel de cordero. Es un argumento similar al de
todos los defeaters acudiendo únicamente a propiedades Bruno Celano27. Celano sostiene que no hay ningún modo
naturales. Además, dadas las infinitas descripciones posi- estable de llevar a cabo la revisión desde nuestros condi-
bles de las acciones, es plausible pensar que también dicha cionales derrotables hasta los condicionales inderrotables
tarea es irrealizable por razones conceptuales24. Este es el y no lo hay porque la noción de una tesis de relevancia
sentido en el cual el universalismo puede ser adecuado: última está mal formada. La tesis de relevancia última es
hay principios morales, ahora bien –y esta es la concesión aquella que establece cuáles son las propiedades naturales
al particularismo– que contienen necesariamente concep- relevantes de un modo exhaustivo28 y, según Celano, no
tos evaluativos25. Podemos de este modo revisar los ante- hay esperanzas de alcanzar tal objetivo. Según Celano, el
cedentes de nuestros principios prima facie haciéndolos conjunto de las combinaciones de las propiedades poten-
más específicos, pero con el coste de incorporar en ellos, cialmente relevantes, que toman en cuenta la historia del
como defeaters, conceptos evaluativos. caso, no es agotable29.

Así es como, por otra parte, están codificadas las normas Estas consideraciones de Celano (y de Dancy) nos llevan
jurídicas. Si alguien mata a otro, comete un homicidio cas- a otra limitación del universalismo. Concedo, sin más ar-

828 ARBOR  CLXXXVI  745  septiembre-octubre [2010]  821-832  ISSN: 0210-1963 doi: 10.3989/arbor.2010.745n1233
gumentos, que en la codificación de las normas morales sistemas así concebidos la operación de revisión estable es
(y jurídicas) no puede irse más allá de principios o normas alcanzable y permite pasar, en este contexto delimitado,
con defaters evaluativos. Sin embargo, el discurso de apli- de la formulación de defeaters evaluativos a defeaters
cación de dichas codificaciones puede ir más allá. Sería descritos mediante propiedades naturales. Obviamente que
irrazonable suponer que, enfrentados a un problema mo- se trata, por definición, de reconstrucciones incompletas.

José Juan Moreso


ral, debamos reconstruir todos nuestros principios morales También es verdad, como quieren los particularistas, que
válidos para todas las acciones humanas posibles. Es más dichas reconstrucciones dependen del contexto, pero no
sensato considerar que podemos adoptar un determinado dependen –como arguyen los particularistas– del contexto
universo del discurso, es decir, un conjunto determinado de establecido por las circunstancias del caso concreto, sino
acciones humanas. Por ejemplo, el conjunto de las accio- que dependen del contexto del problema normativo selec-
nes de devolver los libros que tenemos en préstamo30. Tal cionado. Es decir, en un contexto normativo así delimita-
vez en este caso, podamos alcanzar algo como una tesis do, hay propiedades naturales uniforme e invariablemente
de relevancia. Son relevantes, con seguridad, el plazo de relevantes.
tiempo por el que tenemos el préstamo, el hecho de que
el que nos lo prestó sea su verdadero propietario (no es un
libro sustraído de la biblioteca universitaria, por ejemplo),
debemos incluir también las justificaciones (tuvimos que 6. Frónesis aristotélica para concluir
ausentarnos de la ciudad por un tiempo) y las excusas (lo
dejamos olvidado en un cambio de domicilio). Podemos Alguien puede considerar, con razón, que esta defensa
también delimitar previamente el conjunto de normas mo- del generalismo concede demasiado al particularismo,
rales a tomar en cuenta, para ignorar casos recalcitrantes dado que sólo tenemos principios morales si los conce-
siempre imaginables. Podemos atender únicamente a la bimos como incorporando conceptos evaluativos y en el
norma moral que obliga a devolver lo prestado bajo ciertas razonamiento práctico el juego de estos principios está
condiciones e ignorar, ahora, la norma moral que prohíbe delimitado por el contexto del universo del discurso se-
inducir a otro al suicidio, y así ignorar el supuesto de que la leccionado y de los principios morales que consideramos
lectura del libro, dado el estado anímico de su propietario, aplicables. Sin embargo, aún con estos límites, la imagen
puede llevarlo a suicidarse. Obviamente que si apreciamos del razonamiento práctico que obtenemos permite la sub-
que este es el caso, debemos introducir esta norma en el sunción de casos individuales en pautas generales y deja
sistema. Pero limitada a un universo de acciones y a un un razonable espacio para la articulación y la consisten-
subconjunto de las normas morales, creo que es razonable cia de las decisiones. Esta concepción del razonamiento
aspirar a obtener una tesis de relevancia manejable. Por práctico vale, en mi opinión, tanto para la moral cuanto
otro lado, así operan los juristas cuando se enfrentan a un para el derecho.
caso real o hipotético: toman en cuenta un universo del
discurso delimitado y un subconjunto de normas jurídicas Quiero decir, para terminar, que me complacería que las
(no todas las normas jurídicas, ya que no es sensato, aun- ideas presentadas aquí sean sólo un desarrollo de la con-
que son un conjunto finito –si nos limitamos a las normas cepción aristotélica de la deliberación práctica, de la fró-
formuladas y no tomamos en cuenta las derivadas–, pensar nesis aristotélica31. El cómo y el porqué deberán, me temo,
en una reconstrucción global de todas ellas). Para micro- aguardar a otra ocasión.

Recibido:  21 de enero de 2009


Aceptado:  23 de febrero de 2009

doi: 10.3989/arbor.2010.745n1233 ARBOR  CLXXXVI  745  septiembre-octubre [2010]  821-832  ISSN: 0210-1963
829
NOTAS Amstrong, 1999; Holton, 2002, y quedado desprotegidos. Más extraña,
Väyrynen, 2006. me parece, la posición de Hernán-
1 BverfGE vol. 86, 1. 11 Estas dos operaciones deben ser com- dez Marín, a la que tampoco pue-
Nº 745
2 Vid. STC 20/1992, de 14 de febrero. prendidas como distintas en un sen- do prestar la atención que merece,
3 En la filosofía contemporánea re- tido sólo heurístico y no conceptual. puesto que sostiene que –sea o no
CONFLICTOS ENTRE DERECHOS CONSTITUCIONALES Y MANERAS DE RESOLVERLOS

presentadas por, por ejemplo, Dancy, Quiero decir que, conceptualmente, subjetivo el juicio de ponderación– el
1993, 2004; McNaughton, 1988, y si un medio A es necesario para ob- juez cumple con el derecho aplicando
McDowell, 1998. tener B también es adecuado. Ahora cualquiera de los dos principios en
4 En virtud, digamos, del principio bien, según creo, hay que interpretar pugna. Tal vez habría que revisar la
moral que nos obliga a no mentir. a Alexy en el sentido siguiente: en noción de aplicación y de cumpli-
Kant pensaba que tal principio es un primer lugar, averiguamos si A se en- miento del derecho que permite esta
principio categórico, absoluto y por cuentra entre los medios adecuados extraña conclusión, a saber, que el
ello escribió (1989, 292): “La mentira para obtener B y, en segundo lugar, juez del caso Titanic, por ejemplo,
(en el sentido ético de la palabra), vemos si no existen medios menos cumple juzgando que se debe recha-
como falsedad deliberada, no precisa lesivos que A de alcanzar B. zar la demanda del militar porque las
perjudicar a otros para que se la con- 12 En realidad, Alexy añade otra compli- publicaciones están protegidas por la
sidere reprobable... Su causa pude ser cación en la fórmula, de la que aquí disposición constitucional que prote-
la ligereza o la bondad, incluso puede prescindiré, relativa a la confiabilidad ge la libertad de expresión y cumple
perseguirse con ella un fin realmente de las asunciones empíricas concer- también admitiendo la demanda por-
bueno, pero el modo de perseguir- nientes a las diversas interferencias que se ha vulnerado la disposición
lo es, por la mera forma, un delito en juego. Alexy, 2003b, 446-448. constitucional que protege el dere-
del hombre contra su propia persona 13 Bernal Pulido, 2003, 188-189, en la cho al honor.
y una bajeza que tiene que hacerle mejor presentación y desarrollo de 15 La he desarrollado previamente en
despreciable a sus propios ojos”. las ideas de Alexy en castellano. Para Moreso, 2002, 2004.
5 Éstas parecen ser las razones que con- la distinción entre ponderación defi- 16 Para esta noción de principios vid.
ducen a algunos autores a rechazar la nicional y ponderación ad hoc vid. Ateinza-Ruiz Manero, 1996, cap. 1.
tesis de los conflictos entre los dere- por ejemplo Aleinikoff, 1987, 948. 17 Alchourrón-Bulygin, 1971, cap. I.
chos y a defender alguna versión del 14 Por ejemplo, Guastini, 1999, 169- 18 Vid., para esta función de los casos
enfoque universalista y subsuntivo. 171; también, 1998a, 1998b, 302- paradigmáticos en el ámbito del ra-
Vid., por ejemplo, Habermas, 1992, 304; Hernández Marín, 2005, 322- zonamiento jurídico, Dworkin, 1986,
cap. 6 y Ferrajoli, 2001, 353-354. 327, y García Amado, 2007a, 2007b. 255-257; Hurley, 1989, 212, y Endi-
6 En Dancy, 2004, 3-12, se halla la mis- Lo que parece incomodar a García cott, 1998.
ma idea como “rango de opciones” y Amado, especialmente, es el aumen- 19 Se trata de la doctrina del Tribunal
la presentación de algunas de ellas. to del poder de control jurisdiccional Supremo de los Estados Unidos en
7 Shafer-Landau, 1995a, 225. de los tribunales constitucionales New York Times v. Sullivan 376 US
8 Alexy, 1986, 2002, 2003a, 2003b. frente a los tribunales ordinarios, 254 (1964), en donde se aplica el
9 Dancy (Dancy, 2004, 11) llama a esta a su –como dice– “tácito cometido denominado “malice test”: “knowled-
posición “expansionismo”. como superapelación”. Aunque este ge of falsehood or reckless disregard
10 Y que se inspira en algunas ideas punto exigiría un tratamiento más for falsity”, recogida por muchos Tri-
de Hare, 1952, 48-55, 60-65; Hur- detallado, baste decir aquí que si los bunales Constitucionales europeos.
ley, 1989, 1990; Richardson, 1990; derechos constitucionales han de ser Para España [que, por otra parte, en
Bayón, 2001; Atienza-Ruiz Manero, efectivos, alguien deberá velar por el propio texto constitucional, en el
2002, y Scanlon, 2000. Con atención ellos y, me temo, que durante estos art. 20.1.d), reconoce explicítamente
especial al particularismo y al modo treinta años de democracia consti- el derecho a “comunicar o recibir li-
en que una posición como esta puede tucional en España, si no lo hubiese bremente información veraz...”] pue-
preservar un espacio conceptual para hecho el Tribunal Constitucional, al- den verse, por ejemplo, las siguientes
los principios generales, vid. Sinnot- gunos de nuestros derechos habrían sentencias del Tribunal Constitu-

830 ARBOR  CLXXXVI  745  septiembre-octubre [2010]  821-832  ISSN: 0210-1963 doi: 10.3989/arbor.2010.745n1233
cional: STC 6/1988, de 21 de ene- chos fundamentales, Madrid, Centro Ferrajoli, Luigi (2001): Los fundamentos de
ro, STC 40/1992, de 30 de marzo y de Estudios Constitucionales, 1993. los derechos fundamentales, Madrid,
STC 240/1992, de 21 de diciembre. – (2002): “Epílogo a la Teoría de los Trotta.
20 Como es obvio esta operación re- derechos fundamentales”, trad. de García Amado, Juan Antonio (2007a): “El
presenta el cambio de las reglas del ­C. Bernal, Revista española de Dere- juicio de ponderación y sus partes.

José Juan Moreso


sistema normativo y, por lo tanto, cho Constitucional, 22, n.º 66, 13-64. Una crítica,” en R. Alexy, Derechos
del sistema normativo aplicable. Sin – (2003a): “Constitutional Rights, Ba- sociales y ponderación, ed. a cargo
embargo, este cambio es compatible lancing, and Rationality”, Ratio Iuris, de Ricardo García Manrique, Madrid,
con el hecho de que la solución de 16, 131-140. Fundación Coloquio Jurídico Europeo,
los casos realmente ocurridos en el – (2003b): “On Balancing and Sub- 2007, cap. VII.
pasado sea la misma para los dos sis- sumption. A Structural Comparison”, – (2007b): “¿Ponderación o simples
temas normativos. Vid. el desarrollo Ratio Iuris, 16, 433-449. subsunciones? Comentario a la Sen-
de esta idea en Moreso, 2002. Atienza, Manuel y Ruiz Manero, Juan tencia del Tribunal Constitucional
21 Expongo algunas ideas ya presentes (1996): Las piezas del Derecho, Bar- Español 72/2007, de 25 de abril de
en Moreso, 2005b. celona, Ariel. 2007”, Palestra del Tribunal Constitu-
22 Vid. Moreso, 2005a. – (2000): “Rules and Principles Revisi- cional, Revista mensual de jurispru-
23 Sinnot-Amstrong, 1999, 5. ted”, Associations, 4, 147-156. dencia, Lima, 2, n.º 8, 619-627.
24 Dicha sugerencia en Celano, 2007 y Bayón, Juan Carlos (2001): “¿Por qué es Guastini, Riccardo (1998a): “Principi di
Moreso, 2004a (ahora cap. 18). derrotable el razonamiento jurídico”, Diritto e discrezionalità giudiziale”,
25 Esta es la posición de McNaughton- Doxa, 24, 35-62. Diritto Publico, 651-659.
Rawling, 2000. Bernal Pulido, Carlos (2003): El principio de – (1998b): Teoria e dogmatica delle fon-
26 Dancy, 1999, 2001. proporcionalidad y los derechos fun- ti, Milano, Giuffrè.
27 Celano, 2007. damentales, Madrid, Centro de Estu- – (1999): Distinguiendo, Estudios de teo-
28 La noción de tesis de relevancia se dios Políticos y Constitucionales. ría y metateoría del Derecho, trad. cas-
halla en Alchourrón-Bulygin, 1971, Celano, Bruno (2007): “Pluralismo etico, tellana de J. Ferrer, Barcelona, Gedisa.
cap. VI. Vid. también Rodríguez-Sú- particolarismo e caraterizzazioni di Habermas, Jürgen (1992): Facticidad y
car, 1998, y Rodríguez, 2000. desiderabilità: il modello triadico”, validez. Sobre el Derecho y el estado
29 Celano, 2007. Ragion Pratica, 1 (giugno), 133-150. democrático de derecho en términos
30 Por tomar un ejemplo de Dancy, 1993, Dancy, Jonathan (1993): Moral Reasons, de teoría del discurso, trad. de M. Ji-
60. Oxford: Basil Blackwell. ménez Redondo, Madrid, Trotta.
31 En la línea, por ejemplo, del cap. 6 – (1999): “Defending Particularism”, Hare, R. M. (1952): R. M. The Language
de Wiggins, 1987, o del cap. 10 de Metaphilosophy, 30, 24-32. of Morals, Oxford, Oxford University
Nussbaum, 2001. – (2000): Practical Reality, Oxford, Press.
Oxford University Press, 2000. Hernández marín, Rafael (2005): Las obli-
– (2001): “Moral Particularism”, Edward gaciones básicas de los jueces, Madrid,
BIBLIOGRAFÍA N. Zalta (ed.), The Stanford Encyclope- Marcial Pons, 2005.
dia of Philosophy, Summer 2001 edi- Holton, Richard (2002): “Principles and Par-
Alchourrón, Carlos E.; Bulygin, Eugenio tion, http://plato.stanford.edu/archives/ ticularisms”, Proceedings of the Aristo-
(1971): Normative Systems, New sum2001/entries/moral-particularism/. telian Society, suppl., vol. 76, 191-210.
York-Wien, Springer. – (2004): Ethics without Principles, Oxford, Hurley, Susan (1989): Natural Reasons,
Aleinikoff, T. Alexander (1987): “Constitu- Oxford University Press. Oxford, Oxford University Press.
tional Law in the Age of Balancing,” Dworkin, Ronald (1986): Law’s Empire, – (1990): “Coherence, Hypothetical Ca-
Yale Law Journal, 96, 943-1005. Cambridge, Mass., Harvard University ses, and Precedent”, Oxford Journal of
Alexy, Robert (1986): Theorie der Grundre- Press. Legal Studies, 10, 221-255.
chte, Frankfurt, Srukhamp, trad. de Endicott, Timothy (1998): “Herbert Hart Kant, Immanuel (1989): La metafísica de
­E. Garzón Valdés, Teoría de los dere- and the Semantic Sting”, Legal Theo­ las costumbres, trad. de A. Cortina Ors
ry, 4, 283-301. y J. Conill Sancho, Madrid, Tecnos.

doi: 10.3989/arbor.2010.745n1233 ARBOR  CLXXXVI  745  septiembre-octubre [2010]  821-832  ISSN: 0210-1963
831
McDowell, John (1998): Mind, Value, and – (2005a): “Positivismo giruidico e appli- ción del derecho,” en Analisi e Diritto,
Reality, Cambridge, Mass.; Harvard cazione del diritto”, Materiali per una 1998, 277-305.
University Press. storia della cultura giuridica, 35, 225- Ross, W.D. (1930): The Right and the Good,
Nº 745
McNaughton, David (1988): Moral Vision, 244. Oxford, Oxford University Press.
Oxford, Oxford Blackwell. – (2005b): “Cristina Redondo su ragione Scanlon, Thomas (2000): “Intention and
CONFLICTOS ENTRE DERECHOS CONSTITUCIONALES Y MANERAS DE RESOLVERLOS

McNaughton, David y Rawling, Piers (2000): e norme”, Ragion Pratica, 2 (dicem- Permissibility”, Proceedings of the
“Unprincipled Ethics,” en H Hooker, bre), 491-506. Aristotelian Society, suppl., vol. 74,
Brad y Little, Margaret (eds.) (2000), Nussbaum, Martha C. (2001): The Fragility of 301-317.
Moral Particularism, Oxford, Oxford Goodness: Luck and Ethics in Greek Tra- Shafer-Landau, Russ (1995): “Specifying
University Press. gedy and Philosophy, 2 edition, Cam- Absolute Rights”, Arizona Law Review,
Moreso, José Juan (2002): “Conflitti tra bridge, Cambridge University Press. 37, 209-225.
principi constituzionali”, Ragion Pra- Richardson, H. S. (1990): “Specifying Sinnot-Armstrong, Walter (1999): “Some
tica, 18, 201-221. Norms as a Way to Resolve Concre- Varieties of Particularism”, Metaphi-
– (2004): “Dos concepciones de la te Ethical Problems”, Philosophy and losophy, 30, 1-12.
aplicación de las normas de dere- Public Affairs, 19, 279-310. Väyrynen, Pekka (2006), “Moral Genera-
chos fundamentales,” en Betegón, Rodríguez, Jorge L.(2000): “Axiological lism: Enjoy in Moderation”, Ethics,
Jerónimo; Laporta, Francisco; Pára- Gaps and Normative Relevance,” Ar- 116, 707-741.
mo, Juan Ramón, y Prieto Sanchís, chiv für Recths-und Sozialphilosophie, Wiggins, David (1987): “Deliberation and
Luis (comps.) (2004): Constitución 86, 151-167. Practical Reason”, en D. Wiggins,
y derechos fundamentales, Madrid, Rodríguez, Jorge L.; SÚCAR, Germán (1998): Needs, Values, and Truth. Essays in
Centro de Estudios Políticos y Cons- “Las trampas de la derrotabilidad. Ni- the Philosophy of Value, Oxford, Basil
titucionales. veles de análisis de la indetermina- Blackwell, 1987.

832 ARBOR  CLXXXVI  745  septiembre-octubre [2010]  821-832  ISSN: 0210-1963 doi: 10.3989/arbor.2010.745n1233
módulo 6
Lectura Básica
referencia página 127

Albert Calsalmiglia ¿“Por qué es


importante Dworkin? Doxa, 1985, Nº 2.

EDUBP | Abogacía | Filosofía y Lógica Jurídica - pag.190


módulo 6
Lectura Básica
referencia página 128

Caso “Bustos, Alberto Roque y otros c/ Estado


Nacional y otros s/amparo”

EDUBP | Abogacía | Filosofía y Lógica Jurídica - pag.198


Extracto de fallo:
“Bustos, Alberto Roque y otros c/ Estado Nacional y otros s/ amparo” (C.
S. J. N. 26 de octubre de 2004)
DATOS DEL CASO: la Cámara Federal de Paraná, al confirmar el fallo de
primera instancia, hizo lugar a la acción de amparo promovida por Alberto
Roque Bustos y otros contra el Estado Nacional, el Banco Central de la Repú-
blica Argentina, el Banco de Entre Ríos y el BBVA Banco Francés S.A., decla-
rando la inconstitucionalidad de las leyes 25.557 y 25.561, de los decretos
1570/01, 71/02, 141/02, de los arts. 1º, 2º, 4º, 9º y 10 del decreto 214/02,
del art. 3º del decreto 320/02, de las resoluciones 6, 9, 18, 23 y 36/2002
del Ministerio de Economía de la Nación, de las comunicaciones “A” 3446,
3467 y 3468 del Banco Central de la República Argentina, y toda otra norma
que impida, limite o restrinja de cualquier manera a los actores la posibilidad
de disponer inmediatamente de sus depósitos a plazo fijo y en cuentas a
la vista, y ordenó que las entidades financieras intervinientes devolvieran los
depósitos en el signo monetario efectuado, en el plazo de diez días.

Recurrida la sentencia mediante recurso extraordinario federal, la Corte


Suprema de Justicia de la Nación hizo lugar al recurso y revocó el fallo.

Lo que sigue es parte de la argumentación esgrimida por el tribunal -Corte


Suprema- al resolver el recurso extraordinario:

Por cierto, no cabe poner en tela de juicio que el derecho de propiedad


está garantizado por el art.17 de la Constitución, pero lo que sí corres-
ponde examinar es si las medidas económicas –en suma, la suspensión de
la devolución de los depósitos bancarios y la opción para los depositantes
entre el reintegro en determinados plazos, en moneda argentina y con valo-
res actualizados, o en la moneda de origen a plazos mayores y en valores
comercializables – afecta el mencionado derecho. En otros términos, si a
pesar de las normas dictadas a fin de evitar que la situación de emer-
gencia desembocara en la quiebra del sistema bancario y la pérdida de sus
depósitos por la mayor parte de los depositantes, la propiedad de éstos
sólo puede ser defendida mediante el reintegro de sus valores a breve
plazo y en la moneda extranjera en que fueron inscriptos.

Resulta indiscutible – en el somero examen que puede realizarse en un pro-


ceso sin pruebas- que la devaluación de la moneda era una medida ineludi-
ble frente a la grave emergencia resultante del desfase de valor con el dólar
estadounidense y el comienzo de una importante “corrida bancaria”.

En esa situación, parecería a primera vista que la conversión de los créditos


a moneda argentina a un tipo de cambio determinado por el Estado afectase
el derecho de propiedad del acreedor, que tendría derecho de obtener el
reintegro, por lo menos de la cantidad exactamente equivalente a los nuevos
tipos vigentes en el mercado; sería, por otra parte, el resultado de la “ciega”
aplicación de los arts. 617 y 619 del Código Civil. Sin embargo – y esto no
podrían discutirlo ni siquiera los más ardientes defensores de la propiedad
intangible de los dólares – en los contratos de depósito bancario que motivan
litigios como el presente, el propósito del acreedor no era el de conseguir
moneda extranjera específica que no pudiera ser sustituida por ningún

EDUBP | Abogacía | Filosofía y Lógica Jurídica - pag.199


objeto sino el de lograr la estabilidad de la prestación dineraria, es decir,
asegurar un poder adquisitivo constante. Es obvio que si se depositaba en
dólares era porque se dudaba del mantenimiento del poder adquisitivo de
los pesos nacionales y se buscaba mantener el valor intrínseco del capital.
Desde ese punto de vista, puede afirmarse que la supuesta propiedad de los
dólares no era más que una gran “falacia”. En la Argentina nadie ganaba
en dólares norteamericanos, de manera que asentar los depósitos en esa
moneda era, o bien utilizarla simplemente como una moneda de cuenta –
nadie ignora que en gran medida se depositaban pesos y éstos se convertían
a igual cantidad de dólares, “equivalencia cuya falsedad se hacía notoria si
se pretendía la imposible tarea de cambiar pesos por dólares en el exte-
rior” (salvo en algunos países vecinos), y que “hoy se ve claro que era fic-
ticia” – o bien se compraban dólares en el mercado de cambios a un precio
vil, el de la paridad, con consecuencias perniciosas en la economía que a la
corta o a la larga debían desembocar en el desastre económico resultante
de que tuviese que afrontar el perjuicio el Estado, es decir, la comunidad
toda, no los depositantes en dólares sino todos y cada uno de los habitantes
del país. Por tanto, en los contratos concertados en el país y que debían ser
ejecutados en él, el objeto de la obligación no era el dólar como moneda de
los Estados Unidos sino el definido por la ley 23.928 como “convertible” o
canjeable por pesos argentinos de poder adquisitivo constante.

En tal contexto, y en tanto no se trate de moneda extranjera que estu-


viese específicamente destinada al cumplimiento de las obligaciones
en el exterior, la “pesificación” se presenta como razonable mientras
el importe que se devuelva tenga el mismo o mayor poder adquisitivo
que tenía el depósito originario, ya que ello no causa perjuicio alguno
al acreedor. Fue, por lo demás, una medida razonable frente a la situación
de fuerza mayor trasuntada en la emergencia. Por el contrario, pretender la
devolución inmediata en dólares o en su equivalente en moneda argen-
tina en el mercado libre de cambios implica un “desmesurado beneficio
para el acreedor”, que no puede determinarse en su medida aritmética
como consecuencia de la ausencia de producción de prueba en este pro-
ceso pero que en algunos momentos alcanzó alrededor del doble del poder
adquisitivo originario.

“A falta de daño producido por el Estado no hay acción”. La actora no ha


demostrado que el reintegro a $ 1,40 por dólar más el coeficiente de esta-
bilización de referencia no alcance a cubrir el mismo poder adquisitivo del
dinero depositado; y si lo que pretende son dólares, por dos veces el Estado
le ha ofrecido bonos por el monto originario del depósito, que, si bien
tienen plazos más largos – pero, en todo caso, bastante más cortos que los
emitidos con igual fin en 1990 con el aval constitucional de la mayoría de esta
Corte en el ya citado caso de Fallos: 313:1513 -, al ser comercializables en
bolsa permiten en un plazo más reducido convertidos en moneda contante,
con un descuento, sí, pero que después de un tiempo razonable difícil-
mente afecte el poder adquisitivo que tenían en su momento los dólares
“supuestamente” depositados.

En suma, no está probado: 1º) que la actora hubiera efectuado el depó-


sito en dólares efectivamente ganados como tales – lo que deja abierta la
posibilidad de que los haya comprado al “precio vil” que mantenía el

EDUBP | Abogacía | Filosofía y Lógica Jurídica - pag.200


“Estado” o que se haya tratado de un mero asiento contable-, ni que los
dólares estuviesen afectados a operaciones comerciales o financieras
con el exterior que necesariamente debieran afrontarse en esa moneda;
2º) que las alternativas que el Estado le ofrece le ocasionen un perjuicio
patrimonial, ya que no se ha alegado ni probado que el destino del depó-
sito fuese el cumplimiento de obligaciones fuera del mercado interno. En
tales condiciones, la garantía constitucional de la propiedad está a salvo:
los bancos que recibieron los depósitos cumplirán su obligación de reintegro
en las condiciones impuestas por el Estado, hecho del príncipe que no está
en sus facultades evitar, y el Estado sólo responderá del perjuicio ocasio-
nado por las medidas en caso de demostrarse la real existencia de un
perjuicio. Todo ello lleva a la conclusión de que la vía del amparo no era
admisible, no sólo por mera formalidad legal sino por “no estar acreditada
la violación de preceptos constitucionales”, que en este tipo de procedi-
miento debe resultar palmaria e inequívoca.

Contenido abreviado de la normativa principal invocada:


Ley 25.561/2002: en aplicación del art.76 de la CN declara la emergencia
pública en materia social, económica, administrativa, financiera y cambiaria,
delegando al PEN las facultades comprendidas en la ley hasta el 10/12/2003.
Entre otras facultades se delega: 1.- el reordenamiento del sistema finan-
ciero, bancario y del mercado de cambios y 2.- reglamentar la reestructura-
ción de las obligaciones en curso de ejecución afectadas por el nuevo régi-
men cambiario. Deroga disposiciones de la ley de convertibilidad (25.445);

Suspende ley 25.557 del 29/08/01 que establecía la intangibilidad de los


depósitos bancarios hasta la superación de la emergencia.

Decreto 1570/01: establece la conversión de depósitos en pesos a dólares


con consentimiento del deudor y limita el retiro de efectivos a $ 250 (o dóla-
res) por semana. Prohíbe transferencias al exterior.

Decreto 71/02: establece relación de cambio $1,40 por cada unidad de dóla-
res estadounidenses.

Art.17 CN: garantiza derecho de propiedad.

EDUBP | Abogacía | Filosofía y Lógica Jurídica - pag.201


módulo 7
Lectura Básica
referencia página 154

RODRÍGUEZ, Jorge L.; “Naturaleza y lógica de las


proposiciones normativas”, Doxa 26, 2003.

EDUBP | Abogacía | Filosofía y Lógica Jurídica - pag.202


Jorge L. Rodríguez 87

NATURALEZA Y LÓGICA DE LAS


PROPOSICIONES NORMATIVAS.
CONTRIBUCIÓN EN HOMENAJE
A G. H. VON WRIGHT

Jorge L. Rodríguez
Universidad Nacional de Mar del Plata

1. Introducción*

A
l examinar el discurso normativo y su lógica subyacente,
una de las cuestiones de mayor importancia es la necesidad
de apreciar con claridad las consecuencias que se siguen
de la distinción, destacada entre otros por von Wright,
entre normas y proposiciones normativas. Las normas son
expresadas por formulaciones normativas; constituyen el significado de
oraciones usadas para prescribir, esto es, para ordenar, prohibir o permitir
ciertas conductas. Las proposiciones normativas, en cambio, constituirían el
significado de oraciones que califican ciertas acciones como obligatorias,
prohibidas o permitidas de conformidad con ciertas normas.1 Mientras
que las proposiciones normativas son susceptibles de verdad o falsedad,
no parece tener mayor sentido predicar tal cosa de las propias normas. La
dificultad para distinguir estas dos nociones deviene de que la misma ora-
ción (por ejemplo, “Está prohibido fumar”) puede a veces ser utilizada para
formular una norma y otras para referirse a la existencia de una norma, esto
es, puede a veces expresar una norma y otras una proposición normativa. Es
verdaderamente lamentable que, pese a que con denominaciones diversas la
diferencia que esta distinción recoge ha sido reconocida por muchos auto-
res, ella no ha recibido todavía hoy la atención que a mi juicio merece en el
terreno de la lógica del discurso normativo.2

*
Quiero expresar mi profundo agradecimiento a Pablo Navarro, a quien debo la sugerencia
de desarrollar las ideas que he tratado de volcar en las dos últimas secciones del trabajo.
1
Cf. G. H. von Wright, Norma y acción. Una investigación lógica, Madrid, Tecnos, 1970,
título original: Norm and Action. A Logical Inquiry, London, Routledge & Kegan Paul, 1963,
página 109.
2
Cf. G. H von Wright, “Deontic Logic − As I See It”, en P. McNamara y H. Prakken,
Norms, Logics and Information Systems, Amsterdam-Berlin-Oxford-Tokyo-Washington DC,
IOS Press, 1999, página 20.
88 Jorge L. Rodríguez

En el presente trabajo intentaré examinar tres cuestiones vinculadas con


las características distintivas de las proposiciones normativas y su lógica. En
primer lugar, la posibilidad de concebir, junto a proposiciones normativas
relativas a un cierto conjunto de normas, proposiciones normativas de ca-
rácter absoluto, esto es, proposiciones que califican normativamente ciertas
acciones como obligatorias, prohibidas o permitidas no relativizadas con
relación a un determinado conjunto de normas. En segundo lugar, la posibi-
lidad de diferenciar la categoría de los “enunciados normativos”, entendidos
como enunciados acerca de que algo debe o puede ser hecho, de la categoría
de las proposiciones normativas, esto es, proposiciones relativas a la exis-
tencia de una norma. Por último, trataré de reseñar las particularidades de
dos visiones alternativas de reconstrucción de la lógica de proposiciones
normativas, así como de los presupuestos subyacentes a cada una de ellas.

2. El carácter relativo de las proposiciones normativas


A primera vista, los dos rasgos más salientes que justifican diferenciar
a las proposiciones normativas de las normas son, en primer lugar, que
aunque es altamente dudoso que pueda predicarse verdad o falsedad de las
normas, tales calificaciones sí cobran pleno sentido respecto de las propo-
siciones normativas. Y, en segundo lugar, que las proposiciones normativas
son relativas. Una misma acción p puede estar prohibida, ser permitida u
obligatoria, en un cierto sistema normativo y no en otro. Por tal motivo, la
expresión “p está prohibido”, entendida como una proposición normativa,
es incompleta, ya que mientras no se especifique de qué sistema normativo
se trata, carecerá de valor de verdad.3
Este segundo rasgo de las proposiciones normativas ha sido, con todo,
controvertido recientemente por Jan-R. Sieckmann.4 Sieckmann sostiene que
una proposición normativa, entendida como la afirmación de que una norma
es válida de conformidad con cierto sistema de normas, es una afirmación
relativa a su pertenencia al sistema en cuestión, pero no un enunciado que de
manera directa predique que una norma es “válida”, entendiendo aquí esta
última expresión en sentido prescriptivo, esto es, como referida a que deba
hacerse lo que ella dispone. A su criterio, un enunciado normativo tal como
“No se debe fumar” contendría una proposición normativa relativa a que
no se debe fumar, lo cual se revelaría en la posibilidad de que el hablante

3
Cf. E. Bulygin “Lógica deóntica”, en C. Alchourrón et al. (eds.), Enciclopedia Iberoame-
ricana de Filosofía, Volumen 7, Lógica, Madrid, Trotta, 1995, páginas 129−142.
4
Cf. J. R. Sieckmann, “Remarks on Jorge Rodríguez´s ‘Sistemas Jurídicos y Derrotabili-
dad del Derecho’”, 2000, inédito y J. R. Sieckmann, “The Fragmentation of Deontic Logic”,
ponencia presentada en el XIX Congreso Mundial de Filosofía del Derecho y Filosofía Social,
Lundt, 2003.
Naturaleza y lógica de las proposiciones normativas... 89

ponga de manifiesto su carácter proposicional en oraciones como “Es el


caso que no se debe fumar” o “Es verdad que no se debe fumar”. Pero Siec-
kmann sostiene además que si se asume que no es posible predicar verdad o
falsedad de las normas, las proposiciones normativas sólo podrían interpre-
tarse como aquello que expresan oraciones que se refieren a la pertenencia
de una norma a un cierto sistema, y esto último le parece inaceptable por
cuanto, si bien muchas veces se formulan enunciados normativos relativos
a un cierto sistema, la forma más elemental de una proposición normativa
sería a su juicio una afirmación directa o absoluta de lo que es obligatorio,
prohibido o permitido.
Para Sieckmann, la necesidad de una lógica de proposiciones normativas
en forma pura o absoluta se revelaría con claridad cuando se intenta respon-
der a la pregunta acerca de qué se debe hacer en cierta situación, en el senti-
do de qué norma debe guiar nuestras acciones. La respuesta a esta pregunta
requeriría de una proposición normativa absoluta, ya que un enunciado que
dijese que de conformidad con cierto sistema normativo se debe hacer p
sería manifiestamente insuficiente puesto que podría aceptarse su verdad y,
no obstante, preguntarse si realmente debe seguirse esa norma. En cambio,
si una proposición normativa absoluta es verdadera, no podría seriamente
cuestionarse si ha de seguirse la norma a la que ella alude.
Desde mi punto de vista, la relatividad a un cierto sistema normativo
es la característica más significativa de las proposiciones normativas, de
manera que esta categoría de proposiciones normativas absolutas que Siec-
kmann introduce no puede sino ser el producto de una confusión porque
resulta autocontradictoria. Intentaré justificar esta idea. En una primera
interpretación, la idea de una proposición normativa absoluta podría enten-
derse como una proposición referida a una norma de validez absoluta. Bajo
esta interpretación, el calificativo de “absolutas” que Sieckmann atribuye a
las proposiciones normativas no sería sino un referencia al tipo de normas
a cuyo respecto se formulan tales proposiciones. Ello, claro está, si es que
puede asignarse sentido a la expresión “norma absoluta”, cualquiera sea
éste. Pero entonces las proposiciones normativas absolutas serían aquellas
proposiciones normativas relativas al conjunto de las normas absolutas. No
existiría ninguna diferencia significativa entre este tipo de entidades y las
proposiciones normativas relativas a cierto sistema, pues la única peculia-
ridad se encontraría aquí en el dominio de las normas que se toman como
punto de referencia. En particular, los problemas que pudiera suscitar la
identificación de “normas absolutas” semejantes resultarían independientes
de las relaciones entre las proposiciones que a ellas se refieren.
Las mismas distinciones que correspondería establecer en la lógica de
proposiciones normativas ordinaria se reproducirían respecto de las “pro-
90 Jorge L. Rodríguez

posiciones normativas absolutas” así entendidas. Por ejemplo, aquella que


media entre el sentido positivo y el sentido negativo de la permisión. Como
veremos, puede decirse que una conducta está permitida en sentido positivo
de acuerdo con las normas de cierto sistema normativo si, y sólo si, en él
existe una norma que permite dicha conducta. En cambio, una conducta
está permitida en sentido meramente negativo de acuerdo con las normas de
cierto sistema normativo si, y sólo si, no existe en el sistema de referencia
una norma que prohíba tal conducta. Como fuera demostrado por Carlos
Alchourrón, la permisión positiva y la permisión negativa sólo pueden
considerarse equivalentes respecto de sistemas normativos completos (esto
es, que no contienen lagunas) y consistentes (es decir, que no contienen
contradicciones).5 Pero, a excepción de ese supuesto, ambas nociones no
pueden asimilarse. En razón de esta ambigüedad en la expresión “permiti-
do” cuando aparece en proposiciones normativas, que no se verifica cuando
tal expresión es usada para formular una norma (contexto en el cual “per-
mitido” equivale a “no prohibido”), es que una lógica de normas no resulta
equivalente a una lógica de proposiciones normativas. Todo esto, como se
dijo, valdría igualmente para proposiciones normativas relativas a “normas
absolutas”, razón por la cual no podría fundarse sobre esta interpretación
la pretensión de distinguir a las proposiciones normativas absolutas como
entidades conceptualmente distintas de las proposiciones normativas −rela-
tivas− ordinarias.
Descartada esta posibilidad, una interpretación alternativa consistiría
en considerar que el calificativo de “absolutas” no se refiere a las normas
mencionadas en las proposiciones normativas absolutas, sino a las propias
proposiciones normativas. Una proposición normativa relativa calificaría
normativamente una acción de conformidad con un cierto sistema norma-
tivo. A diferencia de ella, una proposición normativa absoluta sería aquella
que califica normativamente a una acción considerando todos los factores
relevantes (all things considered). A primera vista, diría que determinar la ca-
lificación normativa de una acción considerando todos los factores relevantes
sería tanto como hacerlo tomando en cuenta, no un cierto sistema normativo,
sino una pluralidad de sistemas normativos. En tal sentido, me parece que la
perspectiva intersistemática es conceptualmente dependiente de la intrasiste-
mática.6 Pero examinemos más detenidamente esta alternativa.

5
Cf. C. E. Alchourrón, “Logic of Norms and Logic of Normative Propositions”, en
Logique et Analyse 12, Nº 47, 1969, páginas 242-268.
6
Para un punto de vista semejante en la discusión en torno a la derrotabilidad, véase M.
Belzer y B. Loewer, “Deontic Logics of Defeasibility”, en D. Nute (ed.), Defeasible Deontic
Logic, Dordrecht-Boston-London, Kluwer Academic Publishers, 1997, página 45.
Naturaleza y lógica de las proposiciones normativas... 91

Una forma de entender esta idea sería decir que las proposiciones nor-
mativas absolutas son proposiciones normativas relativas a la totalidad de
los sistemas normativos relevantes. Pero para Sieckmann las proposiciones
normativas absolutas deben suministrar una respuesta a la pregunta acerca
de qué se debe hacer en cierta situación. Supóngase que quiero saber si debo
o no realizar la acción p en la situación C; que dos normas resultan relevan-
tes en tal situación −una norma jurídica y una norma moral, por ejemplo−,
y que cada una de ellas brinda una respuesta diferente. Es evidente que una
proposición normativa que se limitase a informar acerca de una disyunción
de normas potencialmente incompatibles no respondería a la pregunta
acerca de lo que se debe hacer en C. Lo que se requeriría es un criterio de
selección entre las diferentes normas en juego. En otras palabras, la idea
de una proposición normativa absoluta, tal como la concibe Sieckmann, no
sólo exige tomar en cuenta todos los sistemas relevantes, sino además las
relaciones de prelación o jerarquía entre las normas de tales sistemas. Las
relaciones de preferencia entre normas de diferentes sistemas normativos
pueden asumir formas diversas. Puede que un sistema normativo S1 sea je-
rárquicamente superior a otro S2 como un todo, esto es, que dada una norma
cualquiera de S1 y una norma cualquiera de S2, siempre se prefiera la pri-
mera sobre la segunda. Puede ocurrir, en cambio, que algunas normas de S1
sean preferidas a algunas normas de S2, pero que asimismo algunas normas
de S2 sean preferidas a algunas normas de S1. Y también puede ocurrir que
ciertas normas de S1 sean en ciertos casos preferidas a ciertas normas de S2,
mientras que en otros se prefiera a las de S2 sobre las de S1.
De todos modos, sumar a la totalidad de los sistemas relevantes las re-
laciones de preferencia entre sus diversas normas todavía sería insuficiente.
Una proposición normativa absoluta sólo estaría en condiciones de ofrecer
una respuesta definitiva a la pregunta acerca de lo que se debe hacer en la
situación C si, además de tomar en cuenta todos los sistemas relevantes y
las relaciones de preferencia entre sus normas, se cumpliera que tales cri-
terios permiten seleccionar una y sólo una norma como aquella que resulta
aplicable en C. La simple existencia de criterios de preferencia no garantiza
que su empleo permita privilegiar a una única norma sobre las restantes.
Considerar que frente a cualquier situación C es posible formular una pro-
posición normativa absoluta requiere presuponer que existe al menos una
norma que regula la cuestión y, en caso de que haya más de una, que es
posible resolver cualquier posible conflicto entre las que se presentan como
prima facie relevantes.
La pregunta que cabe formular es si, así interpretadas, estas “proposi-
ciones normativas absolutas” tendrían el carácter de genuinas proposicio-
nes normativas o más bien el de normas. Si lo que interesa es resolver el
92 Jorge L. Rodríguez

problema práctico de qué se debe hacer en la situación C, puede resultar


valioso que se informe sobre la existencia de una norma. Ello por cuanto
la ignorancia acerca de su existencia puede llevar a escoger una solución
equivocada. En este sentido, puede decirse que una proposición normativa,
esto es, la información sobre la existencia de una norma, puede −al menos
en ciertos casos− tener incidencia en la resolución de un problema práctico.
Pero suministrar información no resulta suficiente para resolver un proble-
ma práctico, dado que frente a la proposición que afirma la existencia de una
norma siempre cabe la pregunta ¿se debe hacer lo que esta norma prescribe?
Lo que se requiere para resolver un problema práctico es usar una norma.
Y si en una cierta situación C concurren varias normas como relevantes, se
debe elegir a una de ellas como guía para la acción. En otras palabras, bajo
la interpretación aquí considerada, una proposición normativa absoluta sería
aquella que, efectuando un balance entre las distintas normas que se presen-
tan como relevantes en la situación C, privilegia a una de ellas como aquella
que debe orientar la conducta. Ahora bien, si ese “debe” es descriptivo, esto
es, si se limita a indicar la existencia de un criterio para escoger entre las
normas en juego, todavía se hallaría abierta la posibilidad de cuestionar por
qué ha de emplearse ese criterio de elección y no otro entre las diversas
normas en juego. Si en cambio ese “debe” se interpreta prescriptivamente,
el enunciado “Entre las diversas normas en juego frente a la situación C,
debe usted elegir la norma N como guía de su conducta” no expresaría una
proposición normativa sino lisa y llanamente una norma. De manera que
bajo esta interpretación, la idea de una proposición normativa absoluta no
sería sino el producto de confundir a las proposiciones referidas a normas
con las propias normas.
Aunque me parece destacable que Sieckmann tome en cuenta la dis-
tinción entre normas y proposiciones normativas, hablar de proposiciones
normativas absolutas resulta inadecuado porque, o bien introduce un pro-
blema que debería mantenerse al margen de ella −el del fundamento del
deber de cumplir las normas a las que las proposiciones normativas hacen
referencia−, o bien conduce en última instancia a confundir lo que pretende
distinguirse.

3. Proposiciones normativas y enunciados normativos

En un trabajo de 1965, von Wright se pregunta por los fundamentos de


los enunciados normativos.7 Allí define “enunciado normativo” como un

7
Cf. G. H. von Wright, “The Foundation of Norms and Normative Statements”, publica-
do originariamente en K. Ajdukiewicz (ed.), The Foundations of Statements and Decisions,
Warsawa, Polish Scientific Publishers, 1965, páginas 351-367 y posteriormente incluido en G.
Naturaleza y lógica de las proposiciones normativas... 93

enunciado acerca de que algo debe o puede ser hecho. El fundamento de


un enunciado normativo sería “el contenido de una respuesta verdadera a la
pregunta acerca de por qué aquello a lo que se refiere el enunciado debe o
puede ser hecho”, por lo que también podría ser calificado como su funda-
mento de verdad. A veces quien formula el enunciado suministra él mismo
un fundamento (“Se debe hacer p porque...”), pero si no se suministra un
fundamento, puede inquirirse por él.
A juicio de von Wright, una posible respuesta a la pregunta acerca de por
qué se debe hacer p es que existe una norma que establece que debe hacerse
p, y ello constituiría el fundamento de verdad del enunciado normativo.
Para von Wright, “norma” alude aquí a una prescripción o regulación, que
un cierto agente (autoridad) da a otro (sujeto normativo), lo cual requiere
el uso del lenguaje. Señala que las normas se diferencian de los enunciados
normativos en el hecho de que respecto de las primeras no cabe predicar ver-
dad o falsedad, mientras que decir que existe una norma es una afirmación
susceptible de verdad o falsedad. Otras veces, el fundamento de verdad de
un enunciado normativo podría deducirse de la existencia de una norma. En
general, las normas se refieren a categorías o clases de actos, mientras que
los enunciados normativos se refieren a actos individuales.
De estas consideraciones de von Wright puede concluirse que el “funda-
mento de verdad” de los enunciados normativos dependería siempre de la
existencia de una norma, ya sea de manera directa o indirecta. Esto último
ocurriría toda vez que la verdad del enunciado normativo dependiese, no
directamente de lo que una norma diga, sino de una consecuencia lógica de
ella. Así, podría afirmarse que los enunciados normativos expresan proposi-
ciones normativas, esto es, proposiciones que predican la existencia de una
norma, sea que ella haya sido directamente promulgada por cierta autoridad
o que pueda derivarse de las promulgadas por cierta autoridad.
Aunque en el planteo que efectúa von Wright parece distinguirse entre
enunciados normativos y proposiciones normativas, si se toma en cuenta lo
dicho hasta aquí, un enunciado normativo no parecería más que una manera
indirecta de referirse a la existencia de una norma. Bulygin ha elaborado con
cierto detalle este punto de vista y, en un trabajo reciente, sostiene que enun-
ciados como (1) “Usted no debe estacionar aquí” y (2) “Hay una norma que
prohíbe estacionar aquí” tienen el mismo significado, siendo (2) una forma
expandida de (1). Un enunciado como (2), que expresaría una proposición
normativa en su forma expandida, tendría la ventaja de que ya no sería
ambiguo. Ello en razón de que no sería posible usarlo para establecer una

H von Wright, Practical Reason − Philosophical Papers Volume I, Ithaca, New York, Cornell
University Press, 1983, páginas 67-82.
94 Jorge L. Rodríguez

prohibición por tratarse de un enunciado característicamente descriptivo,


mientras que el enunciado (1) sería, en cambio, ambiguo porque podría ser
usado para formular una norma o expresar una proposición normativa.8
El fundamento de esta conclusión es que no existiría diferencia entre
informar, por ejemplo, acerca de las regulaciones existentes respecto de la
prohibición de estacionar, y afirmar que hay una norma que prohíbe esta-
cionar. Si la información es verdadera, entonces existiría una norma que
prohíbe estacionar. Si en cambio no existiese esa norma, la afirmación sería
falsa. En su reconstrucción, Bulygin sostiene que una proposición normativa
afirma que existe una norma en un cierto sistema normativo que regula una
determinada acción. Por consiguiente, considera que un enunciado como (2)
−“Hay una norma que prohíbe estacionar aquí”− tampoco exhibiría todo su
contenido conceptual y debería ser expandido en un enunciado como:
(3) “En el sistema normativo α hay una norma que prohíbe estacionar
aquí”.
Desde este punto de vista, una proposición normativa establecería una
relación entre una norma y un sistema normativo y, en tanto ambos tér-
minos de la relación no estuviesen determinados, no existiría proposición
normativa.
Pablo Navarro ha propuesto denominar a los enunciados que predican la
pertenencia de una norma a un sistema “proposiciones sistemáticas”, distin-
guiéndolos de las proposiciones normativas, que afirmarían la existencia de
una norma.9 Sobre tales bases, sostiene que la propuesta de Bulygin equival-
dría a identificar los enunciados normativos con proposiciones normativas
y, a su vez, estas últimas con proposiciones sistemáticas. Para Bulygin, los
enunciados normativos sólo serían una forma elíptica de afirmar una pro-
posición sistemática, un modo transpuesto de discurso (transposed mode of
speech), pues para decir que hay una norma que prohíbe una acción p en un
cierto sistema normativo, se diría que p está prohibida.
A juicio de Navarro, el trabajo de Bulygin sólo probaría que los enuncia-
dos normativos tienen las mismas condiciones de verdad que las proposicio-
nes normativas, pero ello no garantizaría que tuviesen el mismo significa-
do, y menos aún que tuviesen el mismo significado que las proposiciones
sistemáticas. Por mi parte, considero que si bien la primera observación de
Navarro puede admitirse, la segunda es fruto de un malentendido. En efecto,
en la sugerencia de Bulygin no hay un paso de enunciados normativos a

8
Cf. E. Bulygin, “True or False Statements in Normative Discourse”, en R. Eigidi (ed.),
Search of a New Humanism: The Philosophy of G.H. von Wright, Synthese, v. 282, 1999.
9
Cf. P. E. Navarro, “Legal Reasoning and Systematization of Law”, presentado como
ponencia al XX Congreso Mundial de Filosofía del Derecho y Filosofía Social, Amsterdam,
2001.
Naturaleza y lógica de las proposiciones normativas... 95

proposiciones normativas y uno adicional a “proposiciones sistemáticas”.


Identificar un enunciado que se refiere a la existencia de una norma con uno
que predica la pertenencia de una norma a un cierto sistema no es más que
una forma de hacer referencia a que las calificaciones normativas de ciertas
acciones no suelen hacerse tomando en cuenta una única norma sino una
pluralidad de ellas. Pero aún cuando no se asumiera este último presupuesto,
desde un punto de vista estrictamente formal parece más adecuado relativi-
zar la noción de proposición normativa a un sistema normativo en lugar de
hacerlo respecto de una norma, dado que esto último puede ser visto como
un caso límite de lo primero, esto es, un sistema normativo conformado por
una única norma. Por ello, considero que el paso de (2) a (3) es trivial. Por
su parte, el paso de (1) a (2) me parece plenamente justificado siempre que
(1) sea interpretado descriptivamente, ya que la aparente diferencia en el
significado entre uno y otro no es sino el fruto de la ambigüedad descripti-
vo/prescriptivo que afecta a (1) pero no a (2).

4. Lógica de normas y lógica de proposiciones normativas


En el sistema clásico de lógica deóntica ideado por von Wright10 se
toman como interdefinibles a los conceptos de prohibición, obligación y
permisión:
1. Op =def PH∼p 3. PHp =def O∼p
2. PHp =def ∼Pp 4. Op =def ∼P∼p
En una presentación axiomática del sistema tendríamos los si-
guientes tres axiomas:

Sistema LN:

(A0) |− Pp ↔ ∼O∼p
(A1) |− O(p ∧ q) ↔ (Op ∧ Oq)
(A2) |− Op → Pp

Se acepta además la siguiente regla de inferencia:

(RI) De |− (p ↔ q), se sigue |− (Op ↔ Oq)11

10
Cf. G. H. von Wright, “Deontic Logic”, en Mind 60, 1951, páginas 1-15.
11
Cf. C. Alchourrón, “Philosophical Foundations of Deontic Logic and the Logic of
Defeasible Conditionals”, en J. Meyer y R. Wieringa, Deontic Logic in Computer Science:
Normative System Specification, Chichester-New York-Brisbane-Toronto-Singapore, Wiley &
Sons, 1993, páginas 43-84.
96 Jorge L. Rodríguez

El primer axioma indica la interdefinibilidad entre permisión y obliga-


ción. El segundo, la distributividad de la obligación respecto de la conjun-
ción. El tercero, el principio de que obligatorio implica permitido −equiva-
lente al llamado principio de permisión: “Pp ∨ P∼p”−. La regla de inferencia
del sistema establece que toda tautología de la lógica de enunciados es una
fórmula válida del sistema cuando las variables de enunciado son reempla-
zadas por fórmulas deónticas.

El axioma A0 garantiza la validez de las siguientes dos leyes:

5. ∼(O∼p ∧ Pp) principio de consistencia normativa


6. O∼p ∨ Pp principio de completitud normativa

Demostración:

7. Pp → ∼O∼p implicación de izquierda a derecha de A0.


8. O∼p → ∼Pp contraposición en 7. ((p → q) → (∼q → ∼p))
9. ∼(O∼p ∧ Pp) definición del condicional en 8. ((p → q) ↔ ∼(p ∧ ∼q)).

10. ∼O∼p → Pp implicación de derecha a izquierda de A0.


11. O∼p ∨ Pp definición del condicional en 10. ((p → q) ↔ (∼p ∨ q)).

De manera que de acuerdo con este sistema, respecto de cualquier ac-


ción, ella no podría estar prohibida y permitida a la vez (principio de consis-
tencia normativa) y, o bien está prohibida, o bien está permitida (principio
de completitud normativa).
¿Reconstruye este sistema acabadamente las relaciones lógicas entre
expresiones como “obligatorio”, “prohibido” y “permitido”? Para responder
a esta pregunta debe tenerse presente la ya apuntada sistemática ambigüedad
de tales expresiones, esto es, que ellas pueden aparecer tanto en normas en
sentido genuino como en proposiciones normativas.
Resulta muy importante diferenciar claramente estas dos interpretacio-
nes de expresiones como “obligatorio”, “prohibido” y “permitido”, así como
de las oraciones en las que ellas aparecen. Una lógica de estas expresiones
interpretadas prescriptivamente (una genuina lógica de normas) pretenderá
reconstruir la racionalidad en la actividad de dictar normas, la “racionalidad
del legislador”. En cambio, una lógica de estas expresiones interpretadas
descriptivamente (una lógica de proposiciones normativas), buscará recons-
truir las calificaciones normativas de ciertas acciones de conformidad con
un cierto conjunto de normas. De acuerdo con la conocida distinción de
Hart, podría decirse que un sistema de lógica de normas refleja el punto de
Naturaleza y lógica de las proposiciones normativas... 97

vista interno de quien dicta normas o quien las acepta y las usa como guía
de conducta, mientras que un sistema de lógica de proposiciones normativas
refleja el punto de vista externo de quien se sitúa como observador y califica
normativamente ciertas acciones de conformidad con las normas existentes
en un cierto sistema normativo.12
Una forma de explicar las diferencias entre una lógica de normas y una
lógica de proposiciones normativas es a partir del diferente modo en el que
la negación afecta a unas y otras. Aún cuando no se pueda decir que las nor-
mas sean verdaderas o falsas, la negación parece comportarse en el discurso
prescriptivo de modo análogo a la negación ordinaria: la negación de una
norma es también una norma; para cada norma sólo hay una norma que es
su negación; una norma y su norma-negación13 son recíprocas (si “Op” es la
negación de “P∼p”, “P∼p” es la negación de “Op”). Además, una norma y su
norma-negación son mutuamente excluyentes y conjuntamente exhaustivas,
dado que “∼(Pp ∧ ∼Pp)” y “Pp ∨ ∼Pp” son admitidas como fórmulas válidas
en la lógica de normas.14
La negación de las proposiciones normativas es más problemática. “En
el sistema normativo α el acto p está prohibido” es un enunciado metalin-
güístico complejo que afirma que cierta norma pertenece a un sistema nor-
mativo, por lo que su negación puede ser entendida de dos modos diversos:
“En el sistema α el acto p no está prohibido” puede interpretarse como
expresivo de que el sistema α no contiene una norma que prohíbe la reali-
zación del acto p, en cuyo caso la negación opera sobre la pertenencia de la
norma al sistema −se niega la proposición normativa−. Pero también puede
interpretárselo como significando que el sistema α contiene una norma que
establece la no prohibición de p (esto es, una norma que permite p), en cuyo
caso la negación opera directamente sobre la norma. Para dar cuenta de esta
ambigüedad es necesario diferenciar dos nociones de negación para las pro-

12
Cf. H. L. A. Hart, El concepto de derecho, Buenos Aires, Abeledo Perrot, 1963, título
original: The Concept of Law, Oxford, Oxford University Press, traducido por G. Carrió, pá-
ginas 110-111. En realidad, de acuerdo con Hart, no sólo es un enunciado externo aquel que
describe el hecho de que una cierta regla existe, es decir, es aceptada por el grupo social,
sino que también lo sería aquel que se limita a registrar las regularidades de conducta de
quienes cumplen con las reglas y el que registra, además, la reacción hostil regular frente a
las desviaciones del patrón usual de conducta (cf. H. L. A. Hart, obra citada, páginas 308-309
y E. Bulygin, “Norms, Normative Propositions and Legal Statements”, en G. Floistad (ed.),
Philosophy of Action. Chronicles of Philosophy, vol. III, The Hague, Martinus Nijhoff, 1982,
páginas 107-125).
13
La expresión “norma-negación” fue acuñada por G. H. von Wright en “Norms, Truth and
Logic”, en Practical Reason. Philosophical Papers, volume I, Oxford, Basil Blackwell, 1983,
páginas 130-209. En un ejemplo, la norma-negación de “Op” es “∼Op” (o “P∼p”).
14
Cf. C. Alchourrón, “Logic of Norms and Logic of Normative Propositions” y E. Bulygin,
“Lógica Deóntica”, ya citado.
98 Jorge L. Rodríguez

posiciones normativas: la negación externa (que se representará como “∼”)


y la negación interna (que se representará como “¬”). Mientras la negación
externa es una operación que lleva de la proposición normativa que afirma
la pertenencia de una norma a un sistema a la proposición normativa que
afirma la no pertenencia de tal norma al sistema, la negación interna es una
operación que lleva de la proposición normativa que afirma la pertenencia
de una norma a un sistema a la proposición normativa que afirma la perte-
nencia de su norma-negación al sistema. Por ejemplo:

Negación externa: ∼PHαp =def “PHp” ∉ α


Negación interna: ¬PHαp =def “∼PHp” ∈ α

Esto es, la negación externa de la proposición que afirma que la conducta


p está prohibida en el sistema normativo α equivale a negar la pertenencia
de la norma que prohíbe p al sistema, mientras que la negación interna de tal
proposición equivale a afirmar la pertenencia al sistema de una norma que
establece la no prohibición de p.
Ahora bien, adviértase que bajo una interpretación prescriptiva, decir
que una conducta no está prohibida equivale a decir que está permitido
realizarla. Bajo una interpretación descriptiva, en cambio, la expresión
“permitido” se vuelve ambigua: una conducta puede ser calificada como
permitida simplemente en razón de que nadie la ha prohibido, pero también
puede calificársela como permitida en caso de que la autoridad normativa
haya autorizado su realización. Estas dos nociones distintas, indiscernibles
en el marco de una lógica de normas, pueden ser denominadas permisión
negativa (P−αp) y permisión positiva (P+αp).

Permisión negativa: P−αp =def “PHp” ∉ α


Permisión positiva: P+αp =def “∼PHp” ∈ α =def “Pp” ∈ α

Como puede advertirse, la permisión negativa equivale a la negación


externa de la prohibición de p, mientras que la permisión positiva equivale
a la negación interna de la prohibición de p. La permisión negativa equivale
a la no obligatoriedad de la abstención de una conducta, por lo cual se dirá
que una conducta p está permitida en el sentido negativo en α sí, y sólo si, p
no está prohibida en α, es decir, si no existe en α una norma que prohíba p.
La permisión positiva depende, en cambio, de un acto llevado a cabo por la
autoridad normativa, no de una mera omisión. En consecuencia, se dirá que
una conducta está permitida en el sentido positivo en un sistema normativo
α si, y sólo si, una norma que permite p pertenece a α.
Naturaleza y lógica de las proposiciones normativas... 99

La diferencia entre la permisión negativa y la permisión positiva quizás


pueda entenderse con mayor claridad del siguiente modo: si una conducta
p está permitida en sentido negativo en un sistema normativo y una auto-
ridad decide prohibirla, no surgirá ningún conflicto. Como resultado de la
prohibición, la conducta p, antes no prohibida, pasará a estar prohibida en
el sistema. Pero si la conducta p estaba permitida en sentido positivo y la
autoridad decide prohibirla, surgirá en el sistema un conflicto normativo: p
estará permitida y prohibida a la vez.
Aunque la ambigüedad del término “permitido” fue advertida por von
Wright, el profesor finlandés consideró por cierto tiempo que la permisión
positiva tenía carácter normativo, mientras que la negativa no.15 En otras
palabras, von Wright distinguía sólo dos conceptos de permisión, dado que
identificaba la permisión prescriptiva con la permisión positiva. Y como
suponía que la permisión positiva implicaba a la negativa, denominaba a la
primera “permisión fuerte” y a la segunda “permisión débil”. En realidad,
resulta necesario diferenciar tres conceptos de permisión, uno prescriptivo y
dos descriptivos: el permiso positivo y el negativo. Es más: para una lógica
de proposiciones normativas se puede tomar como base al operador obli-
gatorio (o al operador prohibido) de la lógica de normas y definir a partir
de él dos conceptos descriptivos de permisión y dos de facultamiento, pero
también podría tomarse como base al operador permitido y definir a partir
de él dos conceptos de prohibición y dos de obligación:

Base O: Base P:

(1) Oαp =def “Op” ∈ α (1”) O+αp =def “∼P∼p” ∈ α


(2) PHαp =def “O∼p” ∈ α (1’) O−αp =def “P∼p” ∉ α
+
(3) P p =def “∼O∼p” ∈ α
α
(2”) PH+αp =def “∼Pp”∈ α
(3’) P−αp =def “O∼p” ∉ α (2’) PH−αp =def “Pp” ∉ α
(4) F+αp =def “∼Op” ∈ α ∧ “∼O∼p” ∈ α (3”) Pαp =def “Pp” ∈ α

(4’) F p =def “Op”∉ α∧ “O∼p” ∉ α
α
(4”) Fαp =def “Pp”∈ α ∧ “P∼p” ∈ α

Las fórmulas identificadas con números seguidos de comillas dobles


equivalen a las numeradas sin comillas de la otra presentación, mientras
que las numeradas con una comilla no tienen equivalente en la versión
alternativa. De esto resulta que, desde un punto de vista formal, a partir de
la ambigüedad de la negación de las proposiciones normativas, pueden dis-

15
Cf. G. H. von Wright, Norma y Acción. Una investigación lógica, ya citado, página
102.
100 Jorge L. Rodríguez

tinguirse los siguientes cuatro pares de conceptos, con sus correspondientes


equivalencias:

(1) O+αp = ∼P−α∼p = ¬P+α∼p (1’) O−αp = ∼P+α∼p = ¬P−α∼p


(2) PH+αp = ∼P−αp = ¬P+αp (2’) PH−αp = ∼P+αp = ¬P−αp

+
(3) P p = ∼PH αp = ¬PH p
α
+
α
(3’) P−αp = ∼PH+αp = ¬PH−αp
− −
+ +
(4) F p = ∼O αp ∧ ∼O α∼p = ¬O αp ∧ ¬O ∼p
α
+
α
(4’) F−αp = ∼O+αp ∧ ∼O+α∼p = ¬O−αp ∧
¬O−α∼p

Sin embargo, los conceptos de prohibición negativa (PH−α) y obligación


negativa (O−α) no parecen tener correlato en el lenguaje corriente, razón
por la cual se utiliza en general una presentación sobre la base del operador
obligatorio y se habla exclusivamente de dos conceptos descriptivos de
permisión.16
El primer desarrollo de una lógica de proposiciones normativas se debe
a Carlos Alchourrón.17 Su propuesta podría presentarse axiomáticamente del
siguiente modo:

Sistema LPN:

(A1) |− Oα(p ∧ q) ↔ (Oαp ∧ Oαq)


(A2) |− Oαp → P+αp
(A3) |− P+α(p ∧ q) → P+αp

(RI1) De |− (p ↔ q), se sigue |− (Oαp ↔ Oαq)


(RI2) De |− (p ↔ q), se sigue |− (P+αp ↔ P+αq)18

De la comparación entre este sistema y el sistema de lógica de normas


(LN) surge, en primer lugar, que las proposiciones normativas son relativas
a un determinado sistema normativo, lo cual se refleja en la aparición de los
suscriptos (α) en las fórmulas correspondientes. Por contraste, las expresio-
nes de la lógica de normas no están referidas a un sistema normativo, sino

16
“(...) podríamos definir también la noción de prohibición débil, como la ausencia (en α)
de una norma que permita p. Pero esta noción es muy poco usada: sólo en circunstancias muy
especiales una conducta se calificaría como prohibida por el mero hecho de que no estuviera
expresamente permitida. Menos usual todavía es la noción de obligación débil” (C. Alchou-
rrón y E. Bulygin, Introducción a la metodología de las ciencias jurídicas y sociales, Buenos
Aires, Astrea, 1975, título original: Normative Systems, Wien-New York, Springer Verlag,
1971, traducido por los autores, página 177, nota).
17
Cf. C. Alchourrón, “Logic of Norms and Logic of Normative Propositions”, ya citado.
18
Cf. C. Alchourrón, “Philosophical Foundations of Deontic Logic and the Logic of De-
feasible Conditionals”, ya citado.
Naturaleza y lógica de las proposiciones normativas... 101

que se trata en este sentido de conceptos absolutos. En segundo lugar, en el


sistema de lógica de proposiciones normativas no existe nada parecido al
axioma A0 del sistema LN. Por otro lado, en el sistema LPN hay un axioma y
una regla de inferencia (A3 y RI2) que no se encuentran en LN. No obstante,
esta última es una diferencia menor: si se pudiera agregar al sistema de ló-
gica de proposiciones normativas un análogo del axioma A0 de LN, A3 y RI2
resultarían superfluos por cuanto serían deducibles de los restantes axiomas.
El punto en discordia es, pues, el axioma A0.
¿Por qué razón A0 no puede tomarse como axioma en un sistema de
lógica para proposiciones normativas? Porque, como se ha visto, hay dos
formas distintas en las que una proposición normativa puede ser negada, las
cuales originan dos conceptos descriptivos para cada uno de los operadores
deónticos. Y aunque la negación externa satisface todos los requisitos que se
espera que cumpla una negación −los mismos cinco principios que se enun-
ciaron en formulación analógica como satisfechos por la negación de las
normas−, ello no ocurre con la negación interna. Una proposición normativa
y su negación interna pueden ser ambas verdaderas −en cuyo caso el sistema
será inconsistente− tanto como ambas falsas −en cuyo caso el sistema será
incompleto−.19 De ahí que en la lógica de proposiciones normativas no sea
válida la equivalencia expresada por el axioma A0 de la lógica de normas.
Pese a las diferencias apuntadas, puede demostrarse que bajo ciertos
presupuestos los sistemas LN y LPN serían equivalentes. Si un sistema
normativo satisface las dos condiciones siguientes respecto de cualquier
conducta p, puede probarse que sería válido para LPN un análogo del axio-
ma A0 de LN o, lo que es lo mismo, que la permisión positiva P+ equivaldría
a la permisión negativa P−:

12. ∼(Oα∼p ∧ P+αp) (Consαp)


13. Oα∼p ∨ P+αp (Compαp)

Ello en virtud de que

14. ∼(Oα∼p ∧ P+αp) ∧ Oα∼p ∨ P+αp

equivale a

15. P+αp ↔ ∼Oα∼p

19
Cf. E. Bulygin, “Lógica Deóntica”, ya citado.
102 Jorge L. Rodríguez

Demostración:20
16. ∼(Oα∼p ∧ P+αp)
17. ∼(P+αp ∧ Oα∼p) conmutatividad de la conjunción en 16.
18. P+αp → ∼Oα∼p definición del condicional en 17.
19. ∼(Oα∼p ∧ P+αp) → P+αp → ∼Oα∼p introducción del condicional de 16 a 18.

20. Oα∼p ∨ P+αp


21. (∼Oα∼p → P+αp) definición del condicional en 20.
22. Oα∼p ∨ P+αp → (∼Oα∼p → P+αp) introducción del condicional de 20 a 21.

De esto se sigue que:


23. ((Consαp) ∧ (Compαp)) ↔ (P+αp ↔ ∼Oα∼p)

y dado que
24. ∼Oα∼p ↔ P−αp

entonces:
25. ((Consαp) ∧ (Compαp)) ↔ (P+αp ↔ P−αp)

En otras palabras, bajo el supuesto de completitud y consistencia, el


sistema LPN se comporta de modo semejante al sistema LN. Igualmente,
podría decirse que un sistema de lógica de normas equivale a un sistema de
lógica de proposiciones normativas para un sistema completo y consistente.
Una lógica de normas pretendería reconstruir la racionalidad en el dictado
de normas, y un legislador racional no debería contradecirse ni dejar con-
ductas sin determinar normativamente. Por el contrario, desde una lógica de
proposiciones normativas no puede suponerse la completitud ni la consis-
tencia del sistema al que se hace referencia, puesto que de hecho un sistema
normativo puede contener lagunas y contradicciones.
Finalmente, puede demostrarse que:
Proposición 1: Si un sistema normativo es inconsistente respecto de
la acción p, entonces es completo respecto de la acción p: (∼(Consαp) →
(Compαp)).

26. Oα∼p ∧ P+αp


27. Oα∼p eliminación de la conjunción en 26.
28. Oα∼p ∨ P+αp introducción de la disyunción en 27.
29. (Oα∼p ∧ P+αp) → (Oα∼p ∨ P+αp) introducción del condicional de 26 a 28.

20
Aquí, como en las demostraciones de la página siguiente, doy por conocidas las reglas
básicas del cálculo de deducción natural de Gentzen (cf. G. Gentzen, “Untersuchungen über das
Logische Schliessen”, en Mathematische Zeitschrift 39, 1934, páginas 176-210).
Naturaleza y lógica de las proposiciones normativas... 103

Proposición 2: Si un sistema normativo es incompleto respecto de la


acción p, entonces es consistente respecto de la acción p: (∼(Compαp) →
(Consαp)).
30. ∼(Oα∼p ∨ P+αp)
31. ∼Oα∼p ∧ ∼P+αp De Morgan en 30. (∼(p ∨ q) ↔ (∼p ∧ ∼q)).
32. ∼Oα∼p eliminación de la conjunción en 31.
33. Oα∼p ∧ P+αp
34. Oα∼p eliminación de la conjunción en 33.
35. Oα∼p ∧ ∼Oα∼p introducción de la conjunción en 32 y 34.
36. ∼(Oα∼p ∧ P+αp) introducción de la negación de 33 a 35.
37. ∼(Oα∼p ∨ P+αp) → ∼(Oα∼p ∧ P+αp) introducción del condicional de 30 a 36.

Estas dos demostraciones pueden generalizarse debido a la aplicación del


principio ex falso sequitur quodlibet ((p ∧ ∼p) |− q), de lo que se sigue:
Proposición 3: Si un sistema normativo es inconsistente respecto de
una cierta acción, entonces es completo respecto de cualquier acción (∃x
∼(Consαx) → ∀x (Compαx)).
Proposición 4: Si un sistema normativo es incompleto respecto de una
cierta acción, entonces es consistente respecto de cualquier acción (∃x
∼(Compαx) → ∀x (Consαx)).

5. Dos visiones de la lógica de proposiciones normativas


Pese a que, como se dijo, ni a la distinción entre normas y proposiciones
normativas, ni mucho menos a la diferencia entre una genuina lógica de
normas y una lógica de proposiciones normativas, se les ha reconocido la
importancia que a mi criterio ellas merecen, puede decirse que hasta no hace
mucho tiempo el sistema de lógica de proposiciones normativas propuesto
por Alchourrón, y desarrollado posteriormente por él mismo en colabora-
ción con Bulygin en diversos estudios, reinaba como clásico sin tener que
disputar su lugar con ninguna propuesta alternativa.
Sin embargo, en un breve y reciente trabajo, von Wright presentó un
camino diferente para reconstruir una lógica de proposiciones normativas.21
De acuerdo con este sistema, la forma esquemática para enunciados deón-
ticos consistiría en:
a) una variable de enunciado o compuesto molecular de variables de
enunciado a las cuales se ha antepuesto un operador deóntico O o P, o
b) un compuesto molecular de enunciados como los definidos en a).

21
Cf. G. H. von Wright, “On norms and Norm-Propositions. A Sketch”, en W. Krawietz
et al., The Reasonable as Rational? On Legal Argumentation and Justification. Festschrift for
Aulis Aarnio, Berlin, Duncker & Humblot, 2000, páginas 173-178.
104 Jorge L. Rodríguez

Bajo una lectura descriptiva, los enunciados deónticos expresarían


proposiciones normativas, y para ellas valdrían las leyes de la lógica de
enunciados (proposicional) clásica, así como también un principio según el
cual compuestos lógicamente equivalentes de variables de enunciado son
sustituibles salva veritate. No se presupondría que valga ninguna verdad
lógica especial para las proposiciones normativas. En particular, una serie
de fórmulas que en su lectura prescriptiva son consideradas válidas (esto
es, en el marco de una lógica de normas), no resultarían aquí lógicamente
verdaderas. Entre ellas:

38. ∼O∼p → Pp
39. Pp → ∼O∼p
40. Pp ∨ P∼p
41. O (p ∧ q) → ∼P∼p
42. P (p ∧ q) → ∼O∼p

Es fácil advertir, tal como el propio von Wright lo señala, que (38) equi-
vale al principio de completitud y (39) al principio de consistencia. Como
se examinó en el punto anterior, ninguno de estos principios, sin calificacio-
nes, es admitido por Alchourrón en su sistema de lógica de proposiciones
normativas. Se trata en ambos casos de características contingentes de los
sistemas normativos.
Sin embargo, como también puntualiza von Wright, (39) equivale a “Op
→ ∼P∼p” y (40) a “∼P∼p → Pp”. De manera que, conjuntamente, implican
por transitividad “Op → Pp”, cuya satisfacción por un sistema normativo
sería entonces también contingente. Aquí “Pp” debería interpretarse, en la
lectura descriptiva propuesta, como “P+αp”, esto es, como una expresión re-
ferida a la permisión positiva. Ahora bien, esto choca con la asunción, en el
sistema de Alchourrón, del axioma A2. Por su parte, (41) en conjunción con
(38) y (39) implica “O(p ∧ q) → (Op ∧ Oq)”, que constituye el axioma A1 del
sistema de Alchourrón. Y (42) en conjunción con (38) implica “P(p ∧ q) →
Pp”, esto es, el axioma A3 del sistema de Alchourrón. En consecuencia, si tal
como lo sostiene von Wright ninguna de las fórmulas listadas de (38) a (42)
fuesen lógicamente verdaderas, ¡no quedaría en pie ninguno de los axiomas
que Alchourrón asume para una lógica de proposiciones normativas!
Esto, que a primera vista parece tan sorprendente, no debería en realidad
llamar nuestra atención, siendo que von Wright advierte, como se adelantó,
que en su sistema no vale ninguna verdad lógica especial para las propo-
siciones normativas. Lo notable, en todo caso, lo que requiere de alguna
justificación, es la profunda diferencia de enfoques que parece signar a uno
y otro sistema.
Naturaleza y lógica de las proposiciones normativas... 105

Un primer paso para intentar explicar esa diferencia consiste en tomar


en cuenta la distinción entre lo que Alchourrón y Bulygin han denominado
concepción hilética y concepción expresiva de las normas.22 De acuerdo
con la concepción hilética, las normas son entendidas como el significado
de ciertos enunciados −las formulaciones normativas−, por lo que se trataría
de entidades semejantes a las proposiciones. Las formulaciones normativas
constituirían expresiones lingüísticas de normas y las normas serían los
significados de tales enunciados prescriptivos, así como las proposiciones
son −para ciertos autores− el significado de los enunciados descriptivos.
La diferencia radicaría en que los enunciados descriptivos indican que algo
es de una cierta manera, mientras que los enunciados prescriptivos indican
que algo debe o no debe o puede ser hecho. De acuerdo con la concepción
expresiva, las normas son el resultado del uso prescriptivo del lenguaje. Un
mismo enunciado podría ser usado en diferentes ocasiones para hacer cosas
distintas: aseverar, ordenar, interrogar, etc. El resultado de esas acciones
sería una aserción, una orden o una pregunta, etc., por lo que tan sólo en el
nivel pragmático del uso del lenguaje se localizaría la diferencia entre aser-
ciones, órdenes y preguntas, etc., no en el nivel semántico.
Entre los partidarios de la concepción hilética hay quienes consideran
que las normas son susceptibles de verdad o falsedad y, consiguientemente,
aceptan la posibilidad de una lógica de normas junto a una lógica de pro-
posiciones normativas. La lógica de normas se ocuparía de las relaciones
entre proposiciones prescriptivas, mientras que la lógica de proposiciones
normativas se ocuparía de las relaciones lógicas entre las proposiciones
descriptivas acerca de normas. Otros partidarios de esta concepción, en
cambio, no aceptan que se pueda hablar de verdad o falsedad de las normas,
no obstante lo cual creen en la posibilidad de una lógica de normas, pero
para ello es necesario concebir a la lógica como no circunscripta exclusi-
vamente a enunciados verdaderos o falsos. Podría decirse entonces que la
creencia en la existencia de relaciones lógicas entre normas no compromete
con la aceptación de que las normas sean verdaderas o falsas. Para los par-
tidarios de la concepción expresiva en principio sólo es posible una lógica
de proposiciones normativas, es decir, de enunciados que describen normas.
Desde esta perspectiva, no habría lógica de normas sin referencia al sistema
normativo del cual tales normas forman parte.23

22
Cf. C. Alchourrón y E. Bulygin, ”The Expressive Conception of Norms”, en R. Hilpi-
nen (ed.), New Studies in Deontic Logic, Dordrecht-Boston-London, Reidel, 1981, páginas
95-124.
23
Cf. D. Makinson, “On a Fundamental Problem of Deontic Logic”, en P. McNamara y
H. Prakken (eds.), Norms, Logics and Information Systems. New Studies on Deontic Logic and
Computer Science, Amsterdam, IOS Press, 1998.
106 Jorge L. Rodríguez

Según Alchourrón y Bulygin, adoptar la concepción expresiva de las


normas no condenaría al discurso normativo a un completo irracionalismo,
puesto que la racionalidad podría preservarse en el plano de las proposicio-
nes normativas. Si bien puede admitirse esta idea, es importante destacar
que la lógica de proposiciones normativas que desarrollan Alchourrón y
Bulygin respecto de la concepción expresiva de las normas es sustancial-
mente idéntica a la que atribuyen a la concepción alternativa, esto es, a la
concepción hilética. Ambas responden al modelo LPN reseñado en el punto
anterior, y en ambas se presupone una noción de sistema normativo clausu-
rado bajo la noción de consecuencia. Así, en “La concepción expresiva de
las normas”, los autores argentinos sostienen:
“De esta manera, la existencia de una norma (= pertenencia del contenido
normativo al conjunto ordenado) depende de ciertos hechos empíricos (actos de
promulgación en el caso de las prescripciones; ciertas acciones reveladoras de
disposiciones en el caso de las normas consuetudinarias). Por lo tanto, como no
hay relaciones lógicas entre hechos, no hay cabida para una lógica de normas.
Pero esto no precluye la posibilidad de una lógica de proposiciones normativas.
En efecto, tal como ya lo hemos señalado, la proposición de que p es obligatorio
en A es verdadera si p ha sido ordenado por Rex y pertenece, por lo tanto, al
conjunto ordenado A. Pero esto es una condición suficiente, pero no necesaria
para la verdad de la proposición “p es obligatorio en A”. Puede ocurrir que p no
haya sido nunca ordenado por Rex y, sin embargo, p sea obligatorio en A. Su-
pongamos, por ejemplo, que Rex no haya ordenado p, pero sí ha ordenado p&q.
Esta es una proposición diferente de p y, conforme a nuestro criterio, p no perte-
necería a A. Pero como p es una consecuencia lógica de p&q (pues p&q implica
lógicamente p), también es verdad que p es obligatorio en A. La obligatoriedad
de p es una consecuencia lógica de la obligatoriedad de p&q, porque p es una
consecuencia lógica de p&q. Podemos definir ahora el concepto de sistema
normativo como el conjunto de todas las proposiciones que son consecuencias
lógicas de las proposiciones ordenadas expresamente. (Aunque estemos usando
la expresión tradicional “sistema normativo”, hay que tener presente que en la
concepción expresiva un sistema normativo no es un conjunto de normas, sino
un conjunto de contenidos normativos, esto es, de proposiciones). Esto hace po-
sible distinguir entre el conjunto A (formado por todas las proposiciones expre-
samente ordenadas) como la base axiomática del sistema y el sistema normativo
Cn(A), que es el conjunto de las consecuencias de A. Estamos en condiciones de
poder corregir nuestro criterio de verdad para las proposiciones normativas: “p
es obligatorio en A” es verdadera si, y sólo si, p pertenece al sistema Cn(A), es
decir, si, y sólo si, p pertenece a las consecuencias de A”.24
Ahora bien, si la reconstrucción de una lógica de proposiciones norma-
tivas ha de servir tanto a un partidario de la concepción hilética como a un
partidario de la concepción expresiva, la asunción de una noción de sistema
normativo clausurado bajo la noción de consecuencia parece inadmisible,

24
C. Alchourrón y E. Bulygin, “The Expressive Conception of Norms”, ya citado.
Naturaleza y lógica de las proposiciones normativas... 107

puesto que el expresivista no admitiría relaciones lógicas entre normas.


El modo en el que Alchourrón y Bulygin intentan salvar esta dificultad al
presentar su visión de la lógica de proposiciones normativas en el marco de
la concepción expresiva consiste en no reconocer directamente relaciones
lógicas entre normas, pero interpretar al “sistema normativo” como un con-
junto de contenidos normativos, conjunto que sí se hallaría clausurado bajo
la noción de consecuencia. No obstante, con esto las dos versiones de la
lógica de proposiciones normativas confluyen en el sistema LPN.
¿Qué pasaría, en cambio, si se abandona ese presupuesto? En LPN,
como vimos, hay verdades lógicas específicas para las proposiciones nor-
mativas: que obligatorio p en el sistema normativo α implica permitido en
sentido positivo p en α; o que la obligatoriedad de la conjunción de p y q en
α equivale a sostener que es obligatorio p y es obligatorio q en α; o que la
permisión positiva de la conjunción de p y q en α implica la permisión de
cualquiera de los conjuntos en α. Ahora bien, si se piensa en una lógica de
proposiciones normativas relativa a un sistema que no comprende todas sus
consecuencias lógicas, no sólo debería admitirse que un sistema semejante
puede ser inconsistente o incompleto (porque prohibido p en α y permitido
en sentido positivo p en α pueden ser ambas verdaderas tanto como ambas
falsas), al igual que en LPN, sino además que podría ser verdad que es obli-
gatorio p en α y, no obstante, no estar permitido en sentido positivo p en α;
o que sea obligatorio p y q en α y no sea obligatorio p en α; o que sea verdad
que está permitido p y q en α y no esté permitido p en α. Que se satisfagan o
no principios tales como que obligatorio implica permitido, o la distributivi-
dad de la obligación o la permisión por la conjunción, sería una cuestión tan
contingente como la satisfacción de la completitud y la consistencia.
Esto es exactamente lo que ocurre en el sistema que esboza von Wright.
A diferencia de Alchourrón y Bulygin, von Wright toma en cuenta propo-
siciones normativas relativas a un sistema no clausurado bajo la noción
de consecuencia, sino sólo compuesto por las normas de base, esto es, las
expresamente dictadas. El sistema de von Wright se limita a describir actos
normativos, y del hecho de que se haya dictado en α la norma "Op", por
ejemplo, no se sigue que se haya dictado en α la norma "Pp". En la lógica
de normas, “Op” puede definirse como “Pp ∧ ∼P∼p", y ciertamente "Op"
implica "Pp". Pero el operador descriptivo "Oαp" (es obligatorio en el sis-
tema α la acción p) equivale a “Op” ∈ α, donde la norma "Op" está siendo
mencionada, no usada. Por eso, de “Op” ∈ α no se sigue que "Pp" ∈ α,
salvo que se presuponga que α no está sólo conformado por las normas
expresamente dictadas, sino que comprende todas sus consecuencias lógicas
(Cn(α) = α). En otras palabras, del hecho de que una acción sea calificada
como obligatoria según las normas del sistema α se sigue que dicha acción
108 Jorge L. Rodríguez

está permitida en sentido positivo en α sólo bajo el presupuesto de clausura


bajo la noción de consecuencia deductiva. Lo mismo ocurre con la distribu-
tividad de la obligación o la permisión por la conjunción.
Estas dos alternativas de reconstrucción de la lógica de proposiciones
normativas se corresponden entonces con dos diferentes concepciones de
los sistemas normativos: una según la cual se los reconstruye como exclu-
sivamente compuestos por las normas de base, y otra según la cual se los
reconstruye como la totalidad de las consecuencias que se siguen de ciertas
normas de base. Puede resultar instructivo considerar en este punto cómo
se encara un problema semejante en el terreno de los estudios lógicos sobre
dinámica de creencias. En dicho campo es usual el empleo alternativo de
dos estrategias diferentes para enfocar el análisis, a las que se denomina, res-
pectivamente, bases de creencias y teorías. De acuerdo con esta distinción, A
es una base de creencias para una teoría K si y sólo si K = Cn(A). En otros
términos, una teoría es un conjunto de creencias que comprende todas sus
consecuencias lógicas, mientras que una base de creencias es un conjunto
no clausurado bajo la noción de consecuencia.25 Partir de bases de creencias
y no de teorías ofrecería ciertas ventajas, puesto que las bases son finitas, lo
que permite su tratamiento computacional, y poseen además mayor poder
expresivo que las teorías. No obstante, el enfoque de las bases de creencias
tiene la desventaja de que es muy difícil diferenciar entre creencias básicas
y derivadas. Este problema, sin embargo, no se presenta respecto de ciertos
sistemas normativos, tales como los sistemas jurídicos, a cuyo respecto exis-
ten criterios convencionalmente aceptados para diferenciar entre normas de
base y normas derivadas a partir de las primeras.
El sistema de von Wright parece el más adecuado para dar cuenta de
proposiciones normativas respecto de un sistema de normas en el que éstas
se conciben como actos de prescribir y, por consiguiente, que no admite re-
laciones lógicas entre las propias normas. E incluso desde este punto de vista
también es posible preservar la idea que subyace a los desarrollos de lógicas
de normas, interpretándolos como sistemas de lógica de proposiciones norma-
tivas relativas a un conjunto de normas dictado por un legislador racional.26
Marzo de 2004

25
Cf. P. Gärdenfors, “The Dynamics of Normative Systems”, en A. Martino (ed.), Expert
Systems in Law, Elsevier Science Publishers B.V., 1992, páginas 195-200 y S. O. Hansson, A
Textbook of Belief Dynamics. Theory Change and Database Updating, Dordrecht-Boston-Lon-
don, Kluwer Academic Publishers, 1999.
26
Cf. G. H. von Wright, “Norms, Truth and Logic”, ya citado.

DOXA 26 (2003)

módulo 7
Lectura Básica
referencia página 156

Aplicación de la lógica al lenguaje ordinario

EDUBP | Abogacía | Filosofía y Lógica Jurídica - pag.225


Mientras que el nexo entre premisas y conclusión se basa en la lógica, con
frecuencia las premisas y conclusión en sí mismas no se fundan en ella y, en
esta circunstancia reside:

La aplicación de la lógica a dominios distintos de sí misma, por ejem-


plo su aplicación al lenguaje ordinario.

Las premisas y la conclusión pueden versar sobre cualquier tema y se


ejemplifican mejor en lenguaje ordinario o natural que en lenguaje formal.
Introduciendo letras esquemáticas distorsionamos un tanto los enunciados
para resaltar sus estructuras. La paráfrasis a lenguaje formal de enuncia-
dos en lenguaje natural reduce giros lingüísticos a símbolos uniformes, pero
cierta correspondencia cabe establecer. La conjunción, en lenguaje natu-
ral no sólo se expresa mediante ‘y’, sino también por medio de ‘pero’, de
‘aunque`, de signos de puntuación, etc. La diferencia entre el uso de ‘pero’
y de ‘aunque’ hace explícita la distinción entre los aspectos lógicos y los
aspectos retóricos del lenguaje. Pero las diferencias retóricas no bastan para
señalar diferencias entre la verdad o falsedad de la conjunción. En el caso
del condicional hay también variantes en el lenguaje natural para el uso de
‘si p, entonces q’: ‘p sólo si q’, ‘q supuesto que p’, ‘q en caso de que p’
‘no p a menos que q’. Nótese que el antecedente de un condicional ‘p’,
no es siempre en el habla ordinaria la parte del condicional que ocupa el
primer lugar en la oración emitida. Por ejemplo, ‘si’ es signo del antecedente
y, si decimos ‘p si q’ en lenguaje formal se transforma en ‘qvp’. El ‘sólo’ en
cambio es signo del consecuente y, por ende, ‘p sólo si q’ significa ‘si p,
entonces q’. Por ejemplo: ‘se licenciará sólo si paga su matrícula’ significa
‘si se licencia, entonces ha pagado su matrícula’. Hay que tener claro que
‘sólo si’ no tiene el significado de ‘≡’, el cual es ‘si, y solamente sí’ (‘si y sólo
si’). Con respecto a la disyunción ‘a menos que’ es uno de los significados
de ‘o’. La labor de paráfrasis requiere también, en ocasiones, reformular las
oraciones componentes a fin de salvaguardarlas de alteraciones en su signi-
ficado. Esta necesidad se hace evidente en el siguiente ejemplo:
(1) Fue a Villaconejos y yo lo acompañé,
(2) Fue a Vilamarranos, pero yo no lo acompañé.

Estas dos conjunciones pueden ser ambas verdaderas simultáneamente; no


obstante, si las representamos como ‘p. q’ y ‘r. –q’ el compuesto resulta
inconsistente: ‘p. q. r. –q’. Pero debe notarse que el ‘yo le acompañé’ de
(1) no es el de (2), pues en (1) es ‘yo le acompañé a Villaconejos” y en (2)
‘no le acompañé a Villamarranos’. Por lo tanto, lo correcto es reemplazar-
los en lenguaje formal por variables diferentes: ‘p. q’ y ‘r. –s’, con lo cual
la aparente inconsistencia del compuesto desaparece. Es importante en el
análisis formal no asignar diferentes interpretaciones a una misma expresión
en el curso del mismo argumento. La vilación de este principio se denomina
‘falacia de equivocidad’. En relación a la corrección lógica no importa cómo
interpretemos los enunciados que componen un argumento, lo que importa
es que su interpretación de expresiones ambiguas sea uniforme a lo largo de
todo el argumento. Por último, cuando un enunciado es complejo, es conve-
niente comenzar por la estructura más externa y parafrasear hacia adentro,
paso por paso. Por ejemplo:

EDUBP | Abogacía | Filosofía y Lógica Jurídica - pag.226


(3) “Si Pérez está enfermo o Ruiz está ausente entonces ni se con-
cluirá el asunto Argus ni se reunirán los directores y anunciarán
un dividendo a menos que López entre en razón y tome el asunto
en sus propias manos”

Primero cabe localizar el conector que divide la estructura más externa del
enunciado:

“Si Pérez está enfermo o Ruiz está ausente v ni se concluirá. . . . . .”. Luego
dividiremos la estructura de la parte que comienza con “ni” y termina con
“dividendo”. Su conector principal es “ni-ni”. Como ‘ni r ni s’ se simboliza
como ‘–r . –s’. Siguiendo este procedimiento, transformamos (3) en:

(4) (Pérez está enfermo v Ruiz está ausente) v ( – (se concluirá el asunto
Aarhus) . – (se reunirán los directores. los directores anunciarán un
dividendo ) v ( López entrará en razón . López tomará el asunto en
sus manos) ).

Esquemáticamente formulada, la estructura total es la siguiente:

(5) (p v q) v ( – r . – (s. t) v u . w)

EDUBP | Abogacía | Filosofía y Lógica Jurídica - pag.227


módulo 7
Lectura Básica
referencia página 156

Clases de silogismo

EDUBP | Abogacía | Filosofía y Lógica Jurídica - pag.228


A.- Silogismo Disyuntivo:
O bien Fido se escapó, o bien Fido fue atropellado por un automóvil.
Fido se escapó.
Por tanto, Fido no fue atropellado por un automóvil.

¿Es un silogismo disyuntivo válido? NO, porque niega en la conclusión la


otra disyuntiva y ambas disyuntivas pueden ser verdaderas.

B.- Silogismo Disyuntivo:


O bien Fido se escapó, o bien Fido fue atropellado por un automóvil.
Fido no se escapó.
Por tanto, Fido fue atropellado por un automóvil.
Silogismo disyuntivo válido pues la premisa categórica niega una de las
disyuntivas y la conclusión afirma la otra.

C.- Silogismo hipotético puro:


Si consecuente de primera premisa es igual al antecedente de la segunda
premisa.
Ejemplo de razonamiento válido:
Si el primer nativo es un político, entonces miente.
Si el primer nativo miente, entonces niega ser un político.
Conclusión: Si el primer nativo es un político, entonces niega ser un político.

D.- Silogismo hipotético mixto:


Es el que tiene una premisa condicional y una categórica. Posee dos formas
válidas:
1.- La premisa categórica afirma antecedente de la primera premisa y conclu-
sión afirma su consecuente.
Si el segundo nativo dice la verdad, entonces sólo un nativo es un político.
El segundo nativo dice la verdad.
Conclusión: sólo un nativo es un político.
La forma inválida de este razonamiento sería la falacia de afirmación del
consecuente, la premisa categórica no afirma el antecedente, sino el conse-
cuente de la premisa condicional, por ejemplo:
Si Bacon escribió Hamlet, entonces era un gran escritor.
Bacon era un gran escritor.
Conclusión: Bacon escribió Hamlet.
2.- La premisa categórica niega consecuente de premisa condicional y con-
clusión niega su antecedente.

EDUBP | Abogacía | Filosofía y Lógica Jurídica - pag.229


Si el prisionero tuerto vio dos sombreros rojos, entonces pudo averiguar
el color de su propio sombrero.
El prisionero tuerto no pudo averiguar el color de su propio sombrero.
Conclusión: el prisionero tuerto no vio dos sombreros rojos.
La forma inválida de este razonamiento sería la falacia de negar el antece-
dente, la premisa categórica niega el antecedente y no el consecuente de la
premisa condicional.
Si Carlos desfalcó los fondos del Colegio, entonces Carlos es culpable
de un delito grave.
Carlos no desfalcó los fondos del Colegio.
Conclusión: Carlos no es culpable de un delito grave.

EDUBP | Abogacía | Filosofía y Lógica Jurídica - pag.230

También podría gustarte